You are on page 1of 366

B GIO DC V O TO

(Gio trnh o to gio vin


trung hc h i hc,
Cao ng s ph m)

NH XUT BN GIO DC

B GIO DC V O TO
HONG HUY SN

BI TP

I S
S CP

Gio trnh o to gio vin trung hc


h i hc, Cao ng s ph m
( Ti bn ln th 10)

NH XUT BN GIO DC

512/GD-01/1536/358-00

M s: 7K300T1

LI NI U
Khi bin son ti liu i s s cp chng ti c gng a nhiu v d v thc
hnh gii ton nhm gip sinh vin c iu kin rn k nng thc hnh khi hc l thuyt.
Tuy nhin, qua thc t ging dy, chng ti thy rng khi gii cc bi tp trong sch, sinh
vin gp rt nhiu kh khn. Ngay c khi bit cch gii th vic trnh by li gii sao cho
cht ch v logic th cng cn cha t so vi yu cu. V th, gip sinh vin c mt b
ti liu hon chnh v i s s cp, chng ti tip tc bin son cun Bi tp i s s
cp ny phc v nhu cu hc tp v k c cng vic ging dy ca sinh vin sau khi ra
trng.
Ti liu Bi tp i s s cp gm c hai phn:
Phn I. Tm tt l thuyt v bi.
Phn II. Li gii v hng dn.
Mi phn gm su chng:
1. Chng I: Hm s;
2. Chng II: Phng trnh H phng trnh;
3. Chng III: Bt ng thc Bt phng trnh;
4. Chng IV: Phng trnh, bt phng trnh v t;
5. Chng V: Phng trnh, bt phng trnh m v logarit;
6. Chng VI: Phng trnh lng gic.
Th t cc chng c trnh by theo ng th t cc chng mc trong ti liu
i s s cp. Ti liu c 170 bi tp vi khong gn 550 cu nh. Hu ht cc bi tp
trong ti liu Bi tp i s s cp c chng ti trnh by li gii tng i chi tit
nhm gip sinh vin nht l sinh vin cc lp h o to Lin thng Cao ng ln i hc
d dng trong vic cng c l thuyt v gii cc bi tp tng t. Mt s bi c trnh by
nhiu cch gii, mc ch gip sinh vin c cch tip cn v i n kt qu ca bi ton t
nhiu hng. So vi ti liu i s s cp th trong ti liu ny chng ti c cp nht
thm mt s lng rt ng k cc dng ton rt hay gp trong cc k thi tuyn sinh i
hc v Cao ng theo chng trnh mi ca mn Ton bc Ph thng Trung hc.
Mt li khuyn ca chng ti i vi sinh vin l khi gii cc bi tp trong ti liu
khng nn qu l thuc vo phn li gii c sn trong ti liu, m trc ht hy t mnh c
gng tm ti li gii, sau so snh bi gii ca mnh vi bi gii trong ti liu nhm rt ra
nhng kinh nghim trong gii ton. C nh vy cun ti liu ny mi thc s c ch khi
hc mn i s s cp.
Cui cng, chng ti rt mong nhn c cc kin ng gp qu bu cho ni dung
cng nh hnh thc trnh by trong ti liu ca cc bn ng nghip trong B mn Ton v
Hi ng Khoa hc Khoa S phm cng nh cc bn sinh vin cun sch ny c th
c hon chnh tt hn.
Tc gi

MC LC
Trang
LI NI U

BNG MT S K HIU V CH VIT TT S DNG


TRONG TI LIU

PHN I: TM TT L THUYT V BI

Chng I. Hm s
A. Tm tt l thuyt
B. Bi tp
Chng II. Phng trnh H phng trnh

4
12
17

A. Tm tt l thuyt

17

B. Bi tp

24

Chng III. Bt ng thc Bt phng trnh

31

A. Tm tt l thuyt
B. Bi tp
Chng IV. Phng trnh, Bt phng trnh v t
A. Tm tt l thuyt
B. Bi tp
Chng V. Phng trnh, Bt phng trnh m v lgarit

31
37
43
43
45
51

A. Tm tt l thuyt
B. Bi tp
Chng VI. Phng trnh lng gic

51
55
64

A. Tm tt l thuyt

64

B. Bi tp

71

PHN II: LI GII V HNG DN

76

Chng I. Hm s

76

Chng II. Phng trnh H phng trnh

98

Chng III. Bt ng thc Bt phng trnh

151

Chng IV. Phng trnh, Bt phng trnh v t

188

Chng V. Phng trnh, Bt phng trnh m v lgarit

242

Chng VI. Phng trnh lng gic

312

TI LIU THAM KHO

361

BNG MT S K HIU V CH VIT TT S DNG TRONG TI LIU


: Tp hp cc s t nhin: {0;1; 2;...}.
: Tp hp cc s nguyn: {...; 2; 1; 0;1; 2;...}.

: Tp hp cc s hu t: / a, b , b 0 .
b

: Tp hp cc s thc.

* : Tp hp cc s thc khc khng.


+ : Tp hp cc s thc dng.
n

: Php ly tng t 1 n n.
1

{... / ...} : Tp hp.


T f : Tp (min) gi tr ca hm s f .

Max f ( x) : Gi tr ln nht ca hm s f trn tp D.


xD

Min f ( x) : Gi tr nh nht ca hm s f trn tp D.


xD

: Thuc.
, : Tp con.
: Tp hp rng.
: Mi.

: Khc.

\: Hiu ca hai tp hp.


: Hp ca hai tp hp.
: Giao ca hai tp hp.
n

: Php ly hp t 1 n n.
1
n

: Php ly giao t 1 n n.
1

: Hoc (tuyn ca hai mnh ).


: Php ko theo, phng trnh h qu.
: Php tng ng (khi v ch khi), phng trnh tng ng.
pcm: Kt thc chng minh, iu phi chng minh.

PHN I. TM TT L THUYT V BI
CHNG I
HM S
A. TM TT L THUYT
I. KHI NIM HM S
1. nh ngha

Gi s X v Y l hai tp hp ty . Nu c mt quy tc f cho tng ng m i x X


vi mt v ch mt y Y th ta ni rng f l mt hm t X vo Y , k hiu

f : X Y
x y = f ( x)
Nu X , Y l cc tp hp s th f c gi l mt hm s. Trong chng ny chng ta ch
xt cc hm s thc ca cc bin s thc, ngha l X ; Y .
X c gi l tp xc nh (hay l min xc nh) ca hm s f . (Ngi ta hay dng k
hiu tp xc nh ca hm s l D ).

S thc x X c g i l bin s c lp (gi tt l bin s hay i s). S thc


y = f ( x ) Y c gi l gi tr ca hm s f ti im x. Tp hp tt c cc gi tr f ( x )
khi x ly m i s thc thuc tp hp X gi l tp gi tr (min gi tr) ca hm s f v
c k hiu l T f , (nh vy T f = { f ( x ) | x X } = f ( X )).

Hin nhin T f Y . Ch rng T f c th l mt tp hp con thc s ca Y hoc


bng tp Y .
Trong nhiu trng hp, ngi ta cho hm s f di dng x f ( x ) hoc
y = f ( x ) m khng nu r tp xc nh X v tp hp Y cha tp cc gi tr ca f . Khi
, ta hiu rng Y = v X l tp hp cc s thc x sao cho quy tc cho th
f ( x ) tn ti.
2. th ca hm s

Cho hm s y = f ( x ) c tp xc nh D, ta gi tp hp cc im ( x; f ( x ) ) vi
x D l th ca hm s y = f ( x ) .

Vic biu din cc im ( x; f ( x ) ) thuc th ca hm s y = f ( x ) ln mt phng


ta Oxy gi l v th ca hm s.
Ch rng mt ng ( ) (ng cong hoc ng thng) trong mt phng ta
ch c th l th ca mt hm s no , nu n ct mt ng thng cng phng vi
trc Oy ti khng qu ti mt im.
3. Hm s n iu

3.1. nh ngha. Cho hm s y = f ( x ) c tp xc nh l tp D, khong ( a; b ) l

tp con ca D. Khi ta c
Hm s y = f ( x ) gi l ng bin (hay tng) trn khong ( a; b ) , nu vi
x1 , x2 ( a; b ) , x1 < x2 f ( x1 ) < f ( x2 ) .

Hm s y = f ( x ) gi l nghch bin (hay gim) trn khong ( a; b ) , nu vi


x1 , x2 ( a; b ) , x1 < x2 f ( x1 ) > f ( x2 ) .

Mt hm s ng bin hoc nghch bin trn khong ( a; b ) th ta ni hm s n iu


trn khong .
3.2. Tnh cht
3.3.1. Nu hm s y = f ( x ) ng bin (nghch bin) trn khong ( a; b ) , th

hm s y = f ( x ) + c (c l hng s) cng ng bin (nghch bin) trn khong ( a; b ) .


3.3.2. Nu hm s y = f ( x ) ng bin (nghch bin) trn khong ( a; b ) , th

hm s y = kf ( x ) ng bin (nghch bin) trn khong ( a; b ) nu k > 0 ; hm s


y = kf ( x ) nghch bin (ng bin) trn khong ( a; b ) nu k < 0.

3.3.3. Nu hm s y = f ( x ) v y = g ( x ) ng bin (nghch bin) trn khong

( a; b ) th hm s

y = f ( x ) + g ( x ) ng bin (nghch bin) trn khong ( a; b ) .

3.3.4. Nu hm s y = f ( x ) v y = g ( x ) khng m trn khong ( a; b ) v cng


ng bin (nghch bin) trn khong ( a; b ) , th hm s y = f ( x ) .g ( x ) ng bin (nghch

bin) trn khong ( a; b ) .

Ch . th ca hm s ng bin hoc nghch bin trn khong ( a; b ) ct ng thng


cng phng vi trc Ox nhiu nht ti mt im.
Gi s hm s y = f ( x ) ng bin trn khong ( a; b ) ; hm s y = g ( x ) nghch bin
trn khong ( a; b ) . Khi trn khong (a; b), th ca cc hm s y = f ( x ) v
y = g ( x ) ct nhau khng qu ti mt im.

4. Hm s chn, hm s l
4.1. nh ngha. Cho hm s y = f ( x ) c tp xc nh trn D.

Hm s f gi l hm s chn nu vi m i x D , ta c x D v f ( x ) = f ( x ) .
Hm s f gi l hm s l nu vi m i x D , ta c x D v f ( x ) = f ( x ) .
4.2. th ca hm s chn v hm s l

Gi s hm s y = f ( x ) c tp xc nh D l hm s chn v c th l ( G ) . Vi
5

m i im M ( x0 ; y0 ) thuc th ( G ) , ta xt im i xng vi n qua trc tung l


M ' ( x0 ; y0 ) .

T nh ngha hm s chn, ta c x0 D v f ( x0 ) = f ( x0 ) . Do
M G y0 = f ( x0 ) y0 = f ( x0 ) M ' ( G ) .

iu chng t ( G ) c trc i xng l trc tung.

Nu f l hm s l th l lun tng t, ta cng c ( G ) c tm i xng l gc ta


O.
5. Hm s tun hon
5.1. nh ngha. Hm s y = f ( x ) c tp xc nh D c gi l hm s tun

hon nu tn ti mt s dng T sao cho vi m i x D ta c


i ) x + T D v x T D ;
ii ) f ( x T ) = f ( x ) .

S nh nht (nu c) trong cc s T c cc tnh cht trn gi l chu k ca hm s tun


hon f ( x ) .

Ch . Chng ta c mt s du hiu nhn bit mt hm s cho khng phi l mt hm


s tun hon, chng hn ta c hai du hiu sau.
+ Nu mt hm s c tp xc nh dng D = \ A, vi A l mt tp hp hu hn th hm
s khng phi l mt hm s tun hon.
+ Nu phng trnh f ( x ) = k c nghim, nhng s nghim l mt s hu hn, th hm s

y = f ( x ) khng phi l mt hm s tun hon.


6. Hm s hp
6.1. nh ngha. Cho hm s y = f ( x ) xc nh trn tp D1 v y = g ( x ) xc
nh trn D2 . Khi ta gi hm s hp ca hai hm s f v g k hiu g  f c xc
nh y = ( g  f )( x ) = g f ( x ) xc nh trn tp D = { x D1 | f ( x ) D2 } .
7. Hm s ngc
7.1. nh ngha. Cho hm s

f : X Y
x y = f ( x)
nu vi m i gi tr y T f = f ( X ), c mt v ch mt x X sao cho f ( x ) = y, tc l
phng trnh f ( x ) = y vi n x c nghim duy nht, th bng cch cho tng ng vi m i
y f ( X ) phn t duy nht x X , ta xc nh c hm s

g : f (X ) X
y x = g ( y)
( x tha mn f ( x ) = y ).
Hm s g xc nh nh vy c g i l hm s ngc ca hm s f .
Theo thng l, ngi ta thng k hiu i s l x v hm s l y. Khi hm s ngc
ca hm s y = f ( x ) s c vit li l y = g ( x ) .
Gi s hm s y = f ( x ) c hm s ngc, tm hm s ngc ca hm s y = f ( x )
ta gii phng trnh f ( x ) = y n x, phng trnh ny c nghim duy nht x = g ( y ) , i
k hiu theo cch vit thng thng ta c hm s ngc y = g ( x ) .
Ch .
Ngi ta thng k hiu hm s ngc ca hm s y = f ( x ) l y = f 1 ( x ) .
T nh ngha ca hm s ngc, suy ra rng: Tp xc nh ca hm s ngc
y = f 1 ( x ) l tp gi tr ca hm s y = f ( x ) , tp gi tr ca hm s ngc l tp xc nh
ca hm s y = f ( x ) .
D nhin hm s y = f ( x ) li l hm s ngc ca hm s y = f 1 ( x ) . V vy ta ni
hai hm s y = f ( x ) v y = f 1 ( x ) l hai hm s ngc nhau.
7.2. iu kin hm s c hm s ngc
7.2.1. nh l. M i hm s ng bin (hay nghch bin) trn tp xc nh ca n
u c hm s ngc.
7.3. th ca hm s ngc
7.3.1. nh l. Trong h trc ta Cc vung gc Oxy, th ca hai hm

s ngc nhau y = f ( x ) v y = f 1 ( x ) i xng nhau qua ng phn gic th nht


y = x.
Ch .
T tnh cht ca th hm s ngc ta suy ra rng th ca hai hm s ngc nhau, nu
ct nhau th ct nhau trn ng thng y = x.
T ta c th p dng gii cc phng trnh dng f ( x ) = f 1 ( x ) bng cch a v
phng trnh f ( x ) = x hoc f 1 ( x ) = x.
II. MT S PHP BIN I TH
1. Trc i xng, tm i xng ca th

Chng ta bit th hm s chn nhn trc Oy lm trc i xng, th hm s l


nhn gc ta O lm tm i xng. Sau y chng ta a ra du hiu cho bit th ca

mt hm s c trc i xng, tm i xng. (Trong phn ny chng ta ch xt trc i


xng ca th hm s, cng phng vi trc tung).
1.1. nh l. th ca hm s y = f ( x ) nhn ng thng c phng trnh x =

lm trc i xng khi v ch khi f ( 2 x ) = f ( x ) vi m i x D.


Tht vy, mun cho ng thng c phng trnh x = l trc i xng ca th
y = f ( x ) th t c v l nu im M ( x; y ) thuc th th im M ' i xng vi i m
M qua cng thuc th. y im M ' c ta ( 2 x; y ) , nh vy vi m i
x D ta c f ( 2 x ) = f ( x ) .

1.2. nh l. th hm s y = f ( x ) nhn im I ( ; ) lm tm i xng khi v ch

khi f ( 2 x ) = 2 f ( x ) , x D.
Ch . Trong nh l 1.1 cho = 0 v trong nh l 1.2 cho = = 0, ta c kt qu
+ th ca hm s chn nhn trc tung lm trc i xng.
+ th hm s l nhn gc ta lm tm i xng.
Trong thc t mun chng minh th hm s y = f ( x ) nhn ng thng x = x0 lm trc
i xng th ta c th lm nh sau:
Di h trc ta Oxy v h trc IXY , vi I ( x0 ; 0 ) theo cng thc
x = X + x0

y = Y

Lp hm s mi bng cch thay x = X + x0 ; y = Y vo hm s y = f ( x);


Chng minh hm s mi Y = g ( X ) l hm s chn kt lun x = x0 l trc i xng.
Tng t nh trn, mun chng minh I ( x0 , y0 ) l tm i xng ca th ( C ) ca hm s
y = f ( x ) , ta di h trc ta Oxy sang h trc IXY , bng php t

x = X + x0

y = Y + y0

Sau chng minh hm s mi Y = g ( X ) l hm s l kt lun im I ( x0 ; y0 ) l tm


i xng ca th.
2. Php i xng qua trc ta
2.1. nh l. th ca cc hm s y = f ( x ) v y = f ( x ) i xng nhau qua trc
honh.
2.2. nh l. th ca cc hm s y = f ( x ) v y = f ( x ) i xng nhau qua trc
tung.
3. Php tnh tin song song vi trc tung

3.1. nh l. th ca hm s y = f ( x ) + b ( y = f ( x ) b ) , b > 0 suy ra t th


y = f ( x ) bng mt php tnh tin theo vect Oy Oy mt on bng b.

4. Php tnh tin song song vi trc honh


4.1. nh l. th hm s y = f ( x + a ) ( y = f ( x a ) ) , a > 0 suy c t th

hm s y = f ( x ) bng php tnh tin theo vect Ox Ox mt on bng a.

( )

Ch .

Ngoi

php t nh tin theo cc trc ta ngi ta cn a ra php tnh tin theo vect
v 0.

T th hm s y = f ( x), tnh tin theo vect v = ( a; b ) th c th hm s


y = f ( x a ) + b.

5. th ca mt s hm s cha du gi tr tuyt i
5.1. th hm s y = f ( x )

f ( x ) ;
f ( x) 0
Ta c y = f ( x ) =
f ( x ) ; f ( x ) < 0

Do th ca hm s y = f ( x ) gm
+ Phn t trc honh tr ln ca th hm s y = f ( x ) ;
+ i xng phn th hm s y = f ( x ) pha di trc honh qua trc honh.
5.2. th hm s y = f ( x )

Thy ngay y = f ( x ) l hm s chn nn th c trc i xng l Oy. Vi x 0 th


y = f ( x ) = f ( x ) . Vy th gm hai phn
+ Phn bn phi Oy ca th y = f ( x ) ;
+ i xng phn trn qua Oy.
5.3. th hm s y = u ( x ) .v ( x )

u ( x ) .v ( x ) ; u ( x ) 0
Ta c y = u ( x ) .v ( x ) =
u ( x ) .v ( x ) ; u ( x ) < 0

Do ta v th y = f ( x ) = u ( x ) .v ( x ) v t th y = u ( x ) .v ( x ) gm
+ Phn th y = f ( x ) trn min u ( x ) 0.

+ i xng phn th y = f ( x ) trn min u ( x ) < 0 qua trc honh.


5.4. T th hm s y = f ( x ) suy ra ng biu din y = f ( x ) , ( )

Ta c nhn xt: Gi s im ( x0 ; y0 ) thuc ( ) th ( x0 ; y0 ) cng thuc ( ) .


Vy, ( ) c trc i xng l Ox. Vi y 0 th y = f ( x ) y = f ( x ) .
Do ( ) gm hai phn
+ Phn th t trc honh tr ln ca th y = f ( x ) .
+ i xng phn trn qua trc honh c phn cn li.
III. GI TR LN NHT V GI TR NH NHT CA HM S
1. nh ngha

Cho hm s y = f ( x ) xc nh trn tp D.
a) S M c gi l gi tr ln nht ca hm s y = f ( x ) trn tp D nu
i ) x D : f ( x ) M ;
ii ) x0 D : f ( x0 ) = M .
K hiu M = Max f ( x ) .
xD

b) S m c gi l gi tr nh nht ca hm s y = f ( x ) trn tp D nu
i ) x D : f ( x ) m;
ii ) x0 D : f ( x0 ) = m.
K hiu m = Min f ( x ) .
xD

2. Mt s phng php tm gi tr ln nht, gi tr nh nht ca hm s


2.1. Phng php min gi tr

Ni dung ca phng php ny nh sau.


+ Xem y = f ( x ) l phng trnh i vi n x v y l tham s;
+ Tm iu kin ca y phng trnh y = f ( x ) c nghim;
+ T iu kin trn, bin i a n dng m y M . Xt du = xy ra v kt lun
Minf ( x) = m; Maxf ( x ) = M .
2.2. Phng php o hm

+ Kho st s bin thin ca hm s y = f ( x ) ;


+ Da vo bng bin thin kt lun Maxf ( x); Minf ( x).

10

Ch . Trong trng hp tm gi tr ln nht, gi tr nh nht ca hm s y = f ( x ) trn


on [a; b], ta c th trnh by n gin nh sau.

Bc 1. Tm f ( x ) v tm cc im ti hn x1 , x2 ,..., xn ca f ( x ) trn on [a; b];


Bc 2. Tnh f ( x1 ) , f ( x2 ) ,..., f ( xn ) , f ( a ) , f ( b ) ;
Bc 3. Tm s ln nht M v s nh nht m trong cc s trn, khi
M = Max f ( x ) ; m = Min f ( x ) .
x[ a ;b ]

x[ a ;b ]

(Nu hm s y = f ( x ) lin tc trn on [a; b], th gi tr ln nht v gi tr nh nht ca


hm s trn on [a; b] bao gi cng tn ti).
2.3. Phng php dng bt ng thc

Dng bt ng thc quen thuc chng minh f ( x ) M hoc f ( x ) m.


Phi ch ra tn ti x0 ; x1 D sao cho f ( x0 ) = M , f ( x1 ) = m. Khi
M = Max f ( x ) ; m = Min f ( x ) .
x[ a ;b ]

x[ a ;b ]

Cc bt ng thc quen thuc sau y thng c dng tm gi tr ln nht v gi tr


nh nht ca hm s.
+ Bt ng thc Csi. (Augustin Louis Cauchy,1789 1857. Nh Ton hc Php).
Cho n s thc a1 , a2 ,..., an khng m. Th th
a1 + a2 + ... + an n
a1.a2 ...an
n

Du = xy ra khi v ch khi a1 = a2 = ... = an .


+ Bt ng thc Bunhiacpski. (Victor Yakovlevich Bunyakovsky, 1804 1889. Nh Ton
hc Nga).

Cho n cp s thc (ai ; bi ), i = 1, 2,, n.


Th th
2

n
n 2 n 2
a
b
i i ai bi
i =1
i =1 i =1

Du = xy ra khi v ch khi tn ti k sao cho bi = kai , i = 1, 2,, n.


+ Bt ng thc v du gi tr tuyt i. Cho a, b, ai , i = 1, 2,..., n l cc s thc. Th th
a + b a + b (*); a b a b (**); a1 + a2 + ... + an a1 + a2 + ... + an (***)
Du = trong (*) v (**) xy ra, khi v ch khi ab 0. Du = trong (***) xy ra, khi
v ch khi ai 0 hoc ai 0, i = 1, 2,..., n.
2.4. Phng php ta vc t

11

Ta c cc bt ng thc v vc t nh sau

a + b a + b . Du ng thc xy ra khi v ch khi a , b cng hng.

a b a b . Du ng thc xy ra khi v ch khi a , b cng hng.

a.b a . b . Du ng thc xy ra khi v ch khi a , b cng phng.


B. BI TP
I.1. Tm tp gi tr ca hm s
y=

I.2. Cho hm s y =

2 x 1
.
x + x+4
2

x +1
. Tm cc gi tr a > 0 tp gi tr ca hm s cho cha
x2 + a

on [0;1].
I.3. Tm cc gi tr ca m hm s

y=

1
x (m + 1) x + m
2

l hm s chn.
I.4. Cho hm s y = f ( x ) xc nh trn tha f (a + b) = f (a ) + f (b), a, b . Chng
minh rng

1) f (0) = 0;
2) y = f ( x ) l mt hm s l.
I.5. Cho hm s y = f ( x ) xc nh trn v l hm s l, tha f (0) 0. Chng minh
rng s nghim ca phng trnh f ( x ) = 0 l mt s chn.
I.6. Cho hm s y = f ( x ) xc nh trn tha f ( x ) 0, x v
f ( x1 + x2 ) + f ( x1 x2 ) = 2 f ( x1 ) f ( x2 ), x1 , x2 .

Chng minh rng


1) f (0) = 1;
2) y = f ( x ) l mt hm s chn.
I.7. Chng minh cc hm s cho sau y l hm s tun hon, tm chu k (nu c)

1) y = cos(2 x + 3);
2) y = sin 2 x.
I.8. Chng minh cc hm s cho sau y khng phi l mt hm s tun hon

1) y = x 3 + 2 x 2 ;
2) y = x 1 ;
12

3) y =

x
.
x 1
2

I.9. Chng minh hm s irichl

1, x
f ( x) =
0, x \
l mt hm s tun hon nhng khng c chu k.
I.10. Cho cc hm s y = f ( x) =

x +1
v y = g ( x ) = 2 x 1
x 1

1) Xc nh hm s y = f ( f ( x ));
2) Xc nh hm s y = f ( g ( x )).
I.11. Cho hm s y = f1 ( x ) =

1
. K hiu f n ( x ) = f ( f n 1 ( x )) , vi n v n 2. Xc
1 x

nh hm s y = f100 ( x).

1 2 x, x < 2
x 1, x 1
I.12. Cho cc hm s y = f ( x) =
v y = g ( x ) =
1 x, x < 1.
2 x 1, x 1

2
Xc nh cc hm s hp y = f ( g ( x )), y = g ( f ( x)).
I.13. Cho hm s y = f ( x) = 2 1 x .

Tm hm s ngc y = f 1 ( x) .

I.14. 1) Hy xc nh vc t v = (a; b), sao cho khi tnh tin th ca hm s


y=

x2 + x 3
x+2

theo vc t v ta c th ca hm s cho trong cc trng hp sau y

a) y =

x2 x 7
;
x+2

x2 + 7 x + 9
b) y =
;
x+5
c) y =

x2 + 2x 4
.
x+3

2) T th ca hm s y =

x2 + x 3
, suy ra th ca cc hm s sau bng cc php
x+2

bin i no ?

13

a) y =

x2 x + 3
;
x+2

x2 + 5
b) y =
;
x+2
I.15. T th ca hm s y =

ca hm s y =

1
, bng cc php bin i th no nhn c th
x

3x 7
?
x2

I.16. Cho hm s

y=

x 2 3x + 1
.
x 3

1) Dng th (C) ca hm s cho;


2) T th (C) hy suy ra th ca cc hm s sau
a) y =

x 2 3x + 1
;
x 3

b) y =

x 2 3x + 1
;
x3
2

x 3 x +1
c) y =
;
x 3
2

d) y =

x 3 x +1
x 3

I.17. Chng minh th ca hm s


y=

5
x 4x + 3
2

nhn ng thng x = 2 lm trc i xng.


I.18. Chng minh th ca hm s y = x 4 + 4 x 3 + 3 x 2 2 x

c ng mt trc i xng cng phng vi trc tung.


I.19. Chng minh th ca hm s

y=

x2 + 4 x 2
x2 +1

khng c tm i xng.
I.20. Cho hm s y = x 4 + 4ax 3 2 x 2 12ax.

14

Tm cc gi tr ca a th ca hm s cho c trc i xng cng phng vi trc


Oy.
I.21. Cho hm s y =

x 2 + 2m 2 x + m 2
c th l (Cm ).
x +1

Tm m trn (Cm ) tn ti hai im i xng nhau qua gc to .


I.22. Tm gi tr ln nht v gi tr nh nht ca cc hm s cho sau y

1) y = 2.33 x 4.32 x + 2.3x trn on [1; 1];


3
2) y = cos 3x 15cos x + 8 trn on [ ;
];
3 2

3) y = x 3 3 x 2 + 5 trn on [0; 3].


I.23. Tm gi tr ln nht v gi tr nh nht ca cc hm s cho sau y

1) y =

x2
3
trn on [ ; 2];
2x 1
4

2) y = (cos x + 1) sin x, x [0, 2].


x + y = 2 a
I.24. Gi s ( x, y ) l mt nghim ca h phng trnh 2
2
x + y + x y = 3.

Tm cc gi tr ca a biu thc M = x 2 + y 2 x y t gi tr ln nht, gi tr nh nht.


I.25. Tm gi tr nh nht ca hm s y = ( x + 1)( x + 2)( x + 3)( x + 4).

4 1
5
.
I.26. Cho x > 0, y > 0 tha mn x + y = . Tm gi tr nh nht ca biu thc A = +
4
x 4y
I.27. Tm gi tr nh nht ca hm s y = x + 1 + x 2 + 2 x 5 .
I.28. Cho hai s dng x, y thay i tha mn iu kin x + y 4 . Tm gi tr nh nht ca
biu thc

3x 2 + 4 y 3 + 2
A=
+
.
4x
y2
I.29. Tm gi tr ln nht ca biu thc

T=

1
( yz x 3 + zx y 4 + xy z 5).
xyz

I.30. Xt cc s dng x, y, z tha mn iu kin x + y + z = 1. Tm gi tr nh nht ca


biu thc

x 2 ( y + z ) y 2 ( z + x) z 2 ( x + y )
P=
+
+
.
yz
zx
xy

15

I.31. Cho cc s a, b, c dng thay i tha mn iu kin abc = 1. Tm gi tr nh nht


ca biu thc
A=

a3
b3
c3
+
+
.
(1 + b )(1 + c ) (1 + c )(1 + a ) (1 + a )(1 + b )

I.32. Cho cc s a, b, c dng thay i tha mn iu kin a + b + c 3. Tm gi tr nh


nht ca biu thc

a
b
c
+
+
.
b
c
a

A=

I.33. Cho cc s x, y, z dng thay i tha mn iu kin x 2 + y 2 + z 2 = 1. Tm gi tr nh


nht ca biu thc

S=

xy yz zx
+ + .
z
x
y

I.34. Cho cc s a, b, c dng thay i tha mn iu kin a + b + c = 1. Tm gi tr nh


nht ca biu thc
A=

(1 + a )(1 + b )(1 + c ) .
(1 a )(1 b )(1 c )

I.35. Cho cc s a, b, c dng thay i tha mn iu kin a 2 + b 2 + c 2 = 3. Tm gi tr nh


nht ca biu thc

M=

ab 2 + bc 2 + ca 2

( ab + bc + ca )

.
2

I.36. Cho cc s x, y, z thay i tha mn iu kin ( x 1) + ( y 2 ) + ( z 1) = 1. Tm


gi tr ln nht ca biu thc
A = x + 2 y + 3z 8 .

I.37. Cho cc s a, b, c dng thay i tha mn iu kin a + b + c = 1. Tm gi tr nh


nht ca biu thc
A = a 2 + b2 + b2 + c 2 + c2 + a 2 .

I.38. Cho cc s x, y thay i tha mn iu kin x 2 + y 2 = 1. Tm gi tr ln nht v gi tr


nh nht ca biu thc

A=

xy + y 2
.
1 + 2 x 2 + 2 xy

I.39. Cho x, y, z l cc s thc dng thay i. Tm gi tr nh nht ca biu thc

x 1
y 1 z 1
P = x + + y + + z + .
2 zx 2 xy
2 yz
16

I.40. Cho x, y, z l cc s thc dng thay i tha mn iu kin xyz = 1. Tm gi tr nh


nht ca biu thc
P=

x2 ( y + z )
y y + 2z z

y2 ( z + x )
z z + 2x x

z2 ( x + y)
x x + 2y y

I.41. Tm gi tr ln nht v gi tr nh nht ca hm s

sin x
4

y=
, x ; .
2
2
sin x + 1 + 2cos x
CHNG II
PHNG TRNH H PHNG TRNH
A. TM TT L THUYT
I. CC KHI NIM C BN
1. Phng trnh
1.1. nh ngha

Cho hai hm s ca n bin thc x1 , x2 ,..., xn l f ( x1; x2 ;...; xn ), g ( x1; x2 ;...; xn ). Ta gi b n


s thc ( x1; x2 ;...; xn ) n l mt im trong n . Khi cc hm s
f ( x1; x2 ;...; xn ), g ( x1; x2 ;...; xn )
c xem l cc hm mt bin x trong n .

Ta gi Phng trnh n x l mnh cha bin dng f ( x) = g ( x ) (1)


trong , f ( x ) v g ( x) l nhng biu thc cha x. Ta gi f ( x ) l v tri, g ( x) l v phi
ca phng trnh (1). Nu coi f v g l hm ca n bin trong khng gian th (1) l
phng trnh ca n n x1 , x2 ,..., xn .
Gi s f(x) c tp xc nh l D1, g(x) c tp xc nh l D2 th D = D1 D2 gi l tp
(min) xc nh ca phng trnh (1).
Nu xo D sao cho f ( xo ) = g ( xo ) l mt mnh ng th xo c g i l mt
nghim ca phng trnh (1).
Gii phng trnh (1) l tm tt c cc nghim ca n, tp hp cc nghim ca phng
trnh k hiu l S.
Nu S = th ta ni phng trnh v nghim.
Ch . Trong mt phng trnh (mt hoc nhiu n), ngoi cc ch ng vai tr l cc n
s, cn c th c cc ch khc c xem nh nhng hng s v c g i l tham s. Gi i
v bin lun phng trnh cha tham s, ngha l xt xem vi gi tr no ca tham s th
phng trnh v nghim, c nghim v tm cc nghim .
1.2. Phng trnh tng ng, phng trnh h qu
1.2.1. Phng trnh tng ng. Hai phng trnh c gi l tng ng v i
nhau khi chng c cng tp hp nghim.

17

Khi hai phng trnh f ( x) = g ( x ) ; f1 ( x ) = g1 ( x) tng ng vi nhau ta dng k hiu


f ( x ) = g ( x ) f1 ( x ) = g1 ( x).
Ch . Nu theo nh ngha trn th hai phng trnh v nghim cng c coi l tng
ng vi nhau v c cng tp hp nghim l tp hp . V vy, cch vit sau cng coi
nh l ng, tuy nhin trong thc t t khi gp. Chng hn, x 2 + 3 = 0 cos x = 3.
S tng ng ca hai phng trnh c tnh cht phn x, i xng, bc cu.
1.2.2. Phng trnh h qu

Nu m i nghim ca ca phng trnh f ( x) = g ( x ) u l nghim ca phng trnh


f1 ( x ) = g1 ( x) th phng trnh f1 ( x ) = g1 ( x) c gi l phng trnh h qu ca phng
trnh f ( x) = g ( x ) .
Ta dng k hiu f ( x ) = g ( x ) f1 ( x ) = g1 ( x ).
1.2.3. Cc php bin i tng ng phng trnh

Qu trnh gii mt phng trnh l qu trnh bin i phng trnh i n mt


phng trnh n gin hn m ta bit cch gii. Nu php bin i khng lm thay i
tp xc nh ca phng trnh th phng trnh cho c bin i tng ng, cn nu
lm thay i tp xc nh ca phng trnh th c th tp hp nghim ca phng trnh
cho cng b thay i. Sau y ta xt mt s php bin i tng ng.
1.2.3.1. nh l. Cho phng trnh f ( x) = g ( x ) . Nu h( x) c ngha trong tp xc nh
ca phng trnh cho th f ( x ) = g ( x ) f ( x ) + h( x) = g ( x) + h( x ). (1)
H qu 1. C th chuyn cc hng t t v ny sang v kia ca phng trnh, nhng
phi i du ca n.
H qu 2. Mi phng trnh u c th a v dng m v phi bng khng.

Do vy, ta lun c th k hiu phng trnh l F(x) = 0.


Ch . iu kin h(x) c ngha trong tp xc nh ca phng trnh f(x) = g(x) l iu kin
nhng khng cn. Ni khc i, nu c iu kin y th
f ( x) = g ( x ) f ( x) + h( x ) = g ( x ) + h( x)
l php bin i tng ng, cn nu khng c iu kin y th php bin i trn c th
tng ng hoc c th khng.
1.2.3.2. nh l. Cho phng trnh f(x) = g(x). Nu h(x) c ngha v khc khng trong
tp xc nh ca phng trnh cho th

f ( x) = g ( x ) f ( x)h( x) = g ( x)h( x ).
H qu. C th nhn hai v ca mt phng trnh vi mt s khc khng ty .

Ta cng c nhn xt v h(x) tng t nh nh l 1.2.3.1.


1.2.3.3. nh l. Nu nng hai v ca mt phng trnh ln mt ly tha bc l th ta
c mt phng trnh tng ng vi phng trnh cho.

18

Ch . Php bin i nng hai v ca phng trnh ln mt ly tha bc chn l php bin
i h qu, n ch l php bin i tng ng nu hai v ca phng trnh u khng m
trn tp xc nh.
2k

2k

f ( x) = g ( x ) [ f ( x) ] = [ g ( x)] , ( f ( x ) 0, g ( x) 0).
Nu sau mt php bin i no , tp xc nh ca phng trnh cho m rng ra
th tp hp nghim ca n cng c th m rng ra, khi c th xut hin nhng nghim,
ta gi l nghim ngoi lai ( i vi phng trnh cho). Nhng nghim ngoi lai (nu
c) l nhng nghim ca phng trnh sau khi bin i v thuc vo phn m rng ca tp
xc nh. Nu tp xc nh m rng ra nhng khng c nghim ngoi lai th phng trnh
cho v phng trnh bin i vn tng ng.
Nu sau mt php bin i no , tp xc nh ca phng trnh cho b thu hp li
th tp nghim ca n cng c th b thu hp li, mt s nghim no c th mt i.
Nhng nghim mt i (nu c) l nhng nghim ca phng trnh cho nhng thuc
vo phn b thu hp ca tp xc nh. Nu tt c cc gi tr ca n s b mt i khi tp xc
nh b thu hp khng tha mn phng trnh cho, th phng trnh cho v phng
trnh bin i vn tng ng.
2. H phng trnh Tuyn phng trnh
2.1. nh ngha. Cho m phng trnh

f1 ( x ) = g1 ( x)
f 2 ( x) = g 2 ( x)
.....................
f m ( x) = g m ( x )
(c th coi x = ( x1 ; x2 ;...; xn ) , khi cc fi ( x ), g i ( x), i = 1, 2,..., m l nhng hm n bin).
Gi s m phng trnh cho c tp xc nh ln lt l D1 , D2 ,..., Dm .
Ta gi h m phng trnh k hiu l
f1 ( x ) = g1 ( x)

f 2 ( x) = g 2 ( x)
(1) ....................

f m ( x) = g m ( x )
m

D = Di l tp xc nh ca h (1).
i =1

Mt gi tr a D ca bin x lm cho tng phng trnh ca h (1) u tr thnh ng


thc ng c gi l mt nghim ca h (1). K hiu Si l tp hp nghim ca phng
m

trnh th i ca h (1) th tp hp nghim ca h (1) l S = Si . Khi S = ta ni h v


i =1

nghim.
2.2. nh ngha. Ta cng gi tuyn ca m phng trnh k hiu l

19

f1 ( x ) = g1 ( x)

f 2 ( x) = g 2 ( x)
...................

f m ( x) = g m ( x )

(2)

Tp xc nh ca tuyn phng trnh (2) cng l D = Di , vi Di l tp xc nh ca


i =1

phng trnh th i.
Nu c mt gi tr a D ca x lm cho mt phng trnh no ca tuyn phng
trnh (2) tr thnh ng thc ng th a c g i l mt nghim ca tuyn phng trnh (2).
m

Tp hp nghim ca tuyn phng trnh (2) l S = Si , Si l tp hp nghim ca phng


i =1

trnh th i ca tuyn phng trnh (2).


Khi nim tng ng ca h phng trnh, tuyn phng trnh cng tng t nh
phng trnh.
2.3. Cc nh l v h phng trnh tng ng
2.3.1. nh l. Nu F ( x1; x2 ;...; xn ) = 0 x1 = f1 ( x2 ;...; xn ) th
F1 ( x1; x2 ;...; xn ) = 0
x1 = f1 ( x2 ;...; xn )

F2 ( x1; x2 ;...; xn ) = 0
F2 ( f1 ( x2 ;...; xn ) ; x2 ;...; xn ) = 0

...........................
..........................................
F ( x ; x ;...; x ) = 0
F ( f ( x ;...; x ) ; x ;...; x ) = 0
n
n
2
n
m 1 2
m 1 2

2.3.2. nh l
F1 = 0
F1 = 0

F2 = 0
n12 F1 + n22 F2 = 0
F3 = 0 n13 F1 + n23 F2 + n33 F3 = 0
........... ................................

Fm = 0
n1m F1 + n2 m F2 + ..... + nmm Fm = 0

2.4. nh l v tuyn phng trnh tng ng


F1 = 0

F2 = 0
F1.F2 ...Fm = 0
.......

Fm = 0

II. PHNG TRNH BC NHT, BC HAI MT N


1. Phng trnh bc nht mt n
1.1. nh ngha. Phng trnh bc nht mt n l phng trnh
ax + b = 0, a, b , a 0.

20

b
Phng trnh bc nht c mt nghim duy nht x = .
a

1.2. Gii v bin lun phng trnh dng ax + b = 0 (1)


b
a 0 , phng trnh (1) c mt nghim duy nht x = .
a

a = 0, b 0 , phng trnh (1) v nghim.


a = 0, b = 0 , phng trnh (1) c nghim ty .
1.3. Mt s phng trnh qui v phng trnh bc nht mt n
l cc phng trnh dng:

ax + b
= 0; ax + b = cx + d ; ax + b = cx + d .
cx + d

ax + b
= 0 ta phi t iu kin cho mu khc khng. gii
cx + d
cc phng trnh ax + b = cx + d ; ax + b = cx + d , ta phi kh du gi tr tuyt i bng

Khi gii phng trnh dng

nh ngha v tnh cht ca du gi tr tuyt i. Cho A, B l cc biu thc cha bin, ta c

A; A 0
A =
A; A < 0
A = B
A = B
A = B
B 0

A = B A = B
A = B

2. Phng trnh bc hai mt n


2.1. nh ngha. Phng trnh bc hai mt n l phng trnh c dng ax 2 + bx + c = 0
(1), vi a, b, c l cc tham s thc, a 0 .

Biu thc = b 2 4ac c gi l bit thc ca phng trnh (1).


Xy ra ba trng hp sau:
i) Nu < 0 th phng trnh (1) v nghim;
ii) Nu = 0 th phng trnh (1) c nghim kp x1 = x2 =

b
;
2a

iii) Nu > 0 th phng trnh (1) c hai nghim phn bit x1,2 =

b
.
2a

b
th ' = b '2 ac gi l bit thc thu gn ca phng trnh (1).
2
Ta cng c ba trng hp sau:

Ngoi ra, nu t b ' =

i) Nu ' < 0 th phng trnh (1) v nghim;


21

ii) Nu ' = 0 th phng trnh (1) c nghim kp l x1 = x2 =

b'
;
a

iii) Nu ' > 0 th phng trnh (1) c hai nghim phn bit l x1,2 =

b ' '
.
a

2.2. nh l Viet

Nu phng trnh bc hai ax 2 + bx + c = 0 c nghim x1 , x2 th


x1 + x2 =

b
c
v x1.x2 = .
a
a

o li nu hai s x, y tha mn x + y = S v x.y = P th x, y l nghim ca phng trnh


bc hai X 2 SX + P = 0 (*) (iu kin (*) c nghim l S 2 4 P 0).

T , ta c h qu sau:
2.2.1. Nu a + b + c = 0 th phng trnh (1) c mt nghim bng 1 v nghim kia bng
c
.
a

2.2.2. Nu a b + c = 0 th phng trnh (1) c mt nghim bng 1 v nghim kia


c
bng .
a
3. Mt s phng trnh bc bn c th a v phng trnh bc hai mt n (qua php
t n ph)
3.1. Phng trnh trng phng: ax 4 + bx 2 + c = 0 , t t = x 2 0 , khi phng trnh
cho c a v phng trnh bc hai i vi bin t.
3.2. Phng trnh dng: ( x + a )( x + b)( x + c)( x + d ) = k , vi a + b = c + d .
t t = ( x + a)( x + b), khi phng trnh cho c a v phng trnh bc hai i
vi bin t.
4

3.3. Phng trnh dng: ( x + a ) + ( x + b ) = c. t t = x +

a+b
, phng trnh c a
2

v phng trnh trng phng


2t 4 + 12(

a b 2 2
ab 4
) t + 2(
) = c.
2
2

3.4. Phng trnh dng: ax 4 + bx 3 + cx 2 + bx + a = 0, (a 0) (Phng trnh bc bn h i


quy).

Chia hai v ca phng trnh cho x 2 (v x = 0 khng phi l nghim ca phng


1
1

trnh), phng trnh tr thnh a x 2 + 2 + b x + + c = 0.


x
x

1
t t = x + , t 2, ta c phng trnh bc hai theo bin t
x

22

at 2 + bt + c 2a = 0.
i vi phng trnh dng ax 4 + bx 3 + cx 2 bx + a = 0, (a 0) (Phng trnh bc bn phn
h i quy), ta cng c cch bin i nh trn vi php t
1
t = x , t , khi phng trnh cho c a v phng trnh bc hai theo bin t
x

at 2 + bt + c + 2a = 0.
III. H PHNG TRNH
1. H gm mt phng trnh bc nht v mt phng trnh bc hai hai n

H phng trnh c dng


Ax + By + C = 0
2
2
ax + bxy + cy + dx + ey + f = 0

Phng php gii.


S dng phng php th: Rt x hoc y t phng trnh bc nht ri thay vo phng
trnh bc hai trong h, ta c mt phng trnh mt n. Gii phng trnh mt n ny, sau
tm n cn li.
2. H phng trnh ng cp bc hai

H phng trnh ng cp bc hai i vi hai n x, y l h phng trnh c dng


ax 2 + bxy + cy 2 = d
2
2
a ' x + b ' xy + c ' y = d '

Phng php gii.


Xt xem x = 0 c tha h phng trnh hay khng;
Khi x 0 , t y = kx
+ Th y = kx vo h phng trnh, kh x ta c phng trnh bc hai theo k;
+ Gii phng trnh tm k, sau tm ( x; y ).
3. H phng trnh i xng
3.1. H phng trnh i xng loi I

Ta qui c gi mt h hai phng trnh cha hai n x, y l h phng trnh i xng


loi I, nu ta thay th x bi y v y bi x th m i phng trnh ca h khng thay i.
Phng php gii.
t S = x + y , P = xy a h phng trnh v h phng trnh n S v P.
Tm S, P, khi x, y l nghim ca phng trnh: X 2 SX + P = 0, ch phi c iu
kin S 2 4 P 0 .
3.2. H phng trnh i xng loi II

23

Ta qui c gi mt h hai phng trnh cha hai n x, y l h phng trnh i xng


loi II, nu tro i vai tr ca x, y cho nhau th phng trnh ny chuyn thnh phng
trnh kia.
Phng php gii.
Tr tng v cc phng trnh cho ta c phng trnh mi, a phng trnh ny
v phng trnh tch.
ng vi tng trng hp xy ra, kt hp vi mt trong hai phng trnh ca h c
mt h phng trnh con, gii h phng trnh con ny.
Tng hp nghim.
B. BI TP
II.1. Gii v bin lun cc phng trnh

1) m2 x + 4m 3 = x + m 2 ;
2) (a + b)2 + 2a 2 = 2a(a + b) + (a 2 + b 2 ) x;
3) a 2 x + 2ab = b 2 x + a 2 + b 2 ;
4) a(ax + b) = 4ax + b 2 5.
II.2 Gii v bin lun cc phng trnh

1)

2x + m x + m 1

= 1;
x 1
x

mx 2
2)
m | x |= 2m + 1;
| x | 1
3)

2mx 1
m +1
2 x 1 =
.
x 1
x 1

II.3. Gii v bin lun phng trnh

m2 x 2 m(5m + 1) x (5m + 2) = 0.
II.4. Gii v bin lun phng trnh sau theo hai tham s a v b

(a + b) x 2 (a 2 + 4ab + b 2 ) x + 2ab(a + b) = 0.
II.5. Cho a, b, c l ba s khc nhau v khc 0. Chng minh rng

Nu cc phng trnh x 2 + ax + bc = 0 v x 2 + bx + ca = 0 c ng mt nghim chung th


nghim cn li ca chng tha mn phng trnh x 2 + cx + ab = 0.
II.6. Cho phng trnh

mx 2 2(m 3) x + m 4 = 0
Tm cc gi tr ca m phng trnh c ng mt nghim dng.
24

II.7. Cho phng trnh

(m 1) x 4 + 2(m 3) x 2 + m + 3 = 0
Tm cc gi tr ca m phng trnh trn v nghim.
II.8. Cho phng trnh
x 2 2 x m x 1 + m2 = 0

Tm cc gi tr ca m phng trnh trn c nghim.


II.9. 1) Tm cc gi tr ca k phng trnh sau c bn nghim phn bit

( x 1)2 = 2 | x k | .
2) Tm cc gi tr ca a phng trnh 2 x 2 + 10 x 8 = x 2 5 x + a c bn nghim
phn bit.
II.10. Gii cc phng trnh sau

1) ( x 1)( x + 5)( x 3)( x + 7) = 297;


2) ( x + 2)( x 3)( x + 1)( x + 6) = 36;
3) x( x 2)( x + 2)( x + 4) = 18.
II.11. Gii cc phng trnh sau

1) x 4 + ( x 1)4 = 97;
2) ( x + 3)4 + ( x + 5)4 = 16;
3) ( x + 2)4 + ( x + 6)4 = 2.
II.12. Gii cc phng trnh sau

1) 6 x 4 35 x 3 + 62 x 2 35 x + 6 = 0;
2) x 4 + x 3 4 x 2 + x + 1 = 0;
3) x 4 5 x 3 + 10 x 2 10 x + 4 = 0;
4) 2 x 4 21x 3 + 74 x 2 105 x + 50 = 0 ;
5) 2 x 4 + 5 x3 + x 2 + 5 x + 2 = 0 .
II.13. Bin lun theo m s nghim ca phng trnh

( x + 3)( x 1)( x + 5)( x 3) 40 = m.


II.14. Gii cc h phng trnh

25

2 x + y + 1 x + y = 1
1)
3 x + 2 y = 4;

2 xy 3 y = 15

2)
xy + x = 15;

y
2
x xy = 12
3) 2
y xy = 28;

x + y 3x + 2 y = 1
4)
x + y + x y = 0;

1
1
2
2
x + y + x2 + y 2 = 4

5)
1 1

x + y + + = 4;

x y
x2 + y 2 + 6 x + 2 y = 0
6)
x + y + 8 = 0;

2
2
x + y xy = 13
7)
x + y xy = 3;

x + y + x2 + y 2 = 8
8)
xy ( x + 1)( y + 1) = 12;
x2 + y2 + x + y = 4
9)
x( x + y + 1) + y ( y + 1) = 2;
( x 2 + y 2 ) xy = 78
10)
x 4 + y 4 = 97;

3
3
x y = 7( x y )
11) 2
2
x + y = x + y + 2;

12)

x + 2 + y 3 = 10
4

x + 2 + 4 y 3 = 4;

( x 1)( y 1)( x + y 2 ) = 6
13) 2
2
x + y 2 x 2 y 3 = 0;

26

xy + x + 1 = 7 y
14) 2 2
2
x y + xy + 1 = 13 y ;

x ( x + y + 1) 3 = 0

15)
5
2
( x + y ) 2 + 1 = 0.
x

II.15. Gii cc h phng trnh

x 2 3xy = 4 y
1) 2
y 3 xy = 4 x;

7 x + y
2)
7 y + x

1
=0
x2
1
= 0;
y2

x2 = y 1 + 2x 1

3)
y 2 = x 1 + 2 y 1;

1
1
x = y
x
y
4)
2 y = x3 + 1;

x 4 + 2 x3 y + x 2 y 2 = 2 x + 9
5) 2
x + 2 xy = 6 x + 6;
xy + x + y = x 2 2 y 2
6)
x 2 y y x 1 = 2 x 2 y;
x 4 x3 y + x 2 y 2 = 1
7) 3
2
x y x + xy = 1;

x 3 3 x 2 y + 2 x 6 y = 15
8)
x 3 + x 2 y + 2 x + 2 y = 9;
x 2 + y 2 3x + 4 y = 1
9)
3 x 2 2 y 2 9 x 8 y = 3;
( x y ) 2 y = 2
10) 3
3
x y = 19;

27

x 3 + y 3 = 1
11) 2
2
3
x y + 2 xy + y = 2;
3
3
2
x y = x y
12) 2
2
x + y = x y.

II.16. Gii cc h phng trnh


3 x y = x y
1)
x + y = x + y + 2;
x 2 + y 2 = 25 xy
2)
y ( x + y ) = 10;
x + y = 2
3)
x + 3 + y + 3 = 4;

3 x 2 + xy + 3 x + y = 3
4)
xy 2 + 2 x 2 + y 2 + 2 x = 2;
x 2 y 2 2 x 2 + y 3 2 y = y 2
5)
x 2 + y 2 y 2 = 0;
x + x 2 2 x + 2 = 3 y 1 + 1
6)
y + y 2 2 y + 2 = 3x 1 + 1;
xy 3 x 2 y = 16
7) 2
2
x + y 2 x 4 y = 33;
x 3 + y + 2 = 3
8)
x + y = xy + 2 x 3 y 6 + 4;

( x + y )(2

9)
( x y )(2 +

1
)=
xy
1
)=
xy

9
2
5
;
2

x 2 + xy = 2
10) 3
2
x + 2 xy 2 y = x;

28

x
y
7
+
=
+1

x
xy
11) y

x xy + y xy = 78;
x 1 + y 1 = 3
12)
x + y ( x 1)( y 1) = 5;

x 2 + 6 y = y + 3
13)
x + y + x y = 4.
II.17. Chng minh rng vi a 0, h phng trnh

2
a2
x
=
y
+
2

2
2 y 2 = x + a

c nghim duy nht.


II.18. Cho h phng trnh

x 2 4 xy + y 2 = k
2
y 3 xy = 4

1) Gii h vi k = 1;
2) Chng minh rng h phng trnh cho lun lun c nghim vi m i k .
II.19. Tm cc gi tr ca a h phng trnh sau c nghim

x + 1 + y + 2 = a

x + y = 3a.
II.20. Tm cc gi tr ca m h phng trnh sau c nghim

4 x y m = 0

3 x + x( y + 1) = 1.
II.21. Tm cc gi tr ca m h phng trnh sau c nghim duy nht
2
x = y y + m

2
y = x x + m

II.22. Tm cc gi tr ca m h phng trnh sau c nghim

29

x 2 5 x + 4 0
2
3 x mx x + 16 = 0.
II.23. Tm cc gi tr ca m h phng trnh sau c nghim
x + 1 + y = m

y + 1 + x = 1.

II.24. Cho h phng trnh


x + xy + y = m + 1
2
2
x y + xy = m.

1) Gii h phng trnh vi m = 2;


2) Tm cc gi tr ca m h phng trnh c t nht mt nghim ( x; y ) tha mn
x > 0; y > 0.
II.25. Cho h phng trnh

m( x 2 + 1) + y 2 = m + 1

2
2
x + my = 1

1) Gii h phng trnh khi m = 1;


2) Tm cc gi tr ca m h phng trnh c nghim.
II.26. Cho h phng trnh

x 2 my 2 + 2 x + 2 m = 0

m( x 2 + 2 x + 2) y 2 = m + 2

1) Gii h phng trnh khi m = 1;


2) Tm cc gi tr ca m h phng trnh c nghim.
II.27. Cho h phng trnh
x( x + 2)(2 x + y ) = 9
2
x + 4 x + y = m.

Tm cc gi tr ca m h phng trnh cho c nghim


II.28. Cho h phng trnh
x + 1 + y 2 = m

x 2 + y + 1 = m .

Tm cc gi tr ca m h phng trnh cho c nghim


30

CHNG III
BT NG THC BT PHNG TRNH
A. TM TT L THUYT
I. I CNG V BT NG THC
1. nh ngha

Cho hai s a, b K (K l trng s hu t hay trng s thc ). Ta ni a ln hn


b v k hiu a > b nu a b l mt s dng. Khi , ta cng ni b b hn a v k hiu
b < a.
Ta ni a ln hn hay bng b v vit l a b nu a b l mt s dng hay bng
khng. Khi , ta cng ni b b hn hay bng a v vit b a.
Gi s A( x), B( x ) l hai biu thc ton hc vi tp xc nh chung l D ca bin s x
(hoc c th xem l hai biu thc ton hc ca cng n bin s x1 , x2 ,..., xn nu ta xem
x = ( x1 , x2 ,..., xn ) K n ).

Ta ni A( x ) < B( x ) hay B( x ) > A( x)


( A( x ) B( x ) hay B( x ) A( x ) )
Nu ti m i gi tr ca bin s x D ta u c:
A( x0 ) < B( x0 ) hay B( x0 ) > A( x0 )
( A( x0 ) B( x0 ) hay B( x0 ) A( x0 )) l cc bt ng thc ng.
Ta gi a > b; a b; A( x) < B( x); A( x ) B( x ) l bt ng thc.
2. Tnh cht c bn ca bt ng thc

Ta chng minh c d dng cc tnh cht sau y, trong A, B, C ,... l cc s hoc


cc biu thc ton hc ca cng mt s bin s xt trn cng mt trng s K .
2.1. A < B B > A
2.2. A > B, B > C A > C
2.3. A > B A + C > B + C

A > B
2.4.
A+C > B+ D
C > D
Am > Bm; m > 0
2.5. A > B
Am < Bm; m < 0
A > B
2.6.
AD > BC
C > D

31

A > B > 0
2.7.
AC > BD
C > D > 0
2.8. A > B > 0 An > B n (n * )
2.9. A > B > 0 n A > n B (n * \ {1})
2.10. A > B > 0 hoc B < A < 0

1 1
> .
B A

3. Mt s bt ng thc quan trng

Cc bt ng thc sau y thng c dng gii cc bi ton v bt ng thc.


3.1. Bt ng thc v du gi tr tuyt i. Cho a, b, ai , i = 1, 2,..., n l cc s thc. Th th
a + b a + b (*); a b a b (**); a1 + a2 + ... + an a1 + a2 + ... + an (***).
Du = trong (*) v (**) xy ra, khi v ch khi ab 0.
Du = trong (***) xy ra, khi v ch khi cc s ai 0 hoc ai 0, i = 1, 2,..., n.
3.2. Bt ng thc Csi
Cho n s thc a1 , a2 ,..., an khng m. Th th
a1 + a2 + ... + an n
a1.a2 ...an
n

Du = xy ra khi v ch khi a1 = a2 = ... = an .


3.3. Bt ng thc Bunhiacpski
Cho n cp s thc (ai ; bi ), i = 1, 2,, n.
Th th
2

n
n 2 n 2
a
b
i i ai bi
i =1
i =1 i =1

Du = xy ra khi v ch khi tn ti k sao cho bi = kai , i = 1, 2,, n.


4. Cc phng php chng minh bt ng thc

4.1. Phng php qui v nh ngha


chng minh A > B (hoc A B ), ta chng minh A B > 0 ( hoc A B 0 ).

4.2. Phng php bin i tng ng


chng minh bt ng thc cho l ng, ta bin i bt ng thc cho tng
ng vi mt bt ng thc hin nhin ng. Khi ta c kt lun bt ng thc cho l
ng.

4.3. Phng php vn dng cc bt ng thc bit


32

T cc bt ng thc bit l ng ta suy ra bt ng thc cn chng minh.


4.4. Phng php s dng tam thc bc hai
4.5. Phng php chng minh qui np
4.6. Phng php vec t
Mt s kt qu sau c th suy ra t cc tnh cht ca cc php ton vc t.

Gi s a = (a1 ; a2 ), b = (b1 ; b2 ). Ta c

a = a12 + a22

a b = (a1 b1 ; a2 b2 )

ka = (ka1 ; ka2 )

a.b = a1b1 + a2b2

a.b = a . b .cos(a , b )

2
2
(a) = a 0

a + b a + b . Du ng thc xy ra khi v ch khi a , b cng hng.

a b a b . Du ng thc xy ra khi v ch khi a , b cng hng.

a.b a . b . Du ng thc xy ra khi v ch khi a , b cng phng.


II. BT PHNG TRNH
1. nh ngha

Cho hai hm s f ( x), g ( x), vi x n trong f ( x ), g ( x ) ln lt c min xc nh


l D1 , D2 . Hai hm s f ( x ), g ( x ) c xt trong D = D1 D2 .
Bt phng trnh f ( x) > g ( x) (1) l k hiu ca hm mnh Gi tr ti x ca hm s f
ln hn gi tr ti x ca hm s g .
Gii bt phng trnh l tm cc gi tr x0 D sao cho f ( x0 ) > g ( x0 ) l mt bt ng
thc ng. Gi tr x0 c gi l mt nghim ca bt phng trnh (1).
Ch .
Nu n =1 th ta c bt phng trnh mt n x trn .
Nu n >1 th ta c th xem x = ( x1 , x2 ,..., xn ) n .
Khi , ta c bt phng trnh n n x1 , x2 ,..., xn .
Hon ton tng t nh trn ta nh ngha c khi nim cc bt phng trnh
f ( x ) < g ( x ); f ( x ) g ( x ); f ( x ) g ( x) .
33

Cc khi nim h bt phng trnh, tuyn bt phng trnh c nh ngha tng t


nh trng hp phng trnh.
2. S tng ng ca cc bt phng trnh

Khi nim bt phng trnh tng ng, bt phng trnh h qu cng c nh


ngha tng t nh i vi phng trnh. Sau y ta a ra mt s nh l v bt phng
trnh tng ng.
Ta k hiu cc v ca bt phng trnh bi f , g ,..., khng ghi tn cc n cho gn,
nhng c th hiu l mt n hoc cng n n.
2.1. nh l. f > g g < f .
2.2. nh l. f > g f + h > g + h.

( h c ngha trong min xc nh ca bt phng trnh cho).


2.3. nh l.

fh > gh

h > 0
f >g
fh < gh

h < 0.
2.4. nh l.

f .g > 0

f
> 0.
g

Ch . Tuy nhin, i vi cc h bt phng trnh th cc nh l lm c s cho cc phng


php th v phng php kh trong l thuyt h phng trnh khng cn ng na.
Chng hn, cc h bt phng trnh
F > 0
(I) 1
v (II)
F2 > 0

F1 > 0

F1 + F2 > 0

l khng tng ng.


Tht vy, (II) l h qu ca (I), song (I) li khng phi l h qu ca (II).
3. ng dng ca gi tr ln nht v gi tr nh nht vo vic gii phng trnh v bt
phng trnh

Cho hm s y = f ( x ) c tp xc nh l D, gi s hm s y = f ( x ) c gi tr ln nht
v gi tr nh nht trn D, khi ta c:

Bt phng trnh f ( x ) c nghim x D khi v ch khi


Max f ( x) .
xD

Bt phng trnh f ( x ) nghim ng vi m i x D khi v ch khi


34

Min f ( x) .
xD

Bt phng trnh f ( x ) c nghim x D khi v ch khi


Min f ( x ) .
xD

Bt phng trnh f ( x ) nghim ng vi m i x D khi v ch khi


Max f ( x ) .
xD

Nu hm s y = f ( x) lin tc trn D th phng trnh f ( x ) = c nghim x D khi


v ch khi Min f ( x ) Max f ( x ).
xD

xD

III. BT PHNG TRNH BC NHT, BC HAI MT N


1. Bt phng trnh bc nht mt n

1.1. Bt phng trnh bc nht mt n


nh ngha. Bt phng trnh bc nht mt n l bt phng trnh c dng
ax + b > 0 (1), hoc ax + b < 0; ax + b 0; ax + b 0 (a, b , a 0).

Cc trng hp nghim ca bt phng trnh bc nht ax + b > 0 (1)


b

Nu a > 0, (1) c tp nghim l S = x / x >


;
a

Nu a < 0, (1) c tp nghim l S = x / x <


.
a

1.2. Gii v bin lun bt phng trnh ax + b > 0


Nu a > 0 th (1) x >

b
b

. Vy, tp nghim ca (1) l S = ; + ;


a
a

Nu a < 0 th (1) x <

b
b

.Vy, tp nghim ca (1) l S = ; ;


a
a

Nu a = 0 th (1) tr thnh 0 x > b. Do


(1) v nghim nu b 0;
(1) nghim ng vi m i x nu b > 0.
1.3. nh l v du ca nh thc bc nht f ( x ) = ax + b; a 0
t x 0 =

b
l nghim ca f ( x ). Khi , ta c
a

i) f ( x ) cng du vi h s a khi x0 >


ii) f ( x ) tri du vi h s a khi x0 <

b
;
a

b
.
a

35

Kt qu ca nh l c tm tt trong bng sau

f ( x ) = ax + b

tri du vi a

b
a

+
cng du vi a

Ch .
1. S dng nh l v du ca nh thc bc nht ta c th gii c cc bt phng trnh
dng
P (x )
P (x )
P (x )
P (x )
<0;
> 0;
0;
0.
Q (x )
Q (x )
Q (x )
Q (x )

Trong , P( x ) v Q (x ) l tch ca nhng nh thc bc nht.


2. gii cc bt phng trnh cha du gi tr tuyt i, ta kh du gi tr tuyt i bng
nh ngha v cc tnh cht sau
f ( x) < g ( x )
f ( x ) < g ( x)
f ( x) > g ( x )
f ( x) > g ( x)
f ( x) > g ( x)
f ( x) < g ( x)

f ( x ) > g ( x) [ f ( x)]2 > [ g ( x )]2


f ( x ) < g ( x) [ f ( x)]2 < [ g ( x )]2 .
2. Bt phng trnh bc hai mt n

2.1. nh l v du ca tam thc bc hai f ( x) = ax 2 + bx + c; a 0


nh l. Cho tam thc bc hai f ( x ) = ax 2 + bx + c

+ Nu < 0 th f ( x ) cng du vi h s a vi m i x ;
+ Nu = 0 th f ( x ) cng du vi h s a vi m i x

b
;
2a

+ Nu > 0 th f ( x ) c hai nghim phn bit x1 , x2 , ( x1 < x2 ) .


Khi f ( x ) tri du vi h s a nu x nm trong khong ( x1; x2 ), f ( x ) cng du vi h
s a nu x nm ngoi on [ x1 ; x2 ] .
2.2. Bt phng trnh bc hai mt n
nh ngha. Bt phng trnh bc hai n x l bt phng trnh c dng ax 2 + bx + c > 0
(hoc ax 2 + bx + c 0; ax 2 + bx + c < 0; ax 2 + bx + c 0 ). Vi a, b, c v a 0.

Cch gii.
36

gii bt phng trnh bc hai ta p dng nh l v du ca tam thc bc hai.

Ch . Cng nh trng hp bt phng trnh bc nht, ta cng gii c cc bt phng


trnh dng
P (x )
P (x )
> 0;
<0 ;
Q (x )
Q (x )

P( x )
P (x )
0 ;
0.
Q (x )
Q( x )

Trong P ( x ) ; Q ( x ) l tch cc tam thc bc hai.


2.3. nh l o v du ca tam thc bc hai
Theo nh l v du ca tam thc bc hai th ch trong trng hp f ( x ) c nghim
x1 , x2 th af ( x)< 0 v x2 < x < x1 , do ta c nh l o ca nh l v du ca tam thc
bc hai nh sau.
nh l. Cho tam thc bc hai f ( x ) = ax 2 + bx + c, nu tn ti s thc sao cho af ( ) < 0
th f ( x ) c hai nghim phn bit x1 , x2 ( x1 < x2 ) v nm trong khong ( x1 ; x2 ).

T nh l o v du ca tam thc f ( x ) ta c php so snh nghim ca f ( x ) vi mt s


nh sau.
+ Nu f ( ) = 0 th l nghim ca f ( x);
+ Nu af () < 0 th nm gia hai nghim x1 , x2 ca f ( x);
+ Nu af ( ) > 0 v f ( x) c hai nghim x1 , x2 th nm ngoi on [ x1 ; x2 ] v hn na
< x1 < x2 nu

s
> ;
2

x1 < x2 < nu

s
< .
2

H qu. iu kin tam thc bc hai f ( x) = ax 2 + bx + c c hai nghim, trong c mt


nghim nm trong khong (; ), cn nghim kia nm ngoi on [; ] l f (). f () < 0.
B. BI TP
III.1. 1) Chng minh rng vi m i a, b, c ta c

a) a 2 + b 2 + 1 ab + a + b; ng thc xy ra khi no?


b) a 2 + b 2 + 4 ab + 2 ( a + b ) ; ng thc xy ra khi no?
c)

a2
+ b 2 + c 2 ab ac + 2bc. ng thc xy ra khi no?
4

2) Cho x, y, z l cc s dng. Chng minh rng

x 2 + xy + y 2 + y 2 + yz + z 2 + z 2 + zx + x 2 3 ( x + y + z ) .
3) Cho a, b, c l di ba cnh ca mt tam gic. Chng minh rng

37

1 1 1 3 ( a b )( b c )( c a )
+ + +
9.
abc
a b c

( a + b + c )

4) Cho x 0, y 0. Chng minh rng

( x + y)

x+ y
2x y + 2 y x.
2

III.2. Chng minh rng

1)

a b
c
1 1 1
+ +
2 + , (a, b, c > 0);
bc ca ab
a b c

2) 1 <

a
b
c
d
+
+
+
< 2, (a, b, c, d > 0).
a+b+c b+c+d c+d +a d +a +b

3) 1 <

a
b
c
+
+
< 2 , ( a, b, c l di ba cnh ca mt tam gic).
b+c c+a a +b

III.3. 1) Chng minh rng

a) (a + b)(ab + 1) 4ab (a, b > 0);


b) (a + b)(b + c)(c + a) 8abc (a, b, c > 0);
c) a 2 (1 + b 2 ) + b 2 (1 + c 2 ) + c 2 (1 + a 2 ) 6abc.
2) Cho a > 0, b > 0, c > 0. Chng minh rng

a2
b2
c2
a+b+c
+
+

.
b+c c+a a +b
2
III.4. 1) Cho u , v, x, y tha u 2 + v 2 = x 2 + y 2 = 1 . Chng minh rng

a) ux + vy 1;
b) u ( x + y ) + v( x y ) 2 .
2) Cho x > 0, y > 0, z > 0 v xyz = 1. Chng minh rng

x2
y2
z2
3
+
+
. Khi no ng thc xy ra?
y+ z z+ x x+ y 2
3) Cho a > 0, b > 0, c > 0 v a + b + c = 1. Chng minh rng
1
1
1
1
+
+ + 30. Khi no ng thc xy ra?
2
2
a + b + c ab bc ca
2

III.5. 1) Cho a 2 + b 2 + c 2 + d 2 = 1 . Chng minh rng

( x 2 + ax + b)2 + ( x 2 + cx + d )2 (2 x 2 + 1)2 , x .
2) Cho a, b, c l ba s thc dng tha mn iu kin ab + bc + ca = abc. Chng
minh rng

38

1
1
1
3
+
+
<
.
a + 2b + 3c 2a + 3b + c 3a + b + 2c 1 + 2 + 3 2

III.6. 1) Chng minh (sin 2 x +

1 2
1 2 25
) + (cos 2 x +
) . Khi no ng thc xy
2
sin x
cos 2 x
2

ra?
2) Cho x, y > 0 v tha x 2 + y 2 = 1. Chng minh rng

(1 + x ) 1 +

1
1
+ (1 + y ) 1 + 4 + 3 2. Khi no ng thc xy ra?
y
x

III.7. 1) Chng minh rng vi m i x ; x 0, x

+ k , x k , k , ta lun c
2

tan 2 x cot 2 x
1
+ 1 2 + 1 2 + 1 . Khi no ng thc xy ra?
x2
x
x

2) Cho a, b, c l ba s thc dng tha mn iu kin ab + bc + ca = abc.


Chng minh rng
b 2 + 2a 2
c 2 + 2b 2
a 2 + 2c 2
+
+
3.
ab
bc
ca
Khi no ng thc xy ra?
3) Cho x, y, z > 0 v tha x + y + z 1. Chng minh rng
x2 +

1
1
1
+ y 2 + 2 + z 2 + 2 82.
2
x
y
z

4) Cho x, y, z > 0 v tha xyz = 1. Chng minh rng


1 + x3 + y3
1 + y3 + z 3
1 + z 3 + x3
+
+
3 3.
xy
yz
zx

Khi no ng thc xy ra?


III.8. 1) Chng minh rng vi m i x, y th

x 2 (1 + sin 2 y ) + 2 x (sin y + cos y ) + 1 + cos 2 y > 0 .


2) Cho a b > 0. Chng minh rng
b

a 1 b 1
2 + a 2 + b .
2
2

3) Chng minh: 2sin x + tan x 3 x > 0 , vi 0 < x <

x3
4) Chng minh: x < sin x < x, vi m i x > 0.
6

39

III.9. 1) Cho a, b, c l cc s khng m. Chng minh rng

a 2 (b + c a) + b 2 (c + a b) + c 2 (a + b c ) 3abc.
2) Cho a, b, c, l cc s dng. Chng minh rng
a
b
c
3
+
+
.
2a + b + c 2b + c + a 2c + a + b 4

3) Chng minh rng vi m i s thc dng x, y, z tha mn x ( x + y + z ) = 3 yz ,


3

ta c ( x + y ) + ( x + z ) + 3 ( x + y )( x + z )( y + z ) 5 ( x + z ) .
Khi no ng thc xy ra?
III.10. 1) Cho a, b l cc s dng, n . Chng minh rng
a
b
(1 + )n + (1 + )n 2n +1.
b
a

2) Cho a 0, b 0, n *. Chng minh rng


n

an + bn
a+b

2
2

III.11. Cho a, b, c, l cc s dng. Chng minh rng

1)

1
1
1
1
+ 3 3
+ 3

;
3
3
a + b + abc b + c + abc c + a + abc abc

2)

a2
b2
c2
a +b+c
+
+

.
b+c c+a a+b
2

III.12. Gii cc bt phng trnh sau


1)

2x 5
+ 1 > 0;
| x 3|

2) x 2 1
3)

2
;
x2

| x2|
3;
x 5x + 6 |
2

2 3| x |
1;
1+ x
| x + 2 | x
5)
2;
x
| x 2 4 x | +3
6) 2
1.
x + | x5|
4)

III.13. Gii cc bt phng trnh sau

40

1)

2 x
1 2x
> 3
;
3
2
x +x
x 2x2

2)

x 4 3 x3 + 2 x 2
> 0;
x 2 x 30

3)

x3 3x2 x + 3
0;
2x x2

x 4 4 x2 + 3
4) 2
0;
x 8 x + 15
5)

1
2
2x + 3
+ 2
< 3
;
x +1 x x +1 x +1

6) x 2 + ( x + 1) 2

15
;
x + x +1
2

7) 2 x3 + x 2 5 x + 2 > 0;
8) 2 x3 + x + 3 0.
III.14. Gii v bin lun cc bt phng trnh sau theo tham s m

1) ( m 3) x 2 2 m x + m 6 0;
2) ( m 4) x 2 2( m 2) x + m 1 0;
3) m x 2 2( m 3) x + m 4 < 0.
III.15. Cho tam thc bc hai

f ( x ) = (m + 1) x 2 2(m 1) x + 3m 3
Tm cc gi tr ca m
1) Bt phng trnh f ( x) < 0 v nghim;
2) Bt phng trnh f ( x) 0 c nghim.
III.16. Tm cc gi tr ca m cc bt phng trnh sau c tp hp nghim l

3x 2 mx + 5
< 6;
2x2 x +1
x 2 + mx + 1
2)
< 2.
x2 +1

1) 1

III.17. Tm cc gi tr ca m cc phng trnh sau y c cc nghim x1 , x2 tha iu

kin c ch ra

41

1) x 2 (2m + 3) x + m 2 = 0; x1 < 3 < x2 ;


2) mx 2 + 2(m 1) x + m 5 = 0; x1 < x2 < 2;
3) (m 1) x 2 (m 5) x + m 1 = 0; 1 < x2 < x2 .
III.18. Bin lun theo m v tr ca s 1 vi cc nghim ca phng trnh

(3 m) x 2 2(2m 5) x 2m + 5 = 0.
III.19. Tm cc gi tr ca m cc phng trnh sau c nghim x1 , x2 tha iu kin c

ch ra
1) mx 2 2(m + 1) x + m + 5 = 0; x1 < 0 < x2 < 2;
2) (m 2) x 2 2mx + 2m 3 = 0; 6 < x1 < 4 < x2 .

III.20. Bin lun theo m v tr ca s 0 v s 2 i vi nghim ca phng trnh

mx 2 2(m 1) x + m 3 = 0.
III.21. Tm cc gi tr ca m phng trnh

2 x 2 + (2m 1) x + m 1 = 0
c mt nghim nm trong khong (1;3), cn nghim kia nh hn 1.
III.22. Cho phng trnh

(m 1) x 2 2mx + m + 5 = 0
Tm cc gi tr ca m phng trnh
1) C hai nghim u ln hn 2;
2) C t nht mt nghim ln hn 2.
III.23. Cho f ( x ) = mx 2 2(m + 1) x m + 5 .

Tm cc gi tr ca m f ( x) > 0, x < 1.
III.24. Cho f ( x ) = 2 x 2 (3m + 1) x (3m + 9) .

Tm cc gi tr ca m f ( x) 0, x [ 2;1] .
III.25. Cho f ( x ) = (m 2) 2 x 2 3(m 6) x m 1 .

Tm cc gi tr ca m f ( x ) < 0, x ( 1, 0 ) .
III.26. Cho bt phng trnh

( x + 2)( x + 4)( x 2 + 6 x + 10) m.


Tm cc gi tr ca m bt phng trnh nghim ng vi x .
III.27. Cho bt phng trnh

42

2cos 2 x + 3mcosx +1 0.
Tm cc gi tr ca m bt phng trnh nghim ng vi x [0; ].
III.28. Cho bt phng trnh
x2 +

1
1
+ (2m + 3)( x + ) + 2(m + 2) > 0.
2
x
x

Tm cc gi tr ca m bt phng trnh nghim ng vi x 0.


III.29. Cho bt phng trnh

x 3 (2m + 1) x 2 + 3(m + 4) x m 12 > 0.


Tm cc gi tr ca m bt phng trnh nghim ng vi x > 1.
III.30. Cho bt phng trnh

( x 1)( x + 1)( x + 3)( x + 5) > m.


Tm cc gi tr ca m bt phng trnh nghim ng vi x > 1.
III.31. Cho bt phng trnh

x( x 2)( x + 2)( x + 4) < 2m.


Tm cc gi tr ca m bt phng trnh c nghim x > 0.
III.32. Chng minh rng phng trnh 4 x ( 4 x 2 + 1) = 1 c ng ba nghim phn bit.
CHNG IV.
PHNG TRNH, BT PHNG TRNH V T
A. TM TT L THUYT
I. PHNG TRNH V T
1. nh ngha v cc nh l
1.1. nh ngha

Ta g i phng trnh v t, m i phng trnh c cha n di du cn hay ni khc i


l phng trnh dng f ( x ) = 0, trong f ( x ) l mt hm s c cha cn thc ca bin s.
1.2. Cc nh l. (Cc nh l sau lm c s cho vic gii phng trnh v t).
2 k +1

1.2.1. nh l. f ( x) = g ( x ) [ f ( x)]

= [ g ( x)]2k +1

1.2.2. nh l.

2 k +1

f ( x) = g ( x ) f ( x) = [ g ( x )]2 k +1

1.2.3. nh l.

2 k +1

f ( x) = 2 k +1 g ( x ) f ( x ) = g ( x)

1.2.4. nh l.

2k

g ( x) 0
f ( x ) = g ( x)
2k
f ( x) = [ g ( x)]

43

1.2.5. nh l.

2k

f ( x ) 0 g ( x) 0
f ( x) = 2k g ( x)
f ( x) = g ( x)

(Vi k l s t nhin khc 0).


2. Cc phng php gii phng trnh v t

2.1. Phng php nng ln ly tha


2.2. Phng php t n ph
2.3. Phng php lng gic ha
Trong mt s trng hp, nu chng ta t n ph bi cc hm s lng gic, th vic gii
quyt bi ton tr nn d dng hn. Kin thc cn nh nh sau.
+ Nu trong phng trnh, iu kin ca n x l k x k , k > 0 hay phng trnh c cha

k 2 x 2 th t x = k sin t , t [ ; ]; hoc t x = k cos t , t [0; ].
2 2
+ Nu trong phng trnh, iu kin ca n x l x k , k > 0 hay phng trnh c cha
x 2 k 2 th t x =

3
k

; t [0; ) [; ); hoc t x =
, t [ ; 0) (0; ].
cos t
2
2
sin t
2
2

+ Nu trong phng trnh, n x nhn mi gi tr thuc hay phng trnh c cha



th t x = k tan t , t ; .
2 2

x2 + k 2

Ngoi ra, ty tng trng hp, cng c th t x = cos 2 t; x = sin 2 t ,...


2.4. Mt s phng php khc
II. BT PHNG TRNH V T
1. nh ngha v cc nh l
1.1. nh ngha

Bt phng trnh v t l mt bt phng trnh c cha n di du cn thc. Ni khc


i l mt bt phng trnh c dng f ( x ) > 0, (hoc f ( x) < 0, f ( x ) 0, f ( x) 0 ), trong
f ( x ) l hm s c cha cn thc ca bin s.
1.2. Cc nh l
1.2.1. nh l.

2 k +1

f ( x) g ( x ) f ( x ) g 2 k +1 ( x) .

1.2.2. nh l.

2 k +1

f ( x) g ( x ) f ( x ) g 2 k +1 ( x) .

1.2.3. nh l.

44

2k

g ( x) 0

f ( x) 0
f ( x) g ( x )
g ( x) 0

f ( x ) g 2 k ( x )

1.2.4. nh l.

2k

f ( x) 0

f ( x ) g ( x) g ( x) 0
f ( x) [ g ( x )]2 k

2. Cc phng php gii bt phng trnh v t

2.1. Phng php nng ly tha


2.2. Phng php t n ph
2.3. Mt s phng php khc
B. BI TP
IV.1. Gii cc phng trnh

1) (16 x 2 ) 3 x = 0;
2) (9 x 2 ) 2 x = 0;
3)

4 + 2 x x 2 = x 2;

4) 1 + 4 x x 2 = x 1;
5)

2x +1 + x 3 = 2 x;

6)

x + 1 + 4 x + 13 = 3 x + 12;

7) ( x + 3) 10 x 2 = x 2 x 12;
8)

x + 4 1 x = 1 2x.

IV.2. Gii cc phng trnh

1) 3 x 2 + 15 x + 2 x 2 + 5 x + 1 = 2;
2)

x 2 3x + 3 + x 2 3x + 6 = 3;

3)

x + 2 + 5 x + ( x + 2)(5 x) = 4;

4)

x + 4 + x 4 = 2 x 12 + 2 x 2 16;

5) 1 +

2
x x2 = x + 1 x ;
3

6) 1 + 1 + x x 2 24 = x;
7)

x + x + 11 + x x + 11 = 4;

8) x 3 35 x3 ( x + 3 35 x 3 ) = 30;
9) x 3 + 2 = 3 3 3 x 2;
45

10) 2 3 (1 + x) 2 + 3 3 1 x 2 + 3 (1 x )2 = 0;
11) 2 x + 6 3 1 x + 2 = 0;
12) 3 x + 1 = 3 x 2 8 x + 3;
x + 3 x + 1 = x 2 + x + 1.

13)

IV.3. Gii cc phng trnh

1) x x 2 + 15 x 4 x 2 + 15 = 2;
4
= 2;
2 x +3

2)

2 x +

3)

9 5x = 3 x +
4

4)

x + x2 + x

6
;
3 x
1

3
= ;
x x2 + x x

x 2 + 2 x + 1 + x 2 2 x + 1 = 2;

5)

6) (2 x 2 + 6 x + 10) x 2 + 3 x 11x 2 33 x + 8 = 0.
2 x 2 + 4 x 3 x + 2 2 x + 3 2 x = 0;

7)

8) 2 3 3x 2 + 3 6 5 x 8 = 0;
9)

x + 1 = 32 4 x 4 x + 1 x;

10)

2 x = 1 x 1;

11)

9 x = 2 x 1;

12) 2 3 1 x 2 + 4 x 2 = 4.
IV.4. Gii cc phng trnh

1)

x + 2 x 1 + x 2 x 1 =

x+3
;
2

2) 2 x + 2 + 2 x + 1 x + 1 = 4;
3)

2( x 2 2 x + 4)
= 2 x + 2 + 3 x 2 2 x + 4;
x+2

4) 2( x 2 3x + 2) = 3 x3 + 8;
5) x 3 + (1 x 2 )3 = x 2(1 x 2 ) ;
46

3
6) 1 + 1 x 2 (1 x )

(1 + x )

= 2 + 1 x2 ;

7) 1 x 2 x 1 x 2 2 x 2 + 1 = 0;
8)

9)

x2 + 1 +

x 2 + 1 ( x 2 + 1)2
=
;
2x
2 x (1 x 2 )

2x +1 + 3 2x =

( 2 x 1)

IV.5. Gii cc bt phng trnh

1) ( x 1) x 2 x 2 0;
2) ( x 2 1) x 2 x 2 0;
3)

2 x x 2 < 5 x;

4)

x 2 3x + 2 x 3 > 0;

5)

x + 3 + x + 2 2 x + 4 > 0;

6)

3x 2 + 5 x + 7 3 x 2 + 5 x + 2 > 1;

7)

x + x + 9 x + 1 + x + 4;

8)

5 x 1 x 1 > 2 x 4;

9)

x 2 + 3 x + 2 + x 2 + 6 x + 5 2 x 2 + 9 x + 7;

10)

x 2 + x 2 + x 2 + 2 x 3 x 2 + 4 x 5;

11)

1 1 4 x2
< 3.
x

IV.6. Gii cc bt phng trnh

1) 2 x 2 4 x + 3 x 2 + 4 x 5;
2) 5 x +
3)

5
2 x

2x +

x 1 + x + 3 + 2

1
+ 4;
2x

( x 1)( x + 3) > 4 2 x;

4) x 3 + x 2 + 3 x x + 1 + 2 > 0;
5)

7 x + 7 + 7 x 6 + 2 49 x 2 + 7 x 42 < 181 14 x;

6) 2( x 2)2 + 2 x x 2 + x ;
47

2( x + 2)2 + 2(2 x 1) > x + 2 + 2 x 1;

7)

8) x 2 + 4 x ( x + 4) x 2 2 x + 4;
9) x 2 1 2 x x 2 + 2 x ;
10) ( x 1) 2 x 1 3( x 1);
11) ( x 2 1)

x 2 1 1 + x 2 1 6 > 0;

12) x 2 x 1 5 4 x;
13)

x3 2 x 2 + x < x x + x 2 2 x .

IV.7. Gii cc bt phng trnh

1)

41

16 2
+ 3;
x x

2) x 2 x (2 + 12 2 x x 2 );
3)
4)

4 x2 +

x
0;
x

x 2 13x + 40
19 x x 2 78

0;

5) x 1 x < 0;
6)
7)
8)
9)

2( x 2 16)
x3

+ x 3 >

7
x3

x 2 + 4 x + 4 x + 4 2 x 8 x > 0;
1
3x
+1 >
;
2
1 x
1 x2
x > 1 + 3 x 1;

10) x +

2x
x2 4

> 3 5.

IV.8. Tm cc gi tr ca m phng trnh sau c ng hai nghim phn bit

x 52 x 6 + x 4 x 6 2 = m .
IV.9. Tm cc gi tr ca m cc phng trnh sau c nghim

1) x + 4 x 2 + x 4 x 2 = m;
48

2)

1
4
1
+
+ 4 x = 4( + 2 x ) + m.
2
x
x
x

IV.10. Cho phng trnh


5
x 2 + ( m 2 ) x 2 + 4 + 2 m3 = 0.
3

Chng minh rng phng trnh c nghim vi m i m > 0.


IV.11. Cho phng trnh
x + 1 + x + 4 + x 2 + 5 x + 4 + x + 2m = 0.

Tm cc gi tr ca m phng trnh c nghim khng m.


IV.12. Bin lun theo m s nghim ca phng trnh

5+ x + 7 x + m

( 5 + x )( 7 x ) = 2m + 1.

IV.13. Tm cc gi tr ca m phng trnh sau c nghim


3 x 1 + m x + 1 = 2 4 x2 1 .

IV.14. Tm cc gi tr ca m bt phng trnh

m( x 2 2 x + 2 + 1) x 2 2 x + 6 + x 2 2 x + 2
c nghim thuc on [0; 2].
IV.15. Tm cc gi tr ca m phng trnh
x4 + 4 x + m + 4 x4 + 4 x + m = 6

c hai nghim.
IV.16. Chng minh rng vi m i m > 1, phng trnh sau lun lun c hai nghim phn

bit
x 2 2 x 3 = ( m + 1)( x 3).

IV.17. Cho phng trnh

( x 1)3 + mx = m + 1.
Chng minh rng phng trnh lun lun c mt nghim duy nht vi m i m.
IV.18. Tm cc gi tr ca m bt phng trnh
(4 + x )(6 x ) x 2 2 x + m

nghim ng vi m i x [ 4; 6] .
IV.19. Tm cc gi tr ca m bt phng trnh

49

mx x 3 m + 1
c nghim.
IV.20. Tm cc gi tr ca m phng trnh sau c hai nghim phn bit
x 2 + mx + 2 = 2 x + 1.

IV.21. Cho bt phng trnh

( x + 1)( x + 3) m x 2 + 4 x + 5
1) Gii bt phng trnh khi m = 1;
2) Tm cc gi tr ca m bt phng trnh nghim ng vi m i x [ 2; 2 + 3].
IV.22. Cho bt phng trnh
(3 + x )(7 x ) x 2 4 x + m.

Tm cc gi tr ca m bt phng trnh nghim ng vi m i x [3; 7].


IV.23. Cho bt phng trnh

4 x 2 + 16 4 x m.
Tm cc gi tr ca m bt phng trnh c nghim.
IV.24. Cho bt phng trnh
1 x 2 m x.

Tm cc gi tr ca m bt phng trnh c nghim.


IV.25. Cho bt phng trnh
12 3 x 2 x m.

Tm cc gi tr ca m bt phng trnh c mt nghim duy nht.


IV.26. Cho bt phng trnh
m 2 x 2 + 7 < x + m.
1
1) Gii bt phng trnh khi m = ;
2

2) Tm cc gi tr ca m bt phng trnh nghim ng vi m i x .


IV.27. Cho bt phng trnh
x 2 2mx > 1 x.
1
Tm cc gi tr ca m tp hp nghim ca bt phng trnh cho cha on [ ;1].
4

IV.28. Tm cc gi tr ca m phng trnh m

50

x 2 + 2 4 x2 4 x + 2 = 2 4 x2 4

c nghim thc.
IV.29. Tm cc gi tr ca m bt phng trnh x 3 + 3x 2 1 m

x x 1

c nghim.
IV.30. Tm cc gi tr ca m phng trnh sau c ng hai nghim phn bit
4

2 x + 2 x + 2 4 6 x + 2 6 x = m.

IV.31. Tm cc gi tr ca m bt phng trnh sau c nghim x < 1

x 2 + mx 1 > ( x + m ) x 2 1.
IV.32. Tm cc gi tr ca m phng trnh sau c nghim

x 2 + mx 4 = ( x + m ) x 2 4.
CHNG V
PHNG TRNH, BT PHNG TRNH M V LOGARIT
A. TM TT L THUYT
I. NHC LI LOGARIT
1. nh ngha. Cho a l mt s dng khc 1 v b l mt s dng. S thc sao
cho a = b c gi l logarit c s a ca b v k hiu l log a b tc l
= log a b a = b.

Ch .
Khi vit log a b th phi hiu l a > 0, a 1; b > 0.
Trng hp c s a = 10 th logarit c s 10 ca s dng b ta vit l lg b v c l
logarit thp phn ca b.
Vi a = e th logarit c s e ca s dng b ta vit l ln b v c l logarit t nhin ca
1
b, hay logarit Npe ca b. (S e l gii hn lim (1 + ) x xp x bng 2, 718281828...).
x +
x

T nh ngha ta c mt s kt qu sau.
log a 1 = 0 , log a a = 1;
log a a b = b
a loga b = b
2. Cc tnh cht ca logarit
2.1. nh l.

i ) log a (bc) = log a b + log a c;1 a > 0; b, c > 0


b
ii ) log a = log a b log a c;1 a > 0; b, c > 0
c

51

iii ) log a b = log a b;1 a > 0; b > 0; .

Ch . Trong iii) nu = 2k , k * th log a b 2 k = 2k log a b ;1 a > 0; b 0.


H qu

1
i ) log a = log a b;1 a > 0; b > 0
b

ii ) log a n b =

1
log a b;1 a > 0; b > 0; n , n 2.
n

2.2. nh l

log b c =

log a c
hay log a b.log b c = log a c;1 a > 0;1 b > 0; c > 0.
log a b

H qu

i ) log a b =

1
hay log a b.log b a = 1;1 a > 0;1 b > 0. .
logb a

ii ) log a c =

1
log a c;1 a > 0; c > 0; 0.

iii) a logb c = c logb a ;1 b > 0; a, c > 0.


II. PHNG TRNH, BT PHNG TRNH M
1. nh ngha. Phng trnh, bt phng trnh m l phng trnh, bt phng trnh m
n s c mt s m ca ly tha.

Trong mt s trng hp ta xt thm n s c mt c c s ca ly tha, khi ta


phi xt hai trng hp: c s a > 1 v 0 < a < 1.
2. Mt s phng php gii phng trnh m

2.1. Phng php logarit ha


Cc dng c bn

a f ( x) = a g ( x)

a > 0

a = 1

f ( x ) = g ( x )

a f ( x ) = b f ( x) = log a b, 1 a > 0; b > 0.


2.2. Phng php t n s ph
2.3. Phng php s dng tnh cht n iu ca hm s
S dng tnh cht n iu ca hm s gii phng trnh l cch gii kh quen
thuc. Ta c ba hng p dng nh sau.
1. Bin i phng trnh v dng
52

f ( x ) = k (1)
vi k l hng s.
Nu hm s f ( x ) ng bin (nghch bin) trn khong (a; b) th phng trnh (1) c
nhiu nht mt nghim trn khong (a; b). Do nu tm c x0 thuc khong (a; b) sao
cho f ( x0 ) = k th x0 l nghim duy nht ca phng trnh.
2. Bin i phng trnh v dng
f ( x) = g ( x ) (2)
Nu hm s y = f ( x ) ng bin (nghch bin) trn khong (a; b), nhng hm s
y = g ( x ) nghch bin (ng bin) cng trn khong th phng trnh (2) c nhiu nht
mt nghim trn khong (a; b). Do , nu tm c x0 thuc khong (a; b) sao cho
f ( x0 ) = g ( x0 ) th x0 l nghim duy nht ca phng trnh.
3. Bin i phng trnh v dng
f (u ) = f (v ) (3)
Xt hm s y = f ( x), nu hm s ny n iu trn khong (a; b) th khi phng
trnh (3) tng ng vi u = v; u , v (a; b).
2.4. Mt s phng php khc
3. Mt s phng php gii bt phng trnh m

3.1. Phng php logarit ha


Cc dng c bn

a f ( x) < a g ( x)

a f (x)

a > 1

f ( x) < g ( x )

.
0 < a < 1

f ( x ) > g ( x)

a >1

f ( x ) < log a b
< b (b > 0)
.
0 < a < 1

f ( x) > log a b

a f ( x ) > b (1). (0 < a 1)


i) Nu b 0 th (1) f ( x ) c ngha.
ii) Nu b > 0 th:
+ Trng hp 1: a > 1. Khi (1) f ( x ) > log a b
+ Trng hp 2: 0 < a < 1. Khi (1) f ( x ) < log a b
3.2. Phng php t n s ph

53

3.3. Phng php s dng tnh cht n iu ca hm s


C hai hng p dng nh sau:
1. Bin i bt phng trnh v dng
f ( x) > k (1) ( k l hng s)
Nu hm s f ( x ) n iu trn khong (a; b) (gi s ng bin).
Khi ta c nhn xt: Gi s x0 thuc (a; b) l nghim ca phng trnh f ( x ) = 0, th
Vi x x0 f ( x) f ( x0 ) = k (1) v nghim.
Vi x > x0 f ( x ) > f ( x0 ) = k (1) nghim ng.
Vy, nghim ca bt phng trnh l x > x0 .
2. Bin i bt phng trnh v dng
f (u ) < f (v) (2)
Xt hm s y = f ( x), gi s hm s ng bin trn khong (a; b) , khi
f (u ) < f (v ) u < v; u, v (a, b).
III. PHNG TRNH, BT PHNG TRNH LOGARIT
1. nh ngha. Phng trnh, bt phng trnh logarit l phng trnh, bt phng trnh
c n cha trong biu thc di du logarit.

Trong mt s trng hp c xt c n cha c s ca logarit, khi ta phi xt hai


trng hp ca c s: a > 1 v 0 < a < 1.
2. Mt s phng php gii phng trnh logarit

2.1. Phng php m ha


Cc dng c bn
a > 0, a 1
log a f ( x ) = b
b
f ( x) = a
a > 0, a 1

log a f ( x ) = log a g ( x) f ( x) > 0, ( g ( x ) > 0)

f ( x) = g ( x)

2.2. Phng php t n ph


2.3. Phng php s dng tnh cht n iu ca hm s
3. Mt s phng php gii bt phng trnh logarit

3.1. Phng php m ha


Cc dng c bn

54

a > 1
0 < a < 1
log a f ( x ) < b

b
b
0 < f ( x ) < a f ( x ) > a

log a f ( x) > b

a > 1

f ( x ) > a b
0 < a <1

b
0 < f ( x ) < a

a >1
0 < a <1

log a f ( x ) > log a g ( x ) f ( x ) > g ( x ) f ( x ) < g ( x)


g ( x) > 0
f ( x) > 0

3.2. Phng php t n s ph


3.3. Phng php s dng tnh cht n iu ca hm s
B. BI TP
V.1. Gii cc phng trnh

1) 4 x 10.2 x 1 = 24;
2) 4.22 x 6 x = 18.32x ;
2

3) 3log3 x + x log 3 x =162;


4) 9

log 1 ( 2 x 2 +1)

log 1 ( x +1)

x2 5

5) 4 x
6) 9 x

+ x 1

=5

;
x 2 5

12.2 x 1

10.3x

+ x2

+ 8 = 0;

+ 1 = 0;

7) 3.4 x + (3 x 10).2 x + 3 x = 0;
8) x 2 + (2 x 3) x + 2(1 2 x ) = 0;
9) 4.33 x 3x +1 = 1 9 x .
10) 4 x
11) e

3 x +2


sin x
4

+ 4x

+6 x +5

= 42x

+3 x + 7

+ 1;

= tan x.

V.2. Gii cc bt phng trnh

1) 4 x 2.52 x 10x > 0;


2)

9 x 3x + 2 > 3x 9;

55

3)

4x + 2 x 4
2;
x 1

4) 15.2 x +1 + 1 2 x 1 + 2 x +1 ;
2

5) 2(log 2 x ) + x log 2 x 4;
6) ( 10 3)

x +1
x+3

7) 4 x 2 + x.2 x
8)

+1

( 10 + 3)

x3
x 1

0;
2

+ 3.2 x > x 2 .2 x + 8 x + 12;

8 + 2 x +1 4 x + 2 x +1 > 5;
x+2

1 2 x
> 9.
9)
3
V.3. Gii v bin lun phng trnh
2

2m x +6 24 x +3 m = (4 m 2 ) x + 3m 6.

V.4. Tm cc gi tr ca m phng trnh sau c nghim

4 x + 4 x = m(2 x + 2 x + 1).
V.5. Gii cc h phng trnh
4 x + y = 128
1) 3 x 2 y 3
= 1;
5
8log 9 ( x 4 y ) = 1
2) x 2 y
7.2 x 2 y = 8;
4

23 x = 5 y 2 4 y

3) 4 x + 2 x +1
= y;
x
2 +2
9log 2 ( xy ) = 3 + 2( xy )log 2 3
4)
x 2 + y 2 = 3x + 3 y + 6;
( x 2 + y )2 y x = 1
5)
2
x2 y
9( x + y ) = 6 ;
2

3x 3 y = ( y x )( xy + 8)
6)
x 2 + y 2 = 8;

56

3x + x = 3 + y
7)
3 y + y = 3 + x;
2 x +1 3.2 x = y 2 2
2
8)
2 y 2 3 y = 22 x 2;
4log 3 ( xy ) = 2 + ( xy )log3 2
9)
x 2 + y 2 3 x 3 y = 12;
22 x + 42 y = 2
10)
x
y
x+2 y
= 3;
2 + 4 + 2
y
x2 1
y 2 1
= ln
2 2
x
11)
2
y + 3 + 2 y = 3 + x;

x + y = 1
12) x
y
2 2 = 2;
23 x +1 + 2 y 2 = 3.23 x + y
13)
2
3 x + xy + 1 = x + 1;

32 x + 2 + 22 y + 2 = 17
14) x +1
y
2.3 + 3.2 = 8.
V.6. Gii cc phng trnh

1) log 3 ( 3x 8 ) = 2 x;
2) log x 1 3 = 2;
3)
4)

log 2 ( 9 2 x )
3 x

= 1;

1
1
log 2 ( x + 3) + log 4 ( x 8)8 = log 2 (4 x);
2
4

5) log 2 4 x + 15.2 x + 27 + 2 log 2

1
= 0;
4.2 x 3

6) log 7 ( 2 x 1) + log 7 ( 2 x 7 ) = 1;

7) 2 log 2 x + 1 log 4 x + log 2

1
= 0;
4

57

8) 2log 3 ( 4 x 3) + log 1 ( 2 x + 3) = 2;
3

9) log 3 log 21 x 3log 1 x + 5 = 2;


2
2

10) log x 2 + 2 log 2 x 4 = log


11) log

2x

8;
3

x + 1 log 1 ( 3 x ) log 8 ( x 1) = 0;
2

12) log 3 ( 9 x + 9 ) = x + log 3 ( 28 2.3x ) ;


13) 16log 27 x3 x 3log 3 x x 2 = 0;
14)

log 2 2 x 2 + log 4 16 x = log 4 x 3 ;

15) log 2 (2 x + 1) .log 2 (2 x+1 + 2) = 6;


16) log 2 2 x2 (2 x 2 x 4 ) = 2

1
;
log 3 (2 2 x 2 )
4

17) ( x +1) log 32 x + 4 x log 3 x 16 = 0;


x3 1
3

18) log3 .log 2 x log3


= + log 2 x ;
x
3 2

19) log 34 x2 (9 16 x 4 ) = 2 +

1
;
log 2 (3 4 x 2 )

20) log 2 (4 x +1 + 4) .log 2 (4 x + 1) = log

1
2

1
;
8

9
21) log 2 (2 x 2 ) .log 2 (16 x ) = log 22 x;
2

22) lg x + 1 + 3lg 1 x = lg 1 x 2 ;
23) x + lg(1 + 2 x ) = x lg 5 + lg 6;
24) log 2

2x 1
= 1 + x 2x ;
x

1
25) log 3 ( x 3 x + 2 + 2) +
5
2

26) log 3 ( x + 2) = log 2 ( x + 1);

58

x 2 + 3 x 1

= 2;

27)

3
= 1;
2 + 1 + log 2 x
x

28) log 2
29)
30)

2+ 3

( x 2 2 x 2) = log 2 + 3 ( x 2 2 x 3);

2 lg x = 1 lg x 1;

3 + log 2 ( x 2 4 x + 5) + 2 5 log 2 ( x 2 4 x + 5) = 6;

31) log 22 x + log 2 x + 1 = 1.


V.7. Gii cc bt phng trnh

1) log 1 x + 2 log 1 ( x 1) + log 2 6 0;


2

2) log x log 3 (9 x 72) 1;


3) log 2 x +3 x 2 < 1;
1
4) log 9 x2 ( x 2 + 2 x + 6) ;
2

5) log 1 (4 x + 4) log 1 (22 x +1 3.2 x );


2

6) log x 3 2( x 2 10 x + 24) log x 3 ( x 2 9);


7)

1
1
<
;
log 3 ( x + 1) 2log9 x 2 + 6 x + 9

8)

(log 2 x) 2 + 3
> 2;
log 2 x + 3

9) log 2 (2 x 1) .log 1 (2 x+1 2) > 2;


2

10) log 4 (18 2 x ) .log 2

18 2 x
1;
8

3x 1 3
11) log 4 (3x 1) .log 1
;
4
4 16
12) log x ( 9 x 2 x 1) 1;
13)

log 21 x + 4log 2 x < 2(4 log16 x 4 );


2

14) log x 2 x log x 2 x 3 ;

59

15)

lg 2 x 3lg x + 3
< 1;
lg x 1
log 1 ( x + 3) 2 log 1 ( x + 3) 3

16)

x +1

17) log 5 x + log x


18)

> 0;

x (2 log 3 x) log 5 x
<
;
3
log 3 x

x2 4
< 0;
log 1 ( x 2 1)
2

19) log x 2 x log x (2 x)3 ;


20) log 2 x + log 3 ( x + 1) < 2;
21) log 2 x + 1 + log 3 x + 9 > 1;
V.8. Gii cc h phng trnh
log 4 x log 2 y = 0
1)
x 2 2 y 2 = 8;
1
2
2
x + y = 2 y + 4
2)
log 3 ( x + 2 y ) + log 1 ( x 2 y ) = 1;

3x 2 y = 972
3)
log 3 ( x y ) = 2;
2 x.8 y = 2 2

4)
1 1 1
log 9 + = log3 (9 y );
x 2 2

log x log 2 y = 0
5) 24
2
x 5 y + 4 = 0;
2log 2 x 3 y = 15
6)
3 y.log 2 x = 2log 2 x + 3 y +1 ;
log 2 x + log 4 x = 2 log 1 4

2
7)
log x + log y = 5;
4
2
60

log x ( xy ) = log y x 2

8)
y 2log y x = 4 y + 3;
1

log 1 ( y x) log 4 = 1
y
9)
4

x 2 + y 2 = 25;

log 2 ( x 2 + y 2 ) = 5
10)
2log 4 x + log 2 y = 4;
log y xy = log x y

11)
2 x + 2 y = 3;
log x ( x 3 + 2 x 2 3x 5 y ) = 3

12)
log y ( y 3 + 2 y 2 3 y 5 x) = 3;
x 4 y + 3 = 0

13)
log 4 x log 2 y = 0;

log 4 x log x y =
14)
6

xy = 16;
log 2 x + 2log 2 y = 3
15)
x 2 + y 4 = 16;

x 1 + 2 y = 1

16)
3log 9 (9 x 2 ) log 3 y 3 = 3;
ln(1 + x) ln(1 + y ) = x y
17)
2
2
x 12 xy + 20 y = 0;
x log8 y + y log8 x = 4
18
log 4 x log 4 y = 1;
log 2 x + 3 = 1 + log 3 y

19)
log 2 y + 3 = 1 + log 3 x;

61

3lg x = 4lg y
20)
(4 x)lg 4 = (3 y )lg3 ;
x + log3 y = 3

21)
2
x
( 2 y y + 12 ) .3 = 81 y;
log x 2 + y 2 ) = 1 + log 2 ( xy )
22) x2 2xy(+ y 2
= 81;
3
x 1 1 y
4 =

2
23)
3log 9 x = y ;

3 4 x
x + 1 1 3y =
24)
x
y + log x = 1;

log 4 ( x 2 + y 2 ) log 4 ( 2 x ) + 1 = log 4 ( x + 3 y )

25)
x
2
log 4 ( xy + 1) log 4 ( 4 y + 2 y 2 x + 4 ) = log 4 1.
y

V.9. Cho phng trnh

log 32 x + log32 x + 1 2m 1 = 0 (1)


1) Gii phng trnh khi m = 2;
2) Tm cc gi tr ca m phng trnh (1) c t nht mt nghim thuc on
[1; 3 ] .
3

V.10. Tm cc gi tr ca m phng trnh: 4(log 2 x ) 2 log 1 x + m = 0


2

c nghim thuc khong (0;1).


V.11. Tm cc gi tr ca a phng trnh: 25

1 t 2

(a + 2)5

1t 2

+ 2a + 1 = 0 c nghim.

V.12. Tm cc gi tr ca a phng trnh


2 log 32 x log 3 x + a = 0 c bn nghim phn bit.

V.13. Chng minh rng vi m i gi tr ca a > 0 h phng trnh sau c mt nghim duy
nht

62

e x e y = ln(1 + x ) ln(1 + y )

y x = a.

V.14. Tm cc gi tr ca m h phng trnh sau c nghim.

22 x + 4 2 y = m

2 x + 4 y + 2 x + 2 y = m
V.15. Cho h phng trnh
2 x +1 = y y + 1 + m + 1

y + 1 = 22 x + 2 2 x +1 + m

1) Gii h phng trnh khi m = 0;


2) Tm cc gi tr ca m h phng trnh c nghim;
3) Tm cc gi tr ca m h phng trnh c mt nghim duy nht.
V.16. Cho h phng trnh

2 x 2 y = y x

2
2
2 x 4mx y = 3m
1) Gii h phng trnh khi m = 1;
2) Tm cc gi tr ca m h phng trnh c ng hai nghim.
V.17. Cho h phng trnh

3x + x = 3m + y

3 y + y = 3m + x
1) Gii h phng trnh khi m = 1;
2) Tm cc gi tr ca m h phng trnh v nghim.
V.18. Tm cc gi tr ca m h phng trnh sau c mt nghim duy nht
x 2 + y 2 = 17

log 2 x + log 2 y = m.

V.19. Tm cc gi tr ca m h phng trnh sau c nghim ( x; y ); x > 1, y < 4.


x2 y 4 = 0

x
log 2 = m log y x.
y

V.20. Tm cc gi tr ca m h phng trnh sau c ng bn nghim

63

4 x.4 y = 8.2 xy

2
2
3 + log 2 x + log 2 y = log 2 ( x + y + m).
V.21. Cho h phng trnh

x + 2 lg y = 3m

2
x 3lg y = 1
1) Gii h phng trnh vi m = 1;
2) Tm cc gi tr ca m h phng trnh c nghim ( x; y ); x 1.
V.22. Tm cc gi tr ca m h phng trnh sau c mt nghim duy nht
lg 2 x + lg 2 y = 1

x
lg = m.
y

V.23. Tm cc gi tr ca m h phng trnh sau c mt nghim duy nht


2x
2y
2 + 3 = 1
x
y
2 + 3 = m.

Tm nghim .
V.24. Tm cc gi tr ca m h phng trnh sau c nghim

22 x + 4 2 y = 2
x
y
x+2 y
= 1 m.
2 + 4 2
V.25. Tm cc gi tr ca m h phng trnh sau c ba nghim phn bit
22 x + 22 y = 16
2x
2y
x
y
x + y =1
= 2m2 .
2 + 2 + m 2 + 2 + 2

V.26. Tm cc gi tr ca m h phng trnh sau c mt nghim duy nht

log 32 x + log 22 y 2 = 4m

2
log 3 x + 2 log 2 y = 10
CHNG VI.
PHNG TRNH LNG GIC
A. TM TT L THUYT
I. CC CNG THC BIN I LNG GIC

Ta quy c cc biu thc trong cc cng thc sau u c ngha.


1. Cng thc cng

1) cos(a + b) = cos a cos b sin a sin b


64

2) cos(a b) = cos a cos b + sin a sin b


3) sin(a + b) = sin a cos b + cos a sin b
4)sin(a b) = sin a cos b cos a sin b
tan a + tan b
1 tan a tan b
tan a tan b
6) tan(a b) =
.
1 + tan a tan b

5) tan(a + b) =

2. Cng thc nhn


2.1. Cng thc nhn i

1) cos 2a = cos 2 a sin 2 a


2)sin 2a = 2sin a cos a
3) tan 2a =

2 tan a
.
1 tan 2 a

2.1.1. Cng thc h bc

1 + cos 2a
2
1 cos 2a
2)sin 2 a =
.
2

1) cos 2 a =

2.1.2. Cng thc tnh theo cos 2a

1) cos 2 a =

1
(1 + cos 2a)
2

2) sin 2 a =

1
(1 cos 2a )
2

3) tan 2 a =

1 cos 2a
.
1 + cos 2a

2.1.3. Cng thc tnh theo tan

1) cos a =

1 t2
1+ t2

2)sin a =

2t
1+ t2

3) tan a =

2t
.
1 t2

a
=t
2

2.2. Cng thc nhn ba


1) cos 3a = 4 cos 3 a 3cos a
2)sin 3a = 3sin a 4 sin 3 a

65

3) tan 3a =

3 tan a tan 3 a
.
1 3 tan 2 a

3. Cng thc bin i tch thnh tng

1
1) cos a cos b = [cos(a + b) + cos(a b)]
2
1
2)sin a sin b = [cos(a + b) cos(a b)]
2
1
3) sin a cos b = [sin(a + b) + sin(a b)].
2
4. Cng thc bin i tng thnh tch
a +b
a b
cos
2
2
a+b
a b
2) cos a cos b = 2 sin
sin
2
2
a +b
a b
3) sin a + sin b = 2 sin
cos
2
2
a+b
ab
4)sin a sin b = 2 cos
sin
.
2
2

1) cos a + cos b = 2 cos

Mt s cng thc quen thuc

1) cos a + sin a = 2 cos(a )


4

2) cos a + sin a = 2 sin(a + )


4

3) cos a sin a = 2 cos(a + )


4

4) cos a sin a = 2 sin(a )


4
5) cos 4 a + sin 4 a = 1 2sin 2 a cos 2 a
6) cos 6 a + sin 6 a = 1 3sin 2 a cos 2 a.
II. PHNG TRNH LNG GIC C BN
1. Phng trnh sin x = a (1)

Nu a > 1 th phng trnh (1) v nghim.


Nu a 1 th phng trnh (1) c nghim.
Gi l s o ca gc sao cho sin = a

66

x = + k 2
Ta c (1) sin x = sin
, ( k ).
x = + k 2
(nu cho bng radian).

x = + k .3600
,(k ).
Hay (1)
x = 1800 + k .3600
(nu cho bng ).
Cc trng hp c bit
sin x = 1 x =

+ k 2, ( k ) .
2

sin x = 1 x =

+ k 2, ( k ) .
2

sin x = 0 x = k , ( k ) .
2. Phng trnh cos x = a (2)

Nu a > 1 th phng trnh (2) v nghim.


Nu a 1 th phng trnh (2) c nghim.
Gi l s o gc sao cho cos = a
Ta c ( 2 ) cos x = cos

x = + k 2

,(k ).
x = + k 2
(nu cho bng radian).

x = + k .3600
Hay ( 2 )
,(k ).
x = + k .3600
(nu cho bng ).
Cc trng hp c bit
cos x = 1 x = k 2, ( k ) .
cos x = 1 x = + k 2, ( k ) .
cos x = 0 x =

+ k , ( k ) .
2

3. Phng trnh tan x = a ( 3)

67

(3) xc nh vi m i x

+ k , ( k ) .
2

Gi l s o gc sao cho tan = a, th

( 3) tan x = tan
x = + k , ( k ) .

(nu cho bng radian).


Hay (3) x = + k .1800 , ( k ) .
(nu cho bng ).
Ch . Nu phng trnh ban u dng tan u = tan v (*)

Th iu kin l u

+ k , v + k , ( k ) .
2
2

Khi (*) u = v + k , ( k ) .
4. Phng trnh cot x = a ( 4 )

(4) xc nh vi m i x k , ( k ) .
Gi l s o gc sao cho cot = a, th

( 4 ) cot x = cot
x = + k , ( k ) .

(nu cho bng radian).


Hay (4) x = + k .1800 , ( k ) .
(nu cho bng ).
Ch . Nu phng trnh ban u dng cot u = cot v ( **)

th iu kin l u k , v k , ( k ) , khi (**) u = v + k , ( k ) .


III. MT S PHNG TRNH LNG GIC THNG GP
1. Phng trnh bc nht, bc hai, bc cao i vi mt hm s lng gic

Cch gii.

+ i vi cc phng trnh bc nht i vi mt hm s lng gic ta bin i ngay v


phng trnh lng gic c bn.
+ i vi cc phng trnh bc hai, bc cao i vi mt hm s lng gic ta t n ph,
sau gii phng trnh theo n ph.
Ch . Nu t t = cos x hay t = sin x th iu kin t 1.
2. Phng trnh bc nht i vi sin x v cos x

68

Phng trnh bc nht i vi sin x v cos x l phng trnh c dng


a sin x + b cos x = c (1), a, b, c
Cch gii.
a 2 + b 2 , ta c

Cch 1. Chia hai v ca (1) cho


a
2

a +b
t cos =

sin x +

a +b
a

a +b

cos x =

c
2

a + b2

,sin =

a + b2

Khi (2) tr thnh cos sin x + sin cos x =


Hay sin ( x + ) =
(3) c nghim

c
2

a + b2

a +b

a + b2

1 a 2 + b2 c2

Cch 2. Chia hai v ca (1) cho a ri t

Ta c sin x + tan cos x =

c
a

sin x cos + sin cos x =

c
cos (*)
a

sin ( x + ) =

c
2

( 3)

c
2

( 2)

b
= tan
a

c
cos
a

y l phng trnh xt trong 1.

Ch rng (*) c nghim khi v ch khi

c
cos x 1.
a

3. Phng trnh thun nht bc hai i vi sin x v cos x


l phng trnh dng
a sin 2 x + b sin x cos x + c cos 2 x = 0 ( 2 ) , a, b, c

Cch gii.

Xt x =

+ k xem c phi l mt nghim ca phng trnh khng.


2

Xt x

+ k , khi cos2 x 0, chia hai v ca phng trnh cho cos2 x 0 ta c


2

a tan 2 x + b tan x + c = 0 .

69

y l phng trnh bc hai i vi tan x ta bit cch gii.

Ch .

Nu phng trnh vi v phi khc 0


a sin 2 x + b sin x cos x + c cos2 x = d

Ta vit phng trnh dng


a sin 2 x + b sin x cos x + c cos2 x = d ( cos 2 x + sin 2 x )

ri chuyn v phi sang v tri.


Cng c th gii phng trnh (2) bng cch bin i v phng trnh bc nht i vi
sin 2x v cos 2 x, nh cc cng thc
cos 2 x =

1 + cos 2 x
;
2

sin 2 x =

1 cos 2 x
;
2

1
sin x cos x = sin 2 x.
2

i vi phng trnh thun nht bc ba i vi sin x v cos x


a cos3 x + b cos 2 x sin x + c sin 2 x cos x + d sin 3 x = 0

Ta cng bin i a v phng trnh bc ba i vi tan x.


4. Phng trnh i xng i vi sin x v cos x

Phng trnh i xng i vi sin x v cos x l phng trnh dng


a ( sin x + cos x ) + b sin x cos x + c = 0 ( 3) , a, b, c

Cch gii. t t = sin x + cos x = 2 sin x + , iu kin: t 2.


4

Khi t 2 = 1 + 2sin x cos x


t 2 1
Suy ra sin x cos x =
. Thay vo phng trnh (3) ta c
2
at +

b ( t 2 1)

+ c = 0 hay bt 2 + 2at + ( 2c b ) = 0. (*)

Gii phng trnh (*) tm t v chn nghim tha t 2.


Ch . Phng php gii trnh by trn cng c th p dng cho phng trnh
a ( sin x cos x ) b sin x cos x + c = 0

bng cch t t = sin x cos x = 2 sin x ; iu kin: t 2.


4

70

Khi sin x cos x =

1 t2
.
2

IV. CC PHNG TRNH LNG GIC KHC

C nhiu phng trnh lng gic m gii chng, ta cn s dng cc php bin i
lng gic a v cc phng trnh xt trn.
1. S dng cng thc h bc, gc nhn i, gc nhn ba
2. Dng phn thc
Ch . Khi gii cc phng trnh c cha n di mu, ta phi t iu kin cho mu khc
khng.

3. Dng cha tan x v cot x


Ch . i vi cc phng trnh cha tan x v cot x, ta phi t iu kin cho tan x v
cot x xc nh.

4. Mt s phng trnh gii bng phng php c bit


Ngoi cc phng php c bn gii phng trnh lng gic nu cc mc trn,
chng ta cn c mt s cch gii c bit, s dng cc kt qu sau:
A m

A = m
B m
B = m

A = B

A = 0
A2 + B 2 = 0
B = 0

A A1

A = A1
B B1

B = B1
A + B = A1 + B1

B. BI TP
VI.1. Gii cc phng trnh

1)

3 sin x cos x =

2;

2) cos x + 2cos2 x = 1;
3) cos4 x + 2cos2 x = 0;
4) 2cos 2 x + 4cos x = 3sin2 x;
5) cos x sin x + 3sin2 x 1 = 0;
6) 2sin2 x 3 3 (sin x + cos x ) + 3 3 = 0;
7) sin2 x +

2 sin( x ) = 1;
4

8) sin2 x + 2sin x coss x 2cos2 x =


9) cos x + sin x =

1
;
2

cos2x
;
1 sin 2 x

10) sin3 x cos3 x = 1 + sin x cos x.


71

VI.2. Gii cc phng trnh

1) 2cos2 x 1 = sin3 x ;
2)

1 + tan x
= (sin x + cos x )2;
1 tan x
1 sin 2 x
;
cos 2 2 x

3) 1 + tan2 x =

4) tan3 x tan x = sin2 x ;


5) (sin x sin2 x )(sin x + sin2 x ) = sin23 x ;
6) sin x + sin3 x + 4cos3 x = 0;
7) sin2 x = 1 +
6

2 cos x + cos2 x ;
4

8) 2cos x + sin x + cos2 x = 0;


9) 2cos2 3 x cos2 x cos 2 3 x + sin 2 x 1 = 0 .
VI.3. Gii cc phng trnh

1) sin x + cot

x
= 2;
2

2) sin2 x + cos2 x + tan x = 2;


3) cos 4 x
4)

3 (1 tan 2 x )
1+ tan 2 x

+ 2 = 0;

tan x 1
+ cot 2 x = 0, (0 < x < );
tan x + 1

5) tan 3 x 1 +

3
3cot( x ) = 3, ( < x < );
2
cos x
2
2

6) cos3 x sin x sin3 x cos x =

2
;
8

7) sin23 x cos24 x = sin25 x cos26 x ;


8) cos3 x 4cos2 x + 3cos x 4 = 0, x [0,14];
x
1
9) sin4 x + sin4( + ) + cos4 x = sin22 x ;
2 8
2

10) 2 cos 2 x 8cos x + 7 =

1
.
cos x

VI.4. Gii cc phng trnh

cos 3 x + sin 3x

1) 5 sin x +
= cos 2 x + 3, x (0; 2);
1 + 2sin 2 x

72


2) sin 2 x.cos x = tan 3 x.sin( x + ) cos 2 x.sin x ;
6

3) cot x 1 =

cos 2 x
1
+ sin 2 x sin 2 x ;
1 + tan x
2

4) sin 4 x sin 2 x + sin 9 x sin 3 x = cos 2 x ;


5) cos2 x sin 4 x + cos 2 x = 2cos x(sin x + cos x ) 1 ;
6)

3 cos 4 x + sin 4 x 2cos 3 x = 0 ;

7) 4cos2 x 2 cos2 2 x = 1 + cos 4 x ;

8) 2sin 2 ( x ) = 2sin 2 x tan x ;


4

9) cos 3x + 2 cos 2 x = 1 2 sin x sin 2 x ;


10) (2sin x 1)(2 cos x + sin x ) = sin 2 x cos x;
11)

3 cos 5 x 2 sin 3x cos 2 x sin x = 0;

12) sin x + cos x sin 2 x + 3 cos 3 x = 2 ( cos 4 x + sin 3 x ) .


VI.5. Gii cc phng trnh

1) tan 4 x + 1 =
2)

(2 sin 2 2 x ) sin 3x
;
cos4 x

sin 4 x + cos4 x 1
1
= cot 2 x
;
5sin 2 x
2
8sin 2 x

3) 1 + sin x + cos x + sin 2 x + cos 2 x = 0 ;

1
4) 2 cos 2 x sin 2 x + cos( x ) sin(3 x ) = 0 ;
4
4 2
x
5) cot x + sin x(1 + tan x tan ) = 4 ;
2

6)

2(cos6 x + sin 6 x) sin x cos x


=0;
2 2sin x

7) cos 3x + cos 2 x cos x 1 = 0 ;


8) 13 18 tan x = 6 tan x 3;
9) cos 4 x sin 4 x = cos x + sin x ;
10) cos13 x + sin14 x = 1.
VI.6. Gii cc phng trnh

1) tan x = cot x + 4 cos 2 2 x;


73

2) sin 2 x = sin x +
;
4
4 2

3)

3 ( 2cos2 x + cos x 2 ) + ( 3 2cos x ) sin x = 0;

4) (1 + 2cos 3 x ) sin x + sin 2 x = 2sin 2 2 x + ;


4

x
x
x
5) 1 + sin sin x cos sin 2 x = 2 cos2 ;
2
2
4 2

1
6) cos 2 ( x + ) + sin 2 ( x + ) = 2sin x ;
3
6
4

sin 3x 4 cos x 3
6

7)
= 0;
sin 3x 1
8) cos 3x cos3 x sin 3x sin 3 x =

2+3 2
;
8

9) 4sin 3x sin x + 4 cos 3 x cos x + cos2 2 x + + 1 = 0;


4
4
4

10) sin 3x + 3 cos 3 x + cos 2 x 3 sin 2 x = sin x + 3 cos x;


11) sin 2 x (1 + tan x ) = 3sin x ( cos x sin x ) + 3;
12)

sin 2 x cos 2 x
+
= tan x cot x;
cos x
sin x

13)

1
+
sin x

= 4 sin
x ;
3

sin x
2

14) sin 3 x 3 cos 3 x = sin x cos 2 x 3 sin 2 x cos x;


15) 2sin x (1 + cos 2 x ) + sin 2 x = 1 + 2 cos x;
16)

(1 2sin x ) cos x =
(1 + 2sin x )(1 sin x )

3.

VI.7. Gii cc phng trnh

11 5 x
7 x
3x
1) cos
+ sin
= 2 sin + ;
2
4
4 2
2 2

2)
74

2 sin x
4
1 + sin 2 x = 1 + tan x;
(
)
cos x

3) sin 3 x + cos 3 x = cos 2 x ( 2 cos x sin x ) ;

4) 2sin 2 x = 2sin 2 x tan x ;


4

5)

1
x 1
x
+ cos 2 = sin 2 ;
4
3 2
2

6)

3 sin + cos x =

1
;
cos x

7) 3 tan 2 x + 4 tan x + 4cot x + 3cot 2 x + 2 = 0;


8) sin 2 x tan x + cos 2 x cot x sin 2 x = 1 + tan x + cot x;
9) sin 4 2 x + cos4 2 x = (1 + cos2 4 x ) sin 6 x ;
10) 3 ( 2cos2 x + cos x 2 ) + sin x ( 3 2 cos x ) = 0;

11) tan x tan x + sin 3 x = sin x + sin 2 x;


6
6

12) (1 tan x )(1 + sin 2 x ) = 1 + tan x;


13)

sin 2 x cos 2 x

= tan x cot x;
cos x
sin x

3x
5x
x
14) sin cos = 2 cos ;
2
2 4
2 4

15) 2cos2 x + 2 3 sin x cos x + 1 = 3 sin x + 3 cos x .


VI.8. Tm cc gi tr ca tham s m cc phng trnh cho sau y c nghim

1) tan 2 x + cot 2 x + m(tan x + cot x) + 2m = 0;


2) m(sin x + cos x ) + sin 2 x + m 1 = 0;
3) 4(cos x sin x ) + sin 2 x = m.
VI.9. Tm cc gi tr ca tham s m phng trnh sau c nghim tha

< x<
2
2

cos 2 x 2m cos x + 4 ( m 1) = 0.

VI.10. Gii v bin lun phng trnh sau theo tham s m

sin 2 x + 2sin x cos x 2 cos2 x = m.


VI.11. Tm cc gi tr ca tham s m phng trnh: m cos2 2 x 2sin 2 x + m 2 = 0 c

nghim trong khong 0; .
4

75

PHN II: LI GII V HNG DN


CHNG I.

HM S

I.1. Gi s y0 T f , khi y0 =

2 x 1
(1) c nghim i vi x.
x + x+4
2

(1) y0 ( x 2 + x + 4 ) = 2 x 1
y0 x 2 + ( y0 2 ) x + 4 y0 + 1 = 0 (2)

Xt cc trng hp sau:
+ Xt y0 = 0.
Khi , ( 2 ) 2 x + 1 = 0 x =

1
. Vy, y0 = 0 T f .
2

+ Xt y0 0. Khi , (2) c nghim khi v ch khi


2

= ( y0 2 ) 4 y0 ( 4 y0 + 1) 0
15 y02 8 y0 + 4 0

4 2 19
4 + 2 19
y0
15
15

4 2 19 4 + 2 19
;
Vy, tp gi tr ca hm s l T f =
.
15
15

I.2. Hm s cho c tp xc nh D = .

Gi s y0 T f , khi y0 =

(1) y0 ( x2 + a ) = x + 1

x +1
(1) c nghim i vi x.
x2 + a

y0 x 2 x + ay0 1 = 0 (2)

Xt cc trng hp sau
+ y0 = 0. (2) x = 1 y0 = 0 T f .
+ y0 0. khi , (2) c nghim x khi v ch khi = 1 4 ( ay0 1) y0 0
4ay02 + 4 y0 + 1 0

1 a +1
1+ a +1
y0
.
2a
2a

Nh vy tp gi tr ca hm s cha on [ 0;1] khi v ch khi a > 0 v h iu kin sau


1

1 +

76

1+ a
0
5
2a
0<a .
4
1+ a
1
2a

I.3. Hm s cho c tp xc nh l D = \ {m;1}. Tp xc nh D l tp i xng khi

v ch khi m = 1. Vi m = 1 th hm s tr thnh y =

1
. Hm s ny l mt hm s
x 1
2

chn. Vy, khi m = 1 th hm s cho l hm s chn.


I.4. Hm s cho c tp xc nh l: D = .

1) a D, ta c f (a) = f (0 + a) = f (0) + f (a) f (0) = f (a ) f (a) f (0) = 0.


Vy, f (0) = 0 (pcm).
2) Theo gi thit hm s y = f ( x ) xc nh trn nn tp xc nh ca hm s cho l
tp i xng. Mt khc ta li c: 0 = f (0) = f ( a + a ) = f ( a ) + f (a) f ( a ) = f (a ).
Vy, f l hm s l.
I.5. Trng hp phng trnh f ( x ) = 0 v nghim th s nghim ca phng trnh bng 0.

Gi s phng trnh f ( x ) = 0 c nghim. G i x0 l mt nghim ca phng trnh


f ( x ) = 0, ta c f ( x0 ) = 0 v x0 0. V y = f ( x ) l hm s l nn f ( x0 ) = f ( x0 ) .

Suy ra f ( x0 ) = 0 v do x0 cng l mt nghim ca phng trnh f ( x ) = 0. T y


ta c nu x0 l mt nghim ca phng trnh f ( x ) = 0 th x0 cng l mt nghim ca
phng trnh f ( x ) = 0. Nh vy, s nghim ca phng trnh f ( x ) = 0 l mt s chn.
I.6. a) Ta c: f ( x1 + x2 ) + f ( x1 x2 ) = 2 f ( x1 ) f ( x2 ) (1), x1 , x2 .

Thay x1 = x2 = 0 vo 2 v ca (1) ta c
2

2 f ( 0 ) = 2 f ( 0 ) f ( 0 ) = 0 f ( 0 ) = 1 . Nhng theo bi ra ta c f ( x ) 0 x , do
f ( 0 ) = 1 .
b) Hm s y = f ( x ) xc nh trn nn tp xc nh ca hm s cho l tp i xng.
Thay x1 = 0; x2 = x vo (1) ta c:
f ( x) + f (x) = 2 f ( x)
f ( x ) = f ( x ) . x .

Vy, f ( x ) l mt hm s chn.
I.7. 1) y = cos ( 2 x + 3) . Tp xc nh ca hm s cho l D = .

Hm s y = f ( x) = cos ( 2 x + 3) l hm s tun hon v c T = 2 sao cho

77

ix x 2 .
ix f ( x 2 ) = cos 2 ( x 2 ) + 3 = cos ( 2 x + 3) 4 = cos ( 2 x + 3) = f ( x )

Chu k ca hm s l T0 = . Gi s cn c 0 < l < sao cho


f ( x + l ) = f ( x ) , x
cos 2 ( x + l ) + 3 = cos ( 2 x + 3) (1), x

Chn x =

3
ta c (1) cos 2l = 1 , vi 0 < l < , (V l).
2

Vy, chu k ca hm s l T0 = .
2) y = sin 2 x. Tp xc nh ca hm s cho l D = .
Hm s y = f ( x ) = sin 2 x =

1 cos 2 x
l hm tun hon v c T = 2 sao cho
2

ix x 2

ix f ( x 2 ) =

1 cos 2 ( x 2 )
2

1 cos ( 2 x 4 )
2

1 cos 2 x
= f ( x)
2

Chu k ca hm s l T0 = .
Gi s cn c 0 < l < sao cho
f ( x + l ) = f ( x ) , x

1 cos 2 ( x + l )

1 cos 2 x
, x
2
2
cos 2 ( x + l ) = cos 2 x (1), x

Chn x = ta c (1) cos 2l = 1, vi 0 < l < (V l)


Vy, chu k ca hm s l T0 = .
I.8. 1) y = f ( x ) = x 3 + 2 x 2 . Tp xc nh ca hm s cho l D = . ta c

x = 0
f ( x) = x 3 + 2 x 2 = 0
x = 2.

Gi s f ( x ) l hm s tun hon, khi tn ti s dng T sao cho


f ( x + T ) = f ( x), x D. Chn x = 0, ta c f (0 + T ) = f (0) = 0 T > 0 l nghim ca
phng trnh f ( x ) = 0 (v l). Vy, hm s cho khng phi l hm s tun hon.
2) y = f ( x) = x 1 . Tp xc nh ca hm s cho l D = [1; +).

f ( x ) = 0 x = 1. Gi s f ( x ) l hm s tun hon, khi tn ti s dng T sao cho


f ( x + T ) = f ( x), x D. Chn x = 1, ta c f (1 + T ) = f (1) = 0 T + 1 > 1 l nghim ca
phng trnh f ( x ) = 0 (v l). Vy, hm s cho khng phi l hm s tun hon.

78

3) y = f ( x) =

x
. Tp xc nh ca hm s cho l D = \ {1;1}.
x 1
2

Gi s f ( x ) l hm s tun hon, khi tn ti s dng T sao cho


x D x T D. Do D = \ {1;1} nn1 + T D (1 + T ) T D 1 D (V l).
Vy, hm s cho khng phi l hm s tun hon.

I.9. Tp xc nh ca hm s irichl l D = . Vi T > 0, T ta u c

x T nu x , x T \ nu x \ .

1 , x .
Suy ra f ( x T ) =
0 , x \ .

Nh vy, f ( x T ) = f ( x), x . Suy ra hm s f ( x ) l hm s tun hon. Tuy nhin


trong tp cc s hu t dng khng c s dng b nht, v vy hm s irichl l hm s
tun hon nhng khng c chu k.
x +1
f ( x ) + 1 x 1 +1 2x
I.10. 1) Ta c: y = f ( f ( x ) ) =
=
=
=x
f ( x ) 1 x + 1 1 2
x 1

Vy, y = f ( f ( x ) ) = x.
2) Ta c: y = f ( g ( x ) ) =
Vy, y = f ( g ( x ) ) =

g ( x) +1
g ( x ) 1

2x 1 + 1
2x
x
=
=
2x 1 1 2x 2 x 1

x
.
x 1

I.11. Ta c

f 2 ( x ) = f ( f1 ( x ) ) =

1 x x 1
1
1
, f3 ( x ) = f ( f 2 ( x ) ) =
=
= =x
x 1 1
x
x
1
x
x

1
1 x
1
f4 ( x ) = f ( f3 ( x )) =
= f1 ( x ) .
1 x
1

Nh vy, f1 ( x ) = f 4 ( x ) = f 7 ( x ) = ... = f1+3k ( x ) , k .


Do f100 ( x ) = f1 ( x ) =

1
.
1 x

I.12. Ta c

1 2 x, x < 2
y = f ( x) =
, f ( x ) = 2x 1 .
2 x 1, x 1

2
79

x 1, x 1
y = g (x) =
1 x, x < 1
= x 1

Vy, f [ g ( x )] = 2 x 1 1 , g [ f ( x)] = 2 x 1 1 .
I.13. Hm s y = f ( x ) = 2 1 x xc nh trn na khong ( ;1] .

Trn tp xc nh ( ;1] phng trnh y = 2 1 x c nghim duy nht i vi n x l


2

x = 1 ( 2 y ) = y 2 + 4 y 3.

Vy, hm s ngc cn tm l y = f 1 ( x) = x 2 + 4 x 3, x (; 2].


I.14. 1) a) Ta c y =

x2 x 7 x2 + x 3
=
2 = f ( x) 2
x+2
x+2

x2 + x 3
x2 x 7
T th hm s y = f ( x) =
suy ra th y =
bng php t nh tin
x+2
x+2

theo vc t v = (0; 2).

b) Ta c y =

x 2 + 7 x + 9 ( x + 3)2 + ( x + 3) 3
=
= f ( x + 3)
( x + 3) + 2
x+5

T th hm s y = f ( x) =

theo v = (3; 0).

x2 + x 3
x2 + 7 x + 9
suy ra th y =
bng php t nh tin
x+2
x+5

c) T th hm s y = f ( x) =

x2 + x 3
tnh tin theo vect v = (a; b) c th
x+2

x2 + 2x 4
y=
khi v ch khi
x+3
x 2 + 2 x 4 ( x a) 2 + ( x a ) 3
=
+ b, x 3
x+3
( x a) + 2
( x 2 + 2 x 4) [ ( x a) + 2] = ( x a)2 + ( x a)(1 + b) 3 + 2b ( x + 3) , x 3
x3 + (4 a ) x 2 2ax + 4a 8 = x3 + (4 + b 2a ) x 2 (a 2 7a ab + 5b) x +
+3(a 2 a ab + 2b 3), x 3

4 a = 4 + b 2a
a = 1

2a = (a 2 7 a ab + 5b)

b = 1
4a 8 = 3(a 2 a ab + 2b 3)

Vy, tnh tin th hm s y =


80

x2 + x 3
theo vect v = (1; 1) c th hm s
x+2

y=

x2 + 2x 4
.
x+3

2) a) Ta c y =

x2 + x 3
x2 x + 3
=
= f ( x)
x+2
x+2

x2 + x 3
x2 x + 3
suy ra th hm s y =
bng php i
T th hm s y = f ( x) =
x+2
x+2
xng qua trc honh.

b) Ta c y =

x2 + x 3
x2 + 5
=
+ 1 = f ( x) + 1
x+2
x+2

Do c th hm s y =

x2 + 5
ta thc hin hai bc
x+2

+ Bc 1: i xng th hm s y = f ( x ) qua trc honh ta c th (C1 ) ca hm s


y = f ( x).

x2 + 5
+ Bc 2: Tnh tin (C1 ) theo vect v = (0;1) ta c th hm s y =
.
x+2

3x 7
1
= 3
. Do ta thc hin lin tip cc bc bin i sau:
x2
x2

1
+ Tnh tin th ca hm s y = theo vc t v = (2; 0) th c th hm s
x
1
y=
.
x2

I.15. Ta c y =

1
1
qua trc honh ta c th hm s y =
.
x2
x2

1
+ Tnh tin th ca hm s y =
theo vc t u = (0;3) ta c th hm s
x2

+ i xng th hm s y =

y=

3x 7
.
x2

I.16. 1) y =

x 2 3x + 1
.
x3

Kho st v v th hm s
y=

x 2 3x + 1
x3

dnh cho bn c. th (C) ca hm s y =

x 2 3x + 1
nh sau
x3

81

1
O

x 2 3x + 1
; f ( x) 0
x 2 3 x + 1 x 3
= 2
2) a) y = f ( x ) =
x 3
x 3x + 1 ; f ( x ) < 0
x3

Do th hm s y =

x 2 3x + 1
gm hai phn:
x 3

+ Phn t trc honh tr ln ca th hm s y = f ( x ) ;


+ i xng ca phn th hm s y = f ( x ) pha di trc honh qua trc honh.
th ca hm s y = f ( x) nh sau
y

1
O

b) y =

x 2 3x + 1
;
x3

x 2 3x + 1
; x>3

x 2 3x + 1
x 3
Ta c: y =
2
x3
x 3 x + 1 ; x < 3

x3

82

Do th hm s y =

x 2 3x + 1
gm hai phn:
x3

+ Phn th y = f ( x ) trn min x > 3;


+ i xng ca phn th y = f ( x ) trn min x < 3 qua trc honh.
th hm s y =

x 2 3x + 1
nh sau
x3
y

2
x

O
-1

c) y =

x2 3 x + 1
x 3

; Do hm s y = f ( x ) l hm s chn nn th c trc i xng l trc

Oy. Vi x 0 th y = f ( x ) = f ( x ) .
th hm s y =

x2 3 x + 1
x 3

gm hai phn:

+ Phn bn phi trc tung ca th hm s y = f ( x ) ;


+ i xng ca phn th hm s y = f ( x ) pha bn phi trc tung qua trc tung.
th hm s y = f ( x ) nh sau
y

1
-4

-2

83

x2 3 x + 1 x 2 3 x + 1
0
;

x 3
x2 3 x + 1 x 3
d) Ta c y =
= 2
2
x 3
x 3 x + 1 ; x 3 x + 1 < 0

x 3
x 3

Do th hm s y =

x 3 x +1
x 3

gm hai phn:

+ Phn t trc honh tr ln ca th hm s y = f ( x ) ;


+ i xng ca phn th hm s y = f ( x ) pha di trc honh qua trc honh.
2

th hm s y =

x 3 x +1
x 3

nh sau
y

1
-2

-4

x = X + 2
I.17. t
y = Y

Hm s cho tr thnh
Y=

( X + 2)

4 ( X + 2) + 3

5
5
. Hm s Y = 2
l hm s chn.
X 1
X 1
2

Vy, ng thng x = 2 l trc i xng ca th hm s cho.


I.18. ng thng x = x0 l trc i xng ca th hm s cho khi v ch khi
f ( 2 x0 x ) = f ( x ) , x
4

( 2 x0 x ) + 4 ( 2 x0 x ) + 3 ( 2 x0 x ) 2 ( 2 x0 x ) = x 4 + 4 x 3 + 3x 2 2 x, x

x 4 4 ( 2 x0 + 1) x3 + 3 8 x02 + 8 x0 + 1 x 2 2 16 x03 + 24 x02 + 6 x0 1 x +


4
0

3
0

2
0

+16 x + 32 x + 12 x 4 x0 = x + 4 x + 3x + 3 x 2 x, x

84

x0 = 1
( 2 x0 + 1) = 1

2
x0 = 1 x0 = 0
8 x0 + 8 x0 + 1 = 1
3

( x + 1) 16 x 2 + 8 x 2 = 0 x0 = 1
2
0
0
16
x
+
24
x
+
6
x

1
=
1
0
0
0
0
16 x 4 + 32 x 3 + 12 x 2 4 x = 0
x x + 1 4x2 + 4x 1 = 0
0
0
0
0
0
0( 0 ) 0

Vy, th hm s y = x 4 + 4 x 3 + 3 x 2 2 x c duy nht mt trc i xng cng phng vi


trc tung l x = 1.
I.19. Ta c y =

x2 + 4x 2
4x 3
= 1+ 2
(1)
2
x +1
x +1

Gi s I ( x0 , y0 ) l tm i xng ca th hm s cho.
Di h trc to Oxy v h trc to mi IXY vi I ( x0 , y0 ) bi php t
x = X + x0

y = Y + y0

Thay vo (1) ta c Y = g ( X ) = 1 +

4 ( X + xo ) 3

( X + xo )

+1

yo

Nu th ( C ) nhn I ( xo ; yo ) lm tm i xng th hm s Y = g ( X ) phi l hm s l


tc l

4 ( X + xo ) 3

( X + xo )

+1

+ 1 yo =

4 ( X + xo ) + 3

( X + x0 )

+1

1 + yo ( ) ng vi m i X .

Cho X th ( ) tng ng vi: 2 yo = 2 yo = 1 . Vi yo = 1 ( ) tr thnh


4 ( X + xo ) 3

( X + xo )
3 =

+1

4 ( X + xo ) + 3

( X + xo )

+1

; X . ( ) t X = xo th ( ) tr thnh

8 xo + 3
12 xo2 8 xo + 6 = 0. Phng trnh ny v nghim.
4 xo2 + 1

Vy, I ( xo ; yo ) khng phi l tm i xng ca th hm s cho.


I.20. Gi s x = m l trc i xng ca th hm s cho.

x = X + m
t
y = Y

Khi ta c hm s
Y = f ( X ) = ( X + m)4 + 4a( X + m)3 2( X + m)2 12( X + m)a
= X 4 + 4 X 3m + 6 X 2 m 2 + 4 Xm3 + m 4 + 4a ( X 3 + 3 X 2 m + 3 Xm 2 + m3 )
2( X 2 + m 2 + 2mX ) 12a( X + m)

85

= X 4 + (4m + 4a) X 3 + (6m 2 + 12am 2) X 2 + (4m3 + 12am 2 4m 12a ) X


+ m 4 + 4am3 2m2 12am (2)

ng thng x = m l trc i xng ca th hm s cho khi v ch khi hm s


Y = f ( X ) phi l hm s chn, iu ny tng ng vi
4m + 4 a = 0
3
2
4m + 12am 4m 12a = 0

m + a = 0
3
2
m + 3am m 3a = 0
m = a

3
2
( a) + 3a ( a) ( a) 3a = 0
m = a
m = a

m = a
m = a

a = 0
3
3
a = 0
3
a + 3a + a 3a = 0
2a 2a = 0
a 2 = 1
a = 1

a = 1
a = 0
Vy, vi a = 1 th th hm s c trc i xng cng phng Oy.
a = 1

I.21. Gi s trn th ( Cm ) c hai im M ( x1 ; y1 ) , N ( x2 ; y2 ) i xng nhau qua gc ta


x = x1
, khi ta c 2
y2 = y1

Nh vy ta c
x12 2m 2 x1 + m 2
x 2 + 2m 2 x1 + m 2
x 2 2m 2 x1 + m 2 x12 + 2m 2 x1 + m 2
= 1
1
=
x1 + 1
x1 + 1
x1 1
x1 + 1

2m 2 1 x12 = m 2 (1)

2
x1 1(*)

Trn th ( Cm ) c hai im M ( x1 ; y1 ) , N ( x2 ; y2 ) i xng nhau qua gc ta khi v


ch khi phng trnh (1) c hai nghim phn bit tha iu kin (*), iu ny tng ng
vi
m2

2 1
>0
2
2
m >
m >
2m 1

2
2
2
m

1
m 1
m 1.
2m 2 1

I.22. 1) y = f ( x) = 2.33 x 4.32 x + 2.3x trn on [ 1;1] .

86

t t = 3x > 0. Khi hm s cho tr thnh y = f (t ) = 2t 3 4t 2 + 2t.


1
Vi x [1;1] th t [ ;3].
3

Ly o hm theo bin t ta c
y = 6t 2 8t + 2

t = 1
y = 0 1
t =
3
1 8
Ta c f (1) = 0; f = ; f ( 3) = 24.
3 27

Vy, Maxf ( x ) = 24 v Min f ( x ) = 0.


x[ 1;1]

x[ 1;1]

3
2) y = f ( x ) = cos3 x 15cos x + 8 trn on ; .
3 2
y = f ( x ) = cos3 x 15cos x + 8 = 4 cos 3 x 3cos x 15cos x + 8 = 4 cos 3 x 18 cos x + 8

t t = cos x.

Khi hm s y = f ( x ) = 4 cos3 x 18 cos x + 8 tr thnh


1
3
y = f ( t ) = 4t 3 18t + 8. Vi x ; th t [1; ].
2
3 2

Ly o hm theo bin t ta c:
f ( t ) = 12t 2 18

3
1
t =
. C hai gi tr ny khng thuc [1; ].
2
2
f (t ) = 0

3
t =
2

1
1
f = ; f ( 1) = 22.
2
2
1
Vy, Maxf ( x ) = 22 v Min f ( x ) = .
2
3
x ;

3 2

3
x ;
2 2

3) y = f ( x ) = x 3 3 x 2 + 5 trn on [ 0;3] . Xt hm s y = x3 3x 2 + 5. Ta c
y = 3x2 6 x

87

x = 0 y = 5
y = 0 3x ( x 2) = 0
x = 2 y = 1
Ti x = 3 y = 5. V trn on [ 0;3] hm s y = x 3 3 x 2 + 5 ly gi tr dng, do
Maxf ( x ) = 5 v Minf ( x ) = 1.
x[ 0;3]

x[0;3]

x2
x2
1
3
I.23. 1) y = f ( x ) =
. Tp xc nh ca hm s y = f ( x) =
l D = \ .
2x 1
2x 1
2
2 ( x 1)
Ta c y ' =
, y ' = 0 x = 1.
3 ( 2 x 1) 3 x ( 2 x 1)
3

3
im ti hn ca hm s thuc on ; 2 l x = 1 . Ta c
4
3
3
36
36
3 3 9
f (1) = 1 ; f =
; f ( 2) =
. Vy, Max f ( x ) =
; Min f ( x ) = 1.
3
3
3
3
4 2
x ;2
x ;2
4

2) y = f ( x ) = ( cos x + 1) sin x , x [ 0; 2 ] .
Tp xc nh ca hm s l D = [ 0; 2 ]
1
Ta c y = ( cos x + 1) sin x = sin 2 x + sin x
2

y = cos 2 x + cos x = 2cos 2 x 1 + cos x, y ' = 0 2 cos2 x + cos x 1 = 0


cos x = 1

cos x = 1

. Vi cos x = 1 x = [0; 2 ]
.

x = [ 0; 2 ]

1
3
Vi cos x =
5
2
x=
[ 0; 2]

3 3
5 3 3
f (0) = 0 ; f =
; f ( ) = 0; f ( 2 ) = 0; f =
.
4
4
3
3

Vy, Max f ( x ) =
x[0;2 ]

3 3
3 3
; Minf ( x ) =
.
4
4
x[0;2 ]

I.24. Gi s ( x, y ) l mt nghim ca h phng trnh

88

x + y = 2 a
(1)
2
2
x + y + xy = 3

x + y = 2 a
x + y = 2 a
x + y = 2 a

Ta c (1)
(I )
2
2
2
( x + y ) xy = 3 xy = ( x + y ) 3 xy = a 4a + 1

Khi x; y tha phng trnh X 2 ( 2 a ) X + a 2 4a + 1 = 0 ( )


Phng trnh ( ) c nghim khi v ch khi
2

0 ( 2 a ) 4 a 2 4a + 1 0 3a 2 + 12a 0 0 a 4.
2

Xt M = x 2 + y 2 xy = ( x + y ) 3xy ( 2 )
2

Th ( I ) vo ( 2 ) ta c M = ( 2 a ) 3 a 2 4a + 1 = 2a 2 + 8a + 1.
Khi vic tm gi tr ln nht, gi tr nh nht ca biu thc M = x 2 + y 2 xy tr thnh
vic tm gi tr ln nht, gi tr nh nht ca biu thc N = 2a 2 + 8a + 1 trn on [ 0; 4] .
Xt f ( a ) = 2a 2 + 8a + 1 f ' ( a ) = 4a + 8, f ' ( a ) = 0 4a + 8 = 0 a = 2.
Ta c f ( 2 ) = 9; f ( 0 ) = 1; f ( 4 ) = 1.
a = 0
Vy, khi
th M t gi tr nh nht v khi a = 2 th M t gi tr ln nht.
a = 4
I.25. Tp xc nh ca hm s l D = .

y = ( x + 1)( x + 2 )( x + 3)( x + 4 ) = ( x + 1)( x + 4 ) ( x + 2 )( x + 3)

)(

) (

)(

= x 2 + 5x + 4 x2 + 5x + 6 = x2 + 5 x + 4 x 2 + 5 x + 4 + 2

= ( x 2 + 5x + 4 ) + 2 ( x2 + 5x + 4 ) + 1 1
2

= ( x 2 + 5 x + 5) 1 1, x .
Du " = " xy ra khi v ch khi x 2 + 5 x + 5 = 0 x =

5 5
5 5
x=
.
2
2

Vy, Miny = 1.
I.26. Theo bt ng thc Bunhiascopki ta c
2

2
2
1
1
5 1
=
1
+
+ y.


= x .
x
4 y
4 4
1 2 1 2
2
2

= ( x + y ) . 1 + 1

x + y .
+

x 4 y
x 16 y

( ) ( )

89

25 5 1 4 1
5
. +
(Do x + y = )
4
16 4 4 x 4 y

4 1
+
A 5.
x 4y

Du = xy ra khi v ch khi

1
x=
x

x = 1

y=
1
4 y

y = 4

5
x + y =

4
Vy, gi tr nh nht ca biu thc cho l MinA = 5.

I.27. Xt ba vect u = ( x + 1; 0), v = ( x 2; 0), w = (5 2 x; 0)

Khi u + v + w = (4; 0). Ta c bt ng thc

u + v + w u + v + w x + 1 + x 2 + 5 2 x 42 + 0 = 4.

5
Du ng thc xy ra khi ba vect u , v, w cng hng, khi v ch khi 2 x .
2

Vy, gi tr nh nht ca hm s cho l Miny = 4.


I.28. A =

1 y y x+ y
3
1
2 1
1
x + + y + 2 = x + + 2 2 + + +
.
4
x
y
4
x
8 8
2
y

p dng bt ng thc Csi cho hai s dng

1
1
x v
ta c
4
x

1
1
1 1
x+ 2
x. = 1(1) .
4
x
4 x
Tng t, ta cng p dng bt ng thc Csi cho ba s dng
1 y y
1 y y 3
+ + 3 3 2 . . = ( 2) .
2
y
8 8
y 8 8 4

Ta li c x + y 4

x+ y
2 ( 3) .
2

3
9
T (1), (2), (3), ta suy ra A 1 + 2. + 2 = .
4
2

90

1 y y
; ; ta c
y2 8 8

x 1
4 = x

x = 2
y
1

Du bng xy ra khi v ch khi 2 =


8
y = 2
y
x + y = 4

9
Vy, gi tr nh nht ca biu thc cho l MinA = .
2

I.29. iu kin x 3, y 4, z 5.

x3
+
x

Biu thc c vit li T =

y4
z 5
+
y
z

p dng bt ng thc Csi i vi hai s khng m ( x 3);3 ta c


x 3. 3 = ( x 3).3

x 3+ 3 x
=
2
2

1
x3

.
x
2 3

Lp lun tng t nh trn, ta cng c


y4
1
1

=
y
2 4 4
z 5
1

z
2 5

Nh vy, ta c T

1
2 3

1
1
+
.
4 2 5

ng thc xy ra khi v ch khi

x 3 = 3
x = 6
y 4 = 4

y = 8

z 5 = 5
z = 10.
x 3, y 4, z 5
Vy, MaxT =

1
2 3

1
1
+
.
4 2 5

x2 x2 y 2 y 2 z 2 z 2
I.30. Ta c P = + +
+
+ + (*). Ta nhn thy x 2 + y 2 xy xy , x, y .
y
z
z
x
x
y
Do x 3 + y 3 xy ( x + y ) , x, y > 0 hay

x2 y 2
+
x + y , x, y > 0.
y
x

Tng t ta cng c
91

y2 z2
+ y + z , y , z > 0
z
y
z 2 x2
+
z + x, z , x > 0.
x
z
Cng theo tng v ba bt ng thc va nhn c trn v kt hp vi (*) ta c
1
P 2( x + y + z ) = 2, x, y , z > 0 v x + y + z = 1. Hn na ta c P = 2 khi x = y = z = .
3

Vy, MinP = 2.
I.31. A =

a3
b3
c3
+
+
(1 + b )(1 + c ) (1 + c )(1 + a ) (1 + a )(1 + b )

p dng bt ng thc Csi ta c


a3
1+ b 1+ c
a3
1 + b 1 + c 3a
+
+
33
.
.
= (1)
8
4
(1 + b )(1 + c ) 8
(1 + b )(1 + c ) 8 8
Tng t ta cng c
b3
1 + c 1 + a 3b
+
+
(2)
8
4
(1 + c )(1 + a ) 8
c3

(1 + a )(1 + b )

1 + a 1 + b 3c
+
(3)
8
8
4

Cng cc bt ng thc (1), (2), (3) theo v ta c


A+

3 a + b + c 3 3 abc 3
3

= A .
4
2
2
2
4

3
ng thc xy ra khi a = b = c = 1. Vy, MinA = .
4

I.32. A =

a
b
c
+
+
.
b
c
a

Ta c A2 =

a 2 b 2 c 2 2a b 2 c 2c a
+ + +
+
+
.
b
c a
c
a
b

p dng bt ng thc Csi ta c


a2 a b a b
a2 a b a b
+
+
+ c 44 .
.
.c = 4a(1)
b
b
c
c
c
c
Tng t ta cng c
b2 b c b c
+
+
+ a 4b(2)
c
a
a
92

c2 c a c a
+
+
+ b 4c(3)
a
b
b
Cng cc bt ng thc (1), (2), (3) theo v ta c
A2 3 ( a + b + c ) 9 A 3.

Du ng thc xy ra khi a = b = c = 1. Vy, MinA = 3.


I.33. S =

xy yz zx
+ +
z
x
y

x2 y 2 y 2 z 2 z 2 x 2
Ta c S = 2 + 2 + 2 + 2 x 2 + y 2 + z 2
z
x
y

p dng bt ng thc Csi ta c


x2 y 2 y2 z2
x2 y 2 y2 z2
+

2
. 2 = 2 y 2 (1)
2
2
2
z
x
z
x
Tng t ta cng c
y 2 z2 z2 x2
+ 2 2 z 2 (2)
2
x
y
x 2 y 2 z 2 x2
+ 2 2 x 2 (3)
2
z
y
Cng cc bt ng thc (1), (2), (3) theo v ta c
x2 y 2 y 2 z2 z2 x2
+ 2 + 2 x2 + y 2 + z 2
2
z
x
y
Suy ra S 2 =

x2 y 2 y 2 z2 z2 x2
+ 2 + 2 + 2 x 2 + y 2 + z 2 3 x 2 + y 2 + z 2 = 3 S 3.
z2
x
y

) (

x, y, z > 0
3

ng thc xy ra khi x 2 y 2 y 2 z 2 z 2 x 2 x = y = z =
.
3
z 2 = x2 = y2

Vy, MinS = 3.
I.34. A =

(1 + a )(1 + b )(1 + c ) .
(1 a )(1 b )(1 c )

1 a = b + c > 0

Theo gi thit a + b + c = 1 1 b = a + c > 0


1 c = a + b > 0

p dng bt ng thc Csi ta c


93

1 + a = (1 b ) + (1 c ) 2 (1 b )(1 c ) (1)
Tng t ta cng c
1 + b 2 (1 a )(1 c ) (2)
1 + c 2 (1 a )(1 b ) (3)
Nhn cc bt ng thc (1), (2), (3) theo v ta c

(1 + a )(1 + b )(1 + c ) 8 (1 a )(1 b )(1 c )

(1 + a )(1 + b )(1 + c ) 8
(1 a )(1 b )(1 c )

Suy ra A 8.
1
ng thc xy ra khi a = b = c = .
3

Vy, MinA = 8.
I.35. M =

ab 2 + bc 2 + ca 2

( ab + bc + ca )

p dng bt ng thc Bunhiacpski i vi hai b s

)(

a ; b; c , b a ; c b ; a c

Ta c

( ab + bc + ca )
M

( a + b + c ) ab 2 + bc 2 + ca 2

ab 2 + bc 2 + ca 2
1
=
(1)
( a + b + c ) ab2 + bc 2 + ca 2 a + b + c

Cng theo bt ng thc Bunhiacpski th

(a + b + c)

3 a 2 + b 2 + c 2 = 9 a + b + c 3(2)

1
T (1) v (2) ta suy ra M .
3

ng thc xy ra khi a = b = c = 1.
1
Vy, MinM = .
3

I.36. A = x + 2 y + 3 z 8

Ta c A = x + 2 y + 3 z 8 = 1( x 1) + 2 ( y 2 ) + 3 ( z 1)
p dng bt ng thc Bunhiacpski i vi hai b s

(1; 2;3) , ( x 1; y 2; z 1)
94

Ta c
A = x + 2 y + 3 z 8 14

( x 1) + ( y 2 ) + ( z 1)

= 14

x + 2 y + 3z 8 = 14

ng thc xy ra khi x 1 y 2 z 1 x + 2 y + 3 z 8
14
=
=
=
=

2
3
1+ 4 + 9
14
1
H phng trnh trn c nghim chng hn

14
x = 1+
14

14
. Vy, MaxA = 14.
y = 2 +
7

3 14
z = 1+
14

I.37. A = a 2 + b 2 + b 2 + c 2 + c 2 + a 2

p dng bt ng thc Bunhiacpski i vi hai b s

(1;1) , ( a; b )
Ta c

(1.a + 1.b )

2 a2 + b2 a 2 + b2

1
( a + b ) (1)
2

Tng t ta cng c
b2 + c 2
c2 + a 2

1
2
1
2

( b + c ) (2)
( c + a ) (3)

Cng cc bt ng thc (1), (2), (3) theo v ta c


A

2
( a + b + c ) = 2.
2

1
ng thc xy ra khi a = b = c = .
3

Vy, MinA = 2.
xy + y 2
I.38. A =
.
1 + 2 x 2 + 2 xy

x = sin
Do x 2 + y 2 = 1 nn ta t
, [0; 2 ].
y = cos
95

sin cos + cos 2


.
1 + 2sin 2 + 2sin cos

Khi A =

Ta c 1 + 2 x 2 + 2 xy = ( x + y ) + 2 x 2 , (Do x 2 + y 2 = 1 ). Suy ra A xc nh vi m i x, y tha


x2 + y2 = 1 .
sin cos + cos 2
sin 2 + cos 2 + 1
A=
=
(1)
2
1 + 2sin + 2sin cos 2 ( sin 2 cos 2 + 2 )
(1) ( 2 A 1) sin 2 ( 2 A + 1) cos 2 = 1 4 A (2)

(2) c nghim khi v ch khi


2

( 2 A 1) + ( 2 A + 1)

(1 4 A)

8 A2 8 A 1 0
2 6
2+ 6
.
A
4
4
2+ 6
2 6
MaxA =
, MinA =
.
4
4

x 1
y 1 z 1
I.39. P = x + + y + + z + .
2 zx 2 xy
2 yz
Ta c P =

x2 y 2 z 2 x 2 + y 2 + z 2
+
+ +
.
2
2
2
xyz

Do x 2 + y 2 + z 2 =

x 2 + y 2 y 2 + z 2 z 2 + x2
+
+
xy + yz + zx
2
2
2

x2 1 y 2 1 z2 1
Nn P + + + + + .
2 x 2 y 2 z

Xt hm s f (t ) =

t2 1
+ , t > 0.
2 t

3
9
Lp bng bin thin ca hm f (t ) ta suy ra f (t ) , t > 0. Suy ra P .
2
2

Du = xy ra khi v ch khi x = y = z = 1.
Vy, gi tr nh nht ca P l
I.40. P =

x2 ( y + z )
y y + 2z z

9
.
2

y2 ( z + x )
z z + 2x x

z2 ( x + y )
x x + 2y y

Ta c x 2 ( y + z ) 2 x x ; y 2 ( z + x ) 2 y y ; z 2 ( x + y ) 2 z z .
96

2y y
2x x
2z z
+
+
.
y y + 2z z z z + 2x x x x + 2 y y

Suy ra P

t a = x x + 2 y y ; b = y y + 2 z z ; c = z z + 2 x x
x x=

4c + a 2b
4 a + b 2c
4b + c 2a
;y y =
;z z =
9
9
9

Do
2 4c + a 2b 4a + b 2c 4b + c 2a
P
+
+

9
b
c
a

2 c a b a b c 2
c a b
a b c
= 4 + + + + + 6 4.3 3 . . + 3 3 . . 6 = 2.
9 b c a b c a 9
b c a
b c a

Du = xy ra khi v ch khi x = y = z = 1.
Vy, MinP = 2.

sin x

4

I.41. y =
, x ; .
2
2
sin x + 1 + 2cos x
Ta c y =

2
sin x cos x

.
, x ; .
2
2 sin x + 1 + 2 cos x
2

Xt trng hp x =

: Ta c y =

2
.
4

2
tan x 1
Xt trng hp x ( ; ] : Ta c y =
.
.
2
2 tan x tan 2 x + 3

t t = tan x, vi x ( ; ] th t (; 0].
2

Hm s tr thnh
2
t 1
.
, t (; 0].
2 t t2 + 3

y = f (t ) =

f (t ) =

(t
2
2

t
t 2 + 3 1
( t 1)
2
2
t 2 + 3 + t 1
t +3

=
.
.
2
2
2
2
2
t +3 t t +3
t t +3

f (t ) = 0 t = 1.
Lp bng bin thin ca hm s f (t ) ta kt lun c Maxy =

x ;
2

2
2
, Miny =
.
3 x ; 4
2

97

CHNG II.

PHNG TRNH H PHNG TRNH

II.1. 1) m 2 x + 4m 3 = x + m 2 (1)

(1) m 2 1 x = m 2 4m + 3 (*)

m = 1
+ m2 1 = 0
m = 1
. m = 1, phng trnh (*) tr thnh 0 x = 0 phng trnh (1) c nghim ty .
. m = 1, phng trnh (*) tr thnh 0 x = 8 phng trnh (1) v nghim.

m 1
+ m2 1 0
phng trnh (1) c nghim duy nht.
m 1
m2 4m + 3 m 3
x=
=
.
m2 1
m +1
Kt lun:
+ Nu m = 1 th phng trnh (1) c nghim ty .
+ Nu m = 1 th phng trnh (1) v nghim.

m 1
m3
+ Nu
th phng trnh (1) c nghim duy nht x =
.
m +1
m 1
2

)
+ 2ab + ( a

2) ( a + b ) + 2a 2 = 2a ( a + b ) + a 2 + b 2 x (1)
(1) a 2 + 2ab + b 2 + 2a 2 = 2a 2

+ b2 x

a 2 + b 2 x = a 2 + b2 ( *) .

a = 0
+ Nu a 2 + b 2 = 0
th phng trnh (*) tr thnh 0 x = 0 suy ra phng trnh (1)
b
=
0

c nghim ty .
a 0
a2 + b2
th phng trnh (1) c mt nghim duy nht x = 2
= 1.
+ Nu a 2 + b2 0
2
b

0
a
+
b

Kt lun:

a = 0
+ Vi
th phng trnh (1) c nghim ty .
b = 0
a 0
+ Vi
th phng trnh (1) c mt nghim duy nht x = 1.
b 0
3)a 2 x + 2ab = b 2 x + a 2 + b 2 (1)
2

( a 2 b 2 ) x = ( a b ) (* )

98

a = b
+ a2 b2 = 0
a = b
a = b th (*) tr thnh 0 x = 0 suy ra phng trnh (1) c nghim ty .
a = b th (*) tr thnh 0 x = 4b 2 . Nu b = 0 th phng trnh (1) c nghim ty . Nu
b 0 th phng trnh (1) v nghim.
2

a b
(a b) a b .
+ a b 0
phng trnh (1) c mt nghim duy nht x = 2 2 =
a b
a+b
a b
2

Kt lun:
+ Nu a = b th phng trnh (1) c nghim ty .

a = b
+ Nu
th phng trnh (1) v nghim.
b 0
a b
ab
+ Nu
th phng trnh (1) c mt nghim duy nht x =
a+b
a b

4)a ( ax + b ) = 4ax + b 2 5 (1)


a 2 x + ab = 4ax + b 2 5

a 2 4a x = b 2 ab 5 (*)
a = 0
+ a 2 4a = 0
a = 4
Vi a = 0 th phng trnh (*) tr thnh 0 x = b 2 5.

b 5
Nu b 2 5 0
th phng trnh (1) v nghim.
b 5
b = 5
Nu b 2 5 = 0
th phng trnh (1) c nghim ty .
b = 5
Vi a = 4 th phng trnh (*) tr thnh 0 x = b 2 4b 5

b 1
Nu b 2 4b 5 0
th phng trnh (1) v nghim.
b

b = 1
Nu b 2 4b 5 = 0
th phng trnh (1) c nghim ty .
b = 5
a 0
b 2 4b 5
+ a 2 4a 0
th phng trnh (*) c nghim duy nht x = 2
.
a

4
a

4
a

Kt lun:
99

a = 4
a = 0

+ Nu
b 1 th phng trnh (1) v nghim.
b 5
b 5

a = 0
a = 4

+ Nu b = 5 b = 1 th phng trnh (1) c nghim ty .

b = 5

b = 5

a 0
b 2 4b 5
phng trnh (1) c nghim duy nht x = 2
.
+ Nu
a 4a
a 4
II. 2. 1)

2x + m x + m 1

= 1(1)
x 1
x

x 0
iu kin:
x 1

(1) ( 2 x + m ) x ( x + m 1)( x 1) = x ( x 1)
2 x 2 + mx x 2 + x mx + m + x 1 = x 2 x
3x + m 1 = 0
1 m
x=
.
3
x=

1 m
l nghim ca phng trnh (1) khi v ch khi
3

1 m
3 0
m 1

m 2
1 m 1
3
m 1
1 m
Kt lun: + Nu
th phng trnh (1) c mt nghim duy nht x =
.
3
m 2
m = 2
+ Nu
th phng trnh (1) v nghim.
m = 1

2)

mx 2
m x = 2m + 1(1)
x 1

x 1
iu kin:
x 1
x 0
+ Xt trng hp
th phng trnh (1) tr thnh
x 1

100

mx 2
mx = 2m + 1
x 1
mx 2 mx ( x 1) = ( 2m + 1)( x 1)
( m + 1) x = 2m + 1(*)

Nu m = 1 th (*) v nghim. Nu m 1 th (*) x =


x=

2m + 1
.
m +1

2m + 1
l nghim ca phng trnh (1) khi v ch khi
m +1

2m + 1
1
1
0

m + 1
m < 1 m
m < 1 m

2
2

2
m
+
1

m 0
m 0.
1
m + 1

m < 1 m
Vy, vi
2 th phng trnh (1) c nghim.
m 0
x < 0
th phng trnh (1) tr thnh
+ Xt trng hp
x 1

mx 2
+ mx = 2m + 1
x 1
mx 2 + mx ( x 1) = ( 2m + 1)( x 1)
( m + 1) x = 2m 1(**)
Nu m = 1 th (**) v nghim. Nu m 1 th (**) x =
x=

2 m 1
.
m +1

2m 1
l nghim ca phng trnh (1) khi v ch khi
m +1

2m 1
m 0
m + 1 1 m 0

1
1
2m 1 < 0
m < 1 m > 2
m < 1 m > 2 .
m + 1

Kt lun:
m 0
2m + 1
2m 1

+ Nu
, x2 =
.
1 th phng trnh (1) c hai nghim x1 =
m
+
1
m
+
1
m
<

2
1
(Trng hp m = th hai nghim ny bng nhau v bng 0)
2

101

1 m <

+ Nu
2 th phng trnh (1) v nghim.

m = 0
3)

2mx 1
m +1
2 x 1 =
(1)
x 1
x 1

iu kin x > 1.

(1) 2mx 1 2 ( x 1) = m + 1
( 2m 2 ) x = m(*)
Nu m = 1 th (*) v nghim.
Nu m 1 th (*) x =
x=

m
.
2(m 1)

m
l nghim ca phng trnh (1) khi v ch khi
2(m 1)

m
2m
>1
> 0 1 < m < 2.
2(m 1)
2(m 1)
Kt lun: Khi 1 < m < 2 th phng trnh (1) c mt nghim duy nht x =

m
.
2(m 1)

m 1
phng trnh (1) v nghim.
Khi
m 2
II.3. m 2 x 2 m(5m + 1) x (5m + 2) = 0 (1)

+ Xt m2 = 0 m = 0, (1) tr thnh 2 = 0 (1) v nghim.


+ Xt m 2 0 m 0 th (1) l phng trnh bc hai.
2

= m ( 5m + 1) + 4m 2 ( 5m + 2 ) = 25m4 + 30m3 + 9m 2 = m 2 (5m + 3) 2 0, m .


m = 0
m2 = 0
+ = 0 m (5m + 3) = 0

, kt hp vi gi thit m 0, ta
2
m = 3
(5m + 3) = 0
5

c m =

3
5
, th phng trnh (1) c nghim kp x1 = x2 = .
5
3

m 0

+ > 0 m (5m + 3) > 0


3 th phng trnh (1) c hai nghim phn bit
m 5
2

102

x1 =

5m + 2
1
x2 =
.
m
m

Kt lun:
+ m = 0, phng trnh (1) v nghim.
+ m=

3
5
, phng trnh (1) c nghim kp x1 = x2 = .
5
3

m 0
5m + 2
1

+
; x2 = .
3 , phng trnh (1) c hai nghim phn bit l x1 =
m
m
m 5

II.4. ( a + b ) x 2 a 2 + 4ab + b 2 x + 2ab ( a + b ) = 0 (1)

+ Xt a + b = 0 a = b, phng trnh (1) tr thnh 2b 2 x = 0.


b = 0 : Phng trnh (1) c nghim ty .
b 0 : Phng trnh (1) c mt nghim duy nht x = 0.

+ Xt a + b 0 a b, ta c
2

= ( a 2 + 4ab + b 2 ) 8ab ( a + b ) = ( a 2 + b 2 )

Phng trnh (1) c hai nghim phn bit x1 =

2ab
; x2 = a + b.
a +b

Kt lun:
Nu a = b = 0 th phng trnh (1) c nghim ty .
Nu a = b 0 th phng trnh (1) c mt nghim duy nht x = 0.
Nu a b th phng trnh (1) c hai nghim phn bit x1 =

2ab
; x2 = a + b.
a +b

II.5. Gi s x1 , x2 l nghim ca phng trnh x 2 + ax + bc = 0(1). x1 , x3 l nghim ca

phng trnh x 2 + bx + ca = 0.(2) ( x1 l nghim chung ca cc phng trnh (1) v (2)).


Ta chng minh x2 , x3 tha mn phng trnh x 2 + cx + ab = 0(3).
Theo nh l Viet ta c
x1 + x2 = a
x1 + x3 = b
(*) v
(**)

x1 x2 = bc
x1 x3 = ca

Do x1 l nghim chung ca cc phng trnh (1) v (2) nn ta c


103

x12 + ax1 + bc = x12 + bx1 + ca


(a b) x1 = c (a b)
x1 = c

(Do a, b, c khc nhau i mt v khc 0).


Thay x1 = c vo (*) v (**) ta c
x2 = b = a c
x3 = a = b c
Thay gi tr x2 = b = a c vo v tri phng trnh ( 3) ta c
(a c )2 + c(a c) + ab = a 2 + 2ac + c 2 ac c 2 + a (a c )
= a 2 + 2ac + c 2 ac c 2 a 2 ac = 0
x2 l nghim ca phng trnh ( 3) .

Thay gi tr x3 = a = b c vo v tri phng trnh ( 3) ta c


(b c) 2 + c(b c) + ab = (b c )2 + c(b c ) + (b c)b
= b 2 + 2bc + c 2 bc c 2 b 2 bc = 0
x3 l nghim ca phng trnh ( 3) .

Vy, x2 , x3 l nghim ca phng trnh x 2 + cx + ab = 0. (pcm)


II.6. mx 2 2(m 3) x + m 4 = 0 (1)

+ Xt m = 0, (1) tr thnh 6 x 4 = 0 x =

2
m = 0 tha bi.
3

+ Xt m 0, phng trnh (1) c ng mt nghim dng trong cc trng hp sau


Phng trnh (1) c mt nghim kp dng:

= 9 2m = 0
9

iu kin l
m= .
2 ( m 3)
2
>0
S =
m

Phng trnh (1) c mt nghim dng v mt nghim bng 0:


m4

P = m = 0
iu kin l
m = 4.
2
m

3
(
)
S =
>0

Phng trnh (1) c hai nghim tri du:


iu kin l P =

104

m4
< 0 0 < m < 4.
m

Vy, vi 0 m 4 m =

9
th phng trnh (1) c ng mt nghim dng.
2

II.7. ( m 1) x 4 + 2 ( m 3) x 2 + m + 3 = 0 ( 1)
t X = x 2 0

Khi (1) tr thnh ( m 1) X 2 + 2 ( m 3) X + m + 3 = 0 ( 2 )


+ Nu m 1 = 0 m = 1 khi ( 2 ) tr thnh X = 1 x = 1.
Nh vy m = 1 khng tha yu cu bi.
+ Xt m 1 khi ( 2 ) l phng trnh bc II.

(1)

v nghim khi v ch khi (2) hoc v nghim hoc c hai nghim u m

= ( m 3) 2 ( m 1)( m + 3) < 0
3

m > 2

2
3
= ( m 3 ) ( m 1)( m + 3) 0

m 3
m>

2
2
S = 2 ( m 3 ) < 0

m < 3
m 1
m < 1 m > 3

m < 3 m > 1
P = m + 3 > 0

m 1

Vy, vi m < 3 m >

3
th (1) v nghim.
2

II.8. x 2 2 x m x 1 + m 2 = 0 (1)
2

Ta c (1) ( x 1) m x 1 + m2 1 = 0.
t X = x 1 0 ta c phng trnh X 2 mX + m 2 1 = 0(2)

Phng trnh (1) c nghim khi v ch khi phng trnh (2) c nghim khng m. Ta gii
bi ton ngc tc l tm cc gi tr ca m phng trnh (1) v nghim. Phng trnh
(1) v nghim khi v ch khi phng trnh (2) hoc l v nghim hoc c hai nghim u
m. iu ny xy ra khi v ch khi

2 3
2 3
m<
m>

3
3
= 4 3m 2 < 0

2 3
2 3

2 3
m < 1
m
<

m
>
2
2
3

= 4 3m 0
m

3
3

(*)
3
3
2

m > 2 3

P
=
m

1
>
0

2
3

m < 1 m > 1
m < 1
3

S = m < 0

m < 0

Nh vy, phng trnh (1) v nghim khi v ch khi m tha iu kin (*).

105

Vy, gi tr cn tm ca m l 1 m

2 3
.
3

II.9. 1)( x 1) = 2 x k (1)

( x 1) 2 = 2 ( x k )
x 2 2 x + 1 = 2 x 2k
(1)

2
2

x 2 x + 1 = 2k 2 x
( x 1) = 2 ( k x )

x2 4x +1
1
1
k
=
= x2 + 2x

x 4 x + 1 = 2k
2
2
2
2

x +1 1 2 1
x + 1 = 2k
k = 2 = 2 x + 2
2

1
1
1
1
V cc ng Parabol ( P1 ) : y = x 2 + 2 x , ( P2 ) : y = x 2 + v y = k trn cng mt
2
2
2
2
h trc ta . S nghim ca phng trnh (1) chnh bng s giao im ca ng thng
y = k vi cc Parabol ( P1 ), ( P2 ).

Cc Parabol ( P1 ), ( P2 ) v ng thng y = k c v hnh sau


y

3
2
1
O

y=k

Da vo th ta kt lun c phng trnh (1) c bn nghim phn bit khi v ch khi


1
3
< k < 11 < k < .
2
2

2 ) 2 x 2 + 10 x 8 = x 2 5 x + a ( 2 )
(2) 2 x 2 + 10 x 8 x 2 + 5 x = a . Xt hm s
2
x 5 x + 8; x < 1 x > 4
f ( x) = 2 x 2 + 10 x 8 x 2 + 5 x =
2
3x + 15 x 8;1 x 4

V cc ng Parabol ( P1 ) : y = x 2 5 x + 8, x < 1 x > 4, ( P2 ) : y = 3 x 2 + 15 x 8,1 x 4


v y = a trn cng mt h trc ta . S nghim ca phng trnh (2) chnh bng s giao
im ca ng thng y = a vi cc Parabol ( P1 ), ( P2 ).
106

Cc Parabol ( P1 ), ( P2 ) v ng thng y = a c v hnh sau


y
43
4

y=a
4
5
2
O

Da vo th ta kt lun c phng trnh ( 2 ) c bn nghim phn bit khi v ch khi


4<a<

43
.
4

II.10. 1) ( x 1)( x + 5 )( x 3 )( x + 7 ) = 297 (1) .

(1) ( x 1)( x + 5 ) ( x 3)( x + 7 ) = 297

)(

x 2 + 4 x 5 x 2 + 4 x 21 = 297
2

t t = x 2 + 4 x 5 = ( x + 2 ) 9 9

Khi phng trnh (1) tr thnh


t ( t 16 ) = 297
t 2 16t 297 = 0
t = 27

t = 11
Ta nhn t = 27 v ta c phng trnh
x = 4
x 2 + 4 x 5 = 27 x 2 + 4 x 32 = 0
x = 8.
Vy, nghim ca phng trnh l x = 4 x = 8 .
2) ( x + 2 )( x 3)( x + 1)( x + 6 ) = 36 (1)
(1) ( x + 2 )( x + 1) ( x + 6 )( x 3) = 36

)(

x 2 + 3x + 2 x 2 + 3x 18 = 36
t t = x 2 + 3 x + 2.

Khi phng trnh (1) tr thnh

107

t ( t 20 ) = 36
t 2 20t + 36 = 0
t = 2

t = 18
Vi t = 2 th ta c phng trnh

x 2 + 3x + 2 = 2
x 2 + 3x = 0
x = 0

x = 3
Vi t = 18 th ta c phng trnh
x 2 + 3x + 2 = 18
x 2 + 3 x 16 = 0

3 + 73
x =
2

3 73
x =

Vy, nghim ca phng trnh l x = 0, x = 3, x =


3) x ( x 2 )( x + 2 )( x + 4 ) = 18 (1)
(1) x ( x + 2 ) ( x 2 )( x + 4 ) = 18

)(

x 2 + 2 x x 2 + 2 x 8 = 18
2

t t = x 2 + 2 x = ( x + 1) 1 1

Khi phng trnh (1) tr thnh


t = 4 + 34
t ( t 8) = 18 t 2 8t 18 = 0
t = 4 34
Ta nhn t = 4 + 34 v ta c phng trnh
x 2 + 2 x = 4 + 34
2

( x + 1) = 5 + 34
x = 1 + 5 + 34

x = 1 5 + 34

108

3 73
.
2

Vy, nghim ca phng trnh l x = 1 5 + 34 .


4

II.11. 1) x 4 + ( x 1) = 97 (1)
t t = x

1
1
x = t + . Khi (1) tr thnh
2
2

1 1
t + 2 + t 2 = 97

1
2t 4 + 3t 2 + = 97
8
16t 4 + 24t 2 775 = 0
2 25
t = 4

t 2 = 31

4
Ta nhn t 2 =

25
.
4

5
t = 2
25
2
Vi t =

4
t = 5

2
Khi t =

5
1 5
th x = x = 3.
2
2 2

Khi t =

5
1
5
th x = x = 2.
2
2
2

Vy, nghim ca phng trnh cho l x = 3, x = 2.


4

2) ( x + 3) + ( x + 5 ) = 16 (1)
t t = x + 4 x = t 4. Khi (1) tr thnh
4

( t 1) + ( t + 1)
4

= 16

2t + 12t + 2 = 16
2t 4 + 12t 2 14 = 0
t 4 + 6t 2 7 = 0
t 2 = 1
t = 1
2

t = 1
t = 7

Khi t = 1 th x + 4 = 1 x = 3.

109

Khi t = 1 th x + 4 = 1 x = 5.

Vy, phng trnh cho c nghim l x = 3; x = 5.


4

3) ( x + 2 ) + ( x + 6 ) = 2 (1)
t t = x + 4 x = t 4. Khi (1) tr thnh
4

(t 2) + (t + 2)

=2

2t 4 + 48t 2 + 32 = 2
t 4 + 24t 2 + 15 = 0
t 2 = 12 129

(Loi)
t 2 = 12 + 129
Vy, phng trnh cho v nghim.
II.12. 1) 6 x 4 35 x 3 + 62 x 2 35 x + 6 = 0 (1)

Do x = 0 khng phi l nghim ca phng trnh (1) nn chia hai v ca phng trnh cho
x 2 0, ta c phng trnh tng ng
6( x 2 +

1
1
) 35( x + ) + 62 = 0 (2)
2
x
x

1
t t = x + , iu kin: t 2
x
x2 +

1
= t2 2
x2

Khi , phng trnh (2) tr thnh


6(t 2 2) 35t + 62 = 0
6t 2 35t + 50 = 0
10
t = 3

t = 5
2
Vi t =
x+

10
ta c phng trnh
3

1 10
10
=
x 2 x + 1 = 0 (Do x 0 )
x 3
3

3x 2 10 x + 3 = 0

110

x = 3

x = 1
3

Vi t =
x+

5
2

1 5
5
= x 2 x + 1 = 0 (Do x 0 )
x 2
2

x = 2

x = 1

2
1
1
Vy, phng trnh cho c bn nghim x = 3; x = ; x = 2; x = .
3
2

2) x 4 + x3 4 x 2 + x + 1 = 0 (1)
Do x = 0 khng phi l nghim ca phng trnh (1) nn chia hai v ca phng trnh cho
x 2 0, ta c phng trnh tng ng
x2 + x 4 +
(x2 +

1 1
+
=0
x x2

1
1
) + (x + ) 4 = 0
2
x
x

1
t t = x + , iu kin: t 2
x
x2 +

1
= t2 2
x2

Khi phng trnh tr thnh


t2 + t 6 = 0
t = 2

t = 3

x1 = 1, x2 = 2
Vi t = 2
x+

1
= 2 x2 2x + 1 = 0 x = 1
x

Vi t = 3
x+

1
= 3 x 2 + 3x + 1 = 0
x

111


3 + 5
x =
2

3 5
x =
2

Vy, phng trnh cho c ba nghim l x = 1; x =

3 + 5
3 5
;x =
.
2
2

3) x 4 5 x 3 + 10 x 2 10 x + 4 = 0 (1)
Ta c
(1) x 4 5 x 3 + 10 x 2 10 x + 4 = 0
( x 4 x3 ) + 10( x 2 x ) 4( x3 1) = 0
x3 ( x 1) + 10 x ( x 1) 4( x 1)( x 2 + x + 1) = 0
( x 1)( x 3 + 10 x 4 x 2 4 x 4) = 0
( x 1)( x 3 4 x 2 + 6 x 4) = 0
( x 1)( x 2)( x 2 2 x + 2) = 0

x = 1

x = 2
2
x 2x + 2 = 0
x = 1

x = 2
Vy, phng trnh cho c hai nghim l x = 1; x = 2.
4) 2 x 4 21x3 + 74 x 2 105 x + 50 = 0
Do x = 0 khng phi l nghim ca phng trnh nn chia hai v ca phng trnh cho
x 2 0, ta c phng trnh tng ng
2( x 2 +

25
5
) 21( x + ) + 74 = 0
2
x
x

5
t t = x + ; iu kin: t 2 5
x
x2 +

25 2
= t 10
x2

Khi phng trnh tr thnh


t = 6
2t 21t + 54 = 0 9
t =
2
2

112

Vi t = 6 x +

x = 1
5
=6
x
x = 5.

x = 2
5 9
9
Vi t = x + =
x = 5 .
2
x 2

2
5
Vy, phng trnh cho c bn nghim l x = 1; x = 2; x = 5; x = .
2

5) 2 x 4 + 5 x3 + x 2 + 5 x + 2 = 0
Ta thy x = 0 khng phi l nghim ca phng trnh nn chia hai v ca phng trnh
cho x 2 0, ta c phng trnh tng ng
2( x 2 +

1
1
) + 5( x + ) + 1 = 0
2
x
x

1
t t = x + , iu kin: t 2
x
x2 +

1
= t2 2
2
x

Khi phng trnh tr thnh


2(t 2 2) + 5t + 1 = 0
2t 2 + 5t 3 = 0
t = 3
1
t =
2
Vi t =

1
(Loi)
2

Vi t = 3

1
= 3 x 2 + 3x + 1 = 0
x

3 + 5
x =
2

3 5
x =

2
x+

Vy, phng trnh cho c hai nghim l x =

3 + 5
3 5
;x =
.
2
2

II.13. ( x + 3)( x 1)( x + 5 )( x 3) 40 = m (1)


t t = ( x + 3)( x 1) = x 2 + 2 x 3 ( ) t 12 = x 2 + 2 x 15 = ( x 3)( x + 5 )

113

Khi (1) tr thnh t ( t 12 ) 40 = m t 2 12t 40 = m


Xt hm s t = f ( x ) = x 2 + 2 x 3
f ( x ) = 2 x + 2 = 0 x = 1

Bng bin thin

Xt hm s g ( t ) = t 2 12t 40
g ( t ) = 2t 12 = 0 t = 6

Bng bin thin

Ta c nhn xt:
t = 4, phng trnh (*) c mt nghim x.
t < 4, phng trnh (*) v nghim x.
t > 4, phng trnh (*) c hai nghim x phn bit.

Vy, ta c s nghim ca phng trnh cho nh sau


+ m = 24, (1) c ba nghim phn bit.
+ m > 24 m = 76, (1) c hai nghim phn bit.
+ 76 < m < 24, (1) c bn nghim phn bit.
+ m < 76, (1) v nghim.
2 x + y + 1 x + y = 1(1)
II.14. 1)
3x + 2 y = 4 ( 2)

y +1
x

iu kin:
2 (*)
x y

Vi iu kin (*) th h phng trnh cho c bin i v

114

1
1
2x + y + 1 x + y = 1
x + 3 2 x = 1( 2 )

2
2

3
y = 2 x
y = 2 3 x

(3)
2

( 2)

1
1
x + 3 2 x =1
2
2

1
1
x + 3 = 2 x +1
2
2

x 0
2
x=2
x + 2x 8 = 0

Th x = 2 vo phng trnh (3) ta c y = 1. Cp gi tr (2; 1) tha iu kin (*) nn


l nghim ca h phng trnh cho.
x

2
xy

3
= 15

2)
(1)
xy + x = 15

y
iu kin: y 0
2 xy 2 3 x = 15 y
2 xy 2 3 x = 15 y
2 xy 2 3 x = 15 y
(1)

2
2
2
xy 4 x = 0
xy + x = 15 y
x ( y 4 ) = 0

2 xy 2 3 x = 15 y

x = 0

y = 2
y = 2

Vi x = 0 th y = 0 (Loi)
Vi y = 2 th x = 6
Vi y = 2 th x = 6

Vy, h phng trnh c hai nghim l (6;2) v (6; 2).


x
. Khi h phng trnh cho c a v h phng
y
trnh bc nht i vi hai n u , v.

Ch . C th t u = xy, v =

x 2 xy = 12
3) 2
(1)
y xy = 28
0 = 12
Vi x = 0 thay vo (1), ta c 2
(V l).
y = 28

115

12
12

x( x y ) = 12
y x =
y x =

Vi x 0 , ta c (1)
x
x
y ( y x ) = 28 y ( y x) = 28 12 y = 28 x

12
7
2 18
10 x 2 = 36
x =
3 x + x = x

7
y = 7 x
y = 3 x
y = 7 x

3
3

3 10
3 10
3 10
3 10
x=
x =
x =
x =

5
5
5
5

y = 7 x
y = 7 10 y = 7 10

3
5
5
3 10 7 10 3 10 7 10
;
;
Vy, h phng trnh cho c hai nghim l
,
.
5
5
5
5
x + y 3x + 2 y = 1
4)
x + y + x y = 0;

x + y 0
iu kin:
t u = x + y 0, v = 3 x + 2 y 0. H phng trnh cho tr
3 x + 2 y 0
v = u + 1
u v = 1
v = u + 1

thnh

1
2
2
2
u + 2u 5v = 0
3u + 5u + 2 = 0
u = 2 u = 6
x + y = 2
x + y = 4
x = 1
Ta chn u = 2; v = 3. Ta c

y = 3
3x + 2 y = 3 3 x + 2 y = 9

Vy, h phng trnh cho c mt nghim l (1;3).


1
1
2
2
x + y + x2 + y 2 = 4

5)
(1)
1 1

x+ y+ + = 4

x y
x 0
iu kin:
(*)
y 0

116

1 2
1 2

1
1
(
)
(
)
8
x
+
+
y
+
=
4 ( y + ) + ( y + )2 = 8

x
y

y
y
(1)

( x + 1 ) + ( y + 1 ) = 4
x + 1 = 4 ( y + 1 )

x
y
x
y

1 2
1

y+ =2
2( y + y ) 8( y + y ) + 8 = 0

x + 1 = 4 ( y + 1 )
x + 1 = 2

x
y
x
x 2 2 x + 1 = 0
x = 1
2

tho iu kin (*).


y 2 y + 1 = 0
y =1
Vy, h phng trnh cho c mt nghim l (1;1).
x2 + y2 + 6x + 2 y = 0
6)
x + y + 8 = 0

Ta c x + y + 8 = 0 x = 8 y thay vo phng trnh x 2 + y 2 + 6 x + 2 y = 0 ta c

( 8 y )

+ y 2 + 6 ( 8 y ) + 2 y = 0

64 + 16 y + y 2 + y 2 48 6 y + 2 y = 0
y2 + 6 y + 8 = 0
y = 2 y = 4 .
Vi y = 2 x = 6
Vi y = 4 x = 4
Vy, h phng trnh cho c hai nghim l ( 6; 2 ) , ( 4; 4 ) .
x 2 + y 2 xy = 13
7)
(I )
x + y xy = 3
iu kin: xy 0

( x + y )2 3xy = 13
(I )
x + y xy = 3
t u = x + y , v = xy 0 .

Khi ta c

117

u 2 3v 2 = 13 (1)
(I )
u v = 3 ( 2 )

( 2) u = 3 + v

thay vo (1) ta c ( 3 + v ) 3v 2 = 13.

v 2 3v + 2 = 0.
v = 1
(Nhn)

v = 2
x + y = 4 x = 2 3 x = 2 + 3
Vi v = 1 u = 4. T y ta c h

xy = 1
y = 2 + 3 y = 2 3
x + y = 5
x = 4 x = 1
Vi v = 2 u = 5 . T y ta c h

xy = 2
y =1 y = 4
Vy, h phng trnh cho c bn nghim l

(1; 4 ) , ( 4;1) , ( 2 +

)(

3; 2 3 , 2 3; 2 + 3 .

x + y + x 2 + y 2 = 8
8)
xy ( x + 1)( y + 1) = 12
H phng trnh cho tng ng vi
( x + y ) 2 + x + y 2 xy = 8
(I)

xy ( xy + x + y + 1) = 12

t S = x + y; P = xy, iu kin: S 2 4 P 0 .

S 2 + S 2 P = 8
()
P ( P + S + 1) = 12

(1) P =

S2 + S 8
2

(1)
( 2)

(3)

Th (3) vo (2) ta c

S2 + S 8 S2 + S 8
+ S + 1 = 12

2
2

S 4 + 4 S 3 11S 2 30 S = 0

S S 3 + 4S 2 11S 30 = 0

118

S
S

=0
=3
= 2
= 5

Th ln lt cc gi tr ca S vo (3) ta c
S = 0
S = 3 S = 2 S = 5
(Tho iu kin S 2 4 P 0 )

P = 4 P = 2 P = 3 P = 6
S = 0
x + y = 0
x = 2
x = 2

P = 4 xy = 4
y = 2 y = 2
S = 3 x + y = 3 x = 1 x = 2

P = 2 xy = 2
y = 2 y =1
S = 2 x + y = 2
x = 1
x = 3

P = 3 xy = 3
y = 3 y = 1
S = 5 x + y = 5 x = 3 x = 2

P = 6
xy = 6
y = 2 y = 3
Vy, h phng trnh cho c tm nghim l (2; 2), ( 2;2), (1;2), (2;1), (1; 3),
( 3;1), ( 3; 2),( 2; 3).
Ch . C th t u = x( x + 1); v = y ( y + 1). Khi h phng trnh cho c a v h
u + v = 8

uv = 12
2
2
x + y + x + y = 4
9)
(I )
x ( x + y + 1) + y ( y + 1) = 2

( x + y )2 + x + y 2 xy = 4
(I )
2
2
x + xy + x + y + y = 2
2
( x + y ) 2 + x + y 2 xy = 4
( x + y ) + x + y xy = 2

2
( x + y ) + x + y xy = 2
xy = 2

t S = x + y; P = xy, iu kin S 2 4 P 0 .

Ta c h phng trnh
S 2 + S P = 2

P = 2

119

S = 0
S 2 + S = 0

S = 1

P
=

P = 2

S = 0
S = 1
(Tho iu kin S 2 4 P 0 )

P = 2 P = 2
x = 2 x = 2
S = 0
x + y = 0

P = 2 xy = 2
y = 2
y = 2
S = 1 x + y = 1
x = 1
x = 2
+

P = 2 xy = 2
y = 2 y = 1

) (

Vy, h phng trnh cho c bn nghim l 2; 2 ,

2; 2 , (1; 2 ) , ( 2;1) .

( x 2 + y 2 ) xy = 78
10) 4
4
x + y = 97
Ta c
( x 2 + y 2 ) xy = 78
( x 2 + y 2 ) xy = 78
2
(I )
4
4
2 2
2 2
x + y = 97
( x + y ) 2 x y = 97
T phng trnh th nht ca h cho ta suy ra x. y > 0. t u = x 2 + y 2 , v = xy, iu kin
u > 0, v > 0. Ta c
78

78

v=
v=
78

u
u
uv = 78

v =

u
2
(I ) 2 2
2
u = 169
u 2v = 97
u 2 2 78 97 = 0
u 4 97u 2 12168 = 0

u 2 = 72
u

78

v = u

u = 13

u = 13
2
2
u = 13 x + y = 13
Ta nhn
2 2
( II )
v = 6
x y = 36

X = 9
T ( II ) ta c x 2 , y 2 l hai nghim ca phng trnh bc hai X 2 13 X + 36 = 0
X = 4
Vy ta c

120

x = 3 x = 3

x = 9 x = 4
y = 2 y = 2
2

2
x = 2 x = 2
y = 4 y = 9

y = 3 y = 3
2

Vy, h cho c tm nghim l


( 3; 2 ) , ( 3; 2 ) , ( 3; 2 ) , ( 3; 2 ) , ( 2;3) , ( 2; 3) , ( 2;3) , ( 2; 3) .
x 3 y 3 = 7( x y )
11) 2
2
x + y = x + y + 2
Ta c
3
3
2
2
x y = 7( x y )
( x y )( x + xy + y 7) = 0

2
2
2
2
x + y = x + y + 2
x + y ( x + y ) 2 = 0

x = y
(I )
2
2
x + y ( x + y) 2 = 0
2
2
x + xy + y 7 = 0
( II )
x 2 + y 2 ( x + y ) 2 = 0

Gii ( I )

1+ 5
x =
2

1+ 5
y
=

x = y
2
(I ) 2

x x 1 = 0
1 5
x =
2

y = 1 5

2
Gii ( II )
( x + y )2 xy 7 = 0
( II )
2
( x + y ) 2 xy ( x + y ) 2 = 0
xy + ( x + y ) 5 = 0

2
( x + y ) 2 xy ( x + y ) 2 = 0

t S = x + y, P = xy, ( S 2 4 P )

Nh vy ta c

121

S = 3

P = 5 S
P = 2
( II ) 2

S = 4
S + S 12 = 0

P = 9
S = 3
Ta nhn
suy ra x, y l nghim ca phng trnh bc hai
P = 2
X =1
X 2 3X + 2 = 0
X = 2
x = 1 x = 2
Vy ta c

y = 2 y =1
H phng trnh cho c bn nghim l
1+ 5 1+ 5 1 5 1 5
;
;
,
.
2
2
2
2

(1; 2 ) , ( 2;1) ,

x + 2 + y 3 = 10
12)
(I)
4
4 y 3 = 4.
x
+
2
+

x 2
iu kin:
y 3.
4
u = x + 2
t
(Vi u 0, v 0 ). (*)
4 y 3.
v
=

u 2 = x + 2

2
v = y 3.

H phng trnh (I) tr thnh


u + v = 4
2 2
u + v = 10
(u + v )2 2uv = 10
u = 1
u + v = 4


uv = 3
v = 3
u + v = 4

u = 3

v = 1.

4 x + 2 = 1 x + 2 = 1
u = 1
x = 1

(Nhn).
+
4 y 3 = 3
y

3
=
81
y
=
84

v = 3

4
u = 3
x + 2 = 3 x + 2 = 81 x = 79

(Nhn).
+
y 3 =1
y = 4.
v = 1
4 y 3 = 1

122

Vy, h phng trnh cho c hai nghim l (1;84), (79; 4).


( x 1)( y 1)( x + y 2 ) = 6
13) 2
(I )
2
x + y 2 x 2 y 3 = 0
( x 1)( y 1)( x 1 + y 1) = 6
Ta c ( I )
2
2
( x 1) + ( y 1) = 5

X = x 1
t
, h phng trnh cho tr thnh
Y = y 1
XY ( X + Y ) = 6
XY ( X + Y ) = 6

2
2
X + Y = 5
( X + Y ) 2 XY = 5

S = X + Y 2
t
; S 4 P 0 , ta c h
P = XY
6

P=

S
.
P
=
6

P = 2

P =
S

S
2
S 2 P = 5 S 2 12 = 5 S 3 5S 12 = 0 S = 3

S
X +Y = 3 X =1 X = 2
x 1 = 1 x 1 = 2
x = 2 x = 3
Nh vy ta c

XY = 2
Y = 2 Y = 1
y 1 = 2 y 1 = 1
y = 3 y = 2
Vy, h phng trnh cho c hai nghim l ( 2;3) , ( 3; 2 ) .
xy + x + 1 = 7 y
14) 2 2
2
x y + xy + 1 = 13 y

y = 0 : H phng trnh cho v nghim.


y 0 : H phng trnh cho tng ng vi

x 1

x + y + y = 7
1
x
1
x
1

. t a = x + ; b = a 2 = x 2 + 2 + 2 x 2 + 2 = a 2 2b.

y
y
y
y
y
x 2 + x + 1 = 13
2

y y
Ta c h phng trnh theo n mi
a + b = 7
a + b = 7
a + b = 7
2

2
a = 4 a = 5
a b = 13 a + a 20 = 0

123


1
x + y = 4
x2 4 x + 3 = 0

x = 1

a = 4
x

=3
y

y = 1

3
b = 3
y


2
a = 5
1

x
+
5
x
+
12
=
0

x + = 5

x = 3

(VN )
y
b
=
12


y = 1.
y =
x
12

= 12
y
1
Vy, h phng trnh cho c hai nghim l 1; , ( 3;1) .
3
x ( x + y + 1) 3 = 0

15)
. iu kin: x 0. H phng trnh cho tng ng vi
5
2
( x + y ) 2 + 1 = 0
x

x ( x + y + 1) 3 = 0

x ( x + y + 1) 3 = 0

. t t = x ( x + y ) . Ta c h phng trnh

2
5
2
2
2
( x + y ) + 1 = 2
x ( x + y ) + x = 5
x

t + x = 3
t + x = 3 x = 2 x = 1
t + x = 3

2
2
t = 1 t = 2.
t + x = 5 ( t + x ) 2tx = 5 tx = 2
x = 2
x = 1
x ( x + y ) = 1 x ( x + y ) = 2

Nh vy, ta c
(Tha iu kin)

3
=
1.
y
=

y
x = 2
x = 1

2
3

Vy, h phng trnh cho c hai nghim l 2; , (1;1) .


2

x 2 3xy = 4 y
(1)
II.15. 1) 2
y 3xy = 4 x
2
2
x y = 4 ( x y )
( x y )( x + y + 4 ) = 0
(1) 2
2
y 3xy = 4 x
y 3xy = 4 x

x = y
x = y
x = 0
2
2

y = 0
y 3 xy = 4 x
x + 2 x = 0

x = 2
y = 4 x
y = 4 x

y 2 3 xy = 4 x
x 2 + 4 x + 4 = 0
y = 2.

Vy, h phng trnh cho c hai nghim l ( 0; 0 ) , ( 2; 2 ) .

124

7 x + y x 2 = 0
2)
(1)
7 y + x 1 = 0
y2

iu kin: x 0, y 0.
2
2
3
2
7 x + x y 1 = 0 ( x y ) 7 x + xy + y + xy = 0
(1) 3

2
7 y + xy 1 = 0 7 y 3 + xy 2 1 = 0

1
x = y

x=

x
=
y

2
4 ( x + y ) + 3 x 2 + y 2 = 0(*) 3

8 x = 1 y = 1 .
3
2
7 y + xy 1 = 0

(Do x 0, y 0 nn (*) v nghim)


1 1
Vy, h phng trnh cho c mt nghim duy nht l ; .
2 2
x 2 = y 1 + 2 x 1
3)
(1)
2
y = x 1 + 2 y 1
x 1
iu kin:
y 1
x 2 2 x + 1 = y 1

1
() 2
y 2 y + 1 = x 1
( x 1) 2 = y 1 (1)

2
( y 1) = x 1 ( 2 )

( 2 ) ( y 1)

= x 1

( y 1) + 1 = x
Th gi tr x va tm c vo (1) ta c:
8

( y 1)

t: t =

= y 1 (a)
y 1 (t 0)

( a ) t 16 = t

t t15 1 = 0

t=0
t = 0
y =1
15

t = 1
t =1
y = 2
125

Vy, h phng trnh cho c hai nghim l (1;1) v ( 2; 2 ) .


Ch . C th gii bng cch khc nh sau:
Tr tng v ca hai phng trnh ca h cho ta c
x 2 y 2 = y 1 x 1 + 2 x 2 y x2 2 x + x 1 = y 2 2 y + y 1
f ( x) = f ( y ).(*)

Xt hm s f (t ) = t 2 2t + t 1, t [1; +). Hm s ny ng bin trn tp xc nh, do


ta c (*) x = y. T tm c nghim ca h phng trnh cho l (1;1) , ( 2; 2 ) .
1
1
x x = y y (1)
4)
2 y = x 3 + 1 ( 2 )

x 0
iu kin:
y 0
Ta c

(1) x y =

1 1

x y

( x y ) + 1 = 0
xy
y = x

y = 1
x

+ Th y = x vo phng trnh ( 2 ) ta c

( 2 ) x3 + x 2 = 0
( x 1) ( x 2 + x + 2 ) = 0

x = 1 v x 2 + x + 2 > 0, x

Vy, nghim ca h phng trnh l (1;1).


+ Th y =

1
vo phng trnh ( 2 ) ta c
x

( 2 ) x 4 + x + 2 = 0 ( 3)
1
1
Xt hm s y = f ( x) = x 4 + x + 2 y = 4 x3 + 1 = 0 x = 3 ; f ( 3 ) < 0. Lp bng
4
4
bin thin ca hm s f ( x ) ta kt lun c th ca hm s khng ct trc honh.
Suy ra phng trnh ( 3) v nghim.
126

Vy, h phng trnh cho c mt nghim duy nht l (1;1).


4
3
2 2
x + 2 x y + x y = 2 x + 9
(I)
5) 2
x + 2 xy = 6 x + 6

Xt x = 0, khi h phng trnh tr thnh


0 = 9
(V nghim)

0
=
6

Xt x 0 , khi h phng trnh (I) tng ng vi


x 2 ( x + y )2 = 2 x + 9 (1)

x2 + 6 x + 6
(2)
y
=

2x

Thay (2) vo (1) ta c


2

x2 + 6x + 6
x x+
= 2x + 9
2x

x2 + 6 x + 6
x
= 2x + 9
2x

( x 2 + 6 x + 6)2 = 4(2 x + 9)
2

( x + 3)2 3 = 8( x + 3) + 12 (3)
t t = x + 3, khi (3) tr thnh

(t

3 = 8t + 12

t 4 6t 2 + 9 = 8t + 12
(t + 1)3 (t 3) = 0
t + 1 = 0
t = 1

t 3 = 0
t = 3
Vi t = 3 ta c 3 = x + 3 x = 0 (Loi)
Vi t = 1 ta c 1 = x + 3 x = 4 (Nhn), thay x = 4 vo (2) ta c

(4) 2 + 6(4) + 6 17
y=
=
2(4)
4
17

Vy, h phng trnh cho c nghim l 4; .


4

xy + x + y = x 2 2 y 2
6)
(I )
x 2 y y x 1 = 2 x 2 y
127

x 1
iu kin:
(*)
y 0
2
xy + x + y + y = ( x + y )( x y )
(I )
x 2 y y x 1 = 2 x 2 y

x( y + 1) + y ( y + 1) = ( x + y )( x y )

x 2 y y x 1 = 2 x 2 y
( y + 1)( x + y ) = ( x + y )( x y )

x 2 y y x 1 = 2 x 2 y
( x + y )(2 y x + 1) = 0

x 2 y y x 1 = 2 x 2 y
x + y = 0

x = 2 y + 1

x 2 y y x 1 = 2 x 2 y

Do iu kin (*) nn ta loi trng hp x + y = 0. Nh vy ta ch c trng hp


2 y + 1 = x

x 2 y y x 1 = 2 x 2 y
2 y + 1 = x

(2 y + 1) 2 y y 2 y = 2(2 y + 1) 2 y
2 y + 1 = x

2 y ( y + 1) = 2( y + 1)
2 y + 1 = x

( y + 1)( 2 y 2) = 0
2 y + 1 = x

( y + 1 > 0)
2 y 2 = 0

x = 5
(Nhn)

y = 2

Vy, h phng trnh cho c nghim duy nht l (5; 2) .


4
3
2 2
x x y + x y = 1
7) 3
(I )
2
x y x + xy = 1

( x 2 xy ) 2 + x 3 y = 1

Ta c ( I )
3
2
x y ( x xy ) = 1

128

u = x 2 xy
t
, khi ta c h phng trnh
3
v = x y
u = 1
u 2 + v = 1
u 2 + u 2 = 0

u = 2

v u = 1 v = u 1
v = u 1

u = 1 u = 2

v = 0 v = 3
2
x 2 xy = 1 xy = x 1
u = 1
+
3
2 2
x x 1 = 0
x y = 0
v = 0

x = 1 x = 1

y = 0 y = 0
2
x 2 xy = 2
u = 2
xy = x + 2
+
3
2 2
(H phng trnh v nghim)
x
x
+
2
+
3
=
0
(
)
x y = 3
v = 3

Vy, h phng trnh cho c hai nghim l (1;0); ( 1;0).


x 3 3 x 2 y + 2 x 6 y = 15
8) 3
2
x + x y + 2 x + 2 y = 9

H phng trnh cho tng ng vi


x x 2 + 2 3 y x 2 + 2 = 15

2
2
x x + 2 + y x + 2 = 9

(
(

) (
)
) (
)
( x + 2 ) ( x 3 y ) = 15 (1)

( 2)
( x + 2 ) ( x + y ) = 9
2
2

V x + y = 0 khng tha phng trnh ( 2 ) , nn chia (1) cho ( 2 ) theo v ta c


x 3 y 5
=
x+ y
3
3x 9 y = 5 x 5 y
y = 2 x.
Thay y = 2 x vo ( 2 ) ta c

(x

+ 2 ( x + 2x) = 9

x3 + 2 x 3 = 0
x = 1 y = 2.
129

Vy, nghim ca h phng trnh cho l (1; 2 ) .


2
2
x + y 3 x + 4 y = 1
9) 2
(I )
2
3 x 2 y 9 x 8 y = 3

x 2 3 x + y 2 + 4 y = 1
Ta c ( I )
(2)
2
2
3 ( x 3 x ) 2 ( y + 4 y ) = 3
2
u = x 3 x
t
ta c h phng trnh
2
v = y + 4 y

3 13
3 + 13
x 2 3 x = 1
u + v = 1
u = 1
x =
x=

2
2
y + 4 y = 0
3u 2v = 3
v = 0
y = 0 y = 4

Vy, h phng trnh cho c bn nghim l


3 + 13 3 13 3 + 13
3 13

; 0 ,
; 0 ,
; 4 ,
; 4 .

2
2
2
2

( x y )2 y = 2
10)
(I )
3
3

=
19
x
y

( x y )2 y = 2

Ta c ( I )
2
2
( x y ) x + xy + y = 19(*)

Ta nhn xt rng ( x; y ) / x = y khng l nghim ca h phng trnh cho, do chia


theo v ca hai phng trnh ca h ta c

( x y) y
2

x + xy + y

2
19

19 xy 19 y 2 = 2 x 2 + 2 xy + 2 y 2
2 x 2 17 xy + 21y 2 = 0
2

x
x
2 17 + 21 = 0
y
y
x
y =7

x 3
y = 2

x
= 7 x = 7 y kt hp vi (*) ta c h phng trnh
y

130


x=
x = 7 y
x = 7 y

3
3
3
x y = 19 18 y = 1 y =

7
18
1
3
18

x 3
3
= x = y kt hp vi (*) ta cng c h phng trnh
y 2
2

x = 3
x = y

x 3 y 3 = 19 y = 2

1
7
Vy, h phng trnh cho c hai nghim l 3 ; 3 , ( 3; 2 ) .
18 18
3
3
x + y = 1
(I )
11) 2
2
3
x y + 2 xy + y = 2

( x + y ) x 2 xy + y 2 = 1 ( x + y ) x 2 xy + y 2 = 1

(I )

2
2
2
y x + 2 xy + y = 2
y ( x + y ) = 2
x 2 xy + y 2 1

= 2 x 2 xy + y 2 = xy + y 2
y ( x + y)
2

2 x 2 2 xy + 2 y 2 xy y 2 = 0 ( x y )( 2 x y ) = 0
x = y

2x = y

34 34
+ Trng hp x = y, ta c nghim ca h phng trnh l
;
.
2 2

+ Trng hp 2 x = y, thay vo phng trnh th nht ca h phng trnh cho ta c


3

x3 + ( 2 x ) = 1 9 x3 = 1 x =

1
. Nghim ca h phng trnh l
3
9

3 9 23 9
;

.
3
3

3 4 3 4 3 9 23 9
Vy, h phng trnh cho c hai nghim l
;
;
,
.
3
2 2 3
3
3
2
x y = x y
12) 2
(I )
2
x + y = x y
2
y 3 = y
y = 0
y y 1 = 0
Khi h phng trnh ( I ) tr thnh 2

y = y
y = 1
y ( y + 1) = 0

131

Nh vy, nghim ca h l ( 0; 0 ) , ( 0; 1) .
+ Xt trng hp x 0
Nhn hai v ca phng trnh th hai ca h ( I ) vi x ta c h phng trnh tng
ng
3
3
2
x y = x y
3
2
2
x + xy = x xy

Sau khi tr tng v ca hai phng trnh v bin i tip ta c


x 3 y 3 = x 2 y
x3 y 3 = x 2 y
x 3 y 3 = x 2 y

3
3
3
2
2
2
2
x + xy = x xy
y + xy = y xy
y + xy y + xy = 0
3
3
2
3
3
2
x y = x y
x y = x y

2
y ( y + 1)( x + y 1) = 0
y ( y + xy 1 + x ) = 0

y = 0, suy ra x = 1, do ta c nghim (1;0 ) .


y = 1, suy ra x = 1, do ta c nghim (1; 1) .
3

y = 1 x, ta c phng trnh x 3 (1 x ) = x 2 1 + x x3 2 x 2 + x = 0 x = 1, ( x 0).

Ta c nghim (1;0 ) .
Vy, h phng trnh cho c bn nghim l ( 0; 0 ) , ( 0; 1) , (1;0 ) , (1; 1) .
3 x y = x y
II.16. 1)
( I ). Ta c
x + y = x + y + 2

x y 0, x + y 0
6
6

(I ) 3 x y = x y

( x + y ) 2 = x + y + 2

x y 0, x + y 0

2
( x y ) 1 ( x y ) = 0

x + y = 2

) (
(

x y 0, x + y 0

2
3
( x y ) ( x y ) = 0

2
( x + y ) ( x + y ) 2 = 0

x y 0, x + y 0 x y 0, x + y 0
x=

x = 1

2
x = y
x y = 1

y =1 y = 1

x + y = 2
x + y = 2

2
3 1
Vy, nghim ca h phng trnh cho l (1;1), ( ; ).
2 2

132

x 2 + y 2 = 25 xy
2)
(I )
y ( x + y ) = 10
xy = 25 x 2 y 2
10 y 2 25 + x 2 + y 2 = 0
I

( )

2
2
xy = 10 y
xy = 10 y
x = 15
x = 15

x = 15

2
y 2 + 15 y 10 = 0

x 2 = 15
xy = 10 y

x = 15

2
x = 15
x = 15
xy = 10 y

xy = 10 y
2

2
xy
=
10

y 15 y 10 = 0
x =
x = 15
x = 15

15
55
15
55

+
+
y =
y =
y =

2
2

x =

15
55
15
55

y=
y=


2
2

y =

x = 15

15 + 55
15 + 55
y =

2
2

x = 15
15

15 55
15 55
.
y =

2
2

15

Vy, h phng trnh cho c bn nghim l

15 + 55
15 55
15 + 55
15 55
15;
; 15;
; 15;
; 15;
.
2
2
2
2

x + y = 2(*)
3)
x + 3 + y + 3 = 4

iu kin:

x 0
y 0
x 0

y 0
x + 3 0
y + 3 0

Bnh phng hai v ca cc phng trnh ca h ta c


x + y + 2 xy = 4

x + 3 + y + 3 + 2 ( x + 3)( y + 3) = 16
x + y + 2 xy = 4

x + y + 2 ( x + 3)( y + 3) = 10

Tr v theo v ca hai phng trnh ca h trn ta c

133

( x + 3)( y + 3)

xy = 3

( x + 3)( y + 3) = 3 + xy
( x + 3)( y + 3) = 9 + xy + 6

xy

xy + 3 ( x + y ) + 9 = 9 + xy + 6 xy
x + y 2 xy = 0

x y

=0

x= y
Thay x = y vo ( ) ta c
2 x = 2 x = 1 x = 1. (Tha iu kin)
Vy, h phng trnh cho c mt nghim duy nht l (1;1) .
3 x 2 + xy + 3 x + y = 3
4) 2
(I )
2
2
xy + 2 x + y + 2 x = 2
3 x ( x + 1) + y ( x + 1) = 3
Ta c ( I )
2
2 x ( x + 1) + y ( x + 1) = 2
( x + 1)( 3 x + y ) = 3
(1)

2
( x + 1) 2 x + y = 2 (2)

Ta thy v tri ca cc phng trnh (1) v (2) khc 0 nn chia v theo v ca cc phng
trnh ca h ta c
3x + y 3
=
y2 + 2x 2

2 ( 3x + y ) = 3 y 2 + 2 x

6x + 2 y = 6x + 3 y2
3 y2 2 y = 0
y (3 y 2) = 0

y = 0

y = 2
3

+ Vi y = 0 , thay vo (1) ta c

( x + 1) x = 1
134

x2 + x 1 = 0

1 + 5
x =
2

1 5
x =
2

+ Vi y =

2
, thay vo (1) ta c
3

( x + 1) 3x +

2
=3
3

3x 2 + 3 x +

2
2
x+ =3
3
3

9 x 2 + 11x 7 = 0

11 + 373
x =
18

11 373
x =
18

Vy, h phng trnh cho c bn nghim l


1 + 5 1 5 11 + 373 2 11 373 2
; 0 ,
; 0 ,
; ,
; .

2
18
3
18
3

2

x 2 y 2 2 x 2 + y 3 2 y = y 2
5) 2
2
x + y y 2 = 0
2
2
x = 2 + y y

2
2
2
3
2
2 + y y y 2 2 + y y + y 2 y = y
2
2
x = 2 + y y
4
3
2
y + 2 y + 3 y 4 y 4 = 0

2
2
x = 2 + y y

2
2
( y + 1) ( y 2 ) = 0

x = 0
x2 = 2 + y y 2
x2 = 2 + y y 2

y = 1

( y + 1) = 0 y = 1

x = 0

y = 2
2

( y 2 ) = 0
y = 2
Vy, nghim ca h phng trnh cho l ( 0; 1) v ( 0; 2 ) .
135

x + x 2 2 x + 2 = 3 y 1 + 1
6)
(I )
y + y 2 2 y + 2 = 3x 1 + 1
x 1 + x 2 2 x + 2 = 3 y 1
Ta c ( I )
y 1 + y 2 2 y + 2 = 3x 1
u = x 1
t
th h ( I ) tr thnh
v = y 1
u + u 2 + 1 = 3v (1)
( II )

v + v 2 + 1 = 3u (2)
Ly (1) tr ( 2 ) theo v ta c
u v + u 2 + 1 v 2 + 1 = 3v 3u
u + u 2 + 1 + 3u = v + v 2 + 1 + 3v
f (u ) = f (v) (3)

Xt hm s f (t ) = t + t 2 + 1 + 3t
t

f (t ) = 1 +

t +1

+ 3t ln 3

t2 +1 t

Ta c

t
t 2 +1

f (t ) 0, t .
Do , hm s f (t ) lun lun tng trn ton b .
V vy, ( 3) u = v.
Thay u = v vo (1) ta c
u + u 2 + 1 = 3u

)(

) (

u + u 2 + 1 u u 2 + 1 = 3u u u 2 + 1

1 = 3u u u 2 + 1
3u

u 2 + 1 u = 1(*)

Xt hm s g (u ) = 3u

136

u 2 + 1 u xc nh trn .

g (u ) = 3u ln 3
Ta c
Do

u 2 + 1 u + 3u
1 = 3u
2
u +1

1
u 2 + 1 u ln 3

u2 +1

u2 + 1 u 0

1
u2 + 1

1 m ln 3 > 1 nn ln 3

1
u2 +1

0.

Vy, g (u ) 0.
Suy ra hm s g (u ) lun lun ng bin trn . Do ( ) nu c nghim th c mt
nghim duy nht.
Ta thy u = 0 tha ( ) , nh vy u = v = 0. T ta c x = y = 1.
Vy, h phng trnh cho c nghim duy nht (1;1) .
xy 3 x 2 y = 16
7) 2
(I )
2
x + y 2 x 4 y = 33
(1)
2 xy 6 x 4 y = 32
Ta c ( I ) 2
2
x + y 2 x 4 y = 33 (2)

Cng tng v ca cc phng trnh (1) v (2) ta c


x 2 + 2 xy + y 2 8 x 8 y = 65
2

( x + y ) 8 ( x + y ) 65 = 0

x + y = 13

x + y = 5

x + y = 13 x = 13 y . Thay x = 13 y vo (1), ta c

(13 y ) y 3 (13 y ) 2 y = 16
y 2 14 y + 55 = 0

Phng trnh trn v nghim.


x + y = 5 x = 5 y . Thay x = 5 y vo (1), ta c

( 5 y ) y ( 5 y ) 3 2 y = 16
y2 + 4 y +1 = 0
y = 2 3

y = 2 + 3
Vi y = 2 3 th x = 3 + 3 .
Vi y = 2 + 3 th x = 3 3 .
137

) (

Vy, h phng trnh cho c hai nghim l 3 + 3 ; 2 3 , 3 3; 2 + 3 .


x 3 + y + 2 = 3
8)
x + y = xy + 2 x 3 y 6 + 4
u = x 3
u 0, v 0
t
v = y + 2

Khi h phng trnh cho tr thnh


u + v = 3
2 2
u + v = uv + 3
u + v = 3

2
(u + v ) = 3uv + 3
u + v = 3

uv = 2

u = 1
u = 2

v = 2
v = 1
Vi u = 1, v = 2 , ta c
x 3 = 1
x = 4

y = 2
y + 2 = 2

Vi u = 2, v = 1 , ta c
x 3 = 2
x = 7

y = 1
y + 2 = 1

Vy, h phng trnh c hai nghim l (4; 2), (7; 1).

( x + y ) 2

9)
x y 2+
)
(

1 9
=
xy 2
1 5
=
xy 2

(I )

(1)

x 0
iu kin:
y 0
1 1 9

2 x + 2 y x y = 2

Ta c ( I )
2 x 2 y 1 + 1 = 5
x y 2

138

(1)
( 2)

Cng (1) v (2) theo v ta c


4x

2
1
= 7 4x2 7 x 2 = 0 x = 2 x =
x
4

Thay x = 2 vo (1), ta c 2 y
Thay x =

1
1
1 = 0 2 y2 y 1 = 0 y = 1 y = .
y
2

1
1
1
vo (1), ta c 2 y 1 = 0 2 y 2 y 1 = 0 y = 1 y = .
4
2
y

1 1 1 1

Vy, h phng trnh cho c bn nghim l ( 2;1) , 2; , ;1 , ; .


2 4 4 2

2
x + xy = 2
(I )
10) 3
2
x + 2 xy 2 y = x

Cch 1. Ta c nhn xt rng x = 0 khng tha h phng trnh, do nhn hai v ca


phng trnh th nht ca h ( I ) vi x ta c
x 3 + x 2 y = 2 x
3
2
x + 2 xy 2 y = x
Tr tng v ca hai h phng trnh cho nhau ta c phng trnh
x 2 y 2 xy 2 + 2 y x = 0 ( x 2 y )( xy 1) = 0.

2 3 3 2 3 3
Trng hp x = 2 y, ta c cc nghim
;
;
,
.
3 3
3
3
Trng hp xy = 1, ta c cc nghim (1;1) , ( 1; 1) .

2 3 3 2 3 3
Vy, h phng trnh cho c bn nghim l (1;1) , ( 1; 1) ,
;
;
,
.
3 3
3
3
x 2 1 = 1 xy
(1)
Cch 2. ( I )
2
x ( x 1) = 2 y (1 xy ) (2)

x = 2y
Th (1) vo (2) ta c x (1 xy ) = 2 y (1 xy ) (1 xy )( x 2 y ) = 0
xy = 1

3
y=

1
3
+ Thay x = 2 y vo (1) ta c 4 y 2 1 = 1 2 y 2 y 2 =
3

3
y =
3

2 3 3 2 3 3
T ta c nghim l
;
;
,
.
3 3
3
3
139

x = 1
+ Thay xy = 1 vo (1) ta c x 2 1 = 0
x = 1
T ta c nghim (1;1) , ( 1; 1) .
2 3 3 2 3 3
Vy, h phng trnh cho c bn nghim l (1;1) , ( 1; 1) ,
;
;
,
.
3
3
3
3

x
y
7
+
=
+1

x
xy
11) y
(I )

x xy + y xy = 78

( x + y ) + xy = 7

iu kin: xy > 0. Ta c ( I )
( x + y ) . xy = 78

Suy ra x + y, xy l nghim ca phng trnh bc hai


X = 13
x + y = 13
x + y = 13
X 2 7 X 78 = 0

X = 6
xy = 36
xy = 6
x = 9 x = 4

y = 4 y = 9.
Vy, h phng trnh cho c hai nghim l ( 9; 4 ) , ( 4;9 ) .
x 1 + y 1 = 3
(1)
12)
x + y ( x 1)( y 1) = 5 (2)

iu kin: x 1, y 1. H phng trnh cho tng ng vi

x 1 + 2 ( x 1)( y 1) + y 1 = 9 x + y + 2 ( x 1)( y 1) = 11

x + y ( x 1)( y 1) = 5
x + y ( x 1)( y 1) = 5

11 ( x + y )

11 ( x + y )
( x 1)( y 1) =
( x 1)( y 1) =
2

2
x + y 11 x y = 5
2 ( x + y ) 11 + ( x + y ) = 10

11 ( x + y )
11 ( x + y )

xy ( x + y ) + 1 =
xy ( x + y ) + 1 =

2
2
3 ( x + y ) = 21
x + y = 7

xy 6 = 2
xy = 10
x = 2 x = 5

x + y = 7
y = 5 y = 2
x + y = 7
140

Cc gi tr ca x, y u tha iu kin.
Vy, h phng trnh cho c hai nghim l ( 2;5 ) , ( 5; 2 )
x 2 + 6 y = y + 3
13)
x + y + x y = 4.
x2 + 6 y 0

iu kin: x + y 0
x y 0

H phng trnh cho tng ng vi


y +3 0
y 3
2

x + 6 y = y2 + 6 y + 9
x2 y 2 = 9

2
2
2
2
x + y + x y + 2 x y = 16
x + x y = 8
y 3
y 3
x = 5
2

2
x y = 9 x = 5

y = 4
x = 5
y = 4 y = 4

Vy, h phng trnh cho c mt nghim duy nht l ( 5; 4 ) .


2
a2
2
x
=
y
+

II.17.
(I )
2
a
2 y 2 = x +

x
x > 0
T cc phng trnh ca h ( I ) ta suy ra iu kin ca n l
y > 0
2 x 2 y = y 2 + a 2
(I )
2
2
2
2 xy = x + a

2 x 2 y = y 2 + a2

( x y )( 2 xy + x + y ) = 0

x > 0
ta suy ra 2xy + x + y >0
Do
y > 0
2x 2 y = y 2 + a 2
2x3 x 2 = a 2
Vy, ta c h

x=y
x= y

Xt hm s y = f ( x ) = 2 x 3 x 2 ,
x = 0
y ' = 6 x 2 x, y ' = 0 6 x 2 x = 0
x = 1
3

141

1
1 2 1
Ta c f =
=
27
3 27 9
Lp bng bin thin ca hm s f ( x ) v da vo bng bin thin ta c ng thng y = a 2
ct th f ( x ) ti mt im duy nht c honh x > 0. Vy, h phng trnh cho c
mt nghim duy nht vi m i a 0.
2
2
x 4 xy + y = k
II.18. 2
(I )
y 3 xy = 4

2
2
x 4 xy + y = 1
1) Thay k = 1 vo h ( I ) ta c 2
(2).
y 3 xy = 4

y 2 = 1
(V l).
Vi x = 0, thay vo h (2) ta c 2
y = 4
x 2 xy + y 2 3xy = 1
y 2 = 4 + 3xy
Vi x 0, ta c (2) 2
2
y 3 xy = 4
x xy + 4 = 1
y 2 = 4 + 3xy
2 x 4 + 7 x 2 9 = 0 (*)

x2 + 3
x2 + 3
y
=
y
=

x
x

Gii (*) ta c x = 1 x = 1.
Vi x = 1, th vo y =

x2 + 3
ta c y = 4.
x

Vi x = 1, th vo y =

x2 + 3
ta c y = 4.
x

Vy, khi k = 1 th h phng trnh cho c hai nghim l (1; 4), (1; 4).
2) Cch 1.
x2 = k
Vi y = 0, thay vo (1) ta c
(V l).
0 = 4

2 y 4 ( 40 9k ) y 2 16 = 0 (3)

y2 4

x =
3y
Vi y 0, ta c (1)

y2 4
x
=
x 2 4 xy + y 2 = k

3y

142

Phng trnh trng phng 2 y 4 (40 9k ) y 2 16 = 0 (3) c cc h s a, c tri du nn


chc chn c nghim y 0 vi m i k .
Vy, h phng trnh cho lun lun c nghim vi m i gi tr ca k .
t 2 4t + 1 y 2 = k
Cch 2. T h phng trnh ta c y 0. t x = ty , ta c h
2
(1 3t ) y = 4

Suy ra

3t 2 + 2t 1
t 2 4t + 1 k
t 2 4t + 1
1
= . Xt hm s f (t ) =
, f (t ) =
< 0, t
2
1 3t
4
1 3t
3
(1 3t )

hm s f (t ) lun lun nghch bin, do ng thng y =


hai im phn bit c honh t1 , t2 tha t1 <
y 2 (1 3t1 ) = 4 y =

k
lun ct th y = f (t ) ti
4

1
< t2 . Khi vi t = t1 ta c
3

2
2t1
x=
. (pcm)
1 3t1
1 3t1

x + 1 + y + 2 = a
II.19.
(I )
x + y = 3a
Nu a 0 th h phng trnh cho v nghim nn ta ch xt a > 0. Ta c

x + 1 + y + 3 = a
(I )
(I )
x + 1 + y + 2 = 3a + 3
u = x + 1 0
t
v = y + 2 0

Khi ta c h phng trnh theo bin u, v


u + v = a
v = a u

2 2
2
2
u + v = 3a + 3
f (u ) = 2u 2au + a 3a 3 = 0(*)

H phng trnh cho c nghim khi v ch khi phng trnh (*) c nghim u 0 tha
v = a u 0. Gi s u1 , u2 l hai nghim ca phng trnh (*), th t phng trnh (*) theo
nh l Viet ta c u1 + u2 = a, nh vy nu u1 l nghim ca phng trnh (*) th nghi m
u2 chnh l v v ngc li. T h phng trnh cho c nghim khi v ch khi phng
trnh (*) c hai nghim u1 , u2 u khng m.
iu ny xy ra khi v ch khi

143


3 15 a 3 + 15
= a 2 2(a 2 3a 3) 0

3 + 21
3 21
3 + 21

2
a
a

a 3 + 15
2 f (0) = a 3a 3 0
2
2
2
a

0
a 0

2
4 x y m = 0
II. 20.
(I )
3x + x ( y + 1) = 1

Cch 1.
Ta c
y = 4 x m
y = 4 x m
y = 4 x m
(I )

2
x ( y + 1) = 3x 1 x(4 x m + 1) = 3 x 1 4 x mx + x = 3x 1
y = 4x m
y = 4x m

1
1

x
x
3
3

2
2
4 x mx + x = 9 x 6 x + 1 f ( x ) = 5 x 2 + ( m 7 ) x + 1 = 0(*)

H phng trnh cho c nghim khi v ch khi phng trnh (*) c t nht mt nghim
1
x . iu ny xy ra khi v ch khi phng trnh (*) c nghim x1 , x2 tha
3

1
7

5. f 3 0

0
x1 3 x2
m 7 2 5 m 7 + 2 5

1
1 < x x
m > 7
2
5. f 3 > 0
3 1

11

S 1
m <
2 > 3
3

m 7 2 5.
7 < m 7 2 5
3

Vy, khi m 7 2 5 th h phng trnh cho c nghim.


Cch 2.

y = 4 x m
y = 4 x m
(I )

x ( y + 1) = 3x 1 x(4 x m + 1) = 3 x 1

144

y = 4x m

y = 4 x m
1

x
2
3
4 x mx + x = 3 x 1
2
2
4 x mx + x = 9 x 6 x + 1

y = 4x m
y = 4x m
y = 4x m

1
1
1

x
x
x
3
3
3

2
2
1
5 x 7 x + 1 = mx
5x 7 x + 1

= m

f ( x) = 5 x 7 + x = m
x

1
1 5 x2 1
1
Xt hm s f ( x) = 5 x 7 + , f ( x ) = 5 2 =
, f ( x) = 0 x =
.
2
x
x
x
5
x=

1 1
< (Loi)
5 3

1
1
f
= 7 + 5. Lp bng bin thin ca hm s f ( x ) trn [ 3 ; +),
5
ta c min gi tr T f = [2 5 7; +). T cc gi tr cn tm ca m phi tha

7
1
Ta c f = ;
3
3

m 2 5 7 m 7 2 5.

II.21.

x = y 2 y + m

2
y = x x + m

Ta c nhn xt rng nu ( x0 ; y0 ) l mt nghim ca h phng trnh cho th ( y0 ; x0 )


cng l nghim ca h, do h c nghim duy nht th x0 = y0 . Khi ta c
x0 = x02 x0 + m x02 2 x0 + m = 0(*)

Do x0 duy nht nn phng trnh (*) phi c nghim duy nht. iu ny xy ra khi v ch
khi = 1 m = 0 m = 1.
Vi m = 1, h phng trnh cho tr thnh
2
x = y y + 1

2
y = x x + 1

Cng theo v ca hai phng trnh ca h ta c

x = 1
2
2
x + y = y 2 y + 1 + x 2 x + 1 ( x 1) + ( y 1) = 0
y =1
x = y = 1 tha h phng trnh nn l nghim v l nghim duy nht ca h. Vy, khi m = 1
th h phng trnh cho c nghim duy nht.

145

x 2 5 x + 4 0
II.22. 2
(I )
3 x mx x + 16 = 0
Ta c

1 x 4
(I ) 2
3 x mx x + 16 = 0
t t = x , vi 1 x 4 th 1 t 2. Khi ta c

1 t 2
1 t 2

4
3t 4 + 16
3
3
t

mt
+
16
=
0
m
=

t3

Xt hm s
3t 4 + 16
, 1 t 2
t3
3t 4 48

0, t [1; 2]
f (t ) =
t4

y = f (t ) =

Vy hm s f (t ) nghch bin trn [1; 2]


f (1) = 19, f (2) = 8.
Vy 1 t 2 th 8 f (t ) 19
Vy, cc gi tr ca m cn tm l 8 m 19.
x + 1 + y = m
II.23.
y + 1 + x = 1

iu kin: x 0, y 0

Vi x 0, y 0 ta c y + 1 + x 1. Du bng xy ra khi v ch khi x = y = 0. T


m = 1. Vi m = 1 th h phng trnh cho c nghim. Vy, m = 1 l gi tr cn tm.
x + xy + y = m + 1
II.24. 2
(I )
2
x y + xy = m

S = x + y
vi S 2 4 P 0.
t
P
=
xy

S + P = m + 1 P = m P = 1
Khi ( I ) tr thnh

SP = m
S = 1 S = m
Suy ra ( x; y ) l nghim ca phng trnh
X 2 X + m = 0 ( a ) hoc X 2 mX + 1 = 0 ( b )

146

1) Vi m = 2

(a) X 2 X + 2 = 0

(Phng trnh v nghim)

x = 1
y =1

(b) X 2 2 X + 1 = 0 X = 1

Vy, vi m = 2 h phng trnh cho c nghim l (1;1) .


2) H phng trnh cho c t nht mt nghim ( x; y ) tha mn x > 0; y > 0 khi v ch khi
phng trnh ( a ) hoc phng trnh ( b ) c hai nghim dng.
+ Phng trnh ( a ) c hai nghim dng khi v ch khi

= 1 4m 0
1

1
1

m
4 0<m .
X1 + X 2 = > 0
1
4

m > 0
X1 X 2 = m > 0
+ Phng trnh ( b ) c hai nghim dng khi v ch khi

= m2 4 0
m 2 m 2

m 2.
X1 + X 2 = m > 0
m
>
0

X X =1 > 0
1 2
Vy, vi m 2 0 < m

1
th h phng trnh cho c t nht mt nghim ( x; y )
4

tha x > 0; y > 0.

m x 2 + 1 + y 2 = m + 1
II.25.
(I )
2
2
x
+
my
=
1

x2 +1 + y2 = 2
x2 + y 2 = 1
1) Khi m = 1 h ( I ) tr thnh 2
x 2 + y 2 = 1.

2
2
2
x
+
y
=
1
x
+
y
=
1

Vy, tp hp nghim ca h phng trnh ( I ) ng vi m = 1 l

{( x; y ) / x

+ y 2 = 1 (Tp

hp nhng cp ( x; y ) l ta ca nhng im nm trn ng trn n v).


m ( x 2 + 1) + y 2 = m + 1(1)
2) 2
2
( 2)
x + my = 1

T phng trnh ( 2 ) x 2 = 1 my 2

Th vo phng trnh (1) ta c m 2 my 2 + y 2 = m + 1


2m m2 y 2 + y 2 = m + 1

147

1 m 2 y 2 = 1 m (3)
Xt 1 m 2 = 0 m = 1 m = 1.

Vi m = 1, h phng trnh ( I ) c v s nghim. Tp hp nghim ca h phng trnh ( I )


l

{( x; y ) / x

+ y 2 = 1 . Vi m = 1, (3) tr thnh 0 y 2 = 2 phng trnh (3) v nghim

t h phng trnh ( I ) v nghim.


Xt 1 m 2 0 m 1
1 m
. H phng trnh ( I ) c nghim khi v ch khi
1 m2

2 m 1
m > 1
y = 2
0

m > 1

m 1
m 1

m 1
x 2 = 1 m. m 1 0
m 1
2
2
0
m 1

m 1

(3) y 2 =

Vy, kt hp cc trng hp xt th gi tr cn tm l m > 1.


Ch . Chng ta c th gii cch khc nh sau
t X = x 2 0, Y = y 2 0. Khi ta c h phng trnh

mX + Y = 1
( II )

X + mY = 1
D=

m 1
1 1
m 1
= m 2 1, DX =
= m 1, DY =
= m 1.
1 m
1 m
1 1

m = 1
+ D=0
m = 1
m = 1 . H phng trnh ( II ) c v s nghim nn h phng trnh ( I ) c v s nghim.
m = 1. H phng trnh ( II ) v nghim nn h phng trnh ( I ) v nghim.

+ D 0 m 1.
H phng trnh ( I ) c nghim khi v ch khi h phng trnh ( II ) c nghim
( X ; Y ) / X > 0, Y > 0.
DX
1

X
=
=
>0

D m +1

m > 1.
D
1
Y
Y =
=
>0

D m +1
Vy, kt hp cc trng hp xt th gi tr cn tm l m > 1.

148

x 2 my 2 + 2 x + 2 m = 0
II. 26.
(I )
2
2
m ( x + 2 x + 2 ) y = m + 2

1) Khi m = 1 th h phng trnh cho tr thnh


2
2
x 2 + y 2 + 2 x + 3 = 0
x + y + 2 x + 3 = 0
2

2
2
2
x + 2 x + 2 y = 1 x + y + 2 x + 3 = 0

x 2 + y 2 + 2 x + 3 = 0 ( x + 1) + y 2 + 1 = 0.

Vy, h phng trnh trn v nghim.


2) Ta c

x 2 + 2 x + 2 = m ( y 2 + 1)
( I ) 2
2
y + 2 = m ( x + 2 x + 1)
( x + 1)2 m ( y 2 + 1) = 1

2
2
m ( x + 1) ( y + 1) = 1
u = ( x + 1) , u 0
t
2
v = y + 1, v 1
2

Khi ta c h phng trnh bc nht i vi hai n u, v

u mv = 1
( II )

mu v = 1
Ta c
D=

1 m
1 m
1 m
= m 2 1, Du =
= 1 + m, Dv =
= 1+ m
m 1
1 1
1 1

m = 1
+ Nu D = 0
m = 1
Vi m = 1 th theo Cu 1) h phng trnh v nghim.

u v = 1
Vi m = 1 th ( I ) tr thnh
, h phng trnh v nghim.
u v = 1
+ Nu D 0 m 1 th
Du
m +1

u
=
u
=

1
m2 1
D
u=v=
.

D
m
+
1
m

1
v
v =
v =

D
m2 1

149

H phng trnh ( I ) c nghim khi v ch khi h phng trnh ( II ) c nghim


(u; v), u 0, v 1. Ta phi c
1
1 1 < m 2. Vy, khi 1 < m 2 th h phng trnh cho c nghim.
m 1
2
x( x + 2)(2 x + y ) = 9
( x + 2 x )(2 x + y ) = 9
II.27. 2
( I ) . Ta c ( I ) 2
( x + 2 x ) + (2 x + y ) = m
x + 4x + y = m

u = x 2 + 2 x 1
uv = 9
t
. Khi ta c h phng trnh theo n u, v :
u + v = m
v = 2 x + y

Theo nh l Viet th u, v l hai nghim ca phng trnh f ( X ) = X 2 mX + 9 = 0. (*)


Nh vy h phng trnh ( I ) c nghim khi v ch khi phng trnh (*) c t nht mt
nghim ln hn hoc bng 1.
Yu cu ca bi ton c tha khi v ch khi phng trnh (*) c hai nghim X1 , X 2 tha

f (1) 0
m + 10 0

m 10

X1 1 X 2
m 6 m 6
m 10
0
m 36 0

1 < X X f (1) > 0 m + 10 > 0 m > 10

m 6

1
2

S
m

m > 2

1
<

1
<


2
2
Vy, cc gi tr cn tm ca tham s m l m 10 m 6.
x + 1 + y 2 = m
II.28.
. iu kin:
x 2 + y + 1 = m

x 2

y2
m 0

Tr tng v ca hai phng trnh ca h cho ta c


x +1 x 2 =

f (t ) =

y + 1 y 2(*). Xt hm s f (t ) = t + 1 t 2, t 2. Ta c

1
1
t 2 t +1

=
< 0, t > 2.
2 t +1 2 t 2 2 t +1 t 2

Do , hm s f (t ) nghch bin trn [2; +). V vy, () x = y.


Vi x = y, ta c
h' ( x ) =

x + 1 + x 2 = m . Xt hm s h( x) = x + 1 + x 2

1
1
+
> 0, x > 2 hm s h( x) ng bin trn [2; +).
2 x +1 2 x 2

Ta c h(2) = 3, lim h( x ) = +, suy ra min gi tr Th ca hm s h( x) trn [2; +) l


x +

[ 3; +). T gi tr ca tham s m cn tm phi tha

150

m 3 m 3.

CHNG III.

BT NG THC BT PHNG TRNH

III.1. 1) a) a 2 + b 2 + 1 ab + a + b (1)
(1)

a2
b2 a 2
1 b2
1
ab + + a + + b + 0
2
2
2
2 2
2

1 2
a 2ab + b 2 + a 2 2a + 1 + b 2 2b + 1 0

2
1
2
2
2
( a b ) + ( a 1) + ( b 1) 0

) (

) (

a b = 0

(pcm). Du = xy ra khi v ch khi a 1 = 0 a = b = 1.


b 1 = 0

b) a 2 + b2 + 4 ab + 2 ( a + b ) (2)
(2)

a2
b2 a2
b2
ab + + 2a + 2 + 2b + 2 0
2
2 2
2

1 2
a 2ab + b 2 + a 2 4a + 4 + b 2 4b + 4 0
2
1
2
2
2
( a b ) + ( a 2 ) + ( b 2 ) 0.

) (

) (

a b = 0

(pcm). Du = xy ra khi v ch khi a 2 = 0 a = b = 2.


b 2 = 0

c)

a2
+ b 2 + c 2 ab ac + 2bc (3)
4

a2
a ( b c ) + c 2 2bc + b 2 0
4
a (b c )
a2
2

2
+ (b c ) 0
4
2

(3)

b + c 0.
2

(pcm).
Du = xy ra khi v ch khi
2)

a
b + c = 0 a = 2 (b c ) .
2

x 2 + xy + y 2 + y 2 + yz + z 2 + z 2 + zx + x 2 3 ( x + y + z ) .

Ta c

2
2
( x + y ) 3 (1)
x+ y x y
x 2 + xy + y 2 = 3
+


2
2 2

151

Tng t
y 2 + yz + z 2

( y + z)

(2), z 2 + zx + x 2

( z + x)

(3)

Cng cc bt ng thc (1), (2), (3) theo v ta c


x 2 + xy + y 2 + y 2 + yz + z 2 + z 2 + zx + x 2 3 ( x + y + z ) . (pcm)

1 1 1 3 ( a b )( b c )( c a )
3) ( a + b + c ) + + +
9. (1)
abc
a b c
Ta c (1) tng ng vi

a b
b c
c a
3 ( a b )( b c )( c a )
0
+ 2 + + 2 + + 2 +
abc
b a
c b
a c

c ( a b ) + a ( b c ) + b ( c a ) + 3 ( a b )( b c )( c a ) 0.(*)

Ta s dng kt qu: x 3 + y 3 + z 3 3 xyz = ( x + y + z ) x 2 + y 2 + z 2 xy yz zx

Vi x + y + z = 0 x 3 + y 3 + z 3 = 3xyz. Vi x = a b; y = b c; z = c a.
2

(*) c vit li ( a b ) ( c + a b ) + ( b c ) ( a + b c ) + ( c a ) ( b + c a ) 0. (**)


Do a, b, c l di ba cnh ca mt tam gic nn (**) ng. Vy, ta c iu phi chng
minh.
2

4) ( x + y ) +

x+ y
2x y + 2 y x.
2
2

1
1

Ta c x 0 x x + 0(1)
2
4

Tng t
y y +

1
0(2)
4

Cng cc bt ng thc (1), (2) theo v ta c


x+ y

x+ y +

1
x+ y
2
0 ( x + y) +
( x + y)
2
2
2

M x + y 2 xy , nn (3) ( x + y ) +
III.2. 1)

152

x + y (3)

x+ y
2 x y + 2 y x . (pcm)
2

a b
c
1 1 1
+ +
2 + (1)
bc ca ab
a b c

V a, b, c > 0 nn (1) tng ng vi

a 2 + b 2 + c 2 2 ( bc + ac ab )
a 2 + b 2 + c 2 2bc 2ac + 2ab 0
2

(a + b c) 0

Bt ng cui lun ng nn bt ng thc (1) ng.


2) 1 <

a
b
c
d
+
+
+
<2
a+b+c b+c+d c+d +a a +b+d

+ Trc ht ta chng minh


V d > 0 nn

a
b
c
d
+
+
+
> 1(*)
a+b+c b+c+d c+d +a a+b+d

a
a
>
(1)
a+b+c a +b+c+d

Tng t, v a, b, c > 0 ta cng c


b
b
>
(2)
b+c+d a+b+c+d
c
c
>
(3)
c+d +a a+b+c+d
d
d
>
(4)
a+b+d a +b+c+d

T (1), (2), (3), (4)


+ Ta chng minh

a
b
c
d
a+b+c+d
+
+
+
>
= 1 (pcm).
a+b+c b+c+d c+d +a a +b+d a+b+c+d

a
b
c
d
+
+
+
< 2(**)
a+b+c b+c+d c+d +a a+b+d

Trc tin ta chng minh bt ng thc sau


x, y , z > 0 v y > x ta u c

x x+z
<
(*)
y y+z

Tht vy
(*) x( y + z ) < y ( x + z )
( y x) z > 0 lun ng v y > x ( y x) > 0
p dng bt ng thc (*) ta c
Vi b ba s a, (a + b + c ), d ta c

a
a+d
<
(1)
a+b+c a+b+c+d

Tng t ta cng c

153

b
b+a
(2)
<
b+c+d a+b+c+d
c
c+b
(3)
<
c+d +a a+b+c+d
d
d +c
<
(4)
a+b+d a+b+c+d
T (1), (2), (3), (4)

a
b
c
d
2(a + b + c + d )
+
+
+
<
=2
a+b+c b+c+d c+d +a a+b+d
a+b+c+d

(pcm).
Vy, ta c 1 <

a
b
c
d
+
+
+
< 2.
a+b+c b+c+d c+d +a a +b+d

Ch . C th chng minh bt ng thc (**) n gin hn nh sau


Ta c
a
a
b
b
<
,
<
a+b+c a+c b+c+d b+d
c
c
d
d
<
,
<
c+d +a c+a d +a +b d +b

a
b
c
d
a+c b+d
+
+
+
<
+
= 1 + 1 = 2.
a+b+c b+c+d c+d +a d +a +b c+a b+d

3) Ta c
a
a
2a
<
<
(1)
a+b+c b+c a+b+c
2b
b
b
<
<
(2)
a+b+c a+c a +b+c
c
c
2c
<
<
(3)
a+b+c a+b a+b+c
Cng cc bt ng thc (1), (2), (3) theo v ta c iu phi chng minh.
III.3. 1) a) (a + b)(ab + 1) 4ab(a, b > 0);

p dng bt ng thc Csi cho hai s dng a, b, ta c a + b 2 ab (1).


Tng t, p dng bt ng thc Csi cho hai s dng ab,1, ta c
ab + 1 2 ab.1 = ab (2).
Nhn cc bt ng thc (1) v (2) v theo v ta c
(a + b)(ab + 1) 2 ab .2 ab = 4ab (pcm).

b) (a + b)(b + c)(c + a) 8abc (a, b, c > 0)


p dng bt ng thc Csi cho hai s dng a, b, ta c a + b 2 ab (1) .

154

Tng t, ta c cc bt ng thc b + c 2 bc (2), c + a 2 ca (3) .


Nhn cc bt ng thc (1), (2), (3) v theo v ta c
(a + b)(b + c)(c + a) 8 a 2b 2 c 2 = 8abc (pcm).
c) a 2 (1 + b 2 ) + b2 (1 + c 2 ) + c 2 (1 + a 2 ) 6abc
a 2 + a 2b 2 + b 2 + b 2c 2 + c 2 + c 2 a 2 6abc
p dng bt ng thc Csi cho su s dng a 2 , a 2b 2 , b 2 , b 2c 2 , c 2 , c 2 a 2 ta c
a 2 + a 2b 2 + b 2 + b 2 c 2 + c 2 + c 2 a 2 6 6 a 2 .a 2b 2 .b 2 .b2 c 2 .c 2 .c 2 a 2
a 2 + a 2b 2 + b 2 + b 2 c 2 + c 2 + c 2 a 2 6 6 a 6 .b6 .c 6
a 2 + a 2b 2 + b 2 + b 2c 2 + c 2 + c 2 a 2 6abc (pcm).
2) Theo bt ng thc Csi ta c
a2
b+c
a2 b + c
+
2
.
= a(1)
b+c
4
b+c 4
Tng t
b2
c+a
+
b(2)
c+a
4
c2
a+b
+
c(3)
a+b
4
Cng cc bt ng thc (1), (2), (3) theo v ta c iu phi chng minh.
III.4. 1) a) p dng bt ng thc Bunhiacpski cho hai cp s ( u; x ) v ( v; y ) ta c
ux + vy

(u

+ v 2 )( x 2 + y 2 ) 1 1 ux + vy 1 (pcm).

b) p dng bt ng thc Bunhiacpski cho hai cp s ( u; v ) , ( x + y; x y ) ta c


u ( x + y) + v ( x y)
2)

(u

+ v2

) ( x + y ) + ( x y ) 2 ( x
2

+ y 2 2 (pcm).

x2
y2
z2
3
+
+
(1)
y+ z z+ x x+ y 2

x2
y2
z2
t A =
+
+
.
y+ z z+ x x+ y
p dng bt ng thc Bunhiacpski cho hai b s

x
y
z
y + z ; z + x; x + y ,
;
;
y + z z + x x + y

Ta c
155

( x + y + z)
A

( y + z + z + x + x + y) A

x+ y+z 3 3
3
. xyz = .
2
2
2

Vy, (1) ng. Ta c iu phi chng minh. Du = xy ra khi v ch khi x = y = z = 1.


3)

1
1
1
1
+
+ +
30. (1)
2
2
a + b + c ab bc ca
2

t A =

1
1
1
1
+
+ + .
2
2
a +b +c
ab bc ca
2

p dng bt ng thc Bunhiacpski cho hai b s

1
1
1
1
;
;
;
2

2
2
ab bc ca
a +b +c

a 2 + b 2 + c 2 ;3 ab ;3 bc ;3 ca

Ta c

(1 + 3 + 3 + 3)

( a 2 + b 2 + c 2 + 9ab + 9bc + 9ca ) A

100 ( a + b + c ) + 7 ( ab + bc + ca ) A(*)

M ab + bc + ca

1
1
2
(a + b + c) = .
3
3

Do A 30. (pcm)
1
Du = xy ra khi v ch khi a = b = c = .
3

III.5. 1) p dng bt ng thc Bunhiacpski cho hai b ba s

( x; a; b ) , ( x; x;1)
Ta c

(x

+ ax + b ) ( x 2 + a 2 + b 2 )( x 2 + x 2 + 1)

(1)

Tng t ta cng c ( x 2 + cx + d ) ( x 2 + c 2 + d 2 )( x 2 + x 2 + 1) (2)


Cng (1) v (2) theo v, ta c

(x

+ ax + b ) + ( x 2 + cx + d ) ( x 2 + a 2 + b 2 + x 2 + c 2 + d 2 )( x 2 + x 2 + 1)
2

( x 2 + ax + b ) + ( x 2 + cx + d ) ( x 2 + x 2 + 1)( x 2 + x 2 + 1)
2

( x 2 + ax + b ) + ( x 2 + cx + d ) ( 2 x 2 + 1) .

156

2) T gi thit abc = ab + bc + ca

1 1 1
+ + = 1.
a b c

p dng bt ng thc Bunhiacpski cho hai b ba s

1 1 1
a ; 2b ; 3c ,
;
;

a b c

Ta c

1 + 1 + 1

a2
b2
c2
1
1
1 1 1
= ( a + 2b + 3c ) + + = a + 2b + 3c

.
a + 2b + 3c 1 + 2 + 3 2
a b c

(1 +

2+ 3

a 2 +

( 2b )

( 3c )

i vai tr ca a, b, c v cng cc bt ng thc ta c iu phi chng minh.


III.6. 1) (sin 2 x +

1 2
1 2 25
) + (cos 2 x +
)
2
sin x
cos 2 x
2

p dng bt ng thc Bunhiacpski cho hai cp s

(1;1) , ( sin 2 x +

1
1
; cos 2 x +
) ta c
2
sin x
cos 2 x

2
2

1 2 2
1 2
1
1
2
2
(1 + 1 ) (sin x + sin 2 x ) + cos x + cos2 x 1. sin x + sin 2 x + 1. cos x + cos 2 x

2
2
2

1
1 2
1
1
2
2 sin 2 x + 2 + cos 2 x +

+
+
+
sin
x
cos
x

sin x
cos 2 x
sin 2 x
cos 2 x

1
1 2
1
1 2

2 (sin 2 x + 2 )2 + (cos 2 x +
) (1 + 2 +
)
2
sin x
cos x
sin x cos 2 x

2
2
2

1
1
4
2 sin 2 x + 2 + cos 2 x +

+
1

sin x
cos 2 x sin 2 2 x

(*)

Mt khc do sin 2 2 x 1
1+

4
sin 2 2x

1+ 4
2

1 + 2 (1 + 4)2
sin 2x

Khi (*) tng ng vi


2
2
2
1 2
1
2
2 sin x + 2 + cos x +
(1 + 4 ) = 25
2
sin x
cos x

157

1
1 25

sin 2 x + 2 + cos 2 x +

sin x
cos 2 x
2

1
1 25

Vy sin 2 x + 2 + cos 2 x +
.
sin x
cos 2 x
2

Du ng thc xy ra khi
1
1
2
2
sin x + 2 = cos x +
sin x
cos 2 x

sin 2 2 x = 1

1
1
2
2
sin x + 2 = cos x +

sin x
cos 2 x
sin 2 x = 1
H ny c nghim, chng hn x =

+ k , k .

2) Ta c v tri ca bt ng thc cho bng


1
1 x y 11 1

A = x+ + y+
+ + + + +2
2x
2y y x 2 x y

p dng bt ng thc Csi ta c


1
1 x y 11 1

A = x+ + y+
+ + + + +2
2x
2y y x 2 x y

x y
1
1
x+
2; y +
2; + 2
2x
2y
y x
11 1
1
=
+
2 x y
xy

1
4

x2 y 2

2
= 2.
x + y2
2

Do A 4 + 3 2.
ng thc xy ra khi v ch khi x = y =
III.7. 1)

Ta c

2
.
2

tan 2 x cot 2 x
1
+
1

2 + 1 2 + 1 (1).
x2
x
x

1
tan x.cot x
+1 =
+1
2
x
x2

tan x cot x
p dng bt ng thc Bunhiacopski cho hai cp s
;1 ,
;1 . Ta c
x
x

158

tan x cot x
.
+ 1.1
x
x

Vy,

tan x 2 cot x 2

+ 1
+ 1
x
x

tan 2 x cot 2 x
1
+
1

2 + 1 2 + 1
x2
x
x

tan 2 x cot 2 x
1
+
1

2 + 1 2 + 1 (pcm).
x2
x
x

iu kin ng thc xy ra

tan x cot x
=
x
x
tan x = cot x
x=
2)

+ k , k .

b 2 + 2a 2
c 2 + 2b 2
a 2 + 2c 2
+
+
3 (1).
ab
bc
ca

Ta c ab + bc + ca = abc

1 1 1
+ + = 1.
a b c

(1) tng ng vi
1
2
1 2
1 2
+ 2 + 2 + 2 + 2 + 2 3.
2
b a
c b
a c

1 1 1
1 1 1
1 1 1
t u = ; ; , v = ; ; , w = ; ; . Khi
b a a
c b b
a c c



1 1 1 1 1 1 1 1 1
u +v +w= + + ; + + ; + +
b c a a b c a b c

u+v +w=

1 2
1 2
1 2
+ 2 + 2+ 2 + 2+ 2
2
b a
c b
a c
2

1 1 1 1 1 1 1 1 1
1 1 1
u + v + w = + + + + + + + + = 3 + + = 3.
b c a a b c a b c
a b c

Mt khc ta lun c tnh cht u + v + w u + v + w , t suy ra iu phi chng minh.

u = k .v ; k > 0

Du = xy ra khi v ch khi v = m.w; m > 0


a = b = c = 3.
ab + bc + ca = abc

3)

x2 +

1
1
1
+ y 2 + 2 + z 2 + 2 82.
2
x
y
z

159

Gi S = x 2 +

1
1
1
+ y2 + 2 + z2 + 2
2
x
y
z

Cch 1.

1
p dng bt ng thc Bunhiacopski cho hai cp s (1;9 ) , x;
x
Ta c x +

9
1
1
1 + 81 x 2 + 2 = 82 x 2 + 2 (1)
x
x
x

Tng t
y+

9
1
82 y 2 + 2 (2)
y
y

z+

9
1
82 z 2 + 2 (3)
z
z

Cng cc bt ng thc (1), (2), (3) theo v ta c


1 1 1
1 1 1
S . 82 x + y + z + 9 + + = 81( x + y + z ) + 9 + + 80 ( x + y + z )
x y z
x y z
1 1 1
2 81( x + y + z ) .9 + + 80 = 54
x y z

54 3 3 xyz .3 3

1 1 1
+ + 80
x y z

( x + y + z ).

1
80 = 162 80 = 82
xyz

S 82.

Cch 2.
Trong mt phng Oxy, ta xt cc vc t

1
a = x; a = x 2 + 2
x
x

1
b = y; b = y 2 + 2
y
y

1
c = z; c = z 2 + 2
z
z

1 1 1
a + b + c = x + y + z; + + a + b + c =
x y z

Ta c a + b + c a + b + c

160

( x + y + z)

1 1 1
+ + +
x y z

1
1
1
S = x + 2 + y2 + 2 + z2 + 2
x
y
z
2

( x + y + z)

1 1 1
+ + +
x y z

p dng bt ng thc Csi i vi ba s dng

1 1 1
, , ta c
x y z

1 1 1
+ +
x y z

(x + y + z)

3
3

xyz

3
9
=
S
x+ y+ z x+ y+ z
3

t t = ( x + y + z ) , 0 < t 1. Ta c S t +

Xt hm s f (t ) = t +

81

(x + y + z)

81
.
t

81
81 t 2 81
< 0, t (0;1].
, t (0;1]; f (t ) = 1 2 =
t
t
t2

Lp bng bin thin ca hm s f (t ) trn (0;1] ta c f (t ) 82 S 82. (pcm)


4) Cho x, y, z > 0 v tha xyz = 1. Ta chng minh
1 + x3 + y 3
1 + y3 + z 3
1 + z 3 + x3
+
+
3 3.
xy
yz
zx

t A =

1 + x3 + y 3
1 + y3 + z3
1 + z 3 + x3
+
+
.
xy
yz
zx

Theo bt ng thc Csi ta c


3 3 x3 y3
1 + x3 + y 3

=
xy
xy

3
(1)
xy

Tng t
1 + y3 + z 3

yz

3
(2)
yz

1 + z 3 + x3

zx

3
(3)
zx

Cng cc bt ng thc (1), (2), (3) theo v ta c


A

3
+
xy

3
3
1
+
3 33
= 3 3. (pcm)
xyz
yz
zx

ng thc xy ra khi v ch khi x = y = z = 1.

III.8. 1) x 2 1 + sin 2 y + 2 x ( sin y + cos y ) + 1 + cos 2 y > 0 (1)


2

(1) tng ng vi ( x sin y + 1) + ( cos y + x ) > 0


161

Nh vy ta lun c
2
2
x 2 1 + sin 2 y + 2 x ( sin y + cos y ) + 1 + cos 2 y = ( x sin y + 1) + ( cos y + x ) 0, x, y .

Ta chng minh du = khng xy ra.


Tht vy, gi s du = xy ra. Khi

x sin y + 1 = 0 (1)

cos y + x = 0 ( 2 )
Ly (1) tr cho (2) theo v ta c x sin y cos y + 1 x = 0 ( )
Phng trnh ( ) c nghim y khi v ch khi
2

x 2 + 1 ( x 1) x 2 + 1 x 2 2 x + 1 x 0
Ly (1) cng vi (2) theo v ta c x sin y + cos y + 1 + x = 0 ()
2

Phng trnh (**) c nghim y khi v ch khi x 2 + 1 ( x + 1) 2 x 0 x 0.


Suy ra x = 0, thay x = 0 vo (1) ta c 1 = 0 (V l ). Suy ra du '' = '' khng th xy ra.

Vy, x 2 1 + sin 2 y + 2 x ( sin y + cos y ) + 1 + cos 2 y > 0, x, y .


2) Bt ng thc cho tng ng vi
b

(1 + 4 ) (1 + 4 )
a

Xt hm s f ( x) =

ln 1 + 4a
a

ln 1 + 4 x
x

) ln (1 + 4 )
b

) , x > 0 f ( x) = 4

) (
(1 + 4 )

ln 4 x 1 + 4 x ln 1 + 4 x
x2

) <0

Suy ra hm s f ( x ) nghch bin trn khong (0; +). Do f ( x ) nghch bin trn khong
(0; +) v a b > 0 nn f (a) f (b) t ta c iu phi chng minh.
3) Xt hm s f ( x) = 2sin x + tan x 3 x.
Ta c

( cos x 1) 2 cos 2 x cos x 1


1
f ( x) = 2cos x +
3 =
cos 2 x
cos 2 x
2

( cos x 1) ( 2 cos x + 1) > 0, x 0;


=
cos 2 x


Suy ra hm s f ( x ) = 2sin x + tan x 3 x ng bin trn khong 0; .
2

Do f ( x ) > f (0) = 0, x 0; . (pcm)
2
162

4) Xt hm s f ( x) = x sin x, x > 0. Ta c f ( x) = 1 cos x 0 f ( x) ng bin trn


khong ( 0; + ) . T f ( x) > f (0) = 0 hay sin x < x, x > 0(1).
Xt hm s g ( x ) = sin x +
Ta c g ( x ) =

x3
x, x > 0.
6

x2
+ cos x 1, g ( x ) = x sin x > 0 g ( x ) ng bin trn khong ( 0; + ) .
2

Suy ra g ( x) > g (0) = 0. T y ta c g ( x) ng bin trn khong ( 0; + ) . V vy


g ( x ) > g (0) = 0 hay x

x3
< sin x (2). T (1) v (2) ta c iu phi chng minh.
6

III.9. 1) a 2 ( b + c a ) + b 2 ( a + c b ) + c 2 ( a + b c ) 3abc (1)

Khng mt tnh tng qut ta gi s a b c 0.


Ta c
a 2 ( b + c a ) + b 2 ( a + c b ) + c 2 ( a + b c ) 3abc
a 2b + a 2c a 3 + b 2 a + b 2c b3 + c 2 a + c 2b c 3 3abc 0
a 2b a 2c + a 3 b 2 a b 2c + b3 c 2 a c 2b + c3 + 3abc 0

) (

) (

) (

) (

a 3 + a 2b a 2 c + 2a 2b 2b 2 a + 2abc + b 2 a + b3 b 2c + abc c 2 a + c 2b + c 3 0
a 2 ( a + b c ) 2ab ( a + b c ) + b2 ( a + b c ) + ac ( b c ) c 2 ( b c ) 0

(
(a

)
(
)
2ab + b ) ( a + b c ) + c ( a c )( b c ) 0

a 2 2ab + b 2 ( a + b c ) + ac c 2 ( b c ) 0
2

( a b ) ( a + b c ) + c ( a c )( b c ) 0.
Vy, bt ng thc (1) lun ng vi m i a, b, c khng m.
2) t v tri ca bt ng thc cho l A v ta t
x = 2a + b + c; y = 2b + c + a; z = 2c + a + b x + y + z = 4 ( a + b + c ) . Khi ta c

4.A =

3x ( y + z )
x

3y ( z + x)

x y z
= 9 + + +
y x x

3z ( x + y )
z

x z y
+ + 9 6 = 3
z y z

3
A . (pcm).
4
3

3) ( x + y ) + ( x + z ) + 3 ( x + y )( x + z )( y + z ) 5 ( x + z ) (1)
t a = x + y , b = x + z , c = y + z. iu kin x ( x + y + z ) = 3 yz tr thnh c 2 = a 2 + b 2 ab.

163

Bt ng thc (1) tng ng vi a3 + b3 + 3abc 5c 2 (2) . Ta c


2

c 2 = a 2 + b 2 ab = ( a + b ) 3ab ( a + b )

3
1
2
2
(a + b) = ( a + b )
4
4

a + b 2c(3).

(2) ( a + b ) a 2 + b 2 ab + 3ab 5c 3
( a + b ) c 2 + 3abc 5c 3
( a + b ) c + 3ab 5c 2 (4)
Mt khc t (3) cho ta ( a + b ) c 2c 2 v 3ab

3
2
( a + b ) 3c 2 . T y suy ra (4) ng.
4

Vy, bt ng thc (1) ng. Du ng thc xy ra khi a = b = c x = y = z.


n

a b
III.10. 1) 1 + + 1 + 2n+1 (1)
b a
n

a
b
p dng bt ng thc Csi cho hai s dng 1 + v 1 +
b
a
n

a b
a
Ta c 1 + + 1 + 2 1 +
b a
b

b
1 +
a

n
n
n
n

2
2
a
b
a b

1 + + 1 + 2 1 + 1 +
b a
b a

a b 2
a b
1 + + 1 + 2 1 + 1 +
b a
b a
n

a b
a b ab 2
1 + + 1 + 2 1 + + +
b a
b a ba
n

n
a b 2
a b

1 + + 1 + 2 2 + + 2 ( 2 + 2 ) 2 = 2 n+1.
b a
b a

a n b
1 + = 1 +

Du " = " xy ra khi v ch khi b a a = b.


a b
b = a , (a, b > 0)

Vy, (1) lun ng.


n

an + bn
a +b
2)

(1)

2
2

164

Ta chng minh bng quy np


Vi n = 1: Ta c

a+b a +b

. Vy, (1) ng vi n = 1.
2
2
k

a k + bk
a +b
Gi s (1) ng vi n = k , k , tc l
, ta chng minh (1) ng vi

2
2
*

a +b
n = k + 1, ngha l phi chng minh

k +1

a k +1 + b k +1
.
2

Tht vy:
k +1

k
k
a +b
a+b a+b a +b a +b

=
.
2
2
2
2
2
k +1
k
k
k +1
k +1
k +1
a + ab + a b + b
a +b
ab k + a k b a k +1 b k +1

=
+
4
2
4
k
k
a k +1 + b k +1 ( a b ) ( a b ) a k +1 + b k +1

.
2
4
2

(do ( a b ) a k b k 0 ). Vy, ta c iu phi chng minh.


III.11. 1)

1
1
1
1
+ 3 3
+ 3

(*)
3
3
a + b + abc b + c + abc c + a + abc abc
3

Ta c

(a b)

0 a 2 + b 2 ab ab ( a + b ) ( a 2 + b 2 ab ) ab ( a + b )

a 3 + b 3 ab ( a + b ) a 3 + b3 + abc ab ( a + b + c )

1
1

(1)
3
a + b + abc ab ( a + b + c )
3

Tng t
1
3

b + c + abc
1
3

c + a + abc

1
( 2)
bc ( a + b + c )

1
( 3)
ca ( a + b + c )

Cng v theo v cc bt ng thc (1) , ( 2 ) , ( 3) ta c


1
1
1
1
1
1
+ 3 3
+ 3

+
+
3
3
a + b + abc b + c + abc c + a + abc ab ( a + b + c ) bc ( a + b + c ) ca ( a + b + c )
3

1
1 1
1
1
1
a+b+c
.
+ + =

=
a + b + c ab bc ca a + b + c abc abc

Vy, bt ng thc (*) ng.


2)

a2
b2
c2
a +b+c
+
+

(**)
b+c a+c a+b
2
165

Cch 1. Theo bt ng thc Bunhiacopxki ta c


2

b
c
a

b+c +
a+c +
a+b
(a + b + c) =
a+c
a+b
b+c

a2
a2
b2
c2
b2
c2

+
+
b
+
c
+
a
+
c
+
a
+
b
=
2
+
+
(
)

(a + b + c)
b+c a+c a +b
b+c a+c a +b
2

(a + b + c) = (a + b + c) .
a2
b2
c2

+
+

b + c a + c a + b 2 (a + b + c)
2
Cch 2.
Theo bt ng thc Csi ta c
a2
b+c
a2 b + c
+
2

=a
4
b+c
b+c 4
b2
a+c
b2 a + c
+
2

=b
4
a+c
a+c 4
c2
a+b
c2 a + b
+
2

=c
4
a+b
a +b 4
Cng v theo v ta c
a2
b2
c2
a+b+c
a2
b2
c2
a +b+c
+
+
+
a+b+c
+
+

.
b+c a+c a+b
2
b+c a+c a +b
2
Vy, bt ng thc (**) ng.
III.12. 1)

2x 5
+ 1 > 0 (1) . Ta c (1) tng ng vi
x 3

x > 3
x > 3
x > 3

x
x

+
1
>
0
>
0
x < 8 x > 3 x > 3
x > 2
x 3
x 3
3

2 < x < 3 x 3.
x < 3
x < 3
x < 3

2x 5
x 2
2 < x < 3
+1 > 0
>0

3 x
3 x
x 3
Vy, nghim ca bt phng trnh cho l
x > 2.

1
2
2) x 2 1 2 (1) . Ta c (1)
x
1

166

2
x2
2
x
2
x2
2
x

x4 x2 + 2
0

2
x
4
x + x2 2
0

x2
0 < x2 1
x 0

1 x 1.
x 0
Vy, nghim ca bt phng trnh cho l
1 x 1.
3)

x2
x2 5x + 6

3 (1) . Ta c

3x 2 + 16 x 20
x2

3
x2 5x + 6 0
x2 5x + 6
x2
(1) 2
3
2
x 5x + 6
x 2 3 3x 14 x + 16 0
x 2 5 x + 6
x 2 5 x + 6

10

x 3
3 < x 3

8
10
8 x < 3
3 x 3 .
3
Vy, ta c nghim ca bt phng trnh cho l
x 3

8
10
3 x 3 .
2 3 x
1

23 x
1+ x
4)
(*) .
1(1) . Ta c bt phng trnh (1) tng ng vi
1+ x
2 3 x 1
1 + x
3

1 < x
2 3x
3 2x

2
1 + x 1 1 + x 0

1
3
Vi x 0 th h (*) tr thnh

x < 1 x .
4
2
2 3x 1
1 4 x 0

1
x
1 + x
1 + x
4

Vi 1 x < 0 th h (*) tr thnh

167

x < 1
2 + 3x
3 + 4x

1 + x 1 1 + x 0
x 3
3
1

x .

4
4
2
2 + 3x 1
1 + 2 x 0

1
1 + x
1 + x
1 < x
2

3
1 1
3
Vy, nghim ca bt phng trnh cho l x x .
4
2 4
2

5)

x+2 x
2 (1) . Ta c bt phng trnh (1) tng ng vi
x

x 2
x 2
x 2

x + 2 x 2
2 2 0
2 2x 0

x
x
x

x < 2
x < 2
x < 2
x 2 x
2 x 2
4 x 2
2
2 0
0

x
x
x 2

0 < x 1
0 < x 1
x < 2

0 < x 1.

x
1
2 x < 0

Vy, nghim ca bt phng trnh cho l 0 < x 1.


6)

x2 4x + 3
x2 + x 5

1.

Bt phng trnh cho tng ng vi x 2 4 x + 3 x 2 + x 5 (1).


Ta c bng xt du

+ Vi x < 0 4 x < 5 ( ) , bt phng trnh (1) tr thnh


x 2 4 x + 3 x 2 x + 5 3 x 2 0 x

2
, tha ( )
3

+ Vi 0 x < 4 ( ) , bt phng trnh (1) tr thnh


x 2 + 4 x + 3 x 2 x + 5 2 x 2 + 5 x 2 0
168

1
x 2 , tha ( )
2

+ Vi x 5 ( ) , bt phng trnh (1) tr thnh


8
x 2 4 x + 3 x 2 + x 5 5 x + 8 0 x .
5

Kt hp vi ( ) ta c x .
Hp cc trng hp xt, ta c tp hp nghim ca bt phng trnh cho l
2 1

; ; 2 .
3 2

III.13.

1)

2 x
1 2x
2 x
1 2x
> 3
2
2
>0
3
2
2
x +x
x 2x
x ( x + 1) x ( x 2 )

( 2 x )( x 2 ) (1 2 x )( x + 1) > 0
x 2 ( x + 1)( x 2 )

x2 + 5x 5
>0
x 2 ( x + 1)( x 2 )

5 3 5
x <
2

1 < x < 0

0 < x < 5 + 3 5

x > 2
2)

x 4 3x3 + 2 x2
>0
x 2 x 30

x 2 x 2 3x + 2

x 2 x 30
x < 5
1 < x < 2
x > 6

3)

)>0

x 3 3x 2 x + 3
0
2x x2

( x 1) ( x2 2 x 3)
2x x2

1 x < 0
1 x < 2
x 3

169

4)

x4 4x2 + 3
0
x 2 8 x + 15
( x 2 1)( x 2 3)

x 2 8 x + 15

3 x 1

1 x 3
3 < x < 5

1
2
2x + 3
+ 2
< 3
x +1 x x +1 x +1
1
2
2x + 3

+ 2
3
<0
x +1 x x +1 x +1
x 2 x + 1 + 2 ( x + 1) ( 2 x + 3)

<0
x3 + 1
x2 x
3
<0
x +1
x < 1

0 < x < 1

5)

6 ) x 2 + ( x + 1)

15
x + x +1
2

15
x + x +1
4
3
2 x + 4 x + 5 x 2 + 3 x 14

0
x2 + x + 1
( x 1) ( 2 x3 + 6 x 2 + 11x + 16)

0
x2 + x +1
( x 1)( x + 2 ) ( 2 x 2 + 2 x + 2 )

0
x2 + x +1
2 x 1.
2 x2 + 2 x + 1

7 ) 2 x3 + x 2 5x + 2 > 0

( x 1) ( 2 x 2 + 3x 2 ) > 0
1

2 < x <

x
>
1

Vy, nghim ca bt phng trnh cho l 2 < x <

170

1
x > 1.
2

8 ) 2 x3 + x + 3 0

( x + 1) 2 x 2 2 x + 3 0
x 1.
III.14. 1) f ( x ) = ( m 3) x 2 2mx + m 6 0 (1)

+ Xt m = 3

(1) 6 x 3 0 x

1
.
2

+ Xt m 3
= m2 ( m 3)( m 6 ) = m 2 m2 + 9m 18 = 9m 18

Bng xt du h s a v bit s

a < 0
i) m < 2, khi
< 0

Suy ra f ( x ) < 0 , x .
Nh vy, bt phng trnh (1) nghim ng vi m i x .
a < 0
ii) m = 2, khi
nghim ca bt phng trnh (1) l m i x .
= 0
a < 0
iii) 2 < m < 3, khi
> 0.

Suy ra f ( x ) c hai nghim phn bit


x1 =

m 9m 18
m + 9m 18
; x2 =
m3
m 3

Trng hp ny a < 0 nn x2 < x1.


Nghim ca bt phng trnh (1) l x x2 x x1
a > 0
iv) m > 3, khi
, x1 < x2 .
> 0

Nghim ca bt phng trnh (1) l x1 x x2 .


Kt lun:

171

1
. m = 3 : Nghim ca bt phng trnh cho l x .
2

. m 2 : Nghim ca bt phng trnh cho l x .


. 2 < m < 3 : Nghim ca bt phng trnh cho l x x2 x x1.
. m > 3 : Nghim ca bt phng trnh cho l x1 x x2 .
(Vi x1 =

m 9m 18
m + 9m 18
; x2 =
).
m3
m 3

2) f ( x ) = ( m 4 ) x 2 2 ( m 2 ) x + m 1 0 ( 2)

+ Xt m = 4

( 2 ) 4 x + 3 0

3
x .
4
2

+ Xt m 4, = ( m 2 ) ( m 4 )( m 1) = m 2 4m + 4 m 2 + 5m 4 = m
Bng xt du h s a v bit s

a < 0
i) m < 0, khi
< 0

Suy ra f ( x ) < 0, x .
Nh vy bt phng trnh ( 2 ) v nghim.
a < 0
1
ii) m = 0 , khi
, nghim ca bt phng trnh ( 2 ) l x = .
2
= 0
a < 0
iii) 0 < m < 4, khi
> 0

Suy ra f ( x ) c hai nghim phn bit


x1 =

m2 m
m2+ m
; x2 =
m4
m4

Trng hp ny a < 0 nn x1 > x2 .


Nghim ca bt phng trnh ( 2 ) l x2 x x1.
a > 0
iv) m > 4 , khi
, x1 < x2
> 0

172

Nghim ca bt phng trnh ( 2 ) l x x1 x x2 .


Kt lun:
3
. m = 4: x .
4

. m < 0 : Bt phng trnh cho v nghim.


. m = 0 : Bt phng trnh cho c mt nghim duy nht x =

1
2

. 0 < m < 4 : Nghim ca bt phng trnh cho l x2 x x1.


. m > 4 : Nghim ca bt phng trnh cho l x x1 x x2 .
(Vi x1 =

m2 m
m2+ m
; x2 =
).
m4
m4

3) f ( x ) = mx 2 2 ( m 3) x + m 4 < 0 ( 3)

+ Xt m = 0

( 3) 6 x 4 < 0 x <

2
.
3
2

+ Xt m 0, = ( m 3 ) m ( m 4 ) = m2 6m + 9 m 2 + 4m = 9 2m.
Bng xt du h s a v bit s

a < 0
i) m < 0, khi
> 0

Suy ra f ( x ) c hai nghim phn bit


x1 =

m 3 9 2m
m

x2 =

m 3 + 9 2m
m

Trng hp ny a < 0 nn x1 > x2 .


Nghim ca ( 3) l x < x2 x > x1 .
ii) 0 < m <

9
, khi
2

a > 0
, x1 < x2 .

> 0

173

Nghim ca ( 3) l x1 < x < x2 .


iii) m =

a > 0
9
, khi
2
= 0

Suy ra f ( x ) 0, x .
Nh vy ( 3) v nghim.
iv) m >

9
, khi
2

a > 0

< 0

Suy ra f ( x ) > 0, x .
Nh vy ( 3) v nghim .
Kt lun:
2
. m = 0 : Nghim ca bt phng trnh cho l x < .
3

. m < 0 : Nghim ca bt phng trnh cho l x < x2 x > x1


. 0<m<
. m

9
: Nghim ca bt phng trnh cho l x1 < x < x2 .
2

9
: Bt phng trnh cho v nghim.
2

(Vi x1 =

m 3 9 2m
m 3 + 9 2m
; x2 =
)
m
m

III.15. f ( x) = (m + 1) x 2 2(m 1) x + 3m 3.

a) Ta c f ( x) < 0 tng ng vi
(m + 1) x 2 2(m 1) x + 3m 3 < 0 (1)
Xt cc trng hp
m + 1 = 0 m = 1
3
(1) 4 x 6 < 0 x < .
2

Nh vy, khi m = 1 th bt phng trnh (1) c nghim v nghim ca bt phng trnh l


3
x < . Suy ra m = 1 khng tha bi.
2
m + 1 0 m 1. Bt phng trnh (1) v nghim khi v ch khi
(m + 1) x 2 2(m 1) x + 3m 3 0, x

174

a = m + 1 > 0
m > 1

2
2
2
= (m 1) (m + 1)(3m 3) 0
m 2m + 1 3m + 3 0
m > 1
m > 1

m 1.
2
m 2 m 1
2 m 2 m + 4 0

Vy, vi m 1 th bt phng trnh f ( x) < 0 v nghim.


b) Ta c f ( x) 0 tng ng
(m + 1) x 2 2(m 1) x + 3m 3 0 (2)
Xt cc trng hp
m + 1 = 0 m = 1 khi
3
(2) 4 x 6 0 x .
2

Nh vy, khi m = 1 th bt phng trnh (2) c nghim v nghim ca bt phng trnh


3
l x . Suy ra m = 1 tha bi.
2
m + 1 0 m 1. Khi (2) l bt phng trnh bc hai. Bt phng trnh (2) c
nghim khi v ch khi

m + 1 > 0
m > 1
a > 0


2
2

<
0

(
m

1)

(
m
+
1)(3
m

3)
<
0

2m 2m + 4 < 0

(m 1) 2 (m + 1)(3m 3) 0
2m 2 2m + 4 0
0

m > 1
m > 1

m < 2 m > 1
m 2.
2 m 1

2 m 1

Vy, khi m 2 th bt phng trnh f ( x) 0 c nghim.


Ch . Ta cng c th gii Cu b) nh sau:
Trc ht ta gii bi ton ngc, tc l tm tham s m bt phng trnh (2) v nghim.
Khi cc gi tr m cn li trn trc s l cc gi tr cn tm bt phng trnh f ( x) 0
c nghim.
Bt phng trnh f ( x) 0 v nghim khi v ch khi f ( x) < 0, x . iu ny c tha
khi v ch khi
m + 1 < 0
a < 0
m < 1

m < 2.

2
< 0
m < 2 m > 1
2 m 2 m + 4 < 0

T cc gi tr cn tm ca tham s m l m 2.
III.16. 1) Ta c tam thc bc hai 2 x 2 x + 1 > 0, x .

175

2 x 2 x + 1 3x 2 mx + 5
Do bt phng trnh cho tng ng vi 2
2
3 x mx + 5 < 6 2 x x + 1

x 2 ( m 1) x + 4 0
2
9 x + ( m 6 ) x + 1 > 0

(1)
(2)

Bt phng trnh cho c tp hp nghim l khi v ch khi (1) v ( 2 ) u c tp hp


nghim l . iu ny c tha khi v ch khi
1 = ( m 1) 2 16 0
3 m 5

0 < m 5.

2
2 = ( m 6 ) 36 < 0 0 < m < 12

Vy, vi 0 < m 5 th bt phng trnh cho c tp hp nghim l .


2) V x 2 + 1 > 0, x nn bt phng trnh cho tng ng vi
2
2 x 2 2 < x 2 + mx + 1 3x + mx + 3 > 0 (1)
x + mx + 1 < 2 x + 2 2
2
2
x + mx + 1 < 2 x + 2
x mx + 1 > 0 ( 2 )
2

Bt phng trnh cho c tp hp nghim l khi v ch khi cc bt phng trnh (1) v

( 2 ) u c tp hp nghim l

. iu ny c tha khi v ch khi

1 = m2 36 < 0
6 < m < 6

2 < m < 2.

<
<
2
m
2

<
m
4
0

Vy, vi 2 < m < 2 th bt phng trnh cho c tp hp nghim l .


III.17. 1) x 2 ( 2m + 3) x + m 2 = 0 (1)
t f ( x ) = x 2 ( 2m + 3) x + m 2

Phng trnh (1) c hai nghim x1 , x2 tha x1 < 3 < x2 khi v ch khi
af ( 3) < 0
9 3 ( 2m + 3) + m2 < 0
m 2 6m < 0
0 < m < 6.
Vy, 0 < m < 6 tha yu cu bi ton.
2) mx 2 + 2 ( m 1) x + m 5 = 0 ( 2 )
t f ( x ) = mx 2 + 2 ( m 1) x + m 5

Phng trnh ( 2 ) c hai nghim x1 , x2 tha x1 < x2 < 2 khi v ch khi cc iu kin sau c
tha
176


> 0

af ( 2 ) > 0

S 2 < 0
2
1

< m < 0
3

Vy,

( m 1)2 m ( m 5 ) > 0
3m + 1 > 0
m > 3

m ( 9m 9 ) > 0 m < 0 m > 1


m ( 9m 9 ) > 0
3m + 1

1
( m 1) 2 < 0

m < 0 m >
<0
m
3

m
m > 1.

1
< m < 0 m > 1 tha yu cu bi ton.
3

3) ( m 1) x 2 ( m 5 ) x + m 1 = 0 ( 3)
t f ( x ) = ( m 1) x 2 ( m 5 ) x + m 1

Phng trnh ( 3) c hai nghim x1 , x2 tha 1 < x1 < x2 khi v ch khi

( m 5 ) 2 4 ( m 1)2 > 0

( m 1)( 3m 7 ) > 0
m5

+1 > 0
2 ( m 1)
7

3 < m <

2
3

3m 2m + 21 > 0

( m 1)( 3m 7 ) > 0 m < 1 m >


3m 7

>0
2 ( m 1)
m < 1 m >

> 0

af ( 1) > 0
S
+1 > 0
2

7
3 < m < 1.
3
7
3

Vy, gi tr cn tm ca tham s m l 3 < m < 1.


III.18.

( 3 m ) x2 2 ( 2m 5 ) x 2m + 5 = 0 (1)

1
+ Xt m = 3, phng trnh (1) tr thnh 2 x 1 = 0 x = .
2

Vy, khi m = 3 th phng trnh (1) c mt nghim x =

1
< 1.
2

+ Xt m 3, t f ( x ) = ( 3 m ) x 2 2 ( 2m 5 ) x 2m + 5.
2

f ( x ) = ( 2m 5 ) ( 3 m )( 2m + 5) = 2m2 9m + 10.
af (1) = ( 3 m )( 7 m + 18) = 7 m 2 40m + 54

S
2m 5
3m 8
1 =
1 =
.
2
3 m
3 m

177

18
m = 3.
7
5
f ( x ) = 0 m = 2 m = .
2
S
8
1 = 0 m = .
2
3
af (1) = 0 m =

Lp bng xt du chung f ( x ) , af (1),

S
1 v da vo nh l o du tam thc bc hai ta
2

c kt qu nh sau:
m < 2
5
18
+ Nu < m <
th phng trnh (1) c hai nghim x1 < x2 < 1.
7
2
m > 3

8
18
7 <m<3
, th phng trnh (1) c hai nghim x1 < 1 < x2 .
+ Nu
8 < m < 3
3
+ Nu 2 < m <

5
, th phng trnh (1) v nghim.
2

+ Nu m = 2 , th phng trnh (1) c nghim kp x = 1 < 1.


+ Nu m =

5
, th phng trnh (1) c nghim kp x = 0 < 1.
2

+ Nu m =

18
1
, th phng trnh (1) c hai nghim x1 = , x2 = 1 x1 < x2 = 1.
7
3

+ Nu m =

8
, th phng trnh (1) c hai nghim x1 = 1 2, x2 = 1 + 2 x1 < 1 < x2 .
3

+ Nu m = 3 , th phng trnh (1) c nghim x =

1
< 1.
2

III.19. 1) mx 2 2 ( m + 1) x + m + 5 = 0 (1)

Phng trnh (1) c hai nghim x1 , x2 tha x1 < 0 < x2 < 2 khi v ch khi
af ( 0 ) < 0
m ( m + 5 ) < 0

af ( 2 ) > 0
m 4m 4 ( m + 1) + m + 5 > 0

m 2 + 5m < 0 m 2 + 5m < 0 5 < m < 0


2
2

5 < m < 1.
m < 1 m > 0
m + m > 0
m + m > 0

Vy, vi 5 < m < 1 th phng trnh cho tha mn iu kin bi.


178

2)

( m 2 ) x2 2mx + 2m 3 = 0 (2)

Phng trnh (2) c hai nghim x1 , x2 tha 6 < x1 < 4 < x2 khi v ch khi
af ( 4 ) < 0
( m 2 ) 36 ( m 2 ) + 12m + 2m 3 > 0

( m 2 ) 16 ( m 2 8m + 2m 3) < 0
af ( 6 ) > 0
( m 2 )( 50m 75 ) > 0 ( m 2 )( 50m 75 ) > 0
7

2<m< .
2
( m 2 )(10m 35 ) < 0
( m 2 )(10m 35 ) < 0

Vy, vi 2 < m <

7
th phng trnh cho tha mn yu cu bi ton.
2

III.20. mx 2 2(m 1) + m 3 = 0 (1)


t f ( x ) = mx 2 2(m 1) x + m 3

+ Khi m = 0 (1) tr thnh 2 x 3 = 0 x =

3
3
. Ta c: 0 < < 2
2
2

+ Khi m 0 , ta c = (m 1)2 m(m 3) = m 2 2m + 1 m 2 + 3m = m + 1.


af (0) = m(m 3).
s
m 1
0 =
.
2
m
af (2) = m [ 4m (m 1) + m 3] = m(m + 1).
s
m 1
(m + 1)
2=
2=
.
2
m
m
Vi m = 1 , (1) tr thnh x 2 + 4 x 4 = 0 x = 2.
Vi m = 1 , (1) tr thnh x 2 2 = 0 x1 = 2 x2 = 2
4
Vi m = 3 , (1) tr thnh 3 x 2 4 x = 0 x1 = 0 x2 = .
3

Lp bng xt du chung f ( x ) , af (0), af (2),

S
S
0, 2 v da vo nh l o du tam
2
2

thc bc hai ta c kt qu nh sau:


+ Nu m < 1, th phng trnh (1) v nghim.
+ Nu m = 1, th 0 < x1 = x2 = 2.
+ Nu 1 < m < 0, th 0 < x1 < 2 < x2 .
+ Nu m = 0, th 0 < x =

3
< 2.
2

+ Nu 0 < m < 1, th x1 < 0 < x2 < 2.


179

+ Nu m = 1, th x1 = 2 < 0 < x2 = 2 < 2.


+ Nu 1 < m < 3, th x1 < 0 < x2 < 2.
+ Nu m = 3, th x1 = 0 < x2 =

4
< 2.
3

+ Nu m > 3, th 0 < x1 < x2 < 2.


III. 21. 2 x 2 + (2m 1) x + m 1 = 0 (1). Phng trnh (1) c mt nghim nm trong
khong ( 1;3) nghim kia nh hn 1 , ngha l x1 < 1 < x2 < 3.
t f ( x ) = 2 x 2 + (2m 1) x + m 1.

Yu cu bi ton c tha khi v ch khi


2 f ( 1) < 0
2 [ 2 (2m 1) + m 1] < 0

2 f (3) > 0
2 [18 + 3(2m 1) + m 1] > 0
m + 2 < 0
m > 2

7m + 14 > 0
m > 2
m > 2.
III.22. ( m 1) x 2 2mx + m + 5 = 0 (1)

1) Phng trnh (1) c hai nghim u ln hn 2 khi v ch khi

m 2 ( m 1)( m + 5 ) 0
0

af ( 2 ) > 0 ( m 1) 4 ( m 1) 4m + m + 5 > 0
S

>2
2m > 2
2
2 ( m 1)

4m + 5 0
m 4

5
( m 1)( m + 1) > 0 m < 1 m > 1 1 < m .
4
m + 2
1 < m < 2

>0
m 1

Vy, khi 1 < m

5
th phng trnh c hai nghim u ln hn 2.
4

2) + Xt m 1 = 0 m = 1.
Khi phng trnh (1) tr thnh 2 x + 6 = 0 x = 3 > 2. Nh vy, m = 1 tha bi.
+ Xt m 1 0 m 1.
Khi phng trnh (1) c t nht mt nghim ln hn 2 nm trong cc trng hp: Hoc
l c hai nghim u ln hn 2, hoc ch c mt nghim ln hn 2. iu ny c tha khi
v ch khi
180


0
2 < x1 x2

f ( 2 ) = 0

x
2
x
af
2
0
af
2
0
<
<

>

<

(
)
(
)

1
2

S > 4
S
x1 = 2, x2 > 2
2>0
2

5
+ Trng hp th nht xt Cu 1), gi tr ca tham s m l 1 < m .
4

+ Trng hp ch c mt nghim ln hn 2, ta gii h iu kin


( m 1)( m + 1) < 0

1 < m < 1
m + 1 = 0

1 < m < 1.

2m
m

4>0
m 1

5
Kt hp cc trng hp xt ta c 1 < m .
4

Vy, khi 1 < m

5
th phng trnh trn c t nht mt nghim ln hn 2.
4

III.23. f ( x ) = mx 2 2 ( m + 1) x m + 5 . K hiu D = (;1), S l tp hp nghim ca bt


phng trnh f ( x ) > 0. Ta bit f ( x ) > 0 ng vi m i x D khi v ch khi D S .

+ Xt m = 0. Khi f ( x ) > 0 khi v ch khi 2 x + 5 > 0 x <

5
. Nh vy tp hp
2

5
nghim ca bt phng trnh f ( x ) > 0 l S = (; ). R rng D S . Vy, m = 0 tha
2
yu cu bi ton.

+ Xt m 0. Ta c
2

= ( m + 1) m ( m + 5 ) = m 2 + 2m + 1 + m 2 5m = 2m2 3m + 1. Tp hp nghim ca bt
phng trnh f ( x ) > 0 ty thuc vo du ca h s a v bit s ca f ( x ).
Lp bng xt du chung h s a v bit s ca f ( x ). Ta xt cc trng hp sau
> 0
m < 0. Khi
, tp nghim S ca bt phng trnh f ( x ) > 0 l khong ( x1; x2 ) ,
a < 0
vi x1 , x2 l hai nghim ca f ( x), do bi ton khng tha.
0<m<

1
m > 1. Khi
2

x < x1
a > 0
, nghim ca bt phng trnh f ( x ) > 0 l

> 0
x > x2

Do f ( x ) > 0 ng vi m i x D khi v ch khi

181

3
af (1) 0 m ( 3 2m ) 0

0 m
1 x1 < x2 S
1

2 0 < m . Kt hp vi iu
2
2 1 > 0
>0
m > 0
m
1
3
kin ang xt ta c 0 < m < 1 < m .
2
2
1
< m < 1. Khi
2
bi ton tha.

a > 0
, tp hp nghim ca bt phng trnh f ( x ) > 0 l S = , do

< 0

m=

2 , tp hp nghim ca bt phng trnh f ( x ) > 0 ln lt l

m = 1
S = \ {3} , S = \ {2}. Suy ra bi ton c tha.
3
Vy, gi tr ca tham s m cn tm l 0 m .
2

III.24. f ( x ) = 2 x 2 ( 3m + 1) x ( 3m + 9 )

Ta c tam thc bc hai f ( x ) c h s a = 2 > 0 nn f ( x ) 0, x [ 2;1] khi v ch khi


f

( 2 ) 0 3m + 1 0
4
1

m .
3
3
(1) 0
6m 8 0
m 4

4
1
Vy, gi tr ca tham s m cn tm l m .
3
3
2

III.25. f ( x ) = ( m 2 ) x 2 3 ( m 6 ) x m 1
1
+ Nu m 2 = 0 m = 2, th f ( x ) < 0 12 x 3 < 0 x < . Suy ra m = 2 tha yu cu
4
bi ton.

+ Nu m 2 0 m 2 (*). Khi tam thc bc hai f ( x ) c h s a > 0, v vy yu cu


bi ton c tha khi v ch khi
f ( 1) 0

f ( 0 ) 0
( m 2 )2 + 3 ( m 6 ) m 1 0

m 1 0
m 2 4m + 4 + 3m 18 m 1 0

m + 1 0

182

m 2 2m 15 0
3 m 5

m 1
m 1

1 m 5
Kt hp vi iu kin ang xt (*) ta c
m 2

Vy, gi tr cn tm ca tham s m l 1 m 5.
III.26. ( x + 2)( x + 4)( x 2 + 6 x + 10) m(1)

Xt hm s f ( x) = ( x + 2)( x + 4)( x 2 + 6 x + 10) = ( x 2 + 6 x + 8)( x 2 + 6 x + 10).


(1) ng vi m i x khi v ch khi m Min f ( x ) .
x

Ta c f ( x) = (2 x + 6)(2 x 2 + 12 x + 18) = 4( x + 3)3 .


f ( x ) = 0 x = 3.
Bng bin thin

Nh vy Min f ( x ) = 1.
x

Vy, m 1 l gi tr cn tm.
2

Ch . C th t n s ph t = x 2 + 6 x + 8 = ( x + 3) 1 1 . Khi ta c hm s
f (t ) = t ( t + 2 ) = t 2 + 2t , t 1. Nh vy, (1) ng vi m i x khi v ch khi t 2 + 2t m

ng vi m i t 1 , khi v ch khi Min(t 2 + 2t ) m.


t[ 1;+ )

f (t ) = 2t + 2, f (t ) = 0 t = 1. Lp bng bin thin ca hm s f (t ) ta tm c


Min(t 2 + 2t ) = 1. Vy, m 1 l gi tr cn tm.
t[ 1;+ )

III.27.

2cos 2 x + 3m cos x + 1 0

(1)

t t = cos x, x [0; ] t [ 1;1] . Bt phng trnh 2cos 2 x + 3m cos x + 1 0 (1) tr

thnh 2t 2 + 3mt + 1 0 (2). Khi (1) nghim ng vi x [ 0; ] khi v ch khi (2)


nghim ng vi t [1;1].
+ Xt t = 0 . Khi (2) 1 0 (2) ng vi m i m.
+ Xt t [1;0). Khi
(2) m

2t 2 1
.
3t
183

Xt hm s f (t ) =

2t 2 1
, t [1; 0)
3t

t=
[1; 0)

6t + 3
2
, f (t ) = 0
f (t ) =
1
9t 2

t = 2 [1; 0)

1 2 2
Ta c f (1) = 1, f
= 3 , lim f (t ) = +.
t 0
2

Lp bng bin thin ca hm s trn min t [1; 0), ta c m

2 2
.
3

+ Xt t (0;1]. Khi
(2) m

2t 2 1
.
3t

Ta tm c m

2 2
.
3

p s ca bi ton l nhng gi tr ca tham s m tha ng thi ba trng hp trn tc


2 2
2 2
l
m
.
3
3

Ch . C th gii bi ton trn theo phng php nh trnh by Bi III.23.


III.28. x 2 +

1
1

+ ( 2m + 3) x + + 2 ( m + 2 ) > 0 (1)
2
x
x

1
1

(1) x + + ( 2m + 3) x + + 2 ( m + 1) > 0.
x
x

t t = x +

1
, iu kin: t 2.
x

Bt phng trnh (1) tr thnh t 2 + ( 2m + 3) t + 2 ( m + 1) > 0 (2)


t y = f (t ) = t 2 + ( 2m + 3) t + 2 ( m + 1) .

(1) nghim ng vi x 0 khi v ch khi (2) nghim ng vi m i t tha t 2


2

Do f (t ) c = ( 2m + 1) 0 .
Nn (2) nghim ng vi m i t tha t 2 khi v ch khi

184

= 0

2 < t0 = 2m 3 < 2

2

> 0
2 < t1 < t2 < 2

1
m = 2
( 2m + 1)2 = 0

4 < 2m 3 < 4
7 < m < 1

2
2
2

( 2m + 1) 0

m 1
1.
f
(2)
>
0

2

1. f (2) > 0
4 + 2 ( 2m + 3) + 2 ( m + 1) > 0

4 2 ( 2m + 3) + 2 ( m + 1) > 0
2 < 2m 3 < 2


2
4 < 2m 3 < 4
1

m = 2

m 1

2

6m + 12 > 0
2m > 0

7 < m < 1
2
2
1

m = 2

m 1

2

2 < m < 0.
m > 2
m < 0

7 < m < 1
2
2

Vy, vi 2 < m < 0 th bt phng trnh cho nghim ng vi x 0.


III. 29. x 3 ( 2m + 1) x 2 + 3 ( m + 4 ) x m 12 > 0 (1)

Ta c (1) ( x 1) x 2 2mx + m + 12 > 0 .


V x > 1 nn (1) x 2 2mx + m + 12 > 0(2). t f ( x ) = x 2 2mx + m + 12.
(1) ng vi m i x > 1 khi v ch khi x 2 2mx + m + 12 > 0(2) ng vi m i x > 1
185

< 0
m 2 m 12 < 0

0
m m 12 0


3 < m < 4
af (1) 0
13 m 0

m < 4.

m 3

S
m 1
2 1

Vy, gi tr m cn tm l m < 4.
III.30. ( x 1)( x + 1)( x + 3)( x + 5) > m.

Ta c ( x 1)( x + 1)( x + 3)( x + 5) > m


[ ( x 1)( x + 5)][ ( x + 1)( x + 3)] > m ( x 2 + 4 x 5)( x 2 + 4 x + 3) > m (1).

t t = x 2 + 4 x 5 x 2 + 4 x + 3 = t + 8, ta c (1) t (t + 8) > m t 2 + 8t > m .

Ta c t = x 2 + 4 x 5 t = 2 x + 4 = 0 x = 2 .

Vi x > 1 th t > 8, do (1) ng x > 1 (2) ng t > 8 .


Xt f (t ) = t 2 + 8t , ta c y = 2t + 8 = 0 2t + 8 = 0 t = 4 .
Bng bin thin

Da vo bng bin thin ta c (2) ng t > 8 Minf (t ) > m m < 16 .


t( 8;+ )

Vy, m < 16 th bt phng trnh cho nghim ng vi m i x > 1 .


III.31. x ( x 2 )( x + 2 )( x + 4 ) < 2m (1)

(1) ( x 2 + 2 x )( x 2 + 2 x 8) < 2m
t t = x 2 + 2 x. Khi ta c (1) tr thnh t 2 8t < 2m (2).

186

Xt t ( x ) = x 2 + 2 x t ( x ) = 2 x + 2
t ( x ) = 0 x = 1.

Ta c bng bin thin

Nh vy, vi x > 0 th t > 0.


Bt phng trnh (1) c nghim x > 0 khi v ch khi bt phng trnh (2) c nghim t > 0.
iu ny c tha khi v ch khi 2m > Min(t 2 8t ).
t(0;+ )

t g ( t ) = t 2 8t g ( t ) = 2t 8. Cho g ( t ) = 0 t = 4.

Bng bin thin

Da vo bng bin thin ta c 2m > 16 m > 8.


Vy, gi tr cn tm l m > 8.

III.32. 4 x 4 x 2 + 1 = 1. Phng trnh cho tng ng vi 4 x 4 x 2 + 1 1 = 0.

Xt hm s f ( x ) = 4 x 4 x 2 + 1 1 vi x .
Ta c f ( x ) = 4 x ln 4 ( 4 x 2 + 1) + 8 x.4 x = 4 x ln 4 ( 4 x 2 + 1) + 8 x

f ( x ) = 0 ln 4 4 x 2 + 1 + 8 x = 0
( 4 ln 4 ) x 2 + 8 x + ln 4 = 0 (*)

Phng trnh (*) c bit thc > 0 nn c ng hai nghim phn bit.
Lp bng bin thin ca hm s f ( x ) v t bng bin thin ta suy ra phng trnh f ( x ) = 0
c khng qu ba nghim phn bit.
1
37 17 27
Mt khc f = 0, f ( 0 ) = 0, f ( 3) . f ( 2 ) = 1 1 =
< 0.
2
64 16 1024
1
Do phng trnh f ( x ) = 0 c ng ba nghim phn bit x1 = 0, x2 = , x3 ( 3; 2 ) .
2

Vy, phng trnh cho c ng ba nghim phn bit.


187

CHNG IV.

IV.1. 1) 16 x 2

PHNG TRNH, BT PHNG TRNH V T

3 x = 0 (1)

iu kin: 3 x 0 x 3. ( )

16 x 2 = 0
(1)
3 x = 0
x 2 = 16

x = 3
x = 4
x = 4
x = 3
So vi iu kin ( ) , phng trnh cho c hai nghim l x = 4 v x = 3.

2) 9 x 2

2 x = 0 ( 2)

iu kin: 2 x 0 x 2 ( )
9 x 2 = 0
2

( )
2 x = 0
x = 3
x2 = 9

x = 3

x = 2
x = 2

So vi iu kin ( ) , phng trnh cho c hai nghim l x = 3 v x = 2.


3) 4 + 2 x x 2 = x 2 ( 3) . Ta c
x 2
x 2 0
x 2

2
x = 0 x = 3.
( 3)
2
2
4 + 2 x x = x 4 x + 4
2 x 6 x = 0
x = 3

Vy, phng trnh cho c mt nghim l x = 3.


4 ) 1 + 4 x x 2 = x 1( 4 )
Ta c
x 1
x 1
x 1

2
x = 0 x = 3.
( 4)
2
2
1 + 4 x x = x 2 x + 1 2 x 6 x = 0
x = 3

Vy, phng trnh cho c mt nghim l x = 3.

188

5)

2x +1 + x 3 = 2 x

( 5)

x 2
2 x + 1 0

iu kin: x 3 0 x 3 x 3 ( )
x 0
x 0

( 5 ) 2 x + 1 + x 3 + 2 ( 2 x + 1)( x 3) = 4 x
( 2 x + 1)( x 3) = x + 2

4 2 x 2 5 x 3 = x 2 + 4 x + 4 (Do x 3 nn x + 2 > 0)
7 x 2 24 x 16 = 0
x = 4

x = 4
7

So vi iu kin ( ) , ta c nghim ca phng trnh cho l x = 4.


6 ) x + 1 + 4 x + 13 = 3x + 12 ( 6 )
x 1
x +1 0

13

x 1( )
iu kin: 4 x + 13 0 x
4
3 x + 12 0

x 4

( 6 ) x + 1 + 4 x + 13 + 2 ( x + 1)( 4 x + 13) = 3 x + 12
2

( x + 1)( 4 x + 13) = 2 x 2 ( x + 1)( 4 x + 13) = x 1(**)

iu kin ca phng trnh (**) l x 1 . Kt hp vi iu kin (*) ta c x = 1.


x = 1 tha phng trnh (**).

Vy, phng trnh cho c mt nghim l x = 1.


7 )( x + 3) 10 x 2 = x 2 x 12 ( 7 )
iu kin: 10 x 2 0 10 x 10. ( )

( 7 ) ( x + 3)

10 x 2 = ( x + 3)( x 4 )

( x + 3) 10 x 2 ( x 4 ) = 0

x + 3 = 0

2
10 x ( x 4 ) = 0
189

x = 3

2
10 x = x 4
x = 3

x 4 0
10 x 2 = x 2 8 x + 16

x = 3

x 4

x = 3.
x = 1

x = 3
So vi iu kin ( ) , ta c nghim ca phng trnh cho l x = 3.
8) x + 4 1 x = 1 2 x ( 8)

x 4
x
+
4

iu kin: 1 x 0 x 1 4 x ( )
2
1 2 x 0

x
2

Ta c
x + 4 1 x = 1 2x
x + 4 = 1 2x + 1 x
x + 4 = 1 2 x + 1 x + 2 (1 2 x )(1 x )
2x +1 =

(1 2 x )(1 x )

2 x + 1 0
2
2
4 x + 4 x + 1 = 1 3x + 2 x
1

2
x2 7 x = 0

x 2
x = 0

x=0
x = 7

x = 7

So vi iu kin ( ) , ta c nghim ca phng trnh cho l x = 0.


IV.2. 1) 3 x 2 + 15 x + 2 x 2 + 5 x + 1 = 2 (1)

190

t t = x 2 + 5 x + 1, t 0 ( )

(1)

tr thnh 3t 2 + 2t 5 = 0

t = 1 t =

5
3

So vi iu kin ( ) ta nhn t = 1, khi ta c


x = 0
t = x 2 + 5 x + 1 = 1 x 2 + 5x = 0
x = 5.
Vy, nghim ca phng trnh cho l x = 0 x = 5.
2)

x 2 3x + 3 + x 2 3x + 6 = 3 (1)

t t = x 2 3x + 3

(1)

t + t + 3 = 3 ( 2)

tr thnh

iu kin: t 0

Khi ( 2 ) tng ng vi
t + t + 3 + 2 t ( t + 3) = 9
t ( t + 3) = 3 t
3 t 0

2
t ( t + 3) = 9 6t + t
t 3
t 3

9t = 9
t = 1
t = 1.

Vi t = 1, ta c
x 2 3x + 3 = 1 x 2 3 x + 2 = 0 x = 1 x = 2.
Vy, nghim ca phng trnh cho l x = 1; x = 2.
3)

x+ 2 + 5 x +

( x + 2 )( 5 x ) = 4 () . iu kin:

2 x 5.

t t = x + 2 + 5 x , t 0

t2 = x + 2 + 5 x + 2
t2 7 = 2

( )

( x + 2 )( 5 x )

( x + 2 )( 5 x )

tr thnh

191

t+

t = 3
t2 7
= 4 2t + t 2 7 = 8 t 2 + 2t 15 = 0
2
t = 5

V t 0 nn ta nhn t = 3, khi ta c
x + 2 + 5t = 3
x+ 2+5 x+ 2

( x + 2 )( 5 x ) = 9

( x + 2 )( 5 x ) = 1

x 2 + 3x + 10 = 1
x 2 3x 9 = 0

x=

33 5
3+3 5
x=
(Tha iu kin)
2
2

Vy, phng trnh cho c nghim l x =

33 5
3+3 5
;x =
.
2
2

x + 4 + x 4 = 2 x 12 + 2 x 2 16 ( )

4)

iu kin: x 4
t t = x + 4 + x 4, t 0
t 2 = 2 x + 2 x 2 16

( )

tr thnh

t = t 2 12
t 2 t 12 = 0
t = 3 t = 4
V t 0 nn nhn t = 4, ta c
x+4 + x4 = 4
x + 4 + x 4 + 2 x 2 16 = 16
x 2 16 = 8 x
8 x 0
2
2
x 16 = 64 16 x + x
x 8
x 8

16 x = 80
x = 5
x = 5 (Tha iu kin).

Vy, phng trnh cho c mt nghim l x = 5.

192

5) 1 +

2
x x 2 = x + 1 x (1)
3

u = x
t
, iu kin: u , v 0.
v = 1 x
Phng trnh (1) c chuyn thnh h
2 2
2

u + v = 1
( u + v ) 2uv = 1 1 + uv 2uv = 1

3
2
2
2
1 + uv = u + v
u + v = 1 + uv

3
3

u + v = 1 + 3 uv
2

uv = 0
(I )

u + v = 1
uv = 0 uv = 2


3
2
uv =
u + v = 1 + uv
2 ( II )

u + v = 2


+ Trng hp ( I ) ta c

x =0
1 x = 1
x =1

x = 0

x =1

1 x = 0

+ Trng hp ( II ) (V nghim).
Vy, phng trnh cho c hai nghim l x = 0; x = 1.
Ch . C th t n ph nh trnh by Bi IV.2. 3).
6) 1 + 1 + x x 2 24 = x

(1)

Ta c (1) 1 + x x 2 24 = x 1
x 1 0
x 1

2
2
2
2
1 + x x 24 = x 2 x + 1 x x 24 = x 2 x
x 1
x 1

2
x 2 0
x 24 = x 2
x 2 24 = x 2 4 x + 4

x 2
x 2

x=7
4 x = 28 x = 7
Vy, phng trnh cho c mt nghim l x = 7.

193

7)

x + x + 11 + x x + 11 = 4 (1)

x + 11 0

iu kin: x x + 11 0 ( )

x + x + 11 0

(1) x +

x + 11 + x x + 11 + 2 x 2 x 11 = 16

x 2 x 11 = 8 x
8 x 0
2
2
x x 11 = x 16 x + 64
x 8
x 8

x=5
15 x = 75
x = 5

So vi iu kin ( ) ta nhn x = 5.
Vy, phng trnh cho c mt nghim l x = 5.

8) x 3 35 x 3 x + 3 35 x 3 = 30 (1)
u = x
t
3
3
v = 35 x

(1)

uv ( u + v ) = 30
tr thnh 3 3
u + v = 35

( u + v ) uv = 30
( u + v ) uv = 30

3
3
( u + v ) 3 ( u + v ) uv = 35 ( u + v ) = 125
u + v = 5 u = 2 u = 3
( u + v ) uv = 30

uv = 6
v = 3 v = 2
u + v = 5
x = 3
x = 3
u = 3 x = 3
Vi
3

x=3

3
3
3
v = 2
35 x = 2
35 x = 8
x = 27
x = 2
x = 2
x = 2
u = 2
Vi
3

x=2

3
3
3
v = 3
35 x = 3 35 x = 27
x = 8
Vy, phng trnh cho c hai nghim l x = 2; x = 3.
9) x 3 + 2 = 3 3 3x 2 (1)

(1)
194

x3 + 2 3
= 3x 2
3

Hm s y =

x3 + 2
lun ng bin trn nn c
3

x3 + 2
hm s ngc l y = 3x 2 . V vy, honh giao im ca hai th y =
v
3
3

y = 3 3x 2 cng chnh l honh giao im ca hai th y = x v y =

1 3
( x + 2) .
3

Do phng trnh cho tng ng vi


x3 + 2
x=
3
x3 3x + 2 = 0
x = 2 x = 1
Vy, phng trnh cho c hai nghim l x = 2; x = 1.
2

10) 2 3 (1 + x ) + 3 3 1 x 2 + 3 (1 x ) = 0 (1)
V x = 1 khng l nghim ca phng trnh nn chia hai v ca (1) cho

(1 + x )

ta

c phng trnh tng ng


2

2 + 33

1 x 3 1 x
+
= 0 ( )
1+ x
1+ x

t t =

1 x
, khi ( ) tr thnh
1+ x

2 + 3t + t 2 = 0
1 x
1 x
3
= 1

1 + x = 1
t = 1
9
1+ x

x=
1 x
7
t = 2
1 x = 8
= 2
3
1 + x
1+ x
9
Vy, phng trnh cho c mt nghim l x = .
7

11) 2 x + 6 3 1 x + 2 = 0(1)
t t = 3 1 x t 3 = 1 x x = 1 t 3 . Phng trnh (1) tr thnh

2 1 t 3 + 6t + 2 = 0 2t 3 6t 4 = 0
t = 1
t 3 3t 2 = 0
t = 2
+ Vi t = 1 3 1 x = 1 x = 2.
195

+ Vi t = 2 3 1 x = 2 1 x = 8 x = 7.
Vy, phng trnh cho c hai nghim l x = 7; x = 2.
12) 3 x + 1 = 3x 2 8 x + 3(1)
iu kin: x 1. Phng trnh (1) tng ng vi 3 x + 1 3x 2 + 8 x 3 = 0.

Xt hm s f ( x ) = 3 x + 1 3x 2 + 8 x 3, x 1.

3
6x + 8
2 x +1
3
f ( x) =
6 < 0, x > 1
3
4 ( x + 1)
f ( x) =

Suy ra hm s l i trn [1; +). Vy, phng trnh (1) nu c nghim s khng qu hai
nghim, ta c f (0) = f (3) = 0. Do phng trnh cho c hai nghim l x = 0; x = 3.
Ch . Chng ta c th gii bi ton ny bng phng php bin i tng ng, c th:
3x 2 8 x + 3 0
(1)
2
9 ( x + 1) = 3 x 8 x + 3

2
3 x 8 x + 3 0
4
3
2
9 x 48 x + 82 x 57 x = 0

3 x 2 8 x + 3 0
x = 0

x = 3.
2

x ( x 3) 9 x 21x + 19 = 0

13)

x + 3 x + 1 = x 2 + x + 1(1)

iu kin: x 0. Phng trnh (1) vit li di dng

x + 3 x + 1 x 2 x 1 = 0.
Xt hm s f ( x ) = x + 3x + 1 x 2 x 1, x 0.
3
2x 1
2 x 2 3x + 1
1
9
f ( x) =

2 < 0, x > 0.
3
3
4 x
4 ( 3 x + 1)
f ( x) =

Lp lun gi ng Bi IV.3.12) ta cng c phng trnh cho c hai nghim x = 0; x = 1.


Ch . Nu s dng phng php bin i tng ng s nhn c phng trnh bc 8,
khi cho d nhm c hai nghim x = 0, x = 1 th chng ta vn phi thc hin tip vic
gii phng trnh bc 6 v iu ny kh c th i n kt qu.
IV.3. 1) x x 2 + 15 x 4 x 2 + 15 = 2 (1).
iu kin: x 0

196

t t = 4 x 2 ( x 2 + 15) , ( t 0 ) . Khi phng trnh (1) tr thnh

t2 t = 2
t2 t 2 = 0
t = 1

t = 2

Ta loi t = 1 < 0 .
Vi t = 2 th

x 2 ( x 2 + 15) = 2

x2 = 1
x x + 15 = 16 x + 15 x 16 = 0 2
x = 16
2

Ta loi x 2 = 16 < 0.
x = 1
Vi x 2 = 1
x = 1.

So vi iu kin x 0 , ta nhn x = 1.
Vy, phng trnh cho c mt nghim l x = 1.
2)

2 x +

4
= 2 (2).
2 x +3

iu kin: 2 x 0 x 2 .
t t = 2 x , (t 0). Khi phng trnh (2) tr thnh

t+

t = 1
4
= 2 t 2 + 3t 2t 2 = 0 t 2 + t 2 = 0
t +3
t = 2

Ta loi t = 2 < 0 .
Vi t = 1 th
2 x = 1 2 x = 1 x = 1.
So vi iu kin x 2 , ta nhn x = 1.
Vy, phng trnh cho c mt nghim l x = 1.
3)

9 5x = 3 x +

6
( 3)
3 x

iu kin:

9 5 x 0 x
9

5 x (*)

5
3 x > 0
x < 3
Vi iu kin (*) ta c phng trnh (3) tng ng vi
197

( 9 5 x )( 3 x ) = 9 x
5 x 2 24 x + 27 = 9 x
2 x 2 3x 27 = 0
x = 3

x = 9
2

9
So vi iu kin x , ta c nghim ca phng trnh cho l x = 3 .
5

4)

4
x + x2 + x

1
x x2 + x

3
(4)
x

x + x2 + x 0

iu kin: x x 2 + x 0
x 0

(4) 4( x x 2 + x ) x x 2 + x = 3
3 x 5 x 2 + x = 3
5 x 2 + x = 3x + 3
3 x + 3 0
2
2
9 x + 18 x + 9 = 25 x + 25 x
x 1
2
16 x + 7 x 9 = 0
x 1

x = 1

x = 9

16
x = 1

x = 9
16

So vi iu kin, ta c nghim ca phng trnh cho l x = 1, x =


5)

x 2 + 2 x + 1 + x 2 2 x + 1 = 2 .(5)

( 5 ) ( x + 1)
2

( x 1)

+
2

= 2 x + 1 + x 1 = 2

( x + 1) + ( x 1) + 2 x + 1 x 1 = 4 2 x 2 + 2 + 2 x 2 1 = 4

198

9
.
16

1 x 2 0
x + x 1 = 1 x 1 = 1 x 2
2
2
x 1 = 1 x
2

) (

1 x 1.

Vy, nghim ca phng trnh cho l 1 x 1.


6) (2 x 2 + 6 x + 10) x 2 + 3 x 11x 2 33 x + 8 = 0.
t t = x 2 + 3 x 0 , khi phng trnh cho tr thnh
1
2t 3 11t 2 + 10t + 8 = 0. Phng trnh ny c ba nghim t = 2; t = 4; t = . Ta ch nhn
2
t = 2; t = 4.

Vi t = 2, ta c

x = 1
x 2 + 3x = 2 x 2 + 3x 4 = 0
x = 4

Vi t = 4, ta c

3 73
x =
2
x 2 + 3 x = 4 x 2 + 3 x 16 = 0

3 + 73
x =
2

T phng trnh cho c bn nghim l x =

2 x 2 + 4 x 3 x + 2 2 x + 3 2 x = 0 (1). Ta c

7)

(1)

2 x( x + 2) 3 x + 2

2x x + 2 3 x + 2

3 73
; x = 1; x = 4.
2

2x 3

)(

2x

2x

+ 3 2x = 0

+ 3 2x = 0 x + 2

2x 3 2x

2x 3 = 0

x + 2 2x = 0

2x 3 = 0 x + 2 2x = 0 x =

9
x = 2.
2

9
Vy, phng trnh cho c hai nghim l x = , x = 2.
2

8) 2 3 3x 2 + 3 6 5 x 8 = 0. (1)
t3 + 2
t t = 3 x 2 t = 3 x 2 x =
.
3
3

Phng trnh (1) tr thnh


2t + 3 6 5.
2t + 3

18 5t 3 10
t3 + 2
8 = 0 2t + 3
8 = 0
3
3

8 5t 3
8 = 0 2t + 24 15t 3 8 = 0 24 15t 3 = 8 2t
3

199

8 2t 0
t 4

3
3
2
2
24 15t = 64 32t + 4t
15t + 4t 32t + 40 = 0
t 4
t 4

2
( t + 2 ) 15t 26t + 20 = 0 t = 2.

Vi t = 2 ta c

3x 2 = 2 3 x 2 = 8 x = 2.

Vy, phng trnh cho c mt nghim duy nht l x = 2.


9)

x + 1 = 32 4 x 4 x + 1 x

iu kin: x 0.

Nhn xt rng x = 0 khng l nghim ca phng trnh cho nn chia hai v ca phng
trnh cho x 4 x 0 ta c
4

4
x +1
x +1
=
32

4
x4 x
x

1
x

t t =

x +1
x +1
= 32 4
.
x
x
4

x +1 4
1
1
1
= 1 + t 4 = 1 + = t 4 1.
x
x
x
x

Ta c phng trnh t 4 1 t = 32 t t 5 = 32 t = 2.
Suy ra 4 1 +

1
1
1
1
= 2 1 + = 16 = 15 x = . Tha iu kin (*).
x
x
x
15

Vy, phng trnh cho c mt nghim duy nht l x =


10)

2 x = 1 x 1(1)

u = 2 x
t
, suy ra u 3 + v 2 = 1.
v = x 1 0

Khi phng trnh (1) c chuyn thnh h


u 3 + v 2 = 1 v = 1 u
v = 1 u
3
3

2
2
u + (1 u ) 1 = 0
u + v = 1
u + u 2u = 0
v = 1 u

u = 0 u = 1 u = 2
u = 0

v = 1 v = 0 v = 3
u = 1
u = 2

200

1
.
15

3
3
3
2 x = 0 2 x = 1 2 x = 2

x 1 = 1 x 1 = 0 x 1 = 3
x = 2 x = 1 x = 10.

Ch . C th t n ph t = 3 2 x nh trnh by Bi IV.3. 8).


11)

9 x = 2 x 1 . Trnh by tng t nh Bi IV.3. 8).

Phng trnh c ba nghim l x = 1; x = 10; x = 17.


12) 2 3 1 x 2 + 4 x 2 = 4(1)
Gii tng t nh Bi IV.3. 8). Phng trnh c mt nghim duy nht x = 0.
IV.4. 1) x + 2 x 1 + x 2 x 1 =

x+3
(1)
2

t t = x 1 0

(1)

t 2 + 1 + 2t + t 2 + 1 2t =

t2 + 4

( t + 1)

( t 1)

t2 + 4
2

t 1
t 1

2
2
t 1

t
+
4
t +1+ t 1 =
2t = t + 4
2

t +4
2
2
t 4t + 4 = 0
t +1+ t 1 =

t < 1
2
t < 1
t < 1
2

t = 0
t2 + 4
t2 + 4
t + 1 t + 1 =
2 =
2
2

t = 2

t = 0
Vi t = 2 x 1 = 2 x 1 = 4 x = 5.
Vi t = 0 x 1 = 0 x 1 = 0 x = 1.

x =5
Vy, phng trnh c hai nghim l
x = 1.

2) 2 x + 2 + 2 x + 1 x + 1 = 4 ( 2 )
t t = x + 1 0

( 2) 2

t 2 + 1 + 2t t = 4 2

( t + 1)

t = 4 2 ( t + 1) t = 4 t + 2 = 4 t = 2

x +1 = 2 x +1 = 4 x = 3
Vy, phng trnh cho c mt nghim l x = 3 .
201

3)

2 x2 2 x + 4
x+2

)=2

x + 2 + 3 x 2 2 x + 4 (1)

x+2>0
iu kin: 2
x > 2
x 2x + 4 0

(1) 2 ( x2 2 x + 4 ) = 2 ( x + 2 ) + 3

(x

2x + 4) ( x + 2)

2
u = x 2 x + 4 0
t
v = x + 2 0

u = 2v
Phng trnh tr thnh 2u = 2v + 3uv 2u 3uv 2v = 0
u = v .

2
2

Ta ch nhn trng hp u = 2v.


Vi u = 2v ta c x 2 2 x + 4 = 2 x + 2
x = 3 13
x2 2 x + 4 = 4 ( x + 2 ) x 2 6 x 4 = 0
x = 3 + 13
x = 3 13
Vy, phng trnh cho c hai nghim l
x = 3 + 13.

Ch . C th bin i (1) tng ng vi


t t =

x2 2 x + 4
x+2

2 x2 2x + 4
x+2

) = 2 + 3.

x2 2x + 4
.
x+2

> 0,

1
Ta c phng trnh 2t 2 3t 2 = 0 t = 2 t = . Ta ch nhn trng hp t = 2. T y
2
x = 3 13
ta tm c nghim ca phng trnh cho l
x = 3 + 13.

4) 2( x 2 3x + 2) = 3 x 3 + 8 .
Ta c 2( x 2 3x + 2) = 3 x 3 + 8 2( x 2 3x + 2) = 3 ( x + 2)( x 2 2 x + 4)
2( x 2 3x + 2) = 3 ( x + 2)( x 2 3 x + 2 + x + 2) .
t t = x 2 3x + 2 ta c phng trnh tr thnh 2t = 3 ( x + 2) [ t + ( x + 2)] (1).

202

iu kin: t 0.

3
t=
( x + 2)
2
2
2

(1) 4t = 9 ( x + 2)t + ( x + 2) 4t 9( x + 2)t 9( x + 2) = 0


.
4

t = 3( x + 2)
2

Vi t =

3
3
( x + 2) x 2 3 x + 2 = ( x + 2) 4 x 2 9 x + 14 = 0 (V nghim).
4
4

x = 3 13
Vi t = 3( x + 2) x 2 3x + 2 = 3( x + 2) x 2 6 x 4 = 0
x = 3 + 13

Vy, phng trnh cho c hai nghim x = 3 13 x = 3 + 13.


5) x 3 + (1 x 2 )3 = x 2(1 x 2 ) .
iu kin: 1 x 1 .

Ta c x 3 + (1 x 2 )3 = x 2(1 x 2 ) ( x + 1 x 2 )(1 x 1 x 2 ) = x 2. 1 x 2 (1).


t 2 1
t t = x + 1 x , 1 x 1
= x 1 x2 .
2
2

Ta c t = 1

x
1 x2

= 0 1 x2 x = 0 x =

2
.
2

Bng bin thin

Da vo bng bin thin ta c 1 t 2 (*).


t2 1
t 2 1
Vi t = x + 1 x , 1 t 2 th (1) tr thnh t (1
)= 2
(2). Ta c
2
2
2

(2) 2t t (t 2 1) = 2 (t 2 1)
t 3 + 2t 2 3t 2 = 0
203

t = 2

(t 2)(t 2 + 2 2t + 1) = 0 t = 2 + 1 , kt hp vi iu kin (*) ta c

t = 2 1

t = 2 t = 2 +1.
Vi t = 2 x + 1 x 2 = 2
x 2
2 x 0
x 2
2

2 x= 2 .
2
2
2
1 x = 2 + x 2 2 x
2 x 2 2 x + 1 = 0
x =
2

x 1 2
Vi t = 2 + 1 x + 1 x 2 = 2 + 1
2
2
2 x 2(1 2) x + (1 2) 1 = 0
x 1 2

1 2 2 2 1
1 2 2 2 1

.
x =
x=
2

1 2 + 2 2 1

x =
2

Vy, phng trnh cho c hai nghim x =

2
1 2 2 2 1
x=
.
2
2

Ch . Ta c th gii bng phng php lng gic ha bng cch t


x = sin t , t [

; ]. (Do iu kin 1 x 1 )
2 2

Khi phng trnh c a v


sin 3 t +

(1 sin t )
2

= sin t 2 1 sin 2 t

sin 3 t + cos3 t = 2 sin t cos t


3

( sin t + cos t ) 3 ( sin t + cos t ) sin t cos t = 2 sin t cos t


sin t + cos t = 2 (*)

sin t + cos t = 1 2(**)

204

Gii trng hp (*) ta c x =

2
.
2

Gii trng hp (**) ta c x =

1 2 2 2 1
.
2

Vy, phng trnh cho c hai nghim x =

2
1 2 2 2 1
x=
.
2
2

6) 1 + 1 x 2 (1 x)3 (1 + x )3 = 2 + 1 x 2 .

iu kin: 1 x 1 (*).

Ta c: 1 + 1 x 2 (1 x)3 (1 + x )3 = 2 + 1 x 2

1 + 1 x 2 ( 1 x 1 + x )(2 + 1 x 2 ) (2 + 1 x 2 ) = 0

1 + 1 x 2 .( 1 x 1 + x ) 1 (2 + 1 x 2 ) = 0

2 + 1 x 2 = 0 (VN )

1 + 1 x 2 .( 1 x 1 + x ) = 1 (2)

Gii (2): 1 + 1 x 2 ( 1 x 1 + x ) = 1 .
t t = 1 x 1 + x , 2 t 2 (**).

t2
1 = 1 x 2 , ta c
2
t > 0
t2

(2) 1 + 1 .t = 1 2 4 t 2
2
=1
t

2
t > 0
4
2
t + 4t 2 = 0

205

t > 0

t = 2 + 2
t = 2 + 2

t = 2 2
, kt hp vi iu kin (**) ta c: t = 2 2 .

t = 2 2

t = 2 2

t = 2 + 2

Vi t = 2 2 1 x 1 + x = 2 2
1 x > 1 + x
1 x < 0
1 x 1 + x > 0

2 1 x 2 = 1
2
2
1 x =

2 2 1 x = 2 2
2
2

1 x < 0
1

tho iu kin (*).


1
1 x=
2
x = 2 x = 2

Vy, phng trnh cho c mt nghim x =

1
.
2

Ch . Chng ta c th gii bng phng php lng gic ha nh sau:


Do iu kin 1 x 1 nn t x = cos t , t [0; ]. Khi ta c phng trnh
1 + sin t

) ( 2 + sin t ) = 2 + sin t
1 + cos t ) = 1

1 cos t 1 + cos t

1 + sin t

1 cos t

1 cos t 1 + cos t 0
1 cos t 1 + cos t 0

1
2
+

=
1
sin
t
2
2sin
t
1
(
)(
)
1 sin t =

2
1 cos t 1 + cos t 0
1 cos t 1 + cos t 0

1
2
2
cos
t
=
cos t =

cos t =

2
2

cos t =

2
2
x=
.
2
2

7) 1 x 2 x 1 x 2 2 x 2 + 1 = 0. (1)
iu kin: 1 x 1.

206

t x = cos t , t [0; ].

(1)

1 cos t 2 cos t 1 cos 2 t 2 cos 2 t + 1 = 0

2 sin
2 sin

t
2cos t sin t cos 2t = 0
2

t
t
s in2t cos2t = 0 (v t [0; ] sin 0,sin t 0 )
2
2

2 sin 2 cos 2t = 0
2
4

sin

= cos(2t )
2
4

sin

= sin( 2t + )
2
2
4

sin

t
3

= sin
2t
2
4

t 3
2 = 4 2t + k 2

t = 2t + + k 2
2
4
5t 3
2 = 4 + k 2

3 = k 2
2
4
3 k 4
t = 10 + 5

,(k ).
t = k 4

6
3

Do t [0; ] nn ta nhn t =
Hay x = cos

8)

3
.
10

3
l nghim ca phng trnh cho.
10

) (1)
( )

x2 +1
x2 + 1
x +1 +
=
2x
2 x 1 x2
2

x 0
iu kin:
x 1

207


t 2 ; 2


t x = tan t. Khi iu kin ca t l t 0

t
4

tan 2 t + 1
tan 2 t + 1
tan t + 1 +
.
=
2 tan t
2 tan t 1 tan 2 t
2

(1)

1
1
1
+
=
2
cos t 2cos t tan t 2cos 4 t tan t 1 tan 2 t

1
1
1
+
=
cos t 2sin t cos t 2sin t cos t cos 2 t sin 2 t

(
)
2 sin t (1 2 sin t ) 2 sin

2 sin t cos 2 t sin 2 t + cos 2 t sin 2 t = 1


2

t =0

2 sin 3 t + sin 2 t sin t = 0

sin t 2 sin 2 t + sin t 1 = 0

t = 0
sin t = 0

sin t = 1 t = .

sin t =
t =

2
6

So snh iu kin ca t ban u ta thy ch c gi tr t =


Vy, vi t =

ta c x = tan

x=

tha.

1
.
3

Vy, phng trnh cho c mt nghim l x =

1
.
3

IV.5. 1) ( x 1) x 2 x 2 0 (1). Ta c
x2 x 2 = 0
x = 1 x = 2

x = 1

(1) x 2 x 2 > 0 x < 1 x > 2


x 2

x 1
x

Vy, nghim ca bt phng trnh cho l x 2 hoc x = 1 .

208

2) ( x 2 1) x 2 x 2 0 (1). Ta c
x2 x 2 = 0
x = 1 x = 2

x 1

x 1 x 1
(1) x 2 1 0
x 2
2
x < 1 x > 2

>
x
x
2
0

x 1
Vy, nghim ca bt phng trnh cho l
.
x 2

3) 2 x x 2 < 5 x (1). Ta c
5 x > 0
x < 5

2
(1) 2 x x 0
0 x 2
0 x 2.
2 x 2 12 x + 25 > 0

2
2

2 x x < ( 5 x )

Vy, nghim ca bt phng trnh cho l 0 x 2 .


4) x 2 3x + 2 x 3 > 0 (1). Ta c
(1) x 2 3 x + 2 > x + 3
x + 3 < 0
x < 3
x < 3
2

x 3x + 2 0
x 1 x 2
x 3 x < 7

x+3 0
9

x 3
7

x
<

x 2 3 x + 2 > ( x + 3)
9

9 x + 7 < 0

7
Vy, bt phng trnh cho c nghim l x < .
9

5)

x + 3 + x + 2 2 x + 4 > 0 (1)

iu kin: x 2 (*)

(1) x + 3 + x + 2 > 2 x + 4
2 x + 5 + 2 ( x + 3)( x + 2) > 2 x + 4
1
( x + 3)( x + 2) > (2)
2
Bt phng trnh (2) lun ng. Kt hp vi (*) ta c x 2 l nghim ca (1).
6)

3x 2 + 5 x + 7 3x 2 + 5 x + 2 > 1 (1)

t t = 3 x 2 + 5 x + 2 0(*)

(1) tr thnh t 2 + 5 > 1 + t. (2)


209

Do t 0 nn t + 1 > 0. V vy bnh phng hai v ca (2) ta c


t 2 +5 > 1 + 2t + t 2 t < 2 . Nh vy ta c
x 1

2 < x 1
2

x 2
3
x
5
x
2
0
+
+

0 t < 2 3x + 5 x + 2 < 2 2

1
3 2
x< .
3 x + 5 x + 2 < 4

3
1
3
2 < x <
3

2 < x 1
Vy, bt phng trnh cho c nghim l 2
x < 1 .
3
3

7)

x + x + 9 x + 1 + x + 4 (1)

x 0
x 0
x + 9 0
x 9

iu kin:

x 0.
x +1 0
x 1
x + 4 0
x 4
(1) 2 x + 9 + 2 x 2 + 9 x 2 x + 5 + 2 x 2 + 5 x + 4 2 + x 2 + 9 x x 2 + 5 x + 4
4 + 4 x 2 + 9 x + x 2 + 9 x x 2 + 5 x + 4 x 2 + 9 x x (*)

Ta c bt phng trnh (*) ng vi m i x 0 .


Vy, nghim ca bt phng trnh cho l x 0 .
8)

5 x 1 x 1 > 2 x 4 . (1)

x 5
5 x 1 0

iu kin: x 1 0 x 1 x 2
2 x 4 0 x 2

Vi x 2
(1) 5 x 1 > 2 x 4 + x 1

5 x 1 > 2 x 4 + x 1 + 2 (2 x 4)( x 1) (2 x 4)( x 2) < x + 2, ( x 2)


(2 x 4)( x 1) < ( x + 2) 2 x 2 + 4 x + 4 > 2 x 2 6 x + 4
x 2 10 x < 0 0 < x < 10.
Kt hp vi iu kin x 2 ta c 2 x < 10 . Vy, 2 x < 10 l nghim ca bt cho.
9)

210

x 2 + 3x + 2 + x 2 + 6 x + 5 2 x 2 + 9 x + 7 (1)


x 2

x 1
2
x + 3x + 2 0

x 5
x 5
iu kin: x 2 + 6 x + 5 0
(*)

x
x

2 x + 9 x + 7 0.

7
x
2

x 1

Vi iu kin bt phng trnh (1) tng ng vi


2 x 2 + 9 x + 7 + 2 ( x + 1)( x + 2). ( x + 1)( x + 5) 2 x 2 + 9 x + 7
( x + 1) 2 ( x + 2)( x + 5) 0 ( x + 1) 2 ( x + 2)( x + 5) = 0
x = 1
x = 2
x = 5.
Kt hp vi iu kin (*) ta c x = 1 , x = 5 l nghim ca bt phng trnh cho.
10)

x 2 + x 2 + x 2 + 2 x 3 x 2 + 4 x 5.

iu kin:
x2 + x 2 0
x 2 x 1
2
x 5

x + 2 x 3 0 x 3 x 1
2
x 5 x 1 x 1.

x + 4x 5 0

Ta c

x 2 + x 2 + x 2 + 2 x 3 x 2 + 4 x 5.

2 x 2 + 3 x 5 + 2 ( x 2 + x 2)( x 2 + 2 x 3) x 2 + 4 x 5.
x( x 1) + 2 ( x 1)2 ( x + 2)( x + 3) 0 .
Vi x 5 ta c
x( x 1) + 2 ( x 1)2 ( x + 2)( x + 3) 0
x( x 1) 2( x 1) ( x + 2)( x + 3) 0
( x 1)( x 2 ( x + 2)( x + 3) ) 0 (*)

Nhn xt: Vi x 5 suy ra x 1 < 0 v x 2 ( x + 2)( x + 3) < 0 . Suy ra (*) v nghim.


Vi x 1 suy ra x 1 0

Ta c x( x 1) + 2 ( x 1) 2 ( x + 2)( x + 3) 0

211

x( x 1) + 2( x 1) ( x + 2)( x + 3) 0
( x 1)( x + 2 ( x + 2)( x + 3) ) 0 ( x 1)( x + 2 ( x + 2)( x + 3)) = 0.
x = 1. (Do x + 2 ( x + 2)( x + 3) > 0, x 1 )

Vy, bt phng trnh cho c mt nghim duy nht l x = 1.


11)

1 1 4x2
< 3(1)
x

1
1
x<0
1

1 4 x 2 0

2
iu kin:
2

0 < x 1
x 0
x 0

2
+ Xt trng hp

1
x < 0.
2

1 3 x > 0
(1) 1 4 x 2 < 1 3x
2
2
1 4 x < (1 3 x )
1

x <

x < 0.
3
13 x 2 6 x > 0

Kt hp vi iu kin ang xt ta c nghim l

1
x < 0.
2

1
+ Xt trng hp 0 < x .
2
1 3 x < 0

2
1 4 x 0
2
(1) 1 4 x > 1 3x
1 3x 0

1 4 x 2 > (1 3x ) 2

1
x > 3

1
1
1 x 1
3 < x 2
1
2
2
0< x .

2
0 < x 1
x 1
3


3
2
13 x 6 x < 0
1
Kt hp vi iu kin ang xt ta c nghim l 0 < x .
2

212

1
1
Vy, nghim bt phng trnh cho l x < 0 0 < x .
2
2

Ch . Chng ta c th nhn hai v ca bt phng trnh cho vi biu thc lin hp ca


1 1 4 x 2 . Khi (1) tng ng vi

(1

)(

1 4 x2 1 + 1 4 x 2
x

) < 3 1+
(

1 4 x2

4 x < 3 1 + 1 4x2 .
n y bi ton c a v dng c bn.
IV.6. 1) 2 x 2 4 x + 3 x 2 + 4 x 5 (1)
iu kin: x 1 x 3 (*)

(1) 2 x 2 4 x + 3 + x 2 4 x + 5 0

1 + x2 4 x + 3

) + 1 0 ( 2)

Ta thy vi m i x tha (*) th (2) lun lun ng.


Vy, bt phng trnh cho c nghim l x 1 x 3 .
2) 5 x +

5
2 x

2x +

1
+ 4 (1)
2x

iu kin: x > 0.
2

1
1
x
(1) 5 x +

2
+
+
1

0.

2 x
2 x

t t = x +

1
2 x

x.

1
2 x

= 2.

Khi , (1) tr thnh 2t 2 5t + 2 0 t 2 t


t 2 x +

1
2 x

1
. Ta ch nhn t 2.
2
2

2 2 x + 1 4 x ( 2 x + 1) 16 x

3+ 2 2
x
2
4 x 2 12 x + 1 0

3 2 2
x

3 2 2 3 + 2 2

Vy, bt phng trnh cho c nghim l x 0;


; + .

2 2

213

3)

x 1 + x + 3 + 2

( x 1)( x + 3) > 4 2 x (1)

iu kin: x 1

(1)

x 1 + x + 3

+ x 1 + x + 3 6 > 0 ( 2)

t t = x 1 + x + 3 > 0

(2) tr thnh t 2 + t 6 > 0 t > 2 t < 3 ( 3)


Do t > 0 nn t (3) ta nhn t > 2 x 1 + x + 3 > 2
2x + 2 + 2

( x 1)( x + 3) > 4 x + 1 + ( x 1)( x + 3) > 2 x + ( x 1)( x + 3) > 1

Bt phng trnh trn ng vi m i x > 1.


Vy, x > 1 l nghim ca bt phng trnh cho.
4) x 3 + x 2 + 3x x + 1 + 2 > 0 (1)
t t = x x + 1

Xt hm s y = f ( x ) = x x + 1
y =

3x + 2
2
2 3
=0 x= ;y =
3
9
2 x +1

Bng bin thin

T bng bin thin ta suy ra t

2 3
, x 1
9

t < 2
(1) Tr thnh t 2 + 3t + 2 > 0
t > 1
Kt hp vi iu kin ca t ta c t

2 3
.
9

Vy, nghim ca bt phng trnh cho l x 1.


5)

7 x + 7 + 7 x 6 + 2 49 x 2 + 7 x 42 < 181 14 x (1)

iu kin: x

214

6
.
7

t t = 7 x + 7 + 7 x 6 > 0

t 2 = 14 x + 1 + 2

( 7 x + 7 )( 7 x 6) t 2 1 = 14 x + 2

49 x 2 + 7 x 42

t < 13
(1) t 2 + t 1 181 < 0 t 2 + t 182 < 0
t > 14
Do t > 0 0 < t < 13.
Nh vy ta c
6

x 7
7 x + 7 + 7 x 6 < 13
14 x + 1 + 2 49 x 2 + 7 x 42 < 169

x 7

7 x + 84 > 0
49 x 2 + 7 x 42 < 7 x + 84

2
2

49 x + 7 x 42 < ( 7 x + 84 )

6
6
x 12
7
x < 6.
7
x < 6
6
Vy, nghim ca bt phng trnh cho l x ;6 .
7
6) 2( x 2)2 + 2 x x 2 + x (1)
in kin: x 0.
t u = x 2, v = x

(1)

u + v 0
u + v 0
2u 2 + 2v 2 u + v 2

2
2
2
2
u + v 2uv 0
2u + 2v ( u + v )

u + v 0
u + v 0
u + v 0
2u 0
u 0

2
u = v
u = v
u = v
( u v ) 0 u v = 0
x 2 0
x 2
x 2
x 2

x = 4.
2
x = 1 x = 4
( x 2 ) = x
x 5x + 4 = 0
x 2 = x
Vy, nghim ca bt phng trnh cho l x = 4.
7)

2 ( x + 2 ) + 2 ( 2 x 1) > x + 2 + 2 x 1.

215

x 2

2
2 ( x + 2 )2 + 2 ( 2 x 1) > x + 2 + 2 x 1

x 2

2 ( x + 2 )2 + 2 ( 2 x 1) > ( x + 2 ) 2 + 2 x 1 + 2 ( x + 2 ) 2 x 1

2
2

2
( x + 2 ) + ( 2 x 1) 2 ( x + 2 ) 2 x 1 > 0
x + 2 2x 1

x
>0

1
2

1
Vy, nghim ca bt phng trnh cho l x .
2

8) x 2 + 4 x ( x + 4 ) x 2 2 x + 4 (1)
Xt x = 4. (1) ng. Do x = 4 l nghim ca bt phng trnh.

x 0
x 0

Xt x > 4. (1) x x 2 x + 4 x 2 2 x + 4 0
x2
x 2
x 2 x2 2 x + 4

Suy ra x 2 l nghim ca bt phng trnh.


Xt x < 4. (1) x x 2 2 x + 4

Bt phng trnh ng vi m i x < 4. Do , x < 4 l nghim ca bt phng trnh.


Vy, nghim ca bt phng trnh cho l x 2 x 4.
Ch . C th t t = x 2 2 x + 4 3. Khi (1) c a v dng
x 2 ( t 4 ) x 4t 0(2). Coi v tri ca (2) l tam thc bc hai theo bin x, ta c
2

= ( t + 4 ) do v tri ca (2) c cc nghim l x = 4; x = t.

Nh vy, (2) c vit li l ( x + 4 )( x t ) 0 ( x + 4 ) x x 2 2 x + 4 0.


n y vic gii bt phng trnh khng c g kh.

9) x 2 1 2 x x 2 + 2 x (1)
t t = x 2 + 2 x 0, (1) tr thnh
t 2 2 x 1 2 xt t 2 2 xt 2 x 1 0 ( t 2 x 1)( t + 1) 0(2). Do t = x 2 + 2 x 0,

Nn (2) tng ng vi
216

t 2x 1 0 x2 + 2x 2x 1 0
1

x 2
x 2

x2 + 2 x 2 x + 1 x2 + 2 x 0
x 2 x 0 x 0.
2
2
2
x + 2 x ( 2 x + 1)
3 x + 2 x + 1 0

Vy, x 0 l nghim ca bt phng trnh cho.


10) ( x 1) 2 x 1 3 ( x 1)(1)
Xt x = 1, (1) ng suy ra x = 1 l nghim ca bt phng trnh.

2 x 1 0
x
Xt x > 1, (1) 2 x 1 3

2
2 x 1 9
x 5
Suy ra 1 < x 5 l nghim ca bt phng trnh.
Xt x < 1, (1) 2 x 1 3 2 x 1 9 x 5, trng hp ny b loi.

Vy, nghim ca bt phng trnh cho l 1 x 5.

)(

11) x 2 1

x2 1 1 + x 2 1 6 > 0

t t = x 2 1 0 , ta c bt phng trnh
t 2 (t 1) + t 6 > 0 t 3 t 2 + t 6 > 0 (t 2)(t 2 + t + 3) > 0 t > 2 .

Nh vy ta c

x < 5
x2 1 > 2 x 2 1 > 4
x > 5

Vy, nghim ca bt phng trnh cho l x < 5 x > 5.


12) x 2 x 1 5 4 x.
t 2 +1
t t = 2 x 1 0 t = 2 x 1 x =
. Ta c bt phng trnh
2
2

t2 +1
t2 +1
3
2
t

4.

t + t 10 4t 4
2
2
t 3 + 4t 2 + t 6 0 ( t + 2 )( t + 3 )( t 1) 0
t 1 0, (t 0)
t 1.

217

Nh vy ta c

1
x
2 x 1 1
2 x 1.
2
x 1

Vy, nghim ca bt phng trnh cho l


13)

1
x 1.
2

x3 2 x 2 + x < x x + x 2 2 x (1)

x2 2x 0

iu kin: x 3 2 x 2 0 x 2.
x 0

(1) x x 2 2 x + x < x x + x 2 2 x
u = x 2 2 x
t
, iu kin u , v 0. Ta c bt phng trnh
v = x

uv + v 2 < v 3 + u uv + v 2 v 3 u < 0 ( v 1) u v 2 < 0



v 1 < 0

2

u
v
0

>

v 1 > 0

2
u v < 0


x2 2 x x > 0


x 1 > 0


x2 2 x x < 0
x 1 < 0

x <1
x2 2 x > x
x >1
x2 2 x < x

(I )

( II )

Do x 2 nn ( I ) v nghim.
x > 1
x > 1
Ta gii ( II )
2
x > 1.
2
2
x

2
x
<
x
x

2
x
<
x

Kt hp vi iu kin x 2 th nghim ca bt phng trnh cho l x 2.


IV.7. 1)

(1)

218

41

16 2
+ 3 (1)
x x

2
2
x + 3 0
3 x < 0

2

41x 16 0
3 x<0
16

x < 0 x

41
2


x>0
x

x 2 x > 0
x +3 0
2

2
8 x 7 x 1 0

41
x

16
2

+ 3
x(41x 16) = 9 x + 12 x + 4

x
x

2
3 x<0
2

3 x<0
2

x
x>0

3
x 2 x 1

3
x 1 x 1

8
Vy, nghim ca bt phng trnh cho l x < 0 x 1.
2) x 2 x (2 + 12 2 x x 2 ) (1)
iu kin: 12 2 x x 2 0 1 13 x 1 + 13
Xt x = 0. Bt phng trnh (1) ng nn x = 0 l nghim ca bt phng trnh.
Xt 0 < x 1 + 13

Bt phng trnh cho tng ng vi


0 < x 1 + 13

2
x 2 + 12 2 x x
0 < x 1 + 13

2
12 2 x x x 2
2 x 1 + 13

2
2
12 2 x x x 4 x + 4
2 x 1 + 13

2
x x 4 0
2 x 1 + 13

1 17
1 + 17
x
x

2
2

1 + 17
x 1 + 13
2

Xt 1 13 x < 0.

Bt phng trnh cho tng ng vi


1 13 x < 0

2
x 2 + 12 2 x x
1 13 x < 0

2
12 2 x x x 2
1 13 x < 0.
219

Vy, nghim ca bt phng trnh cho l 1 13 x 0


3)

4 x2 +

1 + 17
x 1 + 13.
2

x
0 (1). Ta c bt phng trnh (1) tng ng vi
x

x > 0
x > 0
x > 0

4 x 2 + 1 0
4 x 2 1 4 x 2 0

x<0
x < 0
x < 0

2

2
2
4 x 1
4 x 1 0
4 x 1
x > 0

0 < x 2
3 x 2
2 x 2

x < 0
3 x < 0 x 0

3 x 3
3 x 2
Vy, nghim ca bt phng trnh cho l
x 0
4)

x 2 13 x + 40
19 x x 2 78

0 (1)

iu kin: 19 x x 2 78 > 0 6 < x < 13 (*). Khi

(1) x 2 13x + 40 0
5 x 8.
So vi iu kin (*) th nghim ca bt phng trnh cho l 6 < x 8.
5) x 1 x < 0 (1)
Bt phng trnh (1) tng ng vi
x 0
x 0
x 0

x 0

2
1 x > x
2

1 x > x

x + x 1 < 0 1 5 < x < 1 + 5

x<0
2
2
x < 0
1 x < 0

1 + x 0
1 x < 0
1 + x > x

1 + 5
1 + 5
0 x<

1 x <
.
2

2
1 x < 0
Vy, nghim ca bt phng trnh cho l 1 x <

220

1 + 5
.
2

2 ( x 2 16 )

6)

x 3

7
(1)
x 3

+ x3 >

x 2 16 0
x 4 x 4
iu kin:

x 4(*)
x > 3
x 3 > 0
(1) 2 ( x 2 16 ) + x 3 > 7 2 ( x 2 16 ) > 10 x

10 x < 0
x > 10

2 ( x 16 ) 0
x 4 x 4


x 10
10 x 0

2
2
x 2 + 20 x 132 > 0
2 ( x 16 ) > 100 20 x + x
x > 10
x > 10

x 10

x < 10 232 10 + 232 < x 10


x < 10 232 x > 10 + 232

x < 10 232

x > 10 + 232
Kt hp vi iu kin (*) ta c nghim ca bt phng trnh cho l x > 10 + 232.
x 2 + 4 x + 4 x + 4 2 x 8 x > 0(1)

7)

iu kin: x 0. (1) tng ng vi

x x + 4 + 4 x + 4 2x 8 x > 0 x + 4

x +4

)(

x +4 2 x

x +4 >0

x+4 2 x >0 x+4 2 x >0 x+4 > 2 x

4
x + 4 > 4x x < .
3
4
Vy, nghim ca bt phng trnh cho l 0 x < .
3

8)

1
3x
+1 >
.
2
1 x
1 x2

iu kin: x < 1.

Bt phng trnh cho tng ng vi


1 x2 + x2
3x
x2
3x
+
1
>

+ 2 > 0 (1).

2
2
2
1 x
1 x
1 x
1 x2

221

t t =

t < 1
, khi bt phng trnh (1) tr thnh t 2 3t + 2 > 0
1 x
t > 2.
x

a) Vi t < 1 th

1 x

<1 x < 1 x 2 (2).

Nu 1 < x 0 th bt phng trnh (2) ng.


Nu 0 < x < 1 th bt phng trnh (2) x 2 < 1 x 2 0 < x <

1
.
2

Tp hp nghim ca bt phng trnh (2) l S1 = 1;


.
2

b) Vi t > 2 th

x
1 x2

> 2, do iu kin x < 1, nn ta c bt phng trnh

x > 2 1 x 2 (3).
x > 0
2 5
Bt phng trnh (3) 2
.
x>
2
5
x > 4(1 x )

Kt hp vi iu kin x < 1 ta c tp hp nghim ca bt phng trnh (3) l


2 5
S 2 =
;1 .
5

1 2 5

Tp hp nghim ca bt phng trnh cho l S = S1 S2 = 1;


;1 .

2 5

9)

x > 1 + 3 x 1(1)

Cch 1. Ta nhn thy rng x = 0 khng phi l nghim ca (1) nn iu kin ca n l


x > 0. Vi iu kin x > 0 th hai v ca bt phng trnh (1) u dng nn bnh phng
hai v ca bt phng trnh ta c

x > 1 + 3 x 1

x > 1+ 2 3 x 1 +

x 1

x 1 2 3 x 1

x 1

> 0.

t t = 3 x 1 > 1, ( x > 0) ta c
t 3 t 2 2t > 0 t (t + 1)(t 2) > 0 t (t 2) > 0, (t + 1 > 0)
3 x 1 > 2
t > 2
x > 9

1 < t < 0
0 < x < 1.
1 < 3 x 1 < 0

Cch 2. t t = 3 x 1 t 3 = x 1 x = t 3 + 1. Ta c bt phng trnh


t 3 + 1 > 1 + t (2) . Ta cng nhn thy rng x = 0 khng phi l nghim ca (1) nn iu
kin ca n l x > 0, do 1 + t > 0.
222

Bnh phng hai v ca (2) ta c


t 3 + 1 > t 2 + 2t + 1 t 3 t 2 2t > 0 t ( t + 1)( t 2 ) > 0.
n y tip tc gii nh Cch 1 ta c nghim ca bt phng trnh cho.

10) x +

2x
x2 4

> 3 5(1)

x < 2
iu kin:
x > 2
Trng hp 1: x < 2 . Bt phng trnh (1) v nghim.
Trng hp 2: x > 2 . Bnh phng hai v ca (1) ta c bt phng trnh tng ng
2

2x
4x2
4x2
2
+ 2
> 45
x+ 2
> 45 x + 2
x 4
x 4 x 4

x 2 ( x 2 4 ) + 4 x2
x2 4

4x2
x2 4

> 45

x4
4x2
2
+
45 > 0.
x 4
x2 4
t t =

x2
x2 4

> 0.

t < 9
Ta c bt phng trnh t 2 + 4t 45 > 0
t > 5
Do t > 0 nn ta chn
t > 5

x2
x2 4

>5

x4
> 25 x 4 25 x 2 + 100 > 0
2
x 4

x < 2 5 x > 2 5
x 2 > 20
2

x < 5
5 < x < 5

Kt hp vi iu kin x > 2 ta c nghim ca bt phng trnh cho l


2 < x < 5 x > 2 5.
IV.8.

x 5 2 x 6 + x 4 x 6 2 = m (1)

Ta c (1) tng ng vi

x 6 1 +

x 6 1 +

x6 2

=m

x 6 2 = m.

t t = x 6 0 . Ta c phng trnh t 1 + t 2 = m(2)

223

Xt hm s f ( t ) = t 1 + t 2 vi t 0
2t + 3; 0 t < 1

f (t ) = 1
;1 t < 2
2t 3 ; t 2

V th hm s f ( t )
f(t)
3

1
O

Da vo th ta c, phng trnh cho c ng hai nghim khi v ch khi 1 < m 3.


IV.9. 1) Ta c x + 4 x 2 + x 4 x 2 = m

2( x + 4 x 2 ) + 4 + 2 x 4 x 2 ) = 2m + 4
2( x + 4 x 2 ) + ( x + 4 x 2 )2 = 2m + 4 (1).
iu kin: 4 x 2 0 2 x 2 .
t t = f ( x) = x + 4 x 2 , 2 x 2. Khi (1) tr thnh

t 2 + 2t = 2m + 4 (2)
Ta tm min gi tr ca t.
Ta c

f ( x) = 1

x
4 x2

4 x 2 = x 2
2

4 x x

4 x = x
=0
x 0
x = 2.
2
4 x
2 < x < 2
2 < x < 2

=0

Cc im ti hn ca hm s f ( x ) l x = 2; x = 2; x = 2 .
Ta c: f (2) = 2 ; f (2) = 2 ; f ( 2) = 2 2 .
Maxf ( x ) = 2 2 , Min f ( x) = 2 . Vy, t 2; 2 2 . Nh vy, (1) c nghim
[ 2;2]
[ 2;2]
224

x [2; 2] khi v ch khi (2) c nghim t 2; 2 2 . iu ny xy ra khi v ch khi


2m + 4 thuc min gi tr ca hm s g (t ) = t 2 + 2t trn on [2; 2 2].

g (t ) = t 2 + 2t g (t ) = 2t + 2 = 0 t = 1 .
Bng bin thin ca hm g (t )

Phng trnh (2) c nghim khi 1 2m + 4 8 + 4 2

5
m 2+2 2 .
2

5
Vy, khi m 2 + 2 2 th phng trnh cho c nghim.
2

2)

1
4
1
+
+ 4 x = 4( + 2 x ) + m (1).
2
x
x
x

iu kin: x > 0. Ta c
2

1
1
1
(1) + 2.2 x . + (2 x )2 = 4( + 2 x ) + m
x
x
x
2

1
1

+ 2 x = 4( + 2 x ) + m
x
x

+ 2 x 4 + 2 x = m .
x

t t =

1
+ 2 x ta c phng trnh t 2 4t = m(2)
x

Ta tm min gi tr ca t.
t =

1
1
x2 x
1
+
=
= 0 x = 1. Lp bng bin thin ca hm t = + 2 x trn min
2
2
x
x
x
x x

(0; +) ta c min gi tr ca t l [3; +).

225

Xt hm s f (t ) = t 2 4t , t [3; +). Ta c f (t ) = 2t 4 = 0 t = 2 .
Lp bng bin thin ca hm s f (t ) = t 2 4t , t [3; +) ta c min gi tr ca hm s l
[3; +).
Phng trnh (2) c nghim khi v ch khi m 3. .
Vy, m 3 th phng trnh cho c nghim.
5

IV.10. x 2 + m 2 x 2 + 4 + 2 m3 = 0(1)
3

t t = x 2 + 4 2 .

5
3
t m + 2 = 0(2). Nh vy, phng trnh (1) c nghim x khi v ch
3

khi phng trnh (2) c nghim t 2.

(1) t 2 + m2

Tam thc bc hai f (t ) v tri ca (2) c


2

2 5

=
m

+ 4 ( m3 + 2 ) > 0 , m > 0.

f 2 = m3 + 2m 2 4
( )
3

4
Xt g ( m ) = m3 + 2m 2 .
3

m
=
Ta c g ( m ) = 3m + 4m; g ( m ) = 0
3

m = 0
2

Ta c bng bin thin

Nh vy, vi m i m > 0 th g ( m ) < 0 . Hay f ( 2 ) < 0 vi m i m > 0.


T suy ra vi m i m > 0 th phng trnh ( 2 ) lun lun c hai nghim t1 , t2 phn bit,
v t1 < 2 < t2 .
T ta kt lun c phng trnh cho lun c nghim vi m i m > 0.
226

x + 1 + x + 4 + x 2 + 5 x + 4 + x + 2m = 0 (1)

IV.11.

x +1 0
x 1
iu kin:

x 1.
x + 4 0
x 4
(1)

1
2

x +1 + x + 4

) +(

x + 1 + x + 4 + 2m

5
=0
2

t t = x + 1 + x + 4 , x 1. R rng hm s t = x + 1 + x + 4 ng bin trn na


khong [1; +). Do vi x 0 th t 3.

Phng trnh (1) tr thnh t 2 + 2t + 4m 5 = 0 (2)


(1) c nghim khng m khi v ch khi (2) c nghim t 3. iu ny c tha khi v ch
khi 5 4m thuc min gi tr ca hm s f (t ) = t 2 + 2t trn na khong [3; +).
Lp bng bin thin ca hm s f (t ) ta nhn c min gi tr ca hm s trn na khong
[3; +) l [15; +). Vy gi tr cn tm ca tham s m phi tha
5
5 4m 15 4m 10 m .
2
Vy, vi m

5
th phng trnh cho c nghim khng m.
2

IV.12. 5 + x + 7 x + m (5 + x )(7 x ) = 2m + 1 (1)


t t = 5 + x + 7 x , 5 x 7

Xt t = h( x ) = 5 + x + 7 x (*)
t = h( x ) =

1
1

= 0 x =1
2 5+ x 2 7 x

Bng bin thin ca hm s h( x)

Da vo bng bin thin ca hm s h( x), ta c nhn xt sau:


ng vi t = 2 6 th (*) cho ta mt gi tr x
ng vi m i gi tr t tha 2 3 t < 2 6 th (*) cho ta hai gi tr x
ng vi m i gi tr t tha t < 2 3 t > 2 6 th (*) v nghim x.
t 2 12
Ta c (1) t + m
= 2m + 1
2

227

m=

2t 2
(2)
16 t 2

Xt f (t ) =
f (t ) =

2t 2
vi t 2 3; 2 6
16 t 2

2t 2 4t + 32
> 0, t 2 3; 2 6
(16 t 2 ) 2

Bng bin thin ca hm s f (t )

Da vo nhn xt trn v bng bin thin ca hm s f (t ) ta c kt qu nh sau:


i) Nu m =

1 2 6
th phng trnh (1) c mt nghim.
4

1 2 6
m <
4
ii) Nu
th phng trnh (1) c hai nghim.

2 3 1
m
2

iii) Nu

1 2 6
2 3 1
<m<
th phng trnh (1) v nghim.
4
2

IV.13. 3 x 1 + m x + 1 = 2 4 x 2 1.
iu kin: x 1 .

Vi x 1 ta c 3 x 1 + m x + 1 = 2 4 x 2 1
m=

t t =

2 4 x2 1 3 x 1
x +1
4

= 24

x 1
x 1
3
. (1)
x +1
x +1

x 1
.
x +1

Xt hm s h( x) = t =

x 1
Ta c h( x) =
x +1

1
x 1
2( x + 1) 2 4

x +1

> 0, x > 1

x 1
Do hm s h( x) ng bin trn [1; + ) . Ta c h(1) = 0 ; lim 4
= 1.
x +
x +1
228

Vy, khi x 1 th 0 t < 1


(1) tr thnh m = 3t 2 + 2t
Xt hm s f (t ) = 3t 2 + 2t
f (t ) = 6t + 2.
1
f (t ) = 0 6t + 2 = 0 t = .
3

Bng bin thin ca hm s f (t )

Da vo bng bin thin ta c


Vi 0 t < 1 th 1 < f (t )

1
.
3

1
Vy, gi tr m cn tm l 1 < m .
3

IV.14. m

x 2 2 x + 2 + 1 x 2 2 x + 6 + x 2 2 x + 2 (1)

t t = x 2 2 x + 2

Bt phng trnh (1) tr thnh m ( t + 1) t 2 + 4 + t


Vi x [ 0; 2 ] th t 1; 2
V t 1; 2 nn t + 1 0, do

(1) m

t2 + t + 4
( 2) .
t +1

Nh vy, (1) c nghim x [ 0; 2 ] khi v ch khi ( 2 ) c nghim t 1; 2 . iu ny


c tha khi v ch khi
t2 + t + 4
Min
m.
t1; 2
t +1

Xt f ( t ) =
f (t ) =

t2 + t + 4
, t 1; 2
t +1

t 2 + 2t 3

( t + 1)

t = 1
=0
t = 3
229

Lp bng bin thin ca hm s f (t ) ta kt lun c hm s f ( t ) =

t2 + t + 4
ng bin
t +1

trn on [1; 2]. T ta c Min f (t ) = f (1) = 3.


t1; 2

Vy, gi tr m cn tm l m 3.
IV.15.

x 4 + 4 x + m + 4 x 4 + 4 x + m = 6 (1)

t t = 4 x 4 + 4 x + m 0 t 2 = x 4 + 4 x + m

t = 2
Khi phng trnh (1) thnh t 2 + t = 6
t = 2.
t = 3 < 0
Vi t = 2 ta c
4

x4 + 4 x + m = 2

x 4 + 4 x + m = 16
m = x 4 4 x + 16
Xt hm s f ( x ) = x 4 4 x + 16
f ( x ) = 4 x 3 4
f ( x ) = 0 x = 1
Lp bng bin thin

Da vo bng bin thin ta c gi tr m cn tm l m < 19.


IV.16. x 2 2 x 3 =

( m + 1)( x 3) (1)

V m > 1 nn m + 1 > 0. Do ta phi c iu kin x 3 0 x 3. Khi ta c

(1) ( x + 1)( x 3) = ( m + 1)( x 3)


x 3 ( x + 1) x 3 m + 1 = 0
x 3 = 0
x = 3
x = 3

3
2
( x + 1) x 3 m + 1 = 0
( x + 1) x 3 = m + 1
x x x 3 = m + 1(2)
Nh vy phng trnh (1) lun c mt nghim x = 3. Ta chng minh phng trnh (2) c
mt nghim trong khong (3; +).

230

x = 1
Tht vy, xt hm s f ( x) = x x x 3, f ( x ) = 3 x 2 x 1, f ( x) = 0
x = 1 .
3

Lp bng bin thin ca hm s f ( x ) ta kt lun c vi m i m > 1 th phng trnh (2)


c mt nghim trong khong (3; +).
Vy, vi m i m > 1 phng trnh cho lun c hai nghim thc phn bit.
IV.17.

( x 1)

+ mx = m + 1 (1)

iu kin: x 1. Ta c nhn xt rng x = 1 khng l nghim ca phng trnh (1). Do


nghim ca phng trnh phi tha x > 1.
t t = x 1 > 0.

(1) t 3 + mt 2 1 = 0 ( 2 ) . Ta chng minh (2) c mt nghim duy nht

t > 0 vi m i m.

Tht vy ta c

( 2 ) mt 2 = 1 t 3
m=

1 t3
t2

Xt hm s f ( t ) =
Ta c f ( t ) =
f (t ) = 0

1 t3
, t > 0.
t2

2
1
t3

2
1 = 0 t 3 2 = 0 t = 3 2.
3
t

lim f ( t ) = ; lim+ f ( t ) = +.

t +

t o

Bng bin thin

Da vo bng bin thin, ta c ng thng y = m lun lun ct th hm s


1 t3
, t > 0 ti mt im duy nht. Suy ra phng trnh (2) c mt nghim duy nht
t2
t > 0 vi m i m.
f (t ) =

Vy, vi m i gi tr ca m th phng trnh cho lun c mt nghim duy nht.


IV.18.

(4 + x )(6 x ) x 2 2 x + m(1)

231

Gi s (1) ng vi m i x [4; 6], khi (1) cng ng vi x = 1 , tc l


(4 + 1)(6 1) 1 2 + m m 6.

Vi m 6, suy ra m 1 5 m 1 + ( x 1)2 5. Vi iu kin x [4; 6] th


x + 4 0, 6 x 0, do theo bt ng thc Csi ta c
(4 + x) + (6 x )
(4 + x )(6 x)
= 5. Nh vy ta c
2
(4 + x)(6 x)

(4 + x) + (6 x)
= 5 ( x 1) 2 + m 1 = x 2 2 x + m.
2

Vy, gi tr cn tm ca tham s m l m 6.
Ch . C th gii bi ton bng cch khc nh sau
t t = (4 + x )(6 x ) = x 2 + 2 x + 24 0, t = 0 t ti x = 4 x = 6.

t 2 = x 2 + 2 x + 24
Theo bt ng thc Csi ta c
t = (4 + x)(6 x )

1
(4 + x + 6 x) = 5
2

t = 5 x = 1 [ 4; 6]

Nh vy, vi x [ 4; 6] th t [ 0;5] .
(1) tr thnh t 2 + t 24 m 0(2). (1) ng vi m i x [4; 6] khi v ch khi (2) ng vi
m i t [ 0;5] .
1
Xt hm s g (t ) = t 2 + t 24 m g (t ) = 2t + 1; g = 0 2t + 1 = 0 t = .
2

Suy ra g (t ) ng bin trn [ 0;5]


Nn Max g (t ) = g (5) = 6 m.
[ 0,5]

g (t ) 0, t [ 0.5] Max g (t ) 0
[ 0,5]

6 m 0 m 6.
Vy, vi m 6 th tha yu cu bi ton.
IV.19. mx x 3 m + 1(1)
iu kin: x 3

232

(1) mx m 1 + x 3 m( x 1) 1 + x 3

V x 3 nn x 1 > 0 , do (1) m

1+ x 3
(2)
x 1

(1) c nghim x [3; + ) khi v ch khi (2) c nghim x [3; + ) khi v ch khi
1+ x 3
m.
x[3;+ )
x 1
Max

Xt hm s f ( x) =
f ( x) =

1+ x 3
, ta c
x 1

( x 1) 2 x 3(1 + x 3)
2 x 3( x 1)

x 1 2 x 3 2( x 3)
2 x 3( x 1)

x + 5 2 x 3
2 x 3( x 1)2

f ( x) = 0 x + 5 2 x 3 = 0

x 5
x 5

2
2
(5 x ) = 4( x 3)
25 10 x + x = 4 x 12
x 5
x

2
x = 7 + 2 3 x = 7 2 3
x 14 x + 37 = 0

x = 7 2 3
Bng bin thin

1+ x 3
3 +1
=
.
x[3;+ )
x 1
4

Da vo bng bin thin, ta c Max


Vy, gi tr m cn tm l m
IV.20.

3 +1
.
4

x 2 + mx + 2 = 2 x + 1 (1)
1
2

iu kin: 2 x + 1 0 x

(1) x 2 + mx + 2 = ( 2 x + 1) x 2 + mx + 2 = 4 x 2 + 4 x + 1
3x 2 + ( 4 m ) x 1 = 0(2)

233

t f ( x) = 3 x 2 + ( 4 m ) x 1. Phng trnh (1) c hai nghim phn bit khi v ch khi


1
phng trnh (2) c hai nghim phn bit ln hn hoc bng . iu ny xy ra khi v
2
ch khi

( 4 m )2 + 12 > 0
> 0
16 8m + m 2 + 12 > 0

3 1
1
m 9

af

(
)


0

2
4 2
2 4
S 1
(4 m) 1
m 1
+ >0

6 > 0
+ >0
2 2
6
2

m2 8m + 28 > 0
9

9
9

m
m

2 m .
2
2

m > 1
m 1
6 > 0
Vy, vi m

9
th phng trnh cho c hai nghim phn bit.
2

IV.21. ( x + 1)( x + 3) m x 2 + 4 x + 5 (1)


t t = x 2 + 4 x + 5 1. Khi (1) tr thnh t 2 2 mt t 2 mt 2 0.

a) Khi m = 1 , ta c bt phng trnh


t 2 + t 2 0 2 t 1. Nhng v t 1 nn ta phi c
t = 1 x 2 + 4 x + 5 = 1 x 2 + 4 x + 5 = 1 x 2 + 4 x + 4 = 0 x = 2.

Vy, khi m = 1 th bt phng trnh c nghim x = 2.


b) t = x 2 + 4 x + 5 1. t ( x ) =

x+2
2

x + 4x + 5

=0

x = 2 2; 2 + 3 .
Bng bin thin

234

2x + 4
2 x2 + 4 x + 5

x+2
x2 + 4x + 5

, t ( x ) = 0

Nh vy, vi x 2; 2 + 3 t [1; 2] .
Ta bin i (1) t 2 2 mt m
Xt hm s f ( t ) =
f ' (t ) =

t2 2
, t [1; 2] .
t

t2 2
, t [1; 2] .
t

2t 2 t 2 + 2 t 2 + 2
= 2 > 0, t [1; 2] .
t2
t

Ta c f (1) = 1, f ( 2 ) = 1
Vy, Maxf ( t ) = 1.
t[1;2]

Yu cu bi ton c tha khi v ch khi Maxf ( t ) < m m > 1.


t[1;2]

IV.22.

( 3 + x )( 7 x ) x 2 4 x + m (1)

t t =

( 3 + x )( 7 x ) =

x 2 + 4 x + 21 0

Bt phng trnh (1) tr thnh


t t 2 + 21 + m
t 2 + t 21 m ( 2 )

Xt hm s t ( x ) =
t ( x ) =

( 3 + x )( 7 x ) , x [ 3; 7]

x + 2

( 3 + x )( 7 x )

= 0 x = 2

Bng bin thin

Da vo bng bin thin, ta thy vi x [ 3; 7 ] th t [ 0;5] .


Do , (1) nghim ng x [ 3; 7 ] khi v ch khi ( 2 ) nghim ng t [ 0;5] khi v ch
khi Max ( t 2 + t 21) m.
t[0:5]

Xt hm s g ( t ) = t 2 + t 21
g ( t ) = 2t + 1 > 0, t [ 0;5]

235

Ta c g (0) = 21, g (5) = 9


Vy, Max g (t ) = 9
t[ 0:5]

Yu cu bi ton c tha khi v ch khi Maxg ( t ) m m 9.


t[1;2]

4 x 2 + 16 4 x m (1)

IV. 23.

Cch 1.
1

4 x 2 0
1
x
iu kin:

2 x 4.
2
16 4 x 0
x 4
Bt phng trnh (1) c nghim khi v ch khi
m 0

4 x 2 + 16 4 x m
m 0

2
2
4 2 2 x + 9 x 4 m 14
m 0

m 2 14 0
(I )

2
2
2
32 2 x 9 x + 4 + m 14 0

) (

2
1
1
Do 2 x 2 9 x + 4 > 0 x ; 4 nn 32 2 x 2 9 x + 4 + m 2 14 0, x ; 4
2
2

) (

m 0

m 14.
m 14 0 m 14 m 14

m 0

(I )

Vy, vi m 14 th bt phng trnh cho c nghim.


Cch 2. 4 x 2 + 16 4 x m (1)

(1)

c nghim khi v ch khi Min

1
x ;4
2

Xt hm s y = f ( x ) = 4 x 2 + 16 4 x
y = f ( x ) =

236

2
2

4x 2
16 4 x

4 x 2 + 16 4 x m

16 4 x = 4 x 2
9

y' = 0 1
x=
4
<x<4
2
Ta c
9
1
f = 2 7; f = 14; f ( 4 ) = 14.
4
2
Vy, Min f ( x ) = 14. Do gi tr m cn tm l m 14.
1
x ;4
2

IV.24. 1 x 2 m x (1)
iu kin: 1 x 1

Ta c (1) 1 x 2 + x m
(1) c nghim Max ( 1 x 2 + x ) m
x[ 1;1]

Xt hm s y = f ( x ) = 1 x 2 + x; x [ 1;1]
y' =

x
1 x2

+1

x
+1 = 0

y = 0 1 x 2
1 < x < 1

1 x 2 = x

1 < x < 1
x 0

2 x2 = 1

1 < x < 1
x 0

x = 2

2
2
x=

2
x = 2

1 < x < 1
Ta c f (1) = 1 ; f (1) = 1 ; f (

2
)= 2.
2
237

Suy ra Max ( 1 x 2 + x ) = 2 . Vy, gi tr m cn tm l m 2.


x[ 1;1]

IV.25. 12 3 x 2 x m(1)
iu kin: x [ 2; 2] . Ta c (1) x 12 3x 2 m

Xt hm s y = x 12 3 x 2 vi x [ 2; 2] .
y = 1

3 x
12 3 x 2

12 3x 2 + 3 x
12 3 x 2

2 < x < 2
2 < x < 2
2 < x < 2

y = 0

x 0
2
2
12 3x 2 = 9 x 2
12 3 x + 3x = 0
12 3 x = 3 x

2 < x 0

x = 1.
x = 1 x = 1
Bng bin thin

Da vo bng bin thin, suy ra bt phng trnh cho c nghim duy nht khi v ch khi
m=2.
IV.26. m 2 x 2 + 7 < x + m (1)

1) Khi m =

1
th bt phng trnh (1) tr thnh
2

2 x + 1 > 0
1
1
2x2 + 7 < x + 2x2 + 7 < 2x +1 2
2
2
2
2 x + 7 < ( 2 x + 1)
1

x>
1
1

2
x >
x >

x > 1.
2
2
x < 3
2 x 2 + 7 < 4 x 2 + 4 x + 1 x 2 + 2 x 3 > 0

x > 1
Vy, tp hp nghim ca bt phng trnh cho khi m =
2) Ta c (1) m 2 x 2 + 7 < x + m m

238

1
l (1; + ) .
2

2x2 + 7 1 < x m <

x
2

2x + 7 1

(V

2 x 2 + 7 1 > 0, x )
x

Xt f ( x ) =

2x2 + 7 1

f ( x) =

7 2x2 + 7
2x2 + 7

2 x2 + 7 1

f ( x ) = 0 2 x 2 + 7 = 7 2 x 2 + 7 = 49
2 x 2 = 42 x 2 = 21 x = 21 x = 21.
Ta c Limf ( x ) =
x

f ( 21) =

2
2
, Limf ( x ) =
.
2
2
x +

21
21
, f ( 21) =
.
6
6

Lp bng bin thin ca hm s f ( x ) ta c Min f ( x ) =


x

21
.
6

Vy, bt phng trnh (1) nghim ng vi m i x khi v ch khi


21
.
m < Min f ( x ) m <
6
x
IV.27.

x 2 2mx > 1 x (1)

1
Vi x 1; th (1) tng ng vi
4
2
x 2 2mx > (1 x ) x 2 2mx > 1 2 x + x 2 2 ( m 1) x + 1 < 0.
t f ( x) = 2 ( m 1) x + 1.

Yu cu bi ton c tha khi v ch khi


m 1
1
m < 1
+1 < 0
m + 1 < 0
f( )<0

4
1 m < 1.
2
m

1
<
0
m
<

f (1) < 0

2
2 ( m 1) + 1 < 0
Vy, gi tr m tha mn yu cu bi ton l m < 1.
IV.28. m

x 2 + 2 4 x 2 4 x + 2 = 2 4 x 2 4 (1)

iu kin: x 2.

Vi x = 2, phng trnh (1) khng tha. V vy chia hai v ca (1) cho


2 4 x2 4
m 1 +
=

x 2

x+2
x + 2 2 4 x2 4
+
m 1 + 2 4
=
x2
x 2
x2

x 2 ta c

x+2
x+2
+ 24
.
x2
x2
239

t t =

t 2 + 2t
x+2
, t > 1. Ta c phng trnh m (1 + 2t ) = t 2 + 2t m =
.
x2
2t + 1

Phng trnh (1) c nghim khi v ch khi m thuc min gi tr ca hm s


t 2 + 2t
, t > 1.
y = f (t ) =
2t + 1
Ta c f (t ) =

2t 2 + 2t + 2

> 0, do hm s f (t ) ng bin. Tp gi tr ca hm s f (t )

( 2t + 1)
trn min (1; + ) l (1; + ) . Vy, gi tr cn tm ca tham s

IV.29. x 3 + 3x 2 1 m

x x 1

m l m > 1.

iu kin: x 1. Vi iu kin x 1 bt phng trnh cho tng ng vi


x3 + 3x 2 1

x x 1

m ( x 3 + 3 x 2 1)

x + x 1 m.

Bt phng trnh cho c nghim x 1 khi v ch khi

)(

m Min x 3 + 3x 2 1

Cc hm s y = x 3 + 3x 2 1; y =

)(

s y = x 3 + 3x 2 1

x + x 1 ng bin v khng m trn [1; +) nn hm

x + x 1 cng ng bin trn [1; +).

)(

x + x 1 , x [1; +).

Suy ra Min x 3 + 3 x 2 1

x + x 1 = f (1) = 3. Vy, gi tr cn tm ca tham s m l

m 3.

IV.30.

2 x + 2 x + 2 4 6 x + 2 6 x = m.(1)

iu kin 0 x 6. t

f ( x ) = 4 2 x + 2 x + 2 4 6 x + 2 6 x , f ( x ) =

1
2 4 ( 2x)

1 1
=

2 4 ( 2 x )3

1
4

(6 x)

1
1
1

3
2x 2 4 (6 x )
6 x

+ 1 1 .

2 x
6 x


1
1 1 1
1
= 4
4
+4
+
2
4
2
6 x 4 ( 2x)
2x 6 x
2x

f ( x) = 0 x = 2.

1
4

(6 x)

+ 1 + 1
4 2 x 4 6 x

Ta tnh c f (0) = 2 6 + 2 4 6; f (2) = 3 2 + 6; f (6) = 4 12 + 2 3.


240

Lp bng bin thin ca hm s f ( x ) trn on [ 0; 6] ta c gi tr ca tham s m cn


tm phng trnh (1) c ng hai nghim thc l 2

6 + 4 6 m < 3 2 + 6.

IV.31. x 2 + mx 1 > ( x + m ) x 2 1(1)


t t = x 2 1 t 2 = x 2 1 x 2 = t 2 + 1. (1) c a v dng t 2 ( x + m ) t + mx 0(2)
2

Coi v tri ca (2) l tam thc bc 2 theo t ta c = ( x m ) . Khi f (t ) c cc nghim


t = m, t = x. Vy (2) c dng ( t m )( t x ) > 0

Theo bi ra, do x < 1 nn

x2 1 m

)(

x 2 1 x > 0(3).

x 2 1 x > 0. V vy (3) tng ng vi

x 2 1 m > 0 x 2 1 > m. Xt hm s

y = x 2 1, x < 1 y =

x
2

x 1

< 0, x < 1.

Suy ra hm s nghch bin trn (; 1), ta c Limy = +, Limy = 0.


x

x 1

Vy, vi m i gi tr ca tham s m th bt phng trnh cho lun lun c nghim x


tha x < 1.
IV.32. x 2 + mx 4 = ( x + m ) x 2 4(1)

x 2
iu kin:
. t t = x 2 4 t 2 = x 2 4 x 2 = t 2 + 4.

x
2

(1) c a v dng t 2 ( x + m ) t + mx = 0(2)


2

Coi v tri ca (2) l tam thc bc 2 theo t ta c = ( x m ) . Khi f (t ) c cc nghim


t = m, t = x. Vy (2) c dng

( t m )( t x ) = 0 (

x2 4 m

)(

x2 4 x = 0

x2 4 x = 0
x 2 4 = x(3)

x 2 4 m = 0
x 2 4 = m(4)

Ta c (3) v nghim. i vi phng trnh (4) ta xt hm s


f ( x) = x 2 4, x 2 x 2
f ( x) =

x
x2 4

f ( x) < 0, x < 2; f ( x) > 0, x > 2.

f (2) = f (2) = 0. Ta c Limf ( x ) = +, Lim f ( x ) = +.


x

x +

Vy, phng trnh cho c nghim khi m 0.


241

CHNG V.

PHNG TRNH, BT PHNG TRNH M V LOGARIT

V.1.1) 4 x 10.2 x 1 = 24 (1)

(1) ( 2 x )

5.2 x = 24

t t = 2 x > 0. Phng trnh (1) tr thnh

t 2 5t = 24
t 2 5t 24 = 0
t = 8

t = 3

So vi iu kin ta chn t = 8 2 x = 8 x = 3.
Vy, nghim ca phng trnh cho l x = 3.
2 ) 4.22 x 6 x = 18.32 x (1)
2x

2
2
(1) 4. = 18
3
3
x

2
t t = > 0. Phng trnh (1) tr thnh
3

4t 2 t = 18
4t 2 t 18 = 0
9
t=
4

t = 2
x

9 2 9
So vi iu kin ta chn t = = x = 2.
4 3
4

Vy, nghim ca phng trnh cho l x = 2.


2

3) 3log3 x + x log3 x = 162 (1)


iu kin: x > 0
log3 x

Ta c x = 3
2
log x
3

(1) 3

2
log x
3

2.3
242

2
log x
3

+3

= 162

log 3 x

log3 x

= 3

= 162

log3 x

2
log x
3

=3

, do

2
log x
3

= 81

log 32 x = 4
log 3 x = 2

log 3 x = 2

x = 32 = 9

x = 32 = 1 .

9
1
Vy, nghim ca phng trnh cho l x = 9; x = .
9

4) 9

log 1 2 x 2 +1

log 1 ( x +1)

=5

(1)

x +1 > 0
iu kin: 2
x > 1
2 x + 1 > 0
2log

(1) 3

31

( x +1)

log

=5

32log3 ( x +1) = 5

51

( 2 x +1)

log5 2 x 2 +1

log5 2 x 2 +1

3log3 ( x +1) = 5
2

( x + 1) = ( 2 x 2 + 1)

x2 + 2 x + 1 = 2 x 2 + 1
x2 2 x = 0
x = 0

x = 2
Vy, nghim ca phng trnh cho l x = 0; x = 2.
5) 4 x

x2 5

12.2 x 1

x2 5

+ 8 = 0(1)

iu kin: x 2 5 0 x 5

(1) (2 x

x2 5 2

t t = 2 x

x 2 5

) 6.2 x

x2 5

+8 = 0

t = 4
( t > 0). Khi phng trnh (1) tr thnh t 2 6t + 8 = 0
(Nhn).
t = 2

+ Vi t = 2 2 x

x 2 5

= 2 x x2 5 = 1 x2 5 = x 1

243

x 1 0
x 1
x=3

2
2
2 x = 6
( x 1) = x 5

+ Vi t = 4 2 x

x 2 5

= 4 2 x

x2 5

= 22 x x 2 5 = 2 x 2 5 = x 2

x 2 0
x 2
9
x=

2
2
4
4 x = 9
( x 2) = x 5
9
Vy, nghim ca phng trnh cho l x = 3; x = .
4

6) 9 x

+ x 1

10.3x

(3x

+ x 1 2

t t = 3x

+ x 1

+ x2

+ 1 = 0(1)

10 x2 + x 1
.3
+1 = 0
3

(t > 0). Khi phng trnh (1) tr thnh

t = 3
10
t t + 1 = 0 1 (Nhn).
t =
3
3
2

+ Vi t =

2
2
x = 0
1
1
3x + x 1 = 3x + x x = 31 x 2 + x 1 = 1 x( x + 1) = 0
3
3
x = 1

+ Vi t = 3 3x

+ x 1

x = 1
= 3 x 2 + x 1 = 1 x2 + x 2 = 0
x = 2

Vy, nghim ca phng trnh cho l x = 2; x = 1; x = 0; x = 1.


7) 3.4 x + (3 x 10).2 x + 3 x = 0(1)
3(2 x )2 + (3x 10).2 x + 3 x = 0
t t = 2 x (t > 0). Khi phng trnh (1) tr thnh

3t 2 + (3 x 10)t + 3 x = 0(2). Ta coi v tri ca phng trnh (2) l mt tam thc bc hai
ca bin t , khi ta c
2

= ( 3 x 10 ) 4.3. ( 3 x ) = 9 x 2 60 x + 100 36 + 12 x
2

= 9 x 2 48 x + 64 = ( 3x 8 ) .

244

1
t=
V vy (2) 3

t = 3 x
+ Vi t =

1
1
1
2 x = x = log 2 .
3
3
3

+ Vi t = 3 x, ta c phng trnh 2 x = 3 x 2 x + x 3 = 0(3)


Ta thy x = 1 l nghim ca ( 3) .
Mt khc, ta li c y = 2 x + x 3 l mt hm s tng trn nn phng trnh (3) ch c
mt nghim duy nht x = 1.
1
Vy, nghim ca phng trnh cho l x = log 2 ; x = 1.
3

8) x 2 + (2 x 3) x + 2(1 2 x ) = 0(1). Ta coi (1) l phng trnh bc hai theo n x. Khi


x = 2
(1)
x
x = 1 2

Gii phng trnh x = 1 2 x 2 x + x = 1( 2 )


Ta thy x = 0 l nghim ca ( 2 ) . Tht vy 20 + 0 = 1 (ng).
Mt khc, ta li c y = 2 x + x l hm tng trn , y = 1 l hm hng.
Nn x = 0 l nghim duy nht ca ( 2 ) .
Vy, nghim ca phng trnh cho l x = 0; x = 2.
9) 4.33 x 3x +1 = 1 9 x (1)
iu kin: 1 9 x 0 9 x 1 x 0.

(1) 4.33 x 3.3x = 1 32 x .

Vi iu kin x 0 th 0 < 3x 1, ta t 3x = cos t vi t [0; ) . Khi ta c phng


2
trnh 4 cos 3 t 3cos t = 1 cos 2 t

cos 3t = sin t cos 3t = sin t (Do t [0; ) )


2

245

cos 3t = cos t
2

k
3t = 2 t + k 2
t = 8 + 2
, ( k ).

t = + k
3t = + t + k 2

2
4

Do t [0; ) nn ta chn t = 3x = cos x = log 3 cos .


2
8
8
8
Ta c cos

= 2cos 2

1 cos 2

2+ 2

2+ 2
cos =
.
4
8
2

Vy, nghim ca phng trnh cho l x = log 3


10) 4 x

3 x + 2

+ 4x

+ 6 x +5

= 42 x

+3 x + 7

2+ 2
.
2

+1

Ta vit li phng trnh cho di dng


4x

3 x + 2

4x

+ 4x
3 x + 2

+ 6 x +5

= 4x

)(

1 1 4x

3 x + 2

.4 x

+ 6 x +5

+ 6 x +5

+1

)=0

4 x 3 x + 2 1 = 0
4 x 3 x + 2 = 1 x 2 3 x + 2 = 0
x = 1 x = 2

2
2

2
1 4 x + 6 x+ 5 = 0
4 x +6 x + 5 = 1 x + 6 x + 5 = 0
x = 1 x = 5.
2

Vy, nghim ca phng trnh cho l x = 1; x = 2; x = 1; x = 5.


11) e


sin x
4

= tan x.

iu kin: cos x 0 x

+ k , ( k ) .

D thy sin x = 0 khng tha mn phng trnh.


Phng trnh cho tng ng vi e

2 (sinx - cosx )
2

2 sinx

u = s inx
t
. Ta c u , v ( 1;1) ; u , v 0.
v = cosx
e

T (1) ta c phng trnh

Xt hm s y = f ( x) =
246

2u
2

u
2x
2

2v
2

2cosx

s inx
e 2
e 2
=

=
cosx
sinx
cosx

, vi x (1;0) (0;1).

(1)

2x
1)e
2

y =
x2
v (0;1).
(

2x
2

( 2 x 2)e
2 x2

2x
2

< 0 suy ra hm s nghch bin trn cc khong (1; 0)

Ta thy u, v cng du nn u, v cng thuc mt khong ( 1 ; 0) hoc (0; 1).


T gi thit f (u ) = f (v ) u = v tan x = 1 x =

+ k .

i chiu vi iu kin ta c nghim ca phng trnh cho l


x=

+ k , ( k ) .

V.2.1) 4 x 2.52 x 10 x > 0(1)

4x
52 x

2.
1 > 0
10 x
10 x
x

2
5
2. 1 > 0
5
2
x

2
t t = > 0. Bt phng trnh (1) tr thnh
5
2
t 1 > 0
t

t2 t 2 > 0
t < 1

t > 2
So vi iu kin t > 0 ta nhn t > 2 .
x

2
Nh vy ta c > 2 x < log 2 2 .
5
5

Vy, tp hp nghim ca bt phng trnh cho l ;log 2 2 .


5

2) 9 x 3x + 2 > 3x 9
3x 9 < 0
x x+2
9 3 0

x
3 9 0
9 x 3x + 2 9 x 18.3x + 81

x < 2

x 2 x 2
x 2

x 2

Vy, tp hp nghim ca bt phng trnh cho l [2; +).


247

3)

4x + 2 x 4
2(1)
x 1

1
x 2
4x 2 0
2 x 1

x
4 2
x > 1
1
x > 1
x 1 > 0
(1)
0

x <1
x
2 x 1
x 1
2
x 1
4 2 0

2
x 1 < 0
x < 1
x < 1

1
Vy, tp hp nghim ca bt phng trnh cho l ;1 .
2
4) 15.2 x +1 + 1 2 x 1 + 2 x +1
t t = 2 x > 0

Bt phng trnh cho tr thnh 30t + 1 t 1 + 2t


+ Xt 0 < t < 1
Ta c 30t + 1 t 1 + 2t
30t + 1 1 t + 2t
30t + 1 1 + t
30t + 1 t 2 + 2t + 1
t 2 28t 0
0 t 28
So vi iu kin 0 < t < 1 ta c 0 < t < 1 0 < 2 x < 1 x < 0.
+ Xt t 1
Ta c 30t + 1 t 1 + 2t
30t + 1 t 1 + 2t
30t + 1 3t 1

30t + 1 9t 2 6t + 1 (do t 1 3t 1 > 0)


9t 2 36t 0
0t 4

So vi iu kin t 1 ta c 1 t 4 1 2 x 4 0 x 2.
Vy, tp hp nghim ca bt phng trnh cho l (; 2].
2

5) 2(log2 x ) + x log2 x 4(1)


248

t t = log 2 x x = 2t . Ta c bt phng trnh


2

2t + 2t 4
2

2t 2
t2 1
1 t 1

Suy ra 1 log 2 x 1

1
x2
2

1
Vy, tp hp nghim ca bt phng trnh cho l ; 2 .
2
6)

10 3

x +1
x +3

10 + 3

x 3
x 1

0(1)

Ta c

10 3

)(

10 + 3 = 1

10 3 =

10 + 3

do bt phng trnh (1) tng ng vi

10 + 3

x +1
x+3

x +1
x +3

x 1 x 3

x + 3 x 1

10 + 3

10 + 3

x 3
x 1

10 + 3

x 3
x 1

x +1 x 3
+
0
x + 3 x 1
2 x 2 10

0
( x + 3)( x 1)

3 < x 5

1 < x 5
Vy, tp hp nghim ca bt phng trnh cho l (3; 5] (1; 5].
7) 4 x 2 + x.2 x

+1

+ 3.2 x > x 2 .2 x + 8 x + 12

Bt phng trnh cho tng ng vi


4 x 2 + x.2 x

+1

+ 3.2 x x 2 .2 x 8 x 12 > 0
2

) (

x2 4 2x 2 x 4 2x 3 4 2x

)>0
249

4 2x

)( x

2 x 3) > 0

4 2 x2 > 0
2 x2 < 4

x 2 2 x 3 > 0
x < 1 x > 3

2
x2
x

4 2 < 0
2 > 4
2
1 < x < 3
x 2 x 3 < 0

2 < x < 2

2 < x < 1
x < 1 x > 3

2 < x < 3
x < 2 x > 2
1 < x < 3

) (

Vy, tp hp nghim ca bt phng trnh cho l 2; 1


8)

8 + 2 x +1 4 x + 2 x +1 > 5

t t = 2 x > 0.

Bt phng trnh cho tr thnh


8 + 2t t 2 + 2t > 5
8 + 2t t 2 > 5 2t

5 2t < 0

2
8 + 2t t 0

5 2t 0

8 + 2t t 2 > ( 5 2t ) 2

5
t > 2

2 t 4

t 5
2
5t 2 22t + 17 < 0

5
2 < t 4

1< t 4
1 < t 5

2
Suy ra 1 < 2 x 4 0 < x 2 .
Vy, tp hp nghim ca bt phng trnh cho l ( 0; 2] .
250

2;3 .

x+2

1 2 x
9)
>9
3
Bt phng trnh cho tng ng vi
x+2

< 2

2 x

x + 2 + 2 (2 x )
2 x

<0

x+2 2 x +4
< 0(1)
2 x

+ Xt trng hp x < 2
x 2 + 2x + 4
<0
x+2
1 < 0.

(1)

Trng hp ny (1) khng nghim ng.


+ Xt trng hp 2 < x < 0
x + 2 + 2x + 4
<0
x+2
3 < 0.

(1)

(1) cng khng nghim ng.


+ Xt trng hp x > 0
x + 2 2x + 4
<0
x + 2
x 6

< 0 2 < x < 6.


x2

(1)

Vy, tp hp nghim ca bt phng trnh cho l ( 2; 6 ) .


2

V. 3. 2m x + 6 24 x +3m = 4 m 2 x + 3m 6 (1)
2

(1) 2m x+6 24 x+ 3m = ( m2 x + 6 ) + 4 x + 3m
2

2 m x + 6 + m 2 x + 6 = 24 x +3m + 4 x + 3m
Xt hm s f ( t ) = 2t + t.
Hm s ny ng bin trn .
Phng trnh (1) c vit di dng

f m 2 x + 6 = f ( 4 x + 3m )
251

m 2 x + 6 = 4 x + 3m
( m 2 4 ) x = 3m 6
Vy, ta c
+ Nu m = 2 th phng trnh (1) c nghim ty .
+ Nu m = 2 th phng trnh (1) v nghim.
+ Nu m 2 th phng trnh (1) c nghim l x =

3m 6
.
m2 4

V.4. 4 x + 4 x = m 2 x + 2 x + 1 (1) . t t = 2 x + 2 x ( t 2 )

Suy ra t 2 = 4 x + 4 x + 2
Khi phng trnh (1) tr thnh
t 2 mt m 2 = 0 t 2 2 = m ( t + 1)

t2 2

= m (2)
t +1
t2 2
, t 2 . Phng trnh (1) c nghim khi v ch khi ng thng y = m
t +1
t2 2
ct th hm s y = f (t ) =
( t 2 ) ti t nht mt im.
t +1
Xt f ( t ) =

Ta c f ' ( t ) =

t 2 + 2t + 2

( t + 1)

> 0, t 2

Lp bng bin thin

Da vo bng bin thin ta c vi m

2
th phng trnh (1) c nghim.
3

2
Vy, phng trnh (1) c nghim khi m .
3

4 x + y = 128
V.5. 1) 3 x 2 y 3 .
=1
5
Ta c h phng trnh cho tng ng vi

252

x = 2
22( x + y ) = 27
2( x + y ) = 7
2 x + 2 y = 7
2 x + 2 y = 7

3 x 2 y 3
3
0
=5
3 x 2 y 3 = 0
3 x 2 y = 3
5 x = 10
5
y = 2
3
Vy, nghim ca h phng trnh cho l 2; .
2
8log9 ( x 4 y ) = 1
2) x 2 y
7.2 x 2 y = 8
4
x = 4 y + 1
log 9 ( x 4 y ) = 0
x = 4y +1
x2 y

7.2 x 2 y = 8 42 y +1 7.22 y +1 = 8 22 y +1 7.22 y +1 8 = 0


4

x = 4 y +1
x = 4 y + 1 x = 5

22 y +1 = 8 2 y

y =1
2 = 4
2 y +1
2
=

Vy, nghim ca h phng trnh cho l (5;1).


23 x = 5 y 2 4 y

3) 4 x + 2 x +1
=y
x
2 +2
y = 0
x = 0
23 x = 5 y 2 4 y

3x
2
3
2

2 = 5 y 4 y
y 5 y + 4 y = 0
y =1
y =1
x
x

2x 2x + 2
x = 2
=y

2 = y
2 = y
y = 4
x

2 +2
2 x = y

y = 4

Vy, nghim ca h phng trnh cho l ( 0;1) ; ( 2; 4 ) .


9log2 ( xy ) = 3 + 2( xy )log2 3
4) 2
(I )
2
x + y = 3 x + 3 y + 6
iu kin: xy > 0

9log2 ( xy ) = 3 + 2( xy )log2 3
(I )
2
( x + y ) = 3( x + y ) + 2 xy + 6
t t = ( xy )log2 3 >0 log 3 t = log 2 ( xy )

Khi , phng trnh 9log2 ( xy ) = 3 + 2( xy )log2 3 tr thnh


2
t = 1
9log3 t = 3 + 2t 3log3 t = 3 + 2t t 2 = 3 + 2t t 2 2t 3 = 0
t = 3

Chn t = 3 ( xy )log2 3 = 3 log 2 3log 3 ( xy ) = 1 log 2 ( xy ) = 1 xy = 2


253

Thay xy = 2 vo phng trnh ( x + y )2 = 3( x + y ) + 2 xy + 6 ta c


x + y = 5
( x + y )2 3( x + y ) 10 = 0
x + y = 2
xy = 2

x + y = 5
Suy ra ( I )
xy = 2

x + y = 2
xy = 2
+ Gii h
( I )
x + y = 5
Nh vy, x, y l nghim ca phng trnh

5 + 17
X =
2
X 2 5X + 2 = 0

5 17
X =
2

5 + 17 5 17 5 17 5 + 17
Suy ra h phng trnh ( I ) c hai nghim l
;
;
;
.
2
2
2
2

xy = 2
+ Gii h
( I )
x
+
y
=

Ta c x, y l nghim ca phng trnh X 2 + 2 X + 2 = 0 (Phng trnh v nghim)


Do h phng trnh ( I ) v nghim.
5 + 17 5 17 5 17 5 + 17
Vy, nghim ca h phng trnh cho l
;
;
;
.
2 2
2
2
( x 2 + y )2 y x = 1
5)
(I )
2
x2 y
9( x + y ) = 6
2

x 2 + y = 2 x y
x 2 + y = 2 x
(I )

2
x2 y
x2 y
= 6x y
9.2
9 = 3
2

x 2 + y = 2 x

2
x = 2 + y
y =1
2
x = 2 + y

254

2 + 2 y = 4
2
x = 2 + y

2
x
x + y = 2

2
x y = 2

x = 3
y =1

y =1
y = 1
2
x = 3
x = 3
x = 3
=

x
3

y = 1
Vy, nghim ca h phng trnh cho l

)(

3;1 ; 3;1 .

x
y
3 3 = ( y x)( xy + 8)
6) 2
(I )
2
x + y = 8

3x 3 y = ( y x )( xy + x 2 + y 2 )
(I ) 2
2
x + y = 8
3x 3 y = xy 2 + x 2 y + y 3 x 3 xy 2 x 2 y
3x + x 3 = 3 y + y 3
2
2
2
2
x + y = 8
x + y = 8
Xt hm s f ( u ) = 3u + u 3 , hm s ny ng bin trn .
Do , phng trnh 3x + x 3 = 3 y + y 3 x = y
Khi h phng trnh cho tng ng vi
x = 2
x = y

x = y
x = y

y = 2
2
x = 2
2
2
x = 2
x + y = 8 2 x = 8 x = 2

y = 2
Vy, nghim ca h phng trnh cho l ( 2; 2 ) ; ( 2; 2 ) .
3x + x = 3 + y (1)
7) y
3 + y = 3 + x(2)
Tr (1) cho (2) v theo v ta c 3x + x 3 y y = y x 3x + 2 x = 3 y + 2 y.
Xt hm s c trng f ( t ) = 3t + 2t ( t )
Ta c f ( t ) = 3t ln3 +2 > 0, t .
Suy ra hm s f ( t ) ng bin trn .
Do phng trnh 3x + 2 x = 3 y + 2 y x = y
Khi h phng trnh cho tng ng vi
x = y
x = y
x = 1
x

x
3 + x = 3 + x
3 = 3 y = 1

255

Vy, nghim ca h phng trnh cho l (1;1) .


22 x +1 3.2 x = y 2 2

8)
(I )
2 y 2 3 y = 22 x 2
Ta c h phng trnh ( I ) tng ng vi
2 ( 2 y 2 3 y + 2 ) 3. 2 y 2 3 y + 2 = y 2 2

2 y 2 3 y + 2 = 22 x

y 2 2 y + 2 = 2 y2 3 y + 2

2 y 2 3 y + 2 = 22 x
y 4 4 y3 + 6 y 2 5 y + 2 = 0

2x
2
2 y 3 y + 2 = 2

( y 1)( y 2 ) y 2 y + 1 = 0

2 2 x = 2 y 2 3 y + 2
y 1 = 0

y 2 = 0
2x
2
2 = 2 y 3 y + 2

x = 0
y = 1

y = 1
2 x = 0

x = 1

=
2
y

x = 1
2 x = 2

y = 2
Vy, nghim ca h phng trnh cho l ( 0;1) ; (1; 2 ) ; ( 1; 2 ) .
Ch . C th lm theo cch khc nh sau:
t u = 2 x 1. H phng trnh cho tr thnh h i xng loi II
2
2
2u 3u = y 2
(I )
2
2
2 y 3 y = u 2

Tr theo v ca hai phng trnh ca h cho nhau ta c


u = y
2(u 2 y 2 ) 3(u y ) = (u 2 y 2 ) 3(u y )(u + y 1) = 0
y = 1 u
+ Vi u = y th h phng trnh ( I ) tng ng vi
256

u = y
u = y
u = y = 1

2
2
u = 1 u = 2
u = y = 2
2u 3u = u 2
2 x = 1
x = 0

y = 1
y =1

x = 1 x = 1
x
2 = 2

y = 2
y = 2

+ Vi y = 1 u th h phng trnh ( I ) v nghim.


Vy, nghim ca h phng trnh cho l ( 0;1) ; (1; 2 ) ; ( 1; 2 ) .
4log3 ( xy ) = 2 + ( xy )log3 2
9)
2
2
x + y 3x 3 y = 12

iu kin: xy > 0

H phng trnh cho tng ng vi


22.log3 ( xy ) = 2 + 2log3 ( xy )
22.log3 ( xy ) 2log3 ( xy ) 2 = 0

2
2
( x + y ) 2 xy 3 ( x + y ) = 12
( x + y ) 2 xy 3 ( x + y ) = 12
2log3 ( xy ) = 1 (VN )
log3 ( xy )

=2
2
log3 ( xy )
2

=2
2

( x + y ) 2 xy 3 ( x + y ) = 12
2
( x + y ) 2 xy 3 ( x + y ) = 12

log 3 ( xy ) = 1
xy = 3

2
2
( x + y ) 3 ( x + y ) 18 = 0
( x + y ) 2 xy 3 ( x + y ) = 12
xy = 3

x + y = 6
x + y = 3

xy = 3
x=3+

y =3
xy = 3
x + y = 6

xy = 3
x=3
x+ y =6

(VN )
x + y = 3
y =3+

6
6
6
6

)(

Vy, nghim ca h phng trnh cho l 3 + 6;3 6 ; 3 6;3 + 6 .


2x
2y
2 + 4 = 2
10 ) x
y
x+ 2 y
=3
2 + 4 + 2

257

x
u > 0
u = 2
t
.

u
ki

n:

y
v = 4
v > 0

u 2 + v 2 = 2
H phng trnh cho tr thnh
u + v + uv = 3
2
( u + v ) 2 + 2 ( u + v ) 6 = 2
( u + v ) 2uv = 2

u
v
uv
3
+
+
=

uv = ( u + v ) + 3

u + v = 2
( u + v ) 2 + 2 ( u + v ) 8 = 0

u + v = 4
uv = ( u + v ) + 3
uv = u + v + 3
(
)

u + v = 4
(VN )

u = 1
uv = 7

u + v = 2
v = 1.

uv = 1
x
2 = 1 x = 0
Suy ra y

4 = 1 y = 0.

Vy, h phng trnh cho c mt nghim duy nht l ( 0; 0 ) .


y
x2 1
y 2 1
= ln
(1)
2 2
x
11)
y 2 + 3 + 2 y = 3 + x (2)

iu kin: x > 0, y > 0.

(1)

2
1 x2
(2 2 y ) = ln y ln x
2

2 x 2 y = 2 ( ln y ln x )
2 x 2 y = ln y 2 ln x 2
2

2 x + ln x 2 = 2 y + ln y 2
Xt hm s f (t ) = 2t + ln t ( t > 0 ) . Khi ta c f ( x 2 ) = f ( y 2 ).
Ta c hm s f (t ) tng trn (0; +) nn suy ra f ( x 2 ) = f ( y 2 ) x 2 = y 2 .
Vi x 2 = y 2 , h phng trnh cho tr thnh

258

x y = 0
x 2 = y 2

x + y = 0
2
y + 3 + 2 y = 3 + x
2
y +3 + 2 y = 3+ x

x = y
2
(do x > 0, y > 0 ).
x + 3 = 3 x
Ta c hm s y = 3 x nghch bin trn D = [0; +) v hm s y = x 2 + 3 l mt
hm s ng bin trn , do cng ng bin trn D = [0; +). Theo tnh cht ca hm
s n iu th phng trnh x 2 + 3 = 3 x c nhiu nht mt nghim, kim tra c
x = 1 l nghim ca phng trnh.
Vy, h phng trnh cho c mt nghim duy nht l (1;1).
x + y = 1
12) x
(I )
y

=
2
2
2

y
x
2 x+ y = 2
2 . 2 = 2
(I ) x

y
x
y
2 2 = 2
2 + 2 = 2

x
u = 2
t
, iu kin u > 0, v < 0. Ta c h
y
v = 2

u + v = 2
, khi u, v l nghim ca

u.v = 2

t = 1 + 3
phng trnh t 2 2t 2 = 0
. Nh vy ta c
t = 1 3

u = 1 + 3
2 x = 1 + 3
x = log 2 1 + 3

y
v = 1 3
2 = 1 3
y = log 2 3 1

(
(

)
)

( (

Vy, h phng trnh cho c mt nghim duy nht l log 2 1 + 3 ; log 2

))

3 1 .

23 x +1 + 2 y 2 = 3.23 x + y (1)
13)
2
3 x + xy + 1 = x + 1(2)
x 1
x + 1 0
(2) 2

3 x + xy + 1 = x + 1 x ( 3 x + y 1) = 0
x = 0

x 1
y = 1 3 x

259

+ Vi x = 0, thay vo (1) ta c
2 + 2 y 2 = 3.2 y 8 + 2 y = 12.2 y 2 y =

8
8
y = log 2 .
11
11

x 1
+ Vi
, thay y = 1 3 x vo (1) ta c
y = 1 3x
2
1
23 x +1 + 2 3 x 1 = 6 23 x +1 + 3 x +1 = 6 ( 23 x +1 ) 6.23 x +1 + 1 = 0
2
3 x +1
2
= 3+2 2

3 x +1
= 32 2
2
Do iu kin x 1 nn 23 x +1

1
v vy ta chn 23 x +1 = 3 + 2 2. Ta c
4

23 x +1 = 3 + 2 2 3 x + 1 = log 2 3 + 2 2

3+2 2
1
1
x = log 2 3 + 2 2 1 = log 2

3
3
2

y = 2 log 2 3 + 2 2 .

Vy, h phng trnh cho c hai nghim l


11 1

0;log 2 , log 2 3 + 2 2 1 ; 2 log 2 3 + 2 2 .


8 3

2 x+2
+ 22 y + 2 = 17
3
14) x +1
(I )
y
2.3 + 3.2 = 8

u = 3x
t
, iu kin: u , v > 0.
y
v = 2
Khi h phng trnh ( I ) c bin i v dng
8 3v 2
25v 2 48v 4 = 0
+ 4v 2 = 17
9

9u 2 + 4v 2 = 17

8 3v
6u + 3v = 8
u = 8 3v
u =
6

6
2

v = 2 v = 25
u =
3

u = 8 3v
v = 2, ( v > 0 )

6
x 1
x = 1
3 =
Nh vy ta c
. Vy, h phng trnh cho c nghim l ( 1;1) .
3
y =1
2 y = 2

260

V.6. 1) log 3 3x 8 = 2 x

Phng trnh cho tng ng vi


3x 8 = 32 x 3x 8 =

9
3x

(3x ) 2 8.3x 9 = 0
3 x = 1
x
3x = 9 x = 2 .
3 = 9
Vy, nghim ca phng trnh cho l x = 2.
2) log x 1 3 = 2(1)
x 1 > 0
x > 1
iu kin:

x 1 1
x 2
x = 1+ 3
(1) ( x 1)2 = 3 x 2 2 x + 1 = 3 x 2 2 x 2 = 0
x = 1 3
Ta chn x = 1 + 3
Vy, nghim ca phng trnh cho l x = 1 + 3.
3)

log 2 (9 2 x )
= 1(1)
3 x

x 3
x 3
iu kin:

x
x < log 2 9
9 2 > 0
(1) log 2 (9 2 x ) = 3 x (9 2 x ) = 2(3 x ) 9 2 x =

( )

8
2x

2x = 1
x = 0
9.2 + 8 = 0 x

x = 3
2 = 8
x

Ta chn x = 0.
Vy, nghim ca phng trnh cho l x = 0.
4)

1
1
log 2 ( x + 3) + log 4 ( x 8)8 = log 2 (4 x)(1)
2
4

x + 3 > 0
x > 3
x > 0

iu kin: x 8 0 x 8
x 8
4 x > 0
x > 0

1
(1) log 2 ( x + 3) + log 2 ( x 8)8 = log 2 (4 x )
8

261

log 2 ( x + 3) + log 2 x 8 = log 2 (4 x )


log 2 ( x + 3) x 8 = log 2 (4 x)
( x + 3) x 8 = 4 x

Kt hp vi iu kin bi ton ta c
x 2 + x + 24 = 0

0 < x < 8

2
x 9 x 24 = 0
x > 8

( x + 3)(8 x) = 4 x

0 < x < 8
(1)
( x + 3)( x 8) = 4 x

x > 8

Vy, nghim ca phng trnh cho l x =


5) log 2 (4 x + 15.2 x + 27) + 2 log 2

1 + 97
x =
2

9 + 177
x =

1 + 97
9 + 177
;x =
.
2
2

1
= 0(1)
(4.2 x 3)

x
x
3
3
4 + 15.2 + 27 > 0
iu kin: x
4.2 x 3 > 0 2 x > x > log 2 .
4
4
4.2 3 > 0

(1) log 2

4 x + 15.2 x + 27
= 0.
(4.2 x 3)2

4 x + 15.2 x + 27
= 1 4 x + 15.2 x + 27 = (4.2 x 3) 2
(4.2 x 3) 2

2 2 x + 15.2 x + 27 = 16.22 x 24.2 x + 9


15.2 2 x + 39.2 x + 18 = 0 5.2 2 x + 13.2 x + 6 = 0

2x = 3
x
2 x = 3 x = log 2 3
2 = 2

5
x = log 2 3 tha iu kin. Vy, nghim ca phng trnh cho l x = log 2 3.
6) log 7 (2 x 1) + log 7 (2 x 7) = 1(1)
2 x 1 > 0
iu kin: x
2 x > 7 x > log 2 7
2 7 > 0
(1) log 7 (2 x 1)(2 x 7) = log 7 7

( )

(2 x 1)(2 x 7) = 7 2 x

2x = 0
x
2x = 8 x = 3
2 = 8
262

8.2 x = 0

x = 3 tha iu kin. Vy, nghim ca phng trnh cho l x = 3.


1
= 0 (1)
4

7 ) 2 ( log 2 x + 1) log 4 x + log 2

iu kin: x > 0
1
(1) 2 ( log 2 x + 1) log 2 x 2 log 2 2 = 0
2
( log 2 x + 1) log 2 x 2 = 0
log 22 x + log 2 x 2 = 0

x = 2
log 2 x = 1
(Tha iu kin)

x = 1
log 2 x = 2

4
1
Vy, nghim ca phng trnh cho l x = 2; x = .
4

8 ) 2log 3 ( 4 x 3) + log 1 ( 2 x + 3) = 2 (1)


3

x > 3
4
3
0
x

>

4 x > 3 .(*)
iu kin:

3
4
2 x + 3 > 0
x> 2

(1) log 3 ( 4 x 3) log 3 ( 2 x + 3) = 2


2

log 3 ( 4 x 3) log 3 ( 2 x + 3) = log 3 32


2

( 4 x 3)
log 3
( 2 x + 3)

= log 3 32

( 4 x 3)

( 2 x + 3)

= 9 ( 4 x 3) = 9 ( 2 x + 3)

16 x 2 24 x + 9 = 18 x + 27

x = 3
8 x 21x 9 = 0
x = 3
8

i chiu vi iu kin (*) ta chn x = 3.

Vy, nghim ca phng trnh cho l x = 3.

9 ) log 3 log 2 1 x 3log 1 x + 5 = 2(1)

2
2

263

(1) log 3 log 2 1 x 3log 1 x + 5 = log 3 9

2
2

log 2 1 x 3log 1 x + 5 = 9
2

log 1 x = 1

x 3log 1 x 4 = 0 2
log x = 4
2
12

log 2 1
2

x = 2
log 2 x = 1

x = 1
log 2 x = 4
16

Vy, nghim ca phng trnh cho l x = 2; x =


10) log x 2 + 2 log 2 x 4 = log

2x

8(1)

x > 0; x 1
x > 0

iu kin: 2 x > 0; 2 x 1
x 1

2 x > 0; 2 x 1 x 1
2

(1) log x 2 + 2 log 2 x 22 = 2 log 2 x 23


log x 2 + 4 log 2 x 2 = 6 log 2 x 2

log x 2 = log 2 x 4 log x 2 =


log x 2 =

log x 4
log x 2 x

2log x 2
2
( log x 2 ) = log x 2
log x 2 + 1

log x 2 = 0
log x 2(log x 2 1) = 0
log x 2 1 = 0

log x 2 = 1 x = 2 (Nhn)
Vy, nghim ca phng trnh cho l x = 2.
11) log

x + 1 log 1 (3 x ) log 8 ( x 1)3 = 0(1)


2

x +1 > 0
x > 1

iu kin: 3 x > 0 x < 3 1 < x < 3


x 1 > 0
x > 1

(1) log 2 ( x + 1) + log 2 (3 x) log 2 ( x 1) = 0


264

1
.
16

log 2 ( x + 1)(3 x ) = log 2 ( x 1)


( x + 1)(3 x) = x 1
x2 x 4 = 0

1 + 17
x =
2

1 17
x =

2
Ta chn x =

1 + 17
2

Vy, nghim ca phng trnh cho l x =

1 + 17
.
2

12) log 3 (9 x + 9) = x + log 3 (28 2.3x )(1)


28 2.3x > 0
iu kin: x
28 2.3x > 0 2.3x < 28 3x < 14 x < log3 14
9 + 9 > 0
(1) log 3 (9 x + 9) = log 3 3x + log 3 (28 2.3x )

log 3 (9 x + 9) = log3 3x (28 2.3x )


9 x + 9 = 28.3x 2.9 x
3x = 9
x = 2
3.(3x )2 28.3x + 9 = 0 x 1
(Nhn)
3 =
x = 1

3
Vy, nghim ca phng trnh cho l x = 2; x = 1.
13) 16 log 27 x3 x 3log 3 x x 2 = 0(1)
27 x3 > 0

3
27 x 1
x > 0

iu kin: x > 0

1
3 x > 0
x 3

3 x 1
(1)

16
log 3 x x 6 log 3 x x = 0
3

2
log 3 x x = 0 log 3 x x = 0 x = 1 (Tha iu kin)
3

Vy, nghim ca phng trnh cho l x = 1.


14)

log 2 2 x 2 + log 4 16 x = log 4 x 3 (1)


265

iu kin: x > 0
log x 3 0
(1) 4 2
2 3
log 2 2 x + log 4 16 x = log 4 x

x 1

1
9
2
log 2 2 + 2log 2 x + 2 log 2 2 + 2 log 2 x = 4 log 2 x
x 1

2
9 log 2 x 10 log 2 x 12 = 0

x 1

log x = 5 +
2

log x = 5
2

x 1

133
5 + 133
5+ 133

9
9
x

=
2
x
2
=

5 133
133
x = 2 9

Vy, nghim ca phng trnh cho l x = 2

5+ 133
9

) (
)
( 2 + 1) .log 2 ( 2 + 1) = 6

15) log 2 2 x + 1 log 2 x+1 + 2 = 6(1)


(1) log 2

)(

))

log 2 2 x + 1 . 1 + log 2 2 x + 1 = 6
log 22 (2 x + 1) + log 2 (2 x + 1) 6 = 0

2 x + 1 = 4
log 2 (2 x + 1) = 2

x
x
2 + 1 = 1
log 2 (2 + 1) = 3

8
2x = 3
x
2 x = 3 x = log 2 3
2 = 7

8
Vy, nghim ca phng trnh cho l x = log 2 3.

16) log 22 x 2 2 x 2 x 4 = 2

log 3

1
2 2 x2

2 x 2 x 4 > 0
1 < x < 1

2
iu kin: 2 2 x > 0

x
2
2

2 2 x 1

266

(1)

(1) log 2 2 x

(2 x

log 22 x 2 2 x x
2 x2 x4 =

x 4 = log 2 2 x2 2 2 x 2
4

) = log

4 2 2 x2

4 2 2x2
2 2 x2

log 22 x 2

3
4

3
19 x 29 x + 10 = 0
4

x = 1 x = 1
x2 = 1
2 10
10
10
x =
x=
x=

19
19
19
Ta chn x =

10
10
x=
.
19
19

Vy, nghim ca phng trnh cho l x =

10
10
,x =
.
19
19

17) ( x + 1) log 32 x + 4 x log 3 x 16 = 0 (1)


iu kin: x > 0
t t = log 3 x

Khi (1) tr thnh

( 3 + 1) t
t

+ 4.3t .t 16 = 0 t 2 .3t + t 2 + 4.3t .t 16 = 0

t = 4
t.3t ( t + 4 ) + ( t 2 16 ) = 0 ( t + 4 ) ( t.3t + t 4 ) = 0 t
t.3 + t 4 = 0

+ Vi t = 4 log 3 x = 4 x =

1
81

+ Vi t.3t + t 4 = 0 t (3t + 1) = 4 t > 0 3t + 1 =

4
(2).
t

R rng nghim t ca (2) ch c th dng, do ta ch xt t ( 0; + ) . Ta c hm s


4
nghch bin trn khong ( 0; + ) v hm s y = 3t + 1 cng ng bin trn khong
t
. Do phng trnh ( 2 ) c nhiu nht mt nghim, kim tra c t = 1 l nghim ca
y=

phng trnh ( 2 ) .
T suy ra log 3 x = 1 x = 3.
Vy, nghim ca phng trnh cho l x =

1
, x = 3.
81

267

18) log 3

3
x3 1
log 2 x log 3
= + log 2 x (1)
x
3 2

iu kin: x > 0.

Ta thy rng x = 1 l nghim ca phng trnh (1)


Vi x 1 th phng trnh (1) tng ng vi

(1 log3 x ) log 2 x 3log3 x

1 1 1
= + log 2 x
2 2 2

2 (1 log3 x ) log 2 x log 2 x = 6 log3 x (1 2 log 3 x ) log 2 x = 6 log3 x

log x 2
2
=6
log x 3 2 = 6 log x 2
log x 3
log x 3

3
3
=
x =
3
3
64
8
log x
= 2 x2 =

64
64
3
3
=
x =
64
8

Ta chn x =

3
.
8

Vy, nghim ca phng trnh cho l x =


19) log 3 4 x2 (9 16 x 4 ) = 2 +

3
; x = 1.
8

1
(1)
log 2 (3 4 x 2 )

3
3
3 4 x 2 > 0
< x<

2 3 < x< 3
iu kin: 3 4 x 2 1 2
2
2
9 16 x 4 > 0 x 3

2
(1) log 34 x2 (9 16 x 4 ) = 2 + log3 4 x2 2
log 34 x 2 (9 16 x 4 ) log 34 x 2 2 = 2
log 34 x 2

(9 16 x 4 )
=2
2

9 16 x 4
= (3 4 x 2 )2
2
9 16 x 4 = 2 9 24 x 2 + 16 x 4

48 x 4 48 x 2 + 9 = 0 16 x 4 16 x 2 + 3 = 0

268


2 3
3
3
x=
x =
x = 4
2
2

1
1
1

x2 =
x = 2 x = 2

Ta chn x =

1
1
x= .
2
2

1
1
Vy, nghim ca phng trnh cho l x = ; x = .
2
2

20) log 2 (4 x +1 + 4).log 2 (4 x + 1) = log 1


2

1
(1)
8

(1) log 2 4(4 x + 1) .log 2 (4 x + 1) = 3


2 + log 2 (4 x + 1) .log 2 (4 x + 1) = 3
log 22 (4 x + 1) + 2 log 2 (4 x + 1) 3 = 0
4x + 1 = 2
log 2 (4 x + 1) = 1

x
1
x
log 2 (4 + 1) = 3 4 + 1 =

8
4x = 1
x
4 x = 1 x = 0.
4 = 7

8
Vy, nghim ca phng trnh cho l x = 0.
9
21) log 2 (2 x 2 ).log 2 (16 x) = log 2 2 x(1)
2

iu kin: x > 0
(1) (log 2 2 + log 2 x 2 ).(log 2 24 + log 2 x) =

9
log 2 2 x
2

9
log 22 x
2
9
4 + log 2 x + 8log 2 x + 2 log 22 x = log 22 x
2
5
log 22 x + 9log 2 x + 4 = 0 5log 2 2 x 18log 2 x 8 = 0
2
(1 + 2log 2 x ).(4 + log 2 x ) =

2
2

log
x
=

5
2
x
=
2

x = 16

log 2 x = 4

Vy, nghim ca phng trnh cho l x = 2 5 ; x = 16.


269

22) lg x + 1 + 3lg 1 x = lg 1 x 2 (1)


x +1 > 0
x > 1

iu kin: 1 x > 0 x < 1


1 < x < 1
1 x 2 > 0
1 < x < 1

(1) lg

x +1
1 x2

+ 3lg 1 x = 0

x +1

lg
.( 1 x )3 = 0
2
1 x

lg( 1 x )2 = 0 lg(1 x) = 0 x = 0 (Nhn)

Vy, nghim ca phng trnh cho l x = 0.


23) x + lg (1 + 2 x ) = x lg 5 + lg 6(1)
V 2 x + 1 > 0, x nn ta c
5x.6
5 x.6
x

=
10 x 1 + 2 x = 5 x.6
10
1 + 2x
1 + 2x
2x = 2
x
x
2 1+ 2 = 6 x
x =1
2 = 3

(1) x = lg

Vy, nghim ca phng trnh cho l x = 1.


24) log 2

2x 1
= 1 + x 2 x (1)
x

2x 1
iu kin:
> 0 2x 1 > 0 x > 0
x
(1) log 2 (2 x 1) log 2 x = 1 + x 2 x
log 2 (2 x 1) + 2 x 1 = log 2 x + x f (2 x 1) = f ( x )

Vi f (t ) = log 2 t + t (t > 0)
Ta thy f (t ) = 1 +

1
> 0, t > 0
t ln 2

Suy ra, hm s f (t ) ng bin trn khong ( 0; + ) .

270

T suy ra 2 x 1 = x 2 x = x + 1.
Ta c hm s y = 2 x c y = 2 x ln 2, y = 2 x ln 2 2 > 0, x , suy ra th ct ng thng
y = x + 1 khng qu hai im. Kim tra c phng trnh 2 x = x + 1 c x = 1, x = 0 nghi m
ng. Tuy nhin do iu kin nn ta ch nhn x = 1.

Vy, nghim ca phng trnh cho l x = 1.


25) log 3

1
x 2 3x + 2 + 2 +
5

x 2 + 3 x 1

= 2(1)

iu kin: x 2 3 x + 2 0 x 1 x 2

(1) log3 (

x 2 3x + 2 + 2 + 5x

3 x +1

=2

t t = x 2 3x + 2 ( t 0 )

Khi (1) tr thnh log 3 ( t + 2 ) + 5t

= 2 log 3 ( t + 2 ) = 5t

+ 2(2)

Xt hm s f ( t ) = log 3 ( t + 2 )
Ta thy f ( t ) =

1
> 0 t 0
( t + 2) ln 3

f ( t ) ng bin trn [ 0; + )

Xt hm s g ( t ) = 5t
g ( t ) = 2t.5t

+2

0, t 0

g ( t ) nghch bin trn [ 0; + ) .

Do , phng trnh ( 2 ) c nhiu nht mt nghim, kim tra c t = 1 l nghim ca


phng trnh ( 2 ) .

Suy ra

3+ 5
x =
2
x 2 3x + 2 = 1 x 2 3x + 2 = 1 x 2 3 x + 1 = 0
(Nhn)

3 5
x =
2

Vy, nghim ca phng trnh cho l x =


26) log 3

x + 2 = log 2

3+ 5
3 5
;x =
.
2
2

x + 1 (1)

iu kin: x 0

271

t t = log 2

x + 1 (t 0)

x + 1 = 2t x + 2 = 2t + 1.

Phng trnh (1) tr thnh log 3 2t + 1 = t 2t + 1 = 3t


Chia hai v phng trnh cho 3t ta c
t

2t 1
2 1
+ t = 1 + = 1( 2 )
t
3 3
3 3
t

2 1
Ta thy hm s f ( t ) = + nghch bin trn nn phng trnh ( 2 ) c nhiu nht
3 3
mt nghim, kim tra c t = 1 l nghim ca ( 2 ) .

log 2

x + 1 = 1 x + 1 = 2 x = 1.

Vy, nghim ca phng trnh cho l x = 1.


3
= 1(1)
2 + 1 + log 2 x

27)

iu kin: 1 + log 2 x 0 x

(1) 2x +

1
2

1 + log 2 x = 3(2)

V y = 2 x l hm s ng bin trn v y = 1 + log 2 x l hm s ng bin trn


1

x
; + , nn hm s f ( x ) = 2 + 1 + log 2 x l hm s ng bin trn ; + , suy ra
2

( 2 ) c nhiu nht mt nghim, kim tra c x = 1 l nghim ca ( 2 ) .


Vy, nghim ca phng trnh cho l x = 1.
28) log

2 2+ 3

(x

2 x 2 = log 2+

(x

2 x 3 (1)

x 2 2 x 2 > 0
x < 1
iu kin: 2

x > 3
x 2 x 3 > 0
t t = x 2 2 x 3, t > 0

Khi (1) tr thnh


log

2 2+ 3

( t + 1) = log 2+

t=y

log 2 2+ 3 t + 1 = y
t + 1 = 2 2 + 3

t = 2 + 3 y
log 2+ 3 t = y

272

) ( 2)
( 3)

2+ 3

+1 = 2 2 + 3

2+ 3

2 2+ 3


1
+

2 2+ 3

2+ 3
Xt hm s f ( y ) =
2 2+ 3

2+ 3

= 1( 4 )


1
+

2 2+ 3

< 1, nn hm s f ( y ) l hm nghch bin trn , v


2 2+ 3
2 2+ 3
vy ta kt lun c phng trnh ( 4 ) c nhiu nht mt nghim. Kim tra c y = 2 l

Ta c

< 1 v

nghim ca phng trnh ( 4 ) , v l nghim duy nht.

t = 2+ 3

x2 2x 3 = 2 + 3

x = 1 11 + 4 3
(Nhn)
x 2 2 x 10 4 3 = 0
x = 1 + 11 + 4 3

Vy, nghim ca phng trnh cho l x = 1 11 + 4 3; x = 1 + 11 + 4 3.


29)

2 lg x = 1 lg x 1 (1)

x > 0
x > 0
iu kin:

x > 10
lg x 1 0
x > 10
t t = 3 2 lg x t 3 = 2 lg x lg x = 2 t 3

Phng trnh (1) tr thnh

t 1
t 1
t = 1 1 t 3 1 t3 = 1 t
2 3
2
3
1 t = (1 t )
t + t 2t = 0
t 1
t = 0

t = 0


t = 1
t = 1
t = 2
t = 2
+ Vi t = 0 3 2 lg x = 0 lg x = 2 x = 100 (Nhn)
+ Vi t = 1 3 2 lg x = 1 lg x = 1 x = 10 (Nhn)
+ Vi t = 2 3 2 lg x = 2 2 lg x = 8 lg x = 10 x = 1010 (Nhn)
Vy, nghim ca phng trnh cho l x = 100; x = 10; x = 1010.
273

3 + log 2 ( x 2 4 x + 5 ) + 2 5 log 2 ( x 2 4 x + 5 ) = 6(1)

30)

V x 2 4 x + 5 > 0 x v x 2 4 x + 5 = ( x 2 ) + 1 1 nn

log 2 x 2 4 x + 5 0, x .

t t = log 2 x 2 4 x + 5 ( t 0 )

Phng trnh cho tr thnh

3 + t + 2 5 t = 6 ( 2)

t 0

iu kin: 3 + t 0 0 t 5
5 t 0

( 2) 3 + t + 4 ( 5 t ) + 4 ( 3 + t )( 5 t ) = 36
t = 1
4 ( 3 + t )( 5 t ) = 3t + 13 25t + 46t 71 = 0
t = 71
25

Ta chn t = 1

x = 1
log 2 ( x 2 4 x + 5 ) = 1 x 2 4 x + 5 = 2 x 2 4 x + 3 = 0
x = 3
Vy, nghim ca phng trnh cho l x = 1; x = 3.
31) log 22 x + log 2 x + 1 = 1

x > 0
x > 0
1

iu kin:

1 x
2
log 2 x + 1 0
x 2
t t = log 2 x + 1 ( t 0 )
t 2 = log 2 x + 1 log 2 x = t 2 1
2

Phng trnh cho tr thnh ( t 2 1) + t = 1

t 4 2t 2 + t = 0 t t 3 2t + 1 = 0 t ( t 1) t 2 + t 1 = 0

t = 0

t ( t 1) ( t + t 1) = 0 t 1 = 0
t 2 + t 1 = 0
2

274

t = 0
t = 1

t = 1 + 5
2

1 5
t =
2

Ta chn t = 0; t = 1; t =

1 + 5
2

+ Vi t = 0

log 2 x + 1 = 0 log 2 x = 1 x =

1
(Nhn)
2

+ Vi t = 1

log 2 x + 1 = 1 log 2 x = 0 x = 1 (Nhn)


+ Vi t =

1 + 5
2
2

1 5
5 1
5 1
1 5
log 2 x + 1 =
log 2 x =
x = 2 2 (Nhn)
1 log 2 x =
2
2
2
1 5
1
Vy, nghim ca phng trnh cho l x = ; x = 1; x = 2 2 .
2

V.7. 1) log 1 x + 2 log 1 ( x 1) + log 2 6 0(1)


2

x 1 > 0
iu kin:
x >1
x > 0
(1) log 21 x + 2 log 22 ( x 1) + log 2 6 0

log 2 x log 2 ( x 1) + log 2 6 0


log 2 ( x 2 x) + log 2 6 0
log 2
0<

6
0
x x
2

6
1
x x
2

6
x 2 x > 0

6 1
x 2 x
275

x < 0
x( x 1) > 0

x 2

x > 1
2
6 x + x

x 3
x( x 1) 0
x 2

x 3

So vi iu kin th tp hp nghim ca bt phng trnh cho l [ 3; + ) .


2) log x (log 3 (9 x 72)) 1(1)

9 x 72 > 1
log 3 ( 9 x 72 ) > 0
x

9 72 > 0
x
iu kin: 9 72 > 0

x > log 9 73.


x
>
0
x > 0, x 1

x 1

Vi iu kin x > log 9 73 suy ra x > 1, do ta c


(1) log 3 (9 x 72) x

9 x 72 3x
32 x 3x 72 0 8 3x 9

3x 0
x
x2
3 9
So vi iu kin th tp hp nghim ca bt phng trnh cho l ( log 9 73; 2] .
Ch . Chng ta c th xt hai trng hp ca c s x, c th trnh by nh sau:
log x (log 3 (9 x 72)) 1
x > 1
x > 1
x > 1
x x

9 3 72 0
x
x
x
0
<
log
(9

72)

x
<

1
9
72
3
3

9 x > 73

0 < x <1
0 < x < 1

0 < x < 1
x
x
log 3 (9 x 72) x

x x

9
72
3

9 3 72 0
x > 1
x > 1

x
8 3 9
x 2
x > log 73


x > log 9 73 log 9 73 < x 2.
9

0 < x < 1
0 < x < 1
x
x 2 (VN )
3 9

Vy, tp hp nghim ca bt phng trnh cho l ( log 9 73; 2] .


3) log 2 x +3 x 2 < 1(1)

276

+ Nu 2 x + 3 > 1 x > 1(*)

(1) x 2 < 2 x + 3 x2 2 x 3 < 0 1 < x < 3


Kt hp vi (*) ta c 1 < x < 3.
+ Nu 0 < 2 x + 3 < 1

3
< x < 1( **)
2

x < 1
x > 3

(1) x 2 > 2 x + 3 x 2 2 x 3 > 0


Kt hp vi (**) ta c

3
< x < 1.
2

Vy , tp hp nghim ca bt phng trnh cho l ; 1 ( 1;3) .


2

4 ) log 9 x2 x 2 + 2 x + 6

1
(1)
2

1
1
+ Nu 0 < 9 x 2 < 1 < x < x 0.
3
3

(1) x 2 + 2 x + 6

9 x 2 x 2 + 2 x + 6 3x

2
2
x 2 x 6 3 x
x 5 x 6 0
x 2 x 6 3x x + 2 x + 6 2
2
x + 2 x + 6 3 x
x + x 6 0
1 x 6

1 x 2.
3 x 2
2

Kt hp vi iu kin ang xt th nghim ca bt phng trnh trong trng hp ny l


1
1
< x < x 0.
3
3
1

x<

3
+ Nu 9 x 2 > 1
x > 1

3
x 2 + 2 x + 6 > 0
1 7 < x < 1 + 7
1

2
() 2
2
x + 2 x + 6 3 x
x + 2 x + 6 9 x
1 7 < x < 1 + 7
1 7 < x < 1 + 7
1 7 < x < 1 + 7

3 x x 2 + 2 x + 6 x 2 + x 6 0
x 3 x 2

2
x 1 x 6
3 x x 2 x 6
x 5 x 6 0

277

1 7 < x 1

2 x < 1 + 7
Kt hp vi iu kin ang xt th nghim ca bt phng trnh trong trng hp ny l
1 7 < x 1 2 x < 1 + 7.
Vy, nghim ca bt phng trnh cho l
1
1
< x < 1 7 < x 1 2 x < 1 + 7 x 0.
3
3

5) log 1 4 x + 4 log 1 22 x +1 3.2 x (1)


2

iu kin: 22 x +1 3.2 x > 0 x log 2

3
(*) . V c s b hn 1 nn ta c
2

(1) 4 x + 4 22 x +1 3.2 x
2 2 x + 4 22 x.2 3.2 x
2 2 x 3.2 x 4 0
2 x 1
x
2x 4 x 2
2 4
So vi iu kin th tp hp nghim ca bt phng trnh cho l [ 2; + ) .
Ch . V 4 x + 4 > 0, x nn t 4 x + 4 22 x +1 3.2 x 22 x +1 3.2 x > 0, do c th khng
cn t iu kin (*).
6) log x 3 2 x 2 10 x + 24 log x 3 x 2 9 (1)

+ Trng hp 1.
x 3 >1
x > 4
2

(1) x 9 > 0
x < 3 x > 3

x 2 20 x + 57 0
2
2

2 x 10 x + 24 x 9

x > 4

x 10 43 x 10 + 43.

x 10 + 43

+ Trng hp 2.
3 < x < 4
0 < x 3 < 1

2
3 < x < 4
x < 4
(1) x 10 x + 24 > 0

2
x 20 x + 57 0

x > 6
2
2
2
x

10
x
+
24

x 2 20 x + 57 0

278

3 < x < 4

10 43 x < 4.
10 43 x 10 + 43

Vy, tp hp nghim ca bt phng trnh cho l 10 43; 4 10 + 43; + .


7)

1
1
<
(1)
log 3 ( x + 1) 2 log 9 x 2 + 6 x + 9

iu kin:
x +1 > 0
x > 1
x > 1
x > 1
2

x > 1
x + 6x + 9 > 0
x 3
x 3
x 0

x 0
log 3 ( x + 1) 0
x 0
x 0
x 2
2

x 2 + 6 x + 9 1 x 2 + 6 x + 8 0
x 4
log 3 ( x + 6 x + 9) 0

(1)

1
1
<
log 3 ( x + 1) 2. 1 log x 2 + 6 x + 9
3
2

1
1
1
1
<

<
(2)
log 3 ( x + 1) log 3 x 2 + 6 x + 9
log 3 ( x + 1) log 3 x + 3

+ Nu x > 0 th log 3 ( x + 1) > 0, log 3 x + 3 > 0 nn ta c


(2) log 3 ( x + 3) < log3 ( x + 1) x + 3 < x + 1 (VN )

+ Nu 1 < x < 0 th log 3 ( x + 1) < 0, log 3 x + 3 = log 3 ( x + 3) > 0


Do (2) ng.
Vy, tp hp nghim ca bt phng trnh cho l ( 1; 0 ) .
8)

( log 2 x )

+3

log 2 x + 3

> 2(1)

x > 0
x > 0

iu kin:

1
log 2 x + 3 0
x 8
t t = log 2 x. Khi , bt phng trnh (1) tr thnh

3 < t < 1
t2 + 3
t2 + 3
t 2 2t 3
>2
2>0
>0
t +3
t +3
t +3
t > 3
1
1
<
x
<
3 < log 2 x < 1
T suy ra
8
2

log
x
>
3
2
x > 8

279

1 1
So vi iu kin th tp hp nghim ca bt phng trnh cho l ; ( 8; + ) .
8 2

9 ) log 2 2 x 1 log 1 2 x +1 2 > 2 (1)


2

iu kin: 2 x 1 > 0 x > 0

(1) log 2 ( 2 x 1) log 2 2 ( 2 x 1) < 2

)(
( 2 1) )

))
( 2 1) 2 < 0

log 2 2 x 1 1 + log 2 2 x 1 < 2

log 2

+ log 2

2 < log 2 2 x 1 < 1

1
5
5
< 2 x 1 < 2 < 2 x < 3 log 2 < x < log 2 3
4
4
4

So vi iu kin th tp hp nghim ca bt phng trnh cho l log 2 ; log 2 3 .


4

18 2 x
10 ) log 4 18 2 x log 2
8

1(1)

iu kin: 18 2 x > 0 2 x < 18 x < log 2 18

(1)

1
log 2 18 2 x log 2 18 2 x log 2 8 1

log 22 18 2 x 3log 2 18 2 x + 2 0

1 log 2 18 2 x 2 2 18 2 x 4
14 2 x 16 log 2 14 x 4.
So vi iu kin th tp hp nghim ca bt phng trnh cho l log 2 14; 4 .

11) log 4 3x 1 log 1


4

3x 1 3
(1)
16
4

iu kin: 3x 1 > 0 x > 0

(1) log 4 3x 1 ( 1) log 4

3x 1 3

16
4

3
log 4 3x 1 ( 1) log 4 3x 1 log 4 42
4
3
log 24 3x 1 2 log 4 3x 1 + 0
4

log 24 (3x 1) 2 log 4 (3x 1) +

280

3
0
4

x
log 4 (3 1) 2

log (3x 1) 3
4
2
x > 0
0 < 3x 1 2
0 < x 1

x
x 1
3 1 8
x 2
x 2

Vy, tp hp nghim ca bt phng trnh cho l ( 0;1] [ 2; ) .


12) log x

9 x 2 x 1 1(1)

+ Trng hp 1.
x > 1
x > 1
x < 1
(1)

2
2
2
9 x 2 x + 1 9 x 1
9 x x 1 x

x > 1
x < 1
x > 1
x < 1

2
2
2
2
2
9 x 4 x + 4 x + 1 9 x 1 5 x + 4 x 8 0 x 8
x > 1

x < 1
2 2 11
2 + 2 11
x
2 2 x 2 2

5
5

2 2 x < 1.
+ Trng hp 2.

1 < x < 1
0 < x < 1

(1)
9 x2 x 1 > 0
2

0 < 9 x x 1 x
2
9 x x 1 x
0 < x < 1
1 < x < 0

9 x 2 x 1 > 0 9 x2 x 1 > 0

2
2
9 x 2 x + 1
9 x 1
0 < x < 1
1 < x < 0

9 x2 > x +1 9 x2 > x +1

2
2
9 x 2 x + 1 9 x 1
0 < x < 1
1 < x < 0

2
2
9 x > x + 2 x + 1 9 x 2 > x 2 + 2 x + 1
9 x 2 4 x 2 + 4 x + 1 9 x 2 1

281


0 < x < 1
0 < x < 1
1 < x < 0

1 + 17
2
2
1 17
x + x 4 < 0 x + x 4 < 0(VN )
< x<
2
2
5 x 2 + 4 x 8 0 x 2 8 0

2 2 11
2 + 2 11
x
x
5
5

2 + 2 11
x < 1.
5

2 + 2 11
Vy, tp hp nghim ca bt phng trnh cho l 2 2; 1
;1 .
5

13)

log 21 x + 4 log 2 x < 2 4 log16 x 4 (1)


2

iu kin: x > 0.

(1) log 22 x + 2log 2 x < 2 4 log16 x 4

log 22 x + 2log 2 x < 2 ( 4 log 2 x )

2 ( 4 log 2 x ) > 0

log 22 x + 2 log 2 x 0

2
2
log 2 x + 2 log 2 x < 2 ( 4 log 2 x )
log x < 4
2
log 2 x < 2 log 2 x > 0

2
2
log 2 x + 2 log 2 x < 2 16 8log 2 x + log 2 x
0 < log 2 x < 4

2
log 2 x 18log 2 x + 32 > 0
0 < log 2 x < 4

log 2 x < 2 log 2 x > 16


0 < log 2 x < 2 1 < x < 4.

So vi iu kin th tp hp nghim ca bt phng trnh cho l [1; 4] .


14) log x 2 x log x 2 x3 (1)

x > 0
iu kin:
x 1
(1) log x 2 + 1 log x 2 + 3

282

t t = log x 2. Khi , bt phng trnh (1) tr thnh

t + 1 < 0
3 t < 1

t
+
3

0
3 t < 1

t + 1 t + 3
t 1

3 t 1

1
t + 1 0

t 2 + t 2 0

t + 3 (t + 1)2
x > 1
3
x 2
x 2 x

Nh vy ta c 3 log x 2 1
0 < x 1
0 < x <1

2
x 3 2 x

1
So vi iu kin th tp hp nghim ca bt phng trnh cho l 0; 3 [ 2; + ) .
2

15)

lg 2 x 3lg x + 3
< 1(1)
lg x 1

t t = lg x

Bt phng trnh (1) tr thnh


t 2 3t + 3
t 2 3t + 3
t 2 4t + 4
<1
1 < 0
< 0 t < 1.
t 1
t 1
t 1

lg x < 1 0 < x < 10.


Vy, tp hp nghim ca bt phng trnh cho l ( 0;10 ) .
2

log 1 ( x + 3) log 1 ( x + 3 )
16)

x +1

> 0(1)

( x + 3 )3 > 0
x > 3

x > 3.
iu kin:

2
x

x
+
3
>
0
(
)
2

(1)

log 2 ( x + 3) + log 3 ( x + 3)
x +1
3

log3 ( x + 3)
3
+ log 3 ( x + 3)
log 3 2
>0
>0
x +1
2

log 3 2.log 3 ( x + 3) log3 ( x + 3)


3log3 2.log 3 ( x + 3) 2 log 3 ( x + 3)

>0
>0
x +1
x +1
log 3 ( x + 3) . ( 3log 3 2 2 )
log3 ( x + 3)

>0
<0
x +1
x +1
(V 3log 3 2 2 < 0 )

283

log 3 ( x + 3) < 0
0 < x + 3 < 1 3 < x < 2

x + 1 > 0
x > 1
x > 1

2 < x < 1.
x + 3 > 1
x > 2
log 3 ( x + 3) > 0

x + 1 < 0
x
<

x < 1

Vy, tp hp nghim ca bt phng trnh cho l ( 2; 1) .


17) log 5 x + log x

x (2 log 3 x) log 5 x
<
(1)
3
log 3 x

x > 0
iu kin:
x 1
(1) log 5 x + 1 log x 3 < 2 log 5 3 log 5 x
2 log 5 x 2log5 3 + 1 log x 3 < 0
2 log 5 3.log3 x 2 log 5 3 + 1 log x 3 < 0
t t = log3 x ( t 0 ) khi bt phng trnh (1) tr thnh

1
2log 5 3 < 0
t
( 2log 5 3) t 2 + (1 2 log 5 3) t 1

( 2log 5 3) t + 1

( t 1) ( t + log 3

<0

) <0

1
1
1

t < log 3
log 3 x < log 3
0< x<

5
5
5

1 < x < 3
0 < t < 1
0 < log 3 x < 1

1
So vi iu kin th tp hp nghim ca bt phng trnh cho l (1;3) 0;
.
5

18)

x2 4
< 0(1)
log 1 x 2 1

x2 4 < 0
2 < x < 2

2
<
x
<
2
2 < x < 1

2
<
x
<
2
log 1 x 2 1 > 0

2
2
0 < x 1 < 1
x < 1 x > 1
1< x < 2
(1)



2
x < 2 x > 2
x 4 > 0

x < 2 x > 2
x > 2

2
log x 2 1 < 0
x 1 > 1

x < 2
1

x
<

x
>
2

2
284

Vy, tp hp nghim ca bt phng trnh cho l

( ; 2 ) (

) (

2; 1 1; 2 ( 2; + ) .
3

19) log x 2 x log x ( 2 x ) (1)

x > 0
iu kin:
x 1
t t = log x 2 x

Bt phng trnh (1) tr thnh

t < 0
t < 0
t < 0

3t 0
t 0
t 0

t 3t

0t 3
t 0
t 0
t 0

t 2 3t
t 2 3t 0
0 t 3
x > 1

x > 1
2 x 1

x 2
3
x3 2 x 0

1 2 x x

0 log x 2 x 3

0 < x < 1

0 < x <1
0 < x < 1

1 2 x x 3
2 x 1
x3 2 x 0

1
So vi iu kin th tp hp nghim ca bt phng trnh cho l 0; 2; + .
2

20) log 2 x + log 3 ( x + 1) < 2(1)

iu kin: x > 0
t t = log 2 x x = 2t .

Bt phng trnh (1) tr thnh


t + log 3 ( 2t + 1) < 2 log 3 ( 2t + 1) < 2 t 2t + 1 < 32t 2t + 1 <

9
6t + 3t < 9(2)
t
3

Xt hm s f ( t ) = 6t + 3t
Ta thy f ( t ) = 6t ln 6 + 3t ln 3 > 0, t
f ( t ) l hm ng bin trn .

Mt khc f (1) = 9
Suy ra (2) t < 1 log 2 x < 1 x < 2 .
285

So vi iu kin th tp hp nghim ca bt phng trnh cho l ( 0; 2 ) .


21) log 2 x + 1 + log3 x + 9 > 1(1)

iu kin: x > 1

1
1
log 2 ( x + 1) + log 3 ( x + 9 ) > 1
2
2
log 2 ( x + 1) + log 3 ( x + 9 ) > 2

(1)

t t = log 2 ( x + 1) x = 2t 1

Bt phng trnh (1) tr thnh

t + log 3 2t + 8 > 2 log 3 2t + 8 > 2 t 2t + 8 > 32t


2t + 8 >

9
6t + 8.3t > 9(2)
3t

Xt hm s f ( t ) = 6t + 8.3t
Ta thy f ( t ) = 6t ln 6 + 8.3t ln 3 > 0, t .
f ( t ) l hm ng bin trn .

Mt khc f ( 0 ) = 9.
Suy ra (2) t > 0 log 2 ( x + 1) > 0 x + 1 > 1 x > 0.
So vi iu kin th tp hp nghim ca bt phng trnh cho l ( 0; + ) .
log 4 x log 2 y = 0
V. 8. 1) 2
(I )
2
x 2 y = 8

iu kin: x > 0, y > 0.


1
1

log 2 x log 2 y = 0
log 2 x 2 log 2 y = 0
log x = log 2 y

2 2 2
(I ) 2
2
x 2 y = 8
2
x 2 2 y 2 = 8
x 2 y = 8

x = y
x = 4
x = y
x = y
2

2
2
y = 2
x 2 y = 8 x 2 x 8 = 0 x = 4 x = 2
Vy, nghim ca h phng trnh cho l ( 4; 2 ) .
1
2
2
x + y = 2 y + 4
2)
(I )
log 3 ( x + 2 y ) + log 1 ( x 2 y ) = 1
3

286

x + 2 y > 0
iu kin:
x 2 y > 0
1
2
2
x + y = y + 4
2
(I )
log 3 ( x + 2 y ) log 3 ( x 2 y ) = 1

1
2
1
2
2
x
+
y
=
y+4
x + y2 = y + 4
1

2
2

2
2

16 y + y = y + 4

2
log 3 x + 2 y = 1 x + 2 y = 3
x = 4 y
x 2 y
x 2y

x = 2

1
y = 1
y=

2
2

17

4
=
0
y
y

2
y =
x = 32
x = 4 y

17

17

8
x = 4 y
y =
17

x + 2 y > 0
Do iu kin
, nn ta chn
x 2 y > 0

x = 2

1
y = 2

1
Vy, nghim ca h phng trnh cho l 2; .
2
3x 2 y = 972
3)
log 3 ( x y ) = 2
3x 2 y = 972
3x 2 y = 972
3x 2 y = 972
3 y +3 2 y = 972


x y = 3
x = 3 + y
x = y + 3
log 3 ( x y ) = 2
27 6 y = 972
6 y = 36


x = y + 3
x = y + 3

x = 5

y = 2

Vy, nghim ca h phng trnh cho l ( 5; 2 ) .

2 x 8 y = 2 2

4)
(I )
1 1 1
log 9 + = log 3 ( 9 y )
x 2 2

x > 0
iu kin:
y > 0
287

2 x 8 y = 2 2
2 x 8 y = 2 2

(I )

1 1 1
1
log9 + = log 3 ( 9 y )
log 3 + 1 = log 3 ( 9 y )
x 2 2
x

3
=
x
y
x
3 y
x

3
y

=2 2
x 3 y = 2 2

2 2 = 2 2
2
2

1
9 xy = 3
log 3 ( 9 xy ) = 1
y =
3 xy = 1

3x
1 3

2 x 2 3 x 2 = 0
x x = 2

1
1
y
=
y =

3x

3x

x > 0
nn ta chn
Do iu kin
y > 0

1
1

x
=
2
x
=

x
=

x
=

2
2
1

1
1
y =
y = 6 y =

6
6

x = 2

1
y = 6 .

1
Vy, nghim ca h phng trnh cho l 2; .
6
log x log 2 y = 0
5) 42
(I )
2
x 5y + 4 = 0
x > 0
iu kin:
y > 0
x = 1
x=y

x = y
log 4 x = log 2 y

y = 1 (Tha iu kin)

(I ) 2

x
=
1

2
2
2
x = 4
x 5 y + 4 = 0
x 5 y + 4 = 0
x = 4

y = 2
Vy, nghim ca h phng trnh cho l (1;1) , ( 4; 2 ) .

2log x 3 y = 15
(I )
6) y 2
y +1
3 .log 2 x = 2log 2 x + 3
iu kin: x > 0

15 + 3 y
log
x
=
2

(I )
y
y
3 y 15 + 3 = 2 15 + 3

2
2

288

y +1
+3

15 + 3 y
log 2 x =

2
y
15.3 + 32 y = 30 + 2.3 y + 6.3 y


15 + 3 y
log
=
x

15 + 3
2

15 + 3 y
2
x = 29
log 2 x =

log 2 x =

(Nhn)
2
2

y =1
32 y + 7.3 y 30 = 0
3 = 3
y =1

3 y = 10

Vy, nghim ca h phng trnh cho l 29 ;1 .

log 2 x + log 4 x = 2log 1 4

2
7)
(I )
log 4 x + log 2 y = 5
x > 0
iu kin:
y > 0
log x + log 2 x = log 2 16
log x x = log 2 16
2
(I ) 2
log 2 x + log 2 y = 5
log 2 x + log 2 y = 5
3
3
x x = 16
x 2 = 16
x = 4 4

3
log 2 x + log 2 y = 5 log 2 x + log 2 y = 5 log 2 2 2 + log 2 y = 5
x = 4 3 4
3
x = 4 3 4
x = 4 4

32
3
5
3
3
log 2 (2 2 y ) = 5
2 2 y = 2
y = 3 = 8 4
2 2

(Tha iu kin)

Vy, nghim ca h phng trnh cho l 4 3 4;8 3 4 .


log x ( xy ) = log y x 2
8) 2log x
(I )
y
y
=
4
y
+
3

xy > 0
x > 0, x 1

iu kin: x > 0, x 1
y > 0, y 1 y > 0, y 1

log x x + log x y = 2 log y x


( I ) 2log x
y y = 4 y + 3
2

1 + log x y = log y

x
log y x2
= 4y + 3
y

289

y = x
y = x
2
log x y = 1
2

x 4 y 3 = 0

log x y + log x y 2 = 0
1

2
log x y = 2 y = 2

1
x
y = 2
x = 4 y + 3
2

x 4 y 3 = 0
x 2 4 y 3 = 0
x2 4 y 3 = 0

y = x > 0
x = 2 + 7
y = x > 0
2

3
=
0
x
x

2
7
x
=
+

y = 2 + 7

y=

x=2
1

y = 2
x

1
2 4
y =
x = 2, ( x > 0)
x 2 3 = 0

x

1
Vy, nghim ca h phng trnh cho l (2 + 7; 2 + 7 ), (2; ).
4
1

log 1 ( y x ) log 4 y = 1
9) 4
(I )
x 2 + y 2 = 25

y > 0
iu kin:
y > x
log 4 ( y x ) log 4 y 1 = 1 log 4 y log 4 ( y x ) = 1
I

2
( ) 2 2
2
x
+
y
=
25
x + y = 25

3y
3y

y
x=
x=

=4
y = 4 y 4x

4
4
yx
2
2

2
2
x + y = 25
2
2
9 y + y 2 = 25
25 y = 25
x + y = 25
16
16

x = 3
3y

x=
3y

4
x =

y = 4

4
x = 3
y=4
y 2 = 16

y = 4
y = 4
y > 0
Do iu kin
, nn ta chn
y > x

x = 3

y = 4

Vy , nghim ca h phng trnh cho l ( 3; 4 ) .

log 2 x 2 + y 2 = 5
10 )
(I )
2 log 4 x + log 2 y = 4

290

x > 0
iu kin:
(* )
y > 0
2
2
x 2 + y 2 = 32
x 2 + y 2 = 32
x + y = 32

(I )
log 2 x + log 2 y = 4
log 2 xy = 4
2log 22 x + log 2 y = 4

16 2
x + y = 32
+ y 2 = 32
2
2
x + y = 32
y


16
xy = 16
x = y
x = 16

2
y 4 32 y 2 + 256 = 0 y 2 16 = 0 y 2 = 16

16
16
16
x = y
x =
x = y

y = 4

x = 4 x = 4
y = 4

y = 4 y = 4.
x = 16

y
x = 4
Kt hp vi (*) ta c
y = 4
Vy, nghim ca h phng trnh cho l ( 4; 4 ) .
log y xy = log x y (1)
11) x
y
( 2)
2 + 2 = 3

0 < x 1
iu kin:
0 < y 1
1

(1) log y ( xy ) 2 = log x y

1
1
log y x + log y y ) =
(
2
log y x

(1 + log y x ) log y x = 2
log 2y x + log y x 2 = 0
log y x = 1
x = y

2
log y x = 2
x = y

+ Vi x = y thay vo ( 2 ) ta c

291

2x + 2x = 3 2 x =

3
3
x = log 2 > 0 (Nhn)
2
2
1
y2

+ Vi x = y thay vo ( 2 ) ta c 2 + 2 y = 3.(*) Ta chng minh phng trnh (*) v


nghim.
1

2
1
Tht vy, nu 0 < y < 1 th 2 > 1 nn 2 y + 2 y > 2 + 1 = 3 . Suy ra (*) v nghim. Cn nu
y

y > 1 th

2
1
< 1 nn 2 y + 2 y > 1 + 2 = 3. Suy ra (*) v nghim.
2
y

Vy, nghim ca h phng trnh cho l log 2 ; log 2


2

3
.
2

log x x 3 + 2 x 2 3x 5 y = 3

12)
(I )
3
2
log
y
+
2
y

3
y

5
x
=
3
y

(
(

)
)

0 < x 1
0 < y 1

iu kin: 3
2
x + 2 x 3x 5 y > 0
y3 + 2 y2 3 y 5x > 0

log x x 3 + 2 x 2 3 x 5 y = log x x3
( I )
3
2
3
log y y + 2 y 3 y 5 x = log y y

(
(

)
)

2
3
2
3
x + 2 x 3 x 5 y = x
2 x 3 x 5 y = 0 (1)
3
2
2
3
2 y 3 y 5 x = 0 ( 2 )
y + 2 y 3 y 5 x = y

Tr (1) cho (2) theo v ta c

x = y
2 x 2 3x 5 y 2 y 2 3 y 5 x = 0 ( x y )( x + y + 1) = 0
x = y 1

+ Thay x = y vo (1) ta c

x = 0 y = 0
2 x 2 3x 5 x = 0 2 x ( x 4 ) = 0

x = 4 y = 4
So vi iu kin ta nhn x = y = 4.
+ Thay x = y 1 vo ( 2 ) ta c
2 y 2 3 y 5 ( y 1) = 0 2 y 2 + 2 y + 5 = 0 phng trnh v nghim.

Vy, nghim ca h phng trnh cho l ( 4; 4 ) .


292

Ch . C th nhn xt ngay trng hp x + y + 1 = 0, h phng trnh cho v nghim v


x > 0, y > 0.
x 4 y + 3 = 0
(1)
13)
log 4 x log 2 y = 0 ( 2 )
x > 0
y > 0
x 1

iu kin:

y 1
log 4 x 0
log 2 y 0

( 2)

log 4 x = log 2 y

log 4 x = log 2 y log 2 x = 2log 2 y log 2 x = log 2 y 2 x = y 2


Do y 1 nn (1) tr thnh x 4 y + 3 = 0 ( 3)
Thay x = y 2 vo (3) ta c

y =1
y2 4 y + 3 = 0
y = 3.
+ Vi y = 1 x = 1 . (Nhn)
+ Vi y = 3 x = 9. (Nhn)
Vy, nghim ca h phng trnh cho l (1;1) , ( 9;3) .
7

log 4 x log x y = (1)


6
14)
xy = 16
(2)

x > 0

iu kin: x 1
y > 0

( 2) y =

16
x

Thay vo (1) ta c
log 4 x log x

16 7
=
x 6

1
7
log 2 x log x 16 + 1 =
2
6

1
1
log 2 x log x 2 4 = 0
2
6

293

1
1
log 2 x 4 log x 2 = 0
2
6

1
4
1
log 2 x
= 0 (3) (Do x 1 )
2
log 2 x 6

t t = log 2 x

Khi (3) tr thnh

x = 8
log 2 x = 3
t = 3
1
4 1
2

t = 0 3t t 24 = 0
1
8
8

x= 3
2
t 6
t=
log 2 x =

3
4 4

+ Vi x = 8 y = 2
+ Vi x =

1
3

4 4

y = 64 3 4

Vy, nghim ca h phng trnh cho l ( 8; 2 ) ; 3 ;64 3 4 .


4 4

log x + 2 log 2 y = 3
15 ) 2 2 4
(1)
x + y = 16
x > 0
iu kin:
y > 0
log 2 x + log 2 y 2 = 3
(I )
2
2
2
x + y 2 xy = 16
2
log 2 xy = 3

2
2
2
x + y 2 xy = 16
xy 2 = 8

2
2
x + y = 32

xy 2 = 8

(Do x > 0, y > 0 )


2
x + y = 4 2
Khi x; y 2 l cc nghim ca phng trnh

x = 2 2
x = 2 2
X 2 4 2X + 8 = 0 X = 2 2

(Do x > 0, y > 0 )


2
4
y = 2 2
y = 8

Vy, nghim ca h phng trnh cho l 2 2; 4 8 .


294

x 1 + 2 y = 1
16)
(I )
2
3
3log 9 9 x log 3 y = 3

( )

x 1 0, x 0
x 1

iu kin: 2 y 0

0 < y 2
y > 0

x 1 + 2 y = 1 x 1 + 2 y = 1
x 1 + 2 y = 1

3x
(I )
3x
=1
3log 3 3 x 3log3 y = 3 log 3
y =3
y

x = y
x = y
x = y

x 1 + 2 y = 1 x 1 + 2 x = 1 ( x 1)( 2 x ) = 0
x = 1
x = y

y =1
x 1 = 0
x = 2
2 x = 0

y = 2
So vi iu kin th cc gi tr ca x, y u tha.
Vy, nghim ca h phng trnh cho l (1;1) ; ( 2; 2 ) .

ln (1 + x ) ln (1 + y ) = x y (1)
17) 2
2
( 2)
x 12 xy + 20 y = 0
x > 1
iu kin:
y > 1
(1) ln (1 + x ) x = ln (1 + y ) y

Xt hm f ( t ) = ln (1 + t ) t , ( t > 1)
f (t ) =

1
t
1 =
f ( t ) = 0 t = 0; f ( t ) < 0, t ( 0; + ) ; f ( t ) > 0, t ( 1; 0 )
1+ t
1+ t

Suy ra, hm f ( t ) ng bin trn ( 1, 0 ) v nghch bin trn ( 0; + ) . Do , t (1) ta c


x = y hoc xy < 0. Nhng ta thy nu xy < 0 th phng trnh (2) v nghim, nn x = y.

Thay x = y vo ( 2 ) ta c 9 x 2 = 0 x = 0 y = 0
Vy, nghim ca h phng trnh cho l ( 0; 0 ) .
x log8 y + y log8 x = 4
18)
(I )
log 4 x log 4 y = 1

295

x > 0
iu kin:
. Nhn xt rng y = 1 khng tha h phng trnh, do y 1. Khi
y > 0
h phng trnh ( I ) tng ng vi
x log8 y + y log8 x = 4
x log8 y + y log8 x = 4
log8 y
log 4 y
+ y 8( ) = 4

( 4 y )

x
x

x = 4 y
log 4 y = 1
y =4

log ( 4 y )
log ( 4 y )
log ( 4 y )
=4
=2
y 8 + y 8
y 8
log 8 ( 4 y ) = log y 2

x = 4 y
x = 4 y
x = 4 y
1
1

2 1
2
log8 4 + log 8 y =
+ log 2 y =
( log 2 y ) + 2 log 2 y 3 = 0
log 2 y 3 3
log 2 y

x = 4 y
x = 4 y
x = 4 y

log 2 y = 1
=
x
x = 8

2
log 2 y = 3

=
2
y

y = 1
x
=
4
y

1 1
i chiu vi iu kin th nghim ca h phng trnh cho l ( 8; 2 ) , ; .
2 8
log 2 x + 3 = 1 + log 3 y (1)
19)
log 2 y + 3 = 1 + log 3 x (2)

x > 0
iu kin:
y > 0
Ly (1) tr cho (2) theo v ta c
log 2 x + 3 log 2

y + 3 = log 3 y log 3 x

log 2 x + 3 + log 3 x = log 2

y + 3 + log 3 y (3)

Xt hm s c trng
f ( t ) = log 2 t + 3 + log 3 t , t > 0.
f (t ) =

1
1
+
> 0 ( t > 0 )
2 ( t + 3) ln 2 t ln 3

Suy ra f ( t ) ng bin trn ( 0; + ) .


Do (3) x = y. Thay x = y vo (1) ta c
log 2 x + 3 = 1 + log 3 x log 2 x + 3 log 3 x = 1(4)

Xt hm s
296

y = g ( x ) = log 2 x + 3 log 3 x g ( x ) =
=

x ( ln 3 ln 4 ) 6 ln 2
2 x ( x + 3) ln 2 ln 3

x ln 3 x ln 4 6 ln 2
2 x ( x + 3) ln 2 ln 3

< 0, x > 0.

Suy ra hm s y = g ( x ) l hm nghch bin trn khong (0; +), nn (4) c nhiu nht
mt nghim.Ta kim tra c x = 1 l nghim ca (4).
Vy, nghim ca h phng trnh cho l (1;1).
3lg x = 4lg y
(1)
20)
lg 4
lg3
( 4 x ) = ( 3 y ) (2)

x > 0
iu kin:
y > 0

( )

(1) lg 3lg x = lg 4lg y lg x = ( lg y )


(2) lg ( 4 x )

lg 4

= lg ( 3 y )

Thay lg x = ( lg y )

lg3

lg 4
lg 3

( lg 4 )( lg 4 + lg x ) = ( lg 3)( lg 3 + lg y )

lg 4
vo (2) ta c
lg 3

lg 4
lg 4 lg 4 + ( lg y )
= ( lg 3)( lg 3 + lg y )
lg 3

lg 2 4
lg 2 4 + ( lg y )
= lg 2 3 + ( lg y ) lg 3
lg 3
lg 2 4

( lg y )
lg 3 = lg 2 3 lg 2 4
lg 3

( lg
( lg y )

4 lg 2 3
lg 3

) = lg

3 lg 2 4

1
1
lg y == lg 3 lg y = lg y = .
3
3

1 lg 4
1
lg x = lg .
lg x = lg 4 x = .
3 lg 3
4

1 1
Vy, nghim ca h phng trnh cho l ; .
4 3
x + log 3 y = 3
21)
2
x
( 2 y y + 12 ) .3 = 81y.

iu kin: y > 0.

297

Ly logarit c s 3 hai v ca phng trnh th hai ca h ta c


x + log 3 y = 3
x = 3 log3 y

2
2
log 3 2 y y + 12 + x = log 3 y + 4
log 3 2 y y + 12 + 3 log 3 y = log 3 y + 4
x = 3 log 3 y
x = 3 log 3 y
x = 3 log 3 y

2
2

2 y y + 12
2 y y + 12
2
2
log
=
1
=3
log
2
y

y
+
12

log
y
=
1
3
3
3

y2
y2

x = 3 log 3 y
x = 3 log 3 y
x = 3 log 3 y
x = 2
2

2
2
y = 3
y = 3 y = 4
2 y y + 12 = 3 y
y + y 12 = 0

So vi iu kin th h phng trnh cho c mt nghim l ( 2;3) .

log 2 x 2 + y 2 = 1 + log 2 ( xy )
(I )
22) 2
2
3x xy + y = 81

iu kin: xy > 0.
2
2
log 2 x 2 + y 2 = log 2 ( 2 xy )
x + y = 2 xy
(I )
2
2
2
2
x xy + y = 4
x xy + y = 4

( x y )2 = 0 x = y
x = 2 x = 2

xy
y
=
4
=
2
xy
=
4

y = 2

i chiu vi vi iu kin th h phng trnh cho c hai nghim l ( 2; 2 ) , ( 2; 2 ) .

x 1 1 y
4 =

2 (I )
23)
3log9 x = y

3
iu kin: x > 0.

Vit li h phng trnh cho di dng


22 2 = 2 y 1
2 x = y 1
2 x = y 1 2 x = y 1
1

log3 x y log x 1 y 1 y
y
3
2
2
2
=
=
3
3
x =
x=
3

3
3
3

y
2. 3 = y 1 y = 3

x = 1.
x= y

So vi iu kin th h phng trnh cho c mt nghim l (1;3) .

298


3 4 x
(1)
x + 1 1 3y =
24)
x
y + log x = 1
(2)

x +1 0
x 1

iu kin: 4 x 0 x 4 0 < x 4.
x > 0
x > 0

T phng trnh (2) ta c y = 1 log3 x 3 y = 31log3 x =

3
log3 x

3
= .
x

3
vo phng trnh (1) ta c
x
3 3 4x
x +1 1 . =
x + 1 1 = 4 x
x
x

Thay 3 y =

x +1 = 4 x +1 x +1 = 4 x + 2 4 x +1
x 2
4x = x2
2 x = 3 y = 0.
4 x = ( x 2 )

So vi iu kin th h phng trnh cho c mt nghim l ( 3; 0 ) .

log 4 ( x 2 + y 2 ) log 4 ( 2 x ) + 1 = log 4 ( x + 3 y )

25)
x (I )
2
log
xy
+
1

log
4
y
+
2
y

2
x
+
4
=
log
1
(
)
(
)
4
4
4
y

x
y >0

x > 0
2 x > 0

iu kin: x + 3 y > 0
y > 0
xy + 1 > 0
4 y 2 + 2 y 2 x + 4 > 0

2
4 y + 2 y 2 x + 4 > 0

H phng trnh ( I ) c bin i v

2 ( x2 + y2 )
x = y

= x + 3y
2
2

x 3 xy + 2 y = 0
( x y )( x 2 y ) = 0

x
2

x = 2

xy + 1
x

x xy + 2 y 2 x = 0
( x y )( x 2 ) = 0
y = 1
=
2
4 y + 2 y 2 x + 4 4 y
i chiu vi iu kin th nghim ca h phng trnh cho l ( 2;1) , ( a; a ) , vi
a , a > 0.
V. 9. log 32 x + log32 x + 1 2m 1 = 0 (1)

299

a) Khi m = 2 th phng trnh tr thnh log 32 x + log 32 x + 1 5 = 0 ( 2 )


iu kin: x > 0
t t = log 32 x + 1 > 0. Khi ta c phng trnh theo bin t

t=2
t2 + t 6 = 0
t = 3
x=3 3
log 3 x = 3
Ta chn t = 2 log + 1 = 2 log x + 1 = 4 log x = 3

x = 3 3
log3 = 3
2
3

2
3

2
3

Cc gi tr ca x u tha iu kin.
Vy, nghim ca phng trnh cho l x = 3 3 ; x = 3 3.
b) Ta c
1 x 3

0 log 3 x 3 0 log 32 x 3

1 log 23 x + 1 4 1 log 32 x + 1 2.

Nh vy, t [1; 2] .
Phng trnh (1) tr thnh t 2 + t 2 = 2m
t f ( t ) = t 2 + t 2
f ( t ) = 2t + 1 f ( t ) = 0 t =

1
2

Bng bin thin

Da vo bng bin thin, ta suy ra phng trnh cho c nghim x 1;3 3 khi v ch

khi 0 2m 4 0 m 2.
Vy, vi 0 m 2 th phng trnh cho c nghim x 1;3 3 .

V.10. 4(log 2 x ) 2 log 1 x + m = 0


2

Ta c 4(log 2 x ) 2 log 1 x + m = 0 4(log 2 x ) 2 + log 2 x + m = 0


2

(log 2 x) 2 + log 2 x + m = 0(1)

300

t t = log 2 x, ta c 0 < x < 1 t (;0)

(1) tr thnh t 2 + t + m = 0 t 2 + t = m(2)


Bi ton tr thnh: Tm m phng trnh t 2 + t = m(2) c nghim thuc khong
(; 0) . t f (t ) = t 2 + t f (t ) = 2t + 1 = 0 t =

1
2

Bng bin thin

Da vo bng bin thin, suy ra (2) c nghim thuc khong (; 0) khi v ch khi
1
1
m m .
4
4

Vy, vi m
V.11. 25

1 t 2

1
th phng trnh cho c nghim thuc khong (0;1).
4

(a + 2) 5

1t 2

+ 2a + 1 = 0 (1)

iu kin: 1 t 1.
t X = 5

1 t 2

X [1;5]

Phng trnh (1) tr thnh


2

X ( a + 2)

( X 1)
X + 2a + 1 = 0 a =
X 2

a=

X 2 2X +1
X 2

(V X = 2 khng l nghim ca phng trnh X 2 ( a + 2 ) X + 2a + 1 = 0 )


X 2 2 X +1
Xt hm s f ( X ) =
X 2

f ( X ) =

X 2 4X + 3

( X 2)

X =1
f ( X ) = 0
X = 3

Ta tnh c f (1) = 0; f ( 3) = 4; f ( 5 ) =

16
.
3

Bng bin thin


301

a 0
Da vo bng bin thin, suy ra phng trnh cho c nghim khi v ch khi
a 4.
Vy, vi a (;0] [4; +) th phng trnh cho tha mn yu cu bi ton.
V.12. 2 log 32 x log 3 x + a = 0
iu kin: x > 0
2

Phng trnh cho c vit li thnh 2 log 3 x log 3 x + a = 0(1)


t t = log 3 x 0

Khi (1) tr thnh f (t ) = 2t 2 t + a = 0(2)


Ta thy (1) c bn nghim phn bit khi v ch khi (2) c hai nghim dng phn bit

> 0
1 8a > 0

1
f (0) > 0 a > 0
0<a<
8
S
1
>0
>0
2
4
Vy, vi 0 < a <

1
th phng trnh cho tha mn yu cu bi ton.
8

e x e y = ln(1 + x ) ln(1 + y )
V.13.
(I )
y x = a

iu kin: x, y > 1
e x + a e x + ln(1 + x) ln(1 + a + x) = 0(1)
(I )
(2)
y = x + a

Vi a > 0 th h phng trnh ( I ) c nghim duy nht khi v ch khi (1) c nghim duy
nht trong khong ( 1; + )
Xt hm s f ( x) = e x + a e x + ln(1 + x ) ln(1 + a + x ) vi x > 1.
Do f ( x ) lin tc trong ( 1; + ) v c o hm f ( x) == e x (e a 1) +
e a 1 > 0
a > 0
Do

f ( x ) > 0, x ( 1; + )
x > 1 (1 + x)(1 + x + a ) > 0

302

a
(1 + x)(1 + x + a )

Suy ra f ( x ) ng bin trn khong ( 1; + ) . Ta li c lim+ f ( x) = , lim f ( x) = +


x 1

x +

Nn phng trnh f ( x ) = 0 c nghim duy nht trong khong ( 1; + ) .


Vy, h phng trnh cho c nghim duy nht vi m i a > 0 .
2x
2y
2 + 4 = m
(I )
V.14. x
y
x+ 2 y
=m
2 + 4 + 2

u = 2 x
t
(u , v > 0)
y
v = 4
H phng trnh ( I ) tr thnh
u + v = 1 + 1 + 3m

u 2 + v 2 = m
(u + v) 2 2u.v = m

u + v = 1 1 + 3m

u + v + u.v = m
u + v + u.v = m

u + v + uv = m

u + v = 1 + 3m 1

(V u , v > 0 )
u.v = m + 1 1 + 3m
Suy ra u, v l nghim ca phng trnh bc hai
X 2 ( 1 + 3m 1) X + m + 1 1 + 3m = 0 (*)

H ( I ) c nghim khi v ch khi phng trnh (*) c hai nghim dng

= ( 1 + 3m 1)2 4(m + 1 1 + 3m ) 0

S = 1 + 3m 1 > 0

P = m + 1 1 + 3m > 0
1 + 3m 2 1 + 3m + 1 4m 4 + 4 1 + 3m 0

1 + 3m > 1

1 + 3m < m + 1
2 1 + 3m m + 2
0 m 8

m > 0
m > 0
1 < m 8.
m > 1
m > 1

Vy, gi tr m cn tm l 1 < m 8.
2 x +1 = y y + 1 + m + 1
V.15.
(I )
2 x +2
2 x +1 + m
y + 1 = 2

303

u = 2 x +1 2
t
. Khi , ( I ) tr thnh
v = y + 1 0

u = v 2 v + m (1)

2
v = u u + m ( 2 )

v u = 0
v = u
Tr (1) v (2) theo v ta c v 2 u 2 = 0 ( v u )( v + u ) = 0

v + u = 0
v = u
u = 2 x +1 2
nn ta chn v = u. Th v = u vo (1) ta c u 2 2u + m = 0 (3)
V
v = y + 1 0

a) Khi m = 0 ta c (3) tr thnh

v = 0 u = 0
v 2 2v = 0
v = 2 u = 2
2 x +1 = 2
x = 0
x + 1 = 1 x = 0
Ta cng ch nhn v = 2, u = 2

y = 3
y + 1 = 2
y + 1 = 4
y = 3

Vy, nghim ca h phng trnh ( I ) trong trng hp m = 0 l ( 0;3) .


b) Trng hp tng qut th h phng trnh ( I ) c nghim khi v ch khi (3) c nghim
u = 2 x +1 2
u 2, (Khi v v = u nn v 2 v do iu kin
c tha).
v = y + 1 0

Ta c u 2 2u + m = 0 u 2 + 2u = m .
Xt hm s f ( u ) = u 2 + 2u, u 2 f ( u ) = 2u + 2 = 0 u = 1.
Bng bin thin

Da vo bng bin thin suy ra h phng trnh cho c nghim khi v ch khi m 0 .
Vy, vi m 0 th h phng trnh cho c nghim.
c) Da vo bng bin thin cu b) ta thy rng hm s f ( u ) nghch bin trn [2; +)
nn d dng suy ra c phng trnh u 2 2u + m = 0 lun c nghim duy nht u 2 vi
m i m 0 . Vy vi m 0 th h phng trnh cho c nghim duy nht.
x
y
(1)
2 2 = y x
V.16. 2
2
2 x 4mx y = 3m(2)

Ta c (1) 2 x + x = 2 y + y . Xt hm s c trng f (t ) = 2t + t ( t )
Ta c f (t ) = 2t ln 2 + 1 > 0, t . Suy ra, hm s f (t ) ng bin trn .
304

V vy 2 x + x = 2 y + y x = y.
Vi x = y th h phng trnh cho tr thnh
x = y
x = y
x = y

2
2
2
2
2 x 4mx y = 3m
2 x 4mx x = 3m
x 4mx 3m = 0(3)

x = 1
x = y

x = y

y = 1
a) Khi m = 1 ta c h 2
x = 1

x + 4x + 3 = 0
x = 3 x = 3

y = 3
Vy, nghim ca h phng trnh cho khi m = 1 l (1; 1); (3; 3).
b) H phng trnh cho c ng hai nghim khi v ch khi phng trnh (3) c hai
nghim phn bit. iu ny c tha khi v ch khi
3
= 4m 2 + 3m > 0 m > 0 m < .
4

Vy, vi m ; ( 0; + ) th h phng trnh cho c ng hai nghim.


4

3x + x = 3m + y
V.17. y
(I )
3
+
y
=
3
m
+
x

a) Thay m = 1 vo ( I ) ta c h
x
3 x + x = 3 + y
3 x + x = 3 + y
(1)
3 + x = 3 + y

y
x
x
y
y
3 + 2 x = 3 + 2 y ( 2 )
3 + y = 3 + x
3 3 + x y = y x

Hm s f ( t ) = 3t + 2t l hm ng bin trn . Nn phng trnh (2) x = y.


Thay x = y vo (1) ta c 3x + x = 3 + x 3x = 3 x = 1.
Vy, nghim ca h phng trnh cho trong trng hp m = 1 l (1;1) .
x
3x + x = 3m + y
3 + x = 3m + y (1)
b) Trong trng hp tng qut ta cng c y
x
y
3 + 2 x = 3 + 2 y ( 2 )
3 + y = 3m + x

Hm s f ( t ) = 3t + 2t l hm ng bin trn . Nn phng trnh ( 2 ) x = y.


Thay x = y vo (1) ta c 3x + x = 3m + x 3x = 3m(3)
Do 3x > 0, x nn h phng trnh cho v nghim khi v ch khi phng trnh (3)
v nghim. iu ny c tha khi v ch khi 3m 0 m 0.
Vy, vi m 0 th h phng trnh cho v nghim.

305

x 2 + y 2 = 17
(1)
V.18.
(I )
log 2 x + log 2 y = m ( 2 )
iu kin: x > 0, y > 0.

Gi s h ( I ) c nghim ( x, y ) th ( y , x ) cng l nghim ca h ( I )


Nn h phng trnh ( I ) c nghim duy nht th iu kin cn l ( x, y ) = ( y, x ) x = y.

17
x =
2
Thay x = y vo (1) ta c 2 x 2 = 17

17
x =
2

Do iu kin x > 0, y > 0 nn ta chn x =


17
vo ( 2 ) ta c log 2
2

Thay x = y =

Kim tra li ta thy khi m = log 2


Vy, vi m = log 2

17
.
2
17
17
17
+ log 2
= m m = log 2 .
2
2
2

17
th h c nghim duy nht.
2

17
th h phng trnh cho c nghim duy nht.
2

Ch . C th gii bng cch khc nh sau:


Vi iu kin x > 0, y > 0, vit li h phng trnh cho di dng
2
2
2
m
x 2 + y 2 = 17
x + y = 17
( x + y ) 2 xy = 17
x + y = 17 + 2

m
m
m
xy = 2
log 2 ( xy ) = m
xy = 2
xy = 2

Khi x, y l nghim ca phng trnh bc hai


t2

17 + 2m t + 2m = 0(*).

H phng trnh ( I ) c nghim duy nht khi v ch khi (*) c mt nghim kp dng
= 17 + 2m 4.2m = 0
17

b
2m +1 = 17 m + 1 = log 2 17 m = log .
17 + 2m
2
>0
=
2
a
x2 y 4 = 0
(1)

V.19.
x
log 2 y = m log y x(2)

306

x > 0
x = y2

iu kin: y > 0 . Ta c (1) x 2 y 4 = 0


2
x = y
y 1

V x > 0 nn ta ch chn trng hp x = y 2 . Thay x = y 2 vo (2) ta c


log 2

y2
= m log y y 2 log 2 y = 2m y = 2 2m x = y 2 = (22 m )2 = 2 4m
y

4m
x = 2
Nh vy ta c nghim ca h l
2m
y = 2

x = 2 4m > 1
4m > 0
m > 0
Theo yu cu ca bi ta phi c

2m
2m < 2
m < 1
y = 2 < 4
x > 1
Vy, vi 0 < m < 1 th h phng trnh cho c nghim ( x; y ) tha mn
y < 4.
4 x.4 y = 8.2 xy
V.20
(I )
2
2
3 + log 2 x + log 2 y = log 2 ( x + y + m)
22( x + y ) = 23+ xy
2( x + y ) = 3 + xy
2( x + y ) = 3 + xy

2
2
( I ) log 2 (8 xy ) = log 2 ( x 2 + y 2 + m) 8 xy = x + y + m ( x + y )2 10 xy + m = 0
x > 0, y > 0
x > 0, y > 0
x > 0, y > 0

S = x + y 2
t
; S 4 P 0, S > 0, P > 0. Ta c h phng trnh
P = xy
2S = 3 + P
P = 2S 3
P = 2s 3
2

2
2
S 10 P + m = 0
S 10 P + m = 0
f ( S ) = S 20 S + 30 + m = 0(1)
H phng trnh cho c ng bn nghim khi v ch khi phng trnh (1) c hai nghim
S 2 4 P > 0
dng phn bit tha
P > 0

S < 2 S > 6
3
S 2 4 P > 0
S 2 8S + 12 > 0
<S<2

Ta c

2
3

P > 0
2S 3 > 0
S > 2
S > 6
Nh vy, yu cu bi ton c tha khi v ch khi phng trnh (1) c hai nghim phn
bit tha

3
< S < 2 S > 6. Nhn xt rng phng trnh (1) nu c nghim th tng hai
2

nghim bng 20 do yu cu bi ton c tha khi v ch khi


hai nghim S1 , S2 ca (1) tha
307

>
0

m 6 < 0 70 m > 0
f
(2)
<
0
f
3
<
S
<
<
<
S
2
6

1
2
2
f (6) < 0 f (6) > 0 m 54 < 0 m 54 > 0

3
b

10 > 4
9
6 < S1 < S 2
f ( ) > 0
m + > 0
>6
2

4
2a

9
< m < 6 54 < m < 70.
4

9
Vy, vi m ; 6 ( 54; 70 ) th h phng trnh cho tha mn yu cu bi ton.
4
x + 2lg y = 3m

V.21.

2
x 3lg y = 1

x 0
( I ) iu kin:
y > 0

x + 2lg y = 3 x + 2 lg y = 3(1)

1) Thay m = 1 vo ( I ) ta c
2
x 6lg y = 1 (2)
x 3lg y = 1
Nhn hai v ca phng trnh (1) vi 3, ri cng vi phng trnh (2) theo v ta c
x + 3 x = 10 x + 3 x 10 = 0 x = 2 x = 4
Thay x = 4 vo (1) ta c 2 lg y = 1 lg y =

1
y = 10 .
2

Vy, nghim ca h phng trnh cho khi m = 1 l (4; 10).


2) Trong trng hp tng qut, ta tm gi tr ca tham s m h phng trnh cho c
nghim ( x; y ) tha x > 1.

x + 2 lg y = 3m
x + 3 x 9m 1 = 0(3)
Ta c ( I )

(4)
x 6lg y = 1
x 6lg y = 1
t t = x 1 (Do yu cu ca bi l x 1 )

Phng trnh (3) tr thnh f (t ) = t 2 + 3t 9m 1 = 0(5). Ta c nhn xt rng vi m i


x0 th phng trnh (4) lun lun c nghim y0 = 10

x0 1
6

. Do h phng trnh cho

c nghim tha yu cu bi khi v ch khi phng trnh (5) c nghim t 1.

308

Xt f (t ) = t 2 + 3t 9m 1, phng trnh (5) c nghim t 1 (Ch c th c mt nghim


1
tha iu kin ny v t1 + t2 = 3 < 0) khi v ch khi af (1) 0 3 9m 0 m .
3

Vy, vi m

1
th h phng trnh cho c nghim tha mn yu cu bi ton.
3

lg 2 x + lg 2 y = 1
u = lg x

V.22. x
. iu kin: x, y > 0. t
v = lg y
lg y = m.

Khi h phng trnh cho c bin i v dng

u 2 + v 2 = 1 v = u m
v = u m
2
2

2
2
u + ( u m ) = 1 2u 2mu + m 1 = 0(*)
u v = m
H phng trnh cho c nghim duy nht khi v ch khi phng trnh (*) c nghim duy
nht. iu ny c tha khi v ch khi

= m2 2 m2 1 = 0 m = 2 m = 2.
Vy, h phng trnh cho c nghim duy nht khi v ch khi m = 2 m = 2.
2x
2y
2 + 3 = 1
V.23. x
( I ). t
y
2 + 3 = m.

x
u = 2
, u > 0, v > 0.

y
v = 3

Khi h phng trnh ( I ) c bin i v dng


u 2 + v 2 = 1(1)
( II )
. Phng trnh (1) l phng trnh ng trn n v (C ), ta ch ly
u + v = m (2)
nhng im M (u; v) thuc ng trn sao cho u > 0, v > 0. Phng trnh (2) l phng
trnh ng thng (d ) v cng ch xt u > 0, v > 0. H phng trnh ( I ) c nghim duy nht
khi v ch khi trong h ta vung gc Ouv, (d ) tip xc vi (C ) ti im
M (u; v ) (C ), u , v > 0. iu ny c tha khi v ch khi
m
=1
d (O, ( d )) = 1
2
m = 2.

m > 0
m > 0

Nh vy, khi m = 2 th h phng trnh cho c nghim duy nht v th m = 2 vo h

2
u =
( u + v ) 2 2uv = 1 uv = 1
u 2 + v 2 = 1

2
2
( II ) ta c

u + v = 2
u + v = 2
u + v = 2
v = 2

309

x
1

2
x=
2 =

2
Ta c
.

3 y = 2
y = log 2
3

2
2
1
2
Vy, vi m = 2 th h phng trnh cho c nghim duy nht ;log3
.
2
2

2x
2y
2 + 4 = 2
V.24. x
(I )
y
x+2 y
= 1 m.
2 + 4 2

u = 2 x
t
, u > 0, v > 0. Khi h phng trnh ( I ) c bin i v dng
y
v = 4
u 2 + v 2 = 2
(1)
( II )
. Bin i phng trnh (2) v dng
u + v uv = 1 m(2)
2
2
( u + v ) u 2 + v2
(u + v ) 2
u+v
= 1 m u + v
= 1 m (Do u 2 + v 2 = 2)
2
2

2
2

( u + v ) 2 ( u + v ) 2m = 0
.
u + v 1 + 2m + 1 = 0

u + v 1 2m + 1 = 0
Nh vy, h ( II ) tng ng vi

u 2 + v 2 = 2(C )
(1)

u + v 1 + 2m + 1 = 0(d1 )(3) .

u + v 1 2m + 1 = 0(d 2 )(4)
Phng trnh (1) l phng trnh ng trn c tm l gc ta , bn knh 2 , ta ch ly
nhng im M (u; v) thuc ng trn sao cho u > 0, v > 0. Phng trnh (3) v (4) l
phng trnh ng thng v cng ch xt u > 0, v > 0. H phng trnh cho c nghim
khi v ch khi trong h ta vung gc Ouv, (d1 ) hoc (d 2 ) c im chung vi (C ). Cc
ng thng (d1 ) v (d 2 ) u c h s gc bng 1. (d1 ) i qua im A( 2; 0) khi

1 2m + 1 = 2 v tip xc vi (C ) khi 1 2m + 1 = 2. Cng lp lun tng t i vi


(d 2 ) v cu i cng ta c iu kin h phng trnh cho c nghim l

2 < 1 + 2m + 1 2
1 2 < m 0.

2 < 1 2m + 1 2
Vy, gi tr cn tm ca tham s m l 1 2 < m 0.
310

22 x + 2 2 y = 16
V.25. 2 x
(I )
2y
x
y
x + y +1
= 2m 2
2 + 2 + m ( 2 + 2 ) + 2
x
u = 2
t
, u > 0, v > 0. Khi h phng trnh ( I ) c bin i v dng
y
v = 2

u 2 + v 2 = 16(C )
u 2 + v 2 = 16
u 2 + v 2 = 16

u + v m = 0(d1 )

2
2
( u + v m )( u + v + 2m ) = 0
( u + v ) + m ( u + v ) 2m = 0

u + v + 2m = 0(d 2 )
Ta c ng trn (C ) c tm ti gc ta v bn knh bng 4. Cc ng thng (d1 ) v
(d 2 ) u c h s gc bng 1.
H phng trnh cho c ba nghim phn bit khi v ch khi trong h ta vung gc
Ouv, ng trn (C ) giao vi (d1 ) v (d 2 ) ti ba im phn bit c ta dng. iu ny
xy ra khi v ch khi (C ) tip xc vi mt trong hai ng thng v ct ng cn li ti
hai im phn bit.
Ta c (d1 ) i qua im A(4;0) khi m = 4, cn (d1 ) tip xc vi (C ) khi m = 4 2.
Tng t (d 2 ) i qua im A(4;0) khi m = 2, cn (d 2 ) tip xc vi (C ) khi m = 2 2.
D thy rng khng tn ti gi tr no ca tham s m tha mn yu cu ca bi ton.

log 32 x + log 22 y 2 = 4m
V.26.
2
log 3 x + 2 log 2 y = 10
u = log3 x
Cch 1. iu kin: x, y > 0. t
.
v = log 2 y

Khi h phng trnh cho c vit li di dng


2
2
u 2 + 4v 2 = 4m
5u 2 40u + 100 4m = 0(1)
u + 4 ( 5 u ) 4m = 0

v = 5 u
u + v 5 = 0
v = 5 u

H phng trnh cho c nghim duy nht khi v ch khi phng trnh (1) c mt nghim
duy nht. iu ny xy ra khi v ch khi = 20m 100 = 0 m = 5.
Vy, khi m = 5 th h phng trnh cho c nghim duy nht.
u = log3 x
Cch 2. Cng t
, ta c h:
v = log 2 y

u2 v2
u 2 + 4v 2 = 4m
+ = 1( E )

4m m
, (m 0).

u + v 5 = 0
u + v 5 = 0 ( d )

H phng trnh cho c nghim duy nht khi v ch khi (d ) tip xc vi Elip ( E ). Ta c
iu kin l 1.4m + 1.m = 25 5m = 25 m = 5.
Vy, khi m = 5 th h phng trnh cho c nghim duy nht.
311

CHNG VI.
VI.1. 1)

PHNG TRNH LNG GIC

3 sin x cos x = 2

3
1
2
sin x cos x =
2
2
2

sin

sin x cos

(cos

cos x sin

cos x = cos

sin x ) = cos

cos( x + ) = cos
3
4

cos( x + ) = cos
3
4
3

cos( x + ) = cos
3
4
3
x + 3 = 4 + k 2

x + = 3 + k 2

3
4
3

=
+ k 2
x

4 3

x = 3 + k 2

4 3
5

x = 12 + k 2
( k )

x = 13 + k 2

12
Vy, nghim ca phng trnh cho l x =
2) cos x + 2 cos 2 x = 1
4 cos 2 x + cos x 3 = 0

cos x = 1

cos x = 3
4

x = + k 2

3
x = arccos( ) + k 2 , (k )

3
x = arccos( ) + k 2

4
312

5
13
+ k 2 , x =
+ k 2 , ( k ).
12
12

Vy, nghim ca phng trnh cho l


3
3
x = + k 2 , x = arccos( ) + k 2 , x = arccos( ) + k 2 , ( k ).
4
4
3) cos 4 x + 2 cos 2 x = 0
1 + cos 2 x
)=0
2
2 cos 2 2 x + cos 2 x = 0

2 cos 2 2 x 1 + 2(

cos 2 x (2 cos 2 x + 1) = 0

cos 2 x = 0

2cos 2 x + 1 = 0

2 x = 2 + k

cos 2 x = 1 = cos 2

2
3

x = 4 + k 2

2
2x =
+ k 2 , (k ).

2 x = 2 + k 2
3

x = 4 + k 2

x = + k ( k )

x = + k

3
Vy, nghim ca phng trnh cho l x =

+k

,x =

+ k , ( k ).

4) 2cos 2 x + 4cos x = 3sin 2 x


2 cos 2 x + 4 cos x 3(1 cos 2 x) = 0
5 cos 2 x + 4 cos x 3 = 0

2 + 19
cos x =
5

2 19
cos x =
5

cos x =

2 + 19
2 19
(V phng trnh cos x =
v nghim)
5
5
313


2 + 19
) + k 2
x = arccos(
5

(k )

2 + 19
) + k 2
x = arccos(
5

2 + 19
Vy, nghim ca phng trnh cho l x = arccos
+ k 2 , (k ).
5

5) cos x sin x + 3sin 2 x 1 = 0

t t = cos x sin x = 2 cos x + , iu kin: t 2.


4

sin 2 x = 1 t 2 . Phng trnh cho tr thnh


t = 1
3t t 2 = 0
t = 2
3

2
cos(
x
+
) =1

2 cos( x + ) = 2

4
3

cos( x + 4 ) = 2

cos( x + ) =
4
3


x + 4 = 4 + k 2

x + = + k 2
4
4

x + = arccos( 2 ) + k 2

4
3

x = k 2

x = + k 2
, ( k )
2

x = arccos( 2 ) + k 2

3
4
Vy, nghim ca phng trnh cho l
x = k 2 , x =
314

+ k 2 , x = arccos(

2
) + k 2 , (k ).
3
4

6) 2sin 2 x 3 3 ( sin x + cos x ) + 3 3 = 0


4sin x cos x 3 3 ( sin x + cos x ) + 3 3 = 0 (1)

t t = sin x + cos x = 2 sin x + , iu kin: t 2.


4

Khi t 2 = 1 + 2sin x cos x sin x cos x =

t2 1
.
2

Phng trnh (1) tr thnh 2 t 2 1 3 3 t + 3 3 = 0


2t 2 3 3 t + 3 3 2 = 0
t = 1
3 3 2
t =

Chn t = 1 2 sin x + = 1
4

1
2

sin x + =
=
= sin
4
2
4
2


x = k 2
x + 4 = 4 + k 2

,(k ).

x
=
+
k
2
x + = + k 2
2

4
4
Vy, nghim ca phng trnh cho l x = k 2 , x =

+ k 2 , ( k ).

7) sin 2 x + 2 sin x = 1 2sin x cos x + sin x cos x = 1(1)


4

t t = sin x cos x = 2 sin x , iu kin: t 2.


4

Suy ra t 2 = 1 2sin x cos x = 1 s in2x s in2x = 1 t 2


Khi phng trnh (1) tr thnh
1 t2 + t = 1
t2 t = 0
t ( t 1) = 0

t = 0

t = 1
+ Vi t = 0. Ta c
315

2 sin x = 0 sin x = 0 x = k x = + k , (k ).
4
4
4
4

+ Vi t = 1. Ta c

2 sin x = 1 sin x =
4
4
2

x 4 = 4 + k 2
x = + k 2

, ( k ).
2

x = + k 2
x = + k 2

4
4
Vy, nghim ca phng trnh cho l x =
8) sin 2 x + 2sin x cos x 2 cos 2 x =

+ k , x =

+ k 2 , x = + k 2 , ( k ).

1
(1)
2

V cos x = 0 khng l nghim ca (1) nn chia hai v ca phng trnh (1) cho cos 2 x 0
ta nhn c ca phng trnh
tan 2 x + 2 tan x 2 =

1
1 + tan 2 x
2

tan 2 x + 4 tan x 5 = 0

x = + k
tan x = 1

,(k )
4

tan x = 5
x = arctan(5) + k
Vy, nghim ca phng trnh cho l x =
9) cos x + sin x =

cos 2 x
(1)
1 sin 2 x

iu kin: 1 sin 2 x 0 s in2x 1 x

Vi iu kin trn th
(1) cos x + sin x =

cos 2 x
2

( cos x sin x )
( cos x + sin x )( cos x sin x )
cos x + sin x =
2
( cos x sin x )
cos x + sin x =

316

+ k , x = arctan( 5) + k , ( k ).

cos x + sin x
cos x sin x

+ k , ( k )

( cos x + sin x ) 1
=0
cos x sin x
cos x + sin x = 0

cos x sin x = 1

tan x = 1

2 cos x + = 1

x = 4 + k
x = + k

, ( k ) . (Tha iu kin)
x + = + k 2 x = k 2

4 4

x = + k 2
x + = + k 2
2

4
4
Vy, nghim ca phng trnh cho l x =

+ k , x = k 2 , x =

+ k 2 , ( k ).

10) sin 3 x cos 3 x = 1 + sin x cos x


( sin x cos x ) ( sin 2 x + sin x cos x + cos 2 x ) = 1 + sin x cos x
( sin x cos x )(1 + sin x cos x ) = 1 + sin x cos x
(1 + sin x cos x )( sin x cos x 1) = 0


sin 2 x

1 +
2 sin x 1 = 0
2
4

2 sin x 4 1 = 0

sin 2 x
1 + 2 = 0

1
sin x =

4
2

sin 2 x = 2

sin x =
(V phng trnh s in2x = 2 v nghim)
4 2


x 4 = 4 + k 2

x = + k 2

4
4
317

x = + k 2

, ( k )
2

x = + k 2
Vy, nghim ca phng trnh cho l x =

+ k 2 , x = + k 2 , ( k ).

VI.2.1) sin 3x = 2cos 2 x 1

3sin x 4sin 3 x = 1 2sin 2 x


4sin 3 x 2sin 2 x 3sin x + 1 = 0

( sin x 1) 4sin 2 x + 2sin x 1 = 0


sin x = 1

2
4sin x + 2 sin x 1 = 0

sin x = 1

1 + 5
sin x =

sin x = 1 5

x = + k 2

x = arcsin 1 + 5 + k 2

1 + 5
x = arcsin
+ k 2
4

x = arcsin 1 5 + k 2

1 5
x = arcsin
4 + k 2

Vy, nghim ca phng trnh cho l


1 5
1 5

x = + k 2 , x = arcsin
+ k 2 , x = arcsin
+ k 2 , ( k ) .
2
4
4

2)

1 + tan x
2
= ( sin x + cos x ) (1)
1 tan x

318

x + k

cos
x

2
iu kin:

,(k ).
tan x 1
x + k

4
Vi iu kin trn th
(1)

cos x + sin x
2
= ( sin x + cos x )
cos x sin x
2

( cos x + sin x ) ( cos x sin x )( cos x + sin x ) = 0

( cos x + sin x ) 1 cos 2 x + sin 2 x = 0

2 sin + x = 0
cos x + sin x = 0

=
2sin
x
0

sin x = 0

sin + x = 0
+ x = k
x = + k

,(k )
4

sin x = 0
x = k
x = k
Kt hp vi iu kin ta c nghim ca phng trnh cho l
x=

+ k , x = k , ( k ).

3) 1 + tan 2 x =

1 sin 2 x
(1)
cos 2 2 x

iu kin: cos 2 x 0 x

+k

, ( k ).

Vi iu kin trn th
(1) 1 +

sin 2 x 1 sin 2 x
cos 2 x + sin 2 x 1 sin 2 x
=

=
2
cos 2 x
cos 2 x
cos 2 x
cos 2 2 x

cos 2 x ( cos 2 x + sin 2 x ) = 1 sin 2 x

cos 2 2 x + cos 2 x sin 2 x + sin 2 x 1 = 0


s in2x(cos 2 x + 1) + (cos 2 x 1)(cos 2 x + 1) = 0
(cos 2 x + 1)(sin 2 x + cos 2 x 1) = 0

cos 2 x + 1 = 0

cos 2 x + s in2x 1 = 0
cos 2 x = 1

2 cos(2 x ) = 1

4
319


2 x = + k 2


2 x = + k 2 , (k )

4 4

2 x = + k 2
4
4

x = 2 + k

x = + k , ( k )

x
=
k

Kt hp vi iu kin ta c nghim ca phng trnh cho l


x=

+ k , x = k , ( k ).

4) tan 3 x tan x = s in2x (1)

x +
3 x + k

cos 3 x 0

2
6 3 x + k , k .
iu kin:

(
)
6 3
cos x 0
x + k
x + k

2
2

Vi iu kin trn th

(1)

sin 2 x
= sin 2 x
cos 3 x cos x

sin 2 x = sin 2 x cos 3 x cos x


sin 2 x (1 cos 3 x cos x ) = 0
sin 2 x = 0
sin 2 x = 0

1
( cos 4 x + cos 2 x ) 1 = 0
cos 3x cos x 1 = 0
2
sin 2 x = 0
sin 2 x = 0

2
cos 2 x = 1 cos 2 x = 3
2
cos
2
x
+
cos
2
x

3
=
0

sin 2 x = 0
k
sin 2 x = 0 x =
.

2
cos 2 x = 1
i chiu vi iu kin th nghim ca phng trnh cho l x = k , ( k ) .

5) ( sin x sin 2 x )( sin x + sin 2 x ) = sin 2 3 x


sin 2 x sin 2 2 x = sin 2 3 x
320

1 cos 2 x 1 cos 4 x 1 cos 6 x

=
2
2
2

cos 2 x + cos 4 x + cos 6 x 1 = 0

cos 2 x + 2cos 2 2 x 1 + 4 cos3 2 x 3cos 2 x 1 = 0


2 cos3 2 x + cos 2 2 x 2 cos 2 x 1 = 0

cos 2 x = 1

cos 2 x = 1

1
cos 2 x =
2

x = k

x = + k

x = + k
3

2 x = k 2

2 x = + k 2

2
+ k 2
2x =
3

x = 2

, (k )
x = + k

3
Vy, nghim ca phng trnh cho l x =

, x = + k , ( k ).
2
3

6) sin x + sin 3 x + 4cos 3 x = 0


2sin 2 x cos x + 4 cos 3 x = 0
4 cos 2 x(sin x + cos x ) = 0

4 2 cos 2 x cos( x ) = 0
4

cos x = 0

cos( x ) = 0

x = 2 + k
x = 2 + k

, (k )

3
x = + k
x =
+ k

4 2
4
Vy, nghim ca phng trnh cho l x =

+ k , x =

3
+ k , ( k ).
4

321

7) sin 2 x = 1 + 2 cos x + cos 2 x


2sin x cos x = 1 + 2 cos x + 2 cos 2 x 1
2sin x cos x 2 cos x 2 cos 2 x = 0
cos x (2 sin x 2 2 cos x) = 0

cos x = 0
cos x = 0

sin x cos x = 2
2sin x 2 2cos x = 0

2
cos x = 0

2 sin( x ) = 2

4
2
cos x = 0

sin( x ) = 1 = sin
4
2
6

x = 2 + k
x = 2 + k

x = + k 2 x =
+ k 2 , ( k )

4 6
12

x = 5 + k 2
x = 13 + k 2

12
4
6
Vy, nghim ca phng trnh cho l
x=

+ k , x =

5
13
+ k 2 , x =
+ k 2 , ( k ) .
12
12

8) 2cos 6 x + sin 4 x + cos 2 x = 0. Dng cng thc h bc ta a phng trnh cho v


cos3 2 x + 4cos 2 2 x + 5cos 2 x + 2 = 0 cos 2 x = 1 cos 2 x = 2 2 x = + k 2
x=

+ k . Vy, nghim ca phng trnh cho l x =

9) 2 cos 2 3x cos 2 x cos 2 3 x + sin 2 x 1 = 0


cos 2 3x (2cos 2 x 1) cos 2 x = 0
1
cos 2 3x cos 2 x (1 + cos 2 x) = 0
2

1 1

cos 2 x. cos 2 3x = 0
2 2

1 + cos 6 x 1 1
cos 2 x
=0
2
2 2

cos 2 x cos 6 x 1 = 0

322

+ k , ( k ).

1
( cos8 x + cos 4 x ) 1 = 0
2
cos 8 x + cos 4 x 2 = 0

2 cos 2 4 x 1 + cos 4 x 2 = 0
2 cos 2 4 x + cos 4 x 3 = 0

cos 4 x = 1

cos 4 x = 3
2

cos 4 x = 1 4 x = k 2
k
x=
, (k ).
2
Vy, nghim ca phng trnh cho l x =
VI.3. 1) sin x + cot
iu kin: sin

k
, ( k ).
2

x
=2
2

x
0 x k 2 , ( k ) .
2

Ta thy x = + k 2 , ( k ) khng l nghim ca phng trnh (1).

x + k 2
x
Vi
, ( k ) ta t t = tan th phng trnh (1) tr thnh
2
x k 2
2t
1
+ =2
2
1+ t
t

( )
2 (1 + t ) t = 0

2t 2 + 1 + t 2 = 2 1 + t 2 t
2t 2 + 1 + t 2

2t 2 + 1 + t 2 2t 2t 3 = 0
2t 3 3t 2 + 2t 1 = 0
t =1
Khi t = 1 th tan

x
x

= 1 = + k x = + k 2 , ( k ).
2
2 4
2

So vi iu kin ta c nghim ca phng trnh cho l x =

+ k 2 , ( k ) .

2) sin 2 x + cos2 x + tan x = 2(1)


iu kin: x

+ k , ( k ) .

t t = tanx th phng trnh (1) tr thnh

323

2t
1 t2
+
+t = 2
1+ t2 1+ t2

) (

2t + 1 t 2 + t 1 + t 2 = 2 1 + t 2

2t + 1 t 2 + t + t 3 = 2 + 2t 2
t 3 3t 2 + 3t 1 = 0
t =1

Khi t = 1 th tan x = 1 x =

+ k , ( k )

So vi iu kin ta c nghim ca phng trnh cho l x =


3) cos4x

3 1 tan 2 x
2

1 + tan x

iu kin: x

+ k , ( k ) .

) + 2 = 0 (1)

+ k , k . Khi phng trnh (1) tr thnh

cos 4x 3cos 2 x 1 tan 2 x + 2 = 0


1

cos 4x 3cos 2 x 2
+2 = 0
cos 2 x

cos 4x 6cos 2 x + 3 + 2 = 0

2 cos 2 2 x 1 3 (1 + cos 2 x ) + 5 = 0
2 cos 2 2 x 3cos 2 x + 1 = 0
cos 2 x = 1
2 x = k 2
x = k

,(k ).

cos 2 x = 1
2 x = + k 2
x = + k

2
3
6

So vi iu kin ta c nghim ca phng trnh cho l x = k , x =


4)

tan x - 1
+ cot 2 x = 0, ( 0 < x < ) (1)
tan x + 1

x + k
k

cos
x

x
2

2
iu kin: sin2x 0 2 x k

, (k ).

tanx 1

x + k

x + k
4
4

324

+ k 2 , ( k ).


0 < x <

Kt hp vi iu kin 0 < x < ta c iu kin: x


(*)
2

x 4
Khi (1)

tanx 1 cos2x
+
=0
tanx + 1 sin2x

t t = tan x th phng trnh (1) tr thnh


t 1 1 t 2
+
=0
t +1
2t
t +1
1
( t 1)

=0
t + 1 2t
t 1 = 0
1
t =1
t +1

=0
t + 1 2t
tan x = 1
x=

+ k , ( k )

Kt hp vi iu kin (*) ta c nghim ca phng trnh cho l x =


5) tan 3 x 1 +

3cot x = 3(1)
2
cos x
2

Vi iu kin < x <

< x <

3
th phng trnh (1) tr thnh
2

1
3 tan x = 3
cos 2 x
tan 3 x + tan 2 x 3 tan x 3 = 0
tan 3 x 1 +

( tan x + 1) tan 2 x 3 = 0
tan x = 1 tan x = 1
2

tan x = 3
tan x = 3

x = 4 + k

, (k )
x = + k

325

Kt hp vi iu kin < x <

3
4
, ta c nghim ca phng trnh cho l x =
.
2
3

6) cos3 x sin x sin 3 x cos x =

2
(1)
8

(1) cos x sin x(cos 2 x sin 2 x ) =

2
8

2
2
2
sin 2 x cos 2 x =
sin 4 x =
8
4
2

x = 16 + k 2
4 x = 4 + k 2
; (k ).

x = 3 + k
4 x = 3 + k 2

4
16
2
cos x sin x cos 2 x =

Vy, nghim ca phng trnh cho l x =

16

+k

;x =

+ k ; ( k Z ).
16
2

7) sin 2 3 x cos 2 4 x = sin 2 5 x cos 2 6 x(1)


(1) sin 2 3x sin 2 5 x = cos 2 4 x cos 2 6 x
( sin 3x + sin 5 x )( sin 3 x sin 5 x ) = ( cos 4 x + cos 6 x )( cos 4 x cos 6 x )
( 2 sin 4 x cos x )( 2 cos 4 x sin x ) = ( 2 cos 5 x cos x )( 2 sin 5 x sin x )
cos x sin x ( sin 4 x cos 4 x + cos 5 x sin 5 x ) = 0
1
1

cos x sin x sin 8 x + sin10 x = 0


2
2

cos x sin x ( sin 9 x cos x ) = 0


cos 2 x sin x sin 9 x = 0
cos x = 0
sin x = 0
sin 9 x = 0

x = 2 + k
x =

x = k

x =

x =
9

k
2
, ( k ).
k
9

Vy, nghim ca phng trnh cho l x =

k
k
;x =
, ( k Z ).
2
9

8) cos 3x 4 cos 2 x + 3cos x 4 = 0, x [ 0;14]

326

(1) 4 cos3 x 3cos x 4 2cos 2 x 1 + 3cos x 4 = 0


4 cos3 x 8cos 2 x = 0
4 cos 2 x ( cos x 2 ) = 0

cos x = 0

cos x = 2
Ta loi trng hp cos x = 2. Nh vy ta c cos x = 0 x =

Do x [0;14] suy ra 0

+ k ; ( k ) .

+ k 14, ( k Z )

1
14 1
k , k Z k {0;1; 2;3}.
2
2

3 5 7
Vy, nghim ca phng trnh cho l tha yu cu ca bi l x ; ; ; .
2 2 2 2
1
x
9) sin 4 x + sin 4 + + cos 4 x = sin 2 2 x(1)
2
2 8

x
(1) sin 4 x + sin 4 + + cos 4 x = 2sin 2 2 x cos 2 x
2 8
2
x
cos 2 x sin 2 x + sin 4 + = 0
2 8
x
cos 2 2 x + sin 4 + = 0
2 8

cos 2 2 x = 0
cos 2 x = 0

4x
x
sin 2 + 8 = 0
sin 2 + 8 = 0

2 x = 2 + k
x = 4 + 2

x + = m
x = + m
2 8
2
8
k

x = 4 + 2

x = + m2 , ( m ) .
4
x = + m2

4
Vy, nghim ca phng trnh cho l x =
10) 2 cos 2 x 8cos x + 7 =

+ m2 , (m Z ).

1
(1)
cos x

327

iu kin: cos x 0 x

+ k , (k Z)(*)

(1) 2 cos x cos 2 x 8cos 2 x + 7 cos x = 1

2 cos x 2 cos 2 x 1 8cos 2 x + 7 cos x 1 = 0


4 cos3 x 8cos 2 x + 5cos x 1 = 0

x = k 2
cos x = 1

; k .
x = + k 2
cos x = 1
3

Cc gi tr ny u tha iu kin (*)


Vy, nghim ca phng trnh cho l x = k 2 ; x =

+ k 2 ; (k Z ).

cos 3 x + sin 3x

VI.4. 1) 5 sin x +
= cos 2 x + 3, x (0; 2 )
1 + 2sin 2 x

x 12 + k
iu kin: 1 + 2sin 2 x 0
x 7 + k

12

, ( k ).

x ( 0; 2 )

11
23

Kt hp vi iu kin x ( 0; 2 ) ta c iu kin x
,x
(*)
12
12

7
19

x 12 , x 12
Khi phng trnh cho tng ng vi

sin x + 2sin x sin 2 x + cos 3x + sin 3 x


5
= cos 2 x + 3
1 + 2sin 2 x

sin x + cos x + sin 3 x


5
= cos 2 x + 3
1 + 2sin 2 x

2sin 2 x cos x + cos x


5
= cos 2 x + 3
1 + 2sin 2 x

5 cos x = cos 2 x + 3

5cos x = 2cos 2 x 1 + 3

cos x = 2
2 cos x 5cos x + 2 = 0
cos x = 1

328

x = + k 2

1
3
V cos x 1 nn ta chn trng hp cos x =
, (k )
2
x = + k 2

3
So vi iu kin (*) th phng trnh cho c nghim l x =

,x =

5
.
3

2) sin 2 x.cos x = tan 3 x.sin( x + ) cos 2 x.sin x(1)


6
iu kin: cos 3x 0 x

+k

, ( k ) . Khi phng trnh (1) tr thnh

sin 2 x cos x + cos 2 x sin x = tan 3 x sin( x + )


6

sin 3x = tan 3x sin( x + )


6
sin 3 x =

sin 3 x

sin( x + )
cos 3x
6

sin 3x 1
sin( x + ) = 0
6
cos 3x

sin 3 x = 0

1 1 sin( x + ) = 0
6
cos 3x
sin 3 x = 0

cos 3 x sin( x + ) = 0
6

sin 3 x = 0

cos 3 x = sin( x + )
6

sin 3 x = 0

sin( 3 x) = sin( x + )
2
6

3 x = k

, (k )
3 x = x + + k 2
2
6

3 x = ( x + ) + k 2

2
6

329

x = 3

k
x =
, (k )

12 2

x = k

6
So vi iu kin ta c nghim ca phng trnh cho l x =
3) cot x 1 =

cos 2 x
1
+ sin 2 x sin 2 x
1 + tan x
2

k
x

2
iu kin:

, (k ).
2

tan x 1 x + k

4
Khi phng trnh cho tr thnh
cos x sin x
cos 2 x sin 2 x
1
= cos x (
) + sin 2 x sin 2 x
sin x
(cos x + sin x )
2
cos x sin x
1
= cos x(cos x sin x) + sin 2 x sin 2 x
sin x
2
cos x sin x
1

= cos 2 x cos x sin x + sin 2 x sin 2 x


sin x
2

cos x sin x
= 1 sin 2 x
sin x

cos x sin x
= (cos x sin x )2
sin x

(cos x sin x)
(cos x sin x) = 0
sin x

cos x sin x = 0
1

(cos x sin x) = 0
sin x

2 cos( x + 4 ) = 0

1 (cos x sin x) = 0
sin x

x + = + k , ( k )

4 2

2
1 sin x cos x + sin x = 0
330

k
k
,x =
,(k ).
3
12 2

x = 4 + k , ( k )

1 cos x + 1 = 0
sin 2 x sin x

x = + k , (k )

4
2
cot x cot x + 2 = 0
x=

+ k , (k ) (V phng trnh cot 2 x cot x + 2 = 0 v nghim).

So vi iu kin ta c nghim ca phng trnh cho l x =


4) sin 4 x.sin 2 x + sin 9 x.sin 3 x = cos 2 x

+ k , (k ).

1
1
(cos 2 x cos 6 x) + (cos 6 x cos12 x ) = cos 2 x
2
2

1
1 + cos 2 x
(cos 2 x cos12 x) =
2
2
cos12 x = 1

12 x = + k 2
x=

12

k
, (k ).
6

Vy, nghim ca phng trnh cho l x =

12

k
, (k ).
6

5) cos 2 x.sin 4 x + cos 2 x = 2 cos x (sin x + cos x) 1


cos 2 x sin 4 x + cos 2 x = 2 cos x sin x + 2 cos 2 x 1
cos 2 x sin 4 x + cos 2 x = 2 cos x sin x + cos 2 x

1
sin 2 2 x sin 2 x = sin 2 x
4
1

sin 2 x sin 2 x sin 2 x 1 = 0


4

sin 2 x = 0
1
sin 2 x sin 2 x 1 = 0
4
2 x = k , ( k )

2
sin 2 x sin x = 4
x=

k
, (k ) (V s in2x sin 2 x 1 nn phng trnh sin 2 x sin 2 x = 4 v nghim).
2

331

Vy, nghim ca phng trnh cho l x =

k
, (k ).
2

6) 3 cos 4 x + sin 4 x 2 cos 3x = 0


3 cos 4 x + sin 4 x = 2 cos 3x

3
1
cos 4 x + sin 4 x = cos 3 x
2
2

cos

cos 4 x + sin

sin 4 x = cos 3 x

cos 4 x = cos 3x
6

4 x 6 = 3 x + k 2
x = 6 + k 2

,(k ).
4 x = 3x + k 2
x = + k 2

6
42
7
Vy, nghim ca phng trnh cho l x =

+ k 2 , x =

42

k 2
,(k ).
7

7) 4cos 2 x 2 cos 2 2 x = 1 + cos 4 x


4 cos 2 x 2 cos 2 2 x = 2 cos 2 2 x
cos 2 x cos 2 2 x = 0
( cos x cos 2 x )( cos x + cos 2 x ) = 0
3x
x
3x
x
sin cos cos = 0
2
2
2
2
s in3x sin x = 0
sin

s in3x = 0

sin x = 0
3 x = k

x = k
k

x=

x = k
x=

k
, (k ).
3

Vy, nghim ca phng trnh cho l x =

k
, (k ).
3

8) 2sin 2 x = 2sin 2 x tan x (1) . iu kin: x + k , (k ).


4
2

332


sin x
(1) 2 sin x = 2sin 2 x
4
cos x

sin x
2
( sin x cos x ) = 2sin 2 x
cos x
sin x
1 2sin x cos x = 2sin 2 x
cos x
2
2
cos x 2sin x cos x 2sin x cos x + sin x = 0
sin x + cos x 2sin x cos x ( sin x + cos x ) = 0
( sin x + cos x )(1 sin 2 x ) = 0

2 sin x + = 0
sin x + cos x = 0

sin 2 x = 1
sin 2 x = 1

x = 4 + k

,(k )
x = + k

4
So vi iu kin ta c nghim ca phng trnh cho l x =

+ k , x =

+ k , (k ).

9) cos 3x + 2 cos 2 x = 1 2 sin x sin 2 x


cos 3x + 2 cos 2 x = 1 + cos 3x cos x
2 cos 2 x = 1 cos x

2 2cos 2 x 1 = 1 cos x
4 cos 2 x + cos x 3 = 0

cos x = 1 x = + k 2

, k ).
x = arccos 3 + k 2 (
cos x = 3

4
3
Vy, nghim ca phng trnh cho l x = + k 2 , x = arccos + k 2 , (k ).
4
10) ( 2sin x 1)( 2 cos x + sin x ) = sin 2 x cos x
( 2 sin x 1)( 2 cos x + sin x ) = 2 sin x cos x cos x
( 2 sin x 1)( 2 cos x + sin x ) cos x ( 2sin x 1) = 0
( 2 sin x 1)( cos x + sin x ) = 0

333

sin x =

2sin x 1 = 0
2

2 sin x + = 0
cos x + sin x = 0

x = 6 + k 2

5
x =
+ k 2 , (k )

x = + k

4
Vy, nghim ca phng trnh cho l x =
11)

+ k 2 , x =

+ k 2 , x = + k , (k ).
6
4

3 cos 5 x 2 sin 3x cos 2 x sin x = 0(1)

(1) 3 cos 5 x ( sin 5 x + sin x ) sin x = 0


3 cos 5 x sin 5 x = 2sin x

3
1
cos 5 x sin 5 x = sin x
2
2

sin 5 x = sin x
3

3 5 x = x + k 2
6 x = 3 k 2
x = 18 k 3

, k .

5 x = x + k 2 4 x = k 2
x = k
3

3
6
2
Vy, nghim ca phng trnh cho l x =

18

; x = k ,(k ).
3
6
2

12) sin x + cos x sin 2 x + 3 cos 3 x = 2 cos 4 x + sin 3 x (1)


3
1
3sin x sin 3 x
(1) sin x + sin 3x + 3 cos 3x = 2 cos 4 x +
2
2
2
sin 3x + 3 cos 3 x = 2 cos 4 x
1
3
sin 3 x +
cos 3 x = cos 4 x
2
2

cos 4 x = cos 3 x
6

334

4 x = 3 x 6 + k 2
x = 6 + k 2

, ( k ).
4 x = 3 x + + k 2
x = + k 2

6
42
7

Vy, nghim ca phng trnh cho l x =


VI.5.1) tan

( 2 sin
x +1 =

iu kin: x

2 x sin 3 x
4

cos x

+ k 2 , x =

42

+k

2
,(k ).
7

(1)

+ k , ( k ) .

2 sin 2 2 x sin 3 x
sin 4 x
(1)
+1 =
cos 4 x
cos 4 x

sin 4 x + cos 4 x = 2 sin 2 2 x sin 3 x

1 2sin 2 x cos 2 x = 2 sin 2 2 x sin 3 x


1
1 sin 2 2 x = ( 2 sin 2 2 x ) sin 3 x
2
1
2 sin 2 2 x ) ( 2 sin 2 2 x ) sin 3x = 0
(
2

2 sin 2 2 x sin 3x = 0
2

2 sin 2 2 x = 0
sin 2 2 x = 2
1

sin 3x = 0
sin 3 x = 1
2
2

k 2

3 x = + k 2
x= +

1
6
18
3
sin 3x =

, (k ).
2
3 x = 5 + k 2
x = 5 + k 2

6
18
3

(V phng trnh sin 2 2x = 2 v nghim).


So vi iu kin ta c nghim ca phng trnh cho l
x=

18

k 2
5 k 2
,x =
+
, (k ).
3
18
3

sin 4 x + cos 4 x 1
1
2)
= cot 2 x
(1)
5sin 2 x
2
8sin 2 x

335

iu kin: sin 2 x 0 2 x k x

k
,(k ).
2

Vi iu kin trn th
(1)

sin 4 x + cos 4 x cos 2 x


1
=

5sin 2 x
2sin 2 x 8sin 2 x

(
8 (1 2sin

8 sin 4 x + cos 4 x = 20cos 2 x 5


2

x cos 2 x = 20 cos 2 x 5

8 1 sin 2 2 x = 20 cos 2 x 5
2

8 4sin 2 2 x = 20 cos 2 x 5

8 4 1 cos 2 2 x = 20 cos 2 x 5
9

cos 2 x =

2
4 cos 2 2 x 20cos 2 x + 9 = 0
1
cos 2 x =

2 x = + k 2
x = + k

1
3
6
cos 2 x =

,(k ).

2
2 x = + k 2
x = + k

3
6
(V phng trnh cos 2 x =

9
v nghim).
2

So vi iu kin ta c nghim ca phng trnh cho l x =

+ k , (k ).

3) 1 + sin x + cos x + sin 2 x + cos 2 x = 0


(1 + sin 2 x ) + sin x + cos x + cos 2 x = 0
2

( sin x + cos x ) + ( sin x + cos x ) + ( sin x + cos x )( cos x sin x ) = 0


( sin x + cos x )( sin x + cos x + 1 + cos x sin x ) = 0
( sin x + cos x )( 2 cos x + 1) = 0

2
cos(
x

)=0

sin x + cos x = 0
4

2cos x + 1 = 0
cos x = 1

336

x 4 = 2 + k
x = 4 + k

, (k ).
x = 2 + k 2
x = 2 + k 2

3
3

Vy, nghim ca phng trnh cho l x =

3
2
+ k , x =
+ k 2 , (k ).
4
3

4) 2cos 2 x sin 2 x + cos x sin 3 x = 0


4
4 2

2sin 2 x cos 2 x + sin 3x cos x = 0


4
4 2

1
1
1

sin 2 2 x + sin 2 x + sin 4 x = 0


2
2
2 2

1
1
1
sin 2 2 x + ( sin 2 x cos 4 x ) = 0
2
2
2

sin 2 2 x + sin 2 x 1 2sin 2 2 x 1 = 0

sin 2 x = 1
sin 2 2 x + sin 2 x 2 = 0
sin 2 x = 2
s in2x = 1 (V phng trnh s in2x = 2 v nghim).
2x =

+ k 2 x =

+k , (k ).

Vy, nghim ca phng trnh l x =

+ k , (k ).

5) cot x + sin x 1 + tan x tan = 4(1)


2

x 2
k
x
iu kin:

x
,(k ).
2
2
cos x 0
x + k 2

2
Vi iu kin trn th ta c
x

sin x sin

2 =4
(1) cot x + sin x 1 +
x
cos x cos
2

337

cos 2
cot x + sin x
=4
x
cos x cos
2

cot x +

sin x
=4
cos x

4 sin x cos x = 1

2 x = + k 2
x = + k

1
6
12
sin 2 x =

,(k ).
2
2 x = 5 + k 2
x = 5 + k

12
6
So vi iu kin ta c nghim ca phng trnh cho l
x=

6)

12

+ k , x =

5
+ k , (k ).
12

2(cos 6 x + sin 6 x) sin x cos x


= 0(1)
2 2sin x

iu kin:

x 4 + k 2
2
2 2sin x 0 sin x

( k )

3
2
x
+ k 2

Vi iu kin trn th
(1) 2(cos 6 x + sin 6 x ) sin x cos x = 0
2(1 3cos 2 x sin 2 x ) sin x cos x = 0
6 cos 2 x sin 2 x + sin x cos x 2 = 0

sin x cos x = 3
sin 2 x =

sin x cos x = 1
sin 2 x = 1

2
s in2x = 1 (V phng trnh sin2x =

2x =

x=

4
v nghim)
3

+ k 2

+ k , ( k ) .

So vi iu kin ta c nghim ca phng trnh cho l x =

338

5
+ k 2 , (k ).
4

7) cos 3x + cos 2 x cos x 1 = 0


4 cos3 x 3cos x + 2 cos 2 x 1 cos x 1 = 0
4 cos3 x + 2 cos 2 x 4 cos x 2 = 0
(cos 2 x 1)(4 cos x + 2) = 0

x = k 2
cos x = 1

cos x = 1 x = + k 2

1
2
+ k 2
x =
cos x =
2
3

x = k

x = 2 + k 2
3

Vy, nghim ca phng trnh cho l x = k , x =

2
+ k 2 , (k ).
3

8) 13 18 tan x = 6 tan x 3

6 tan x 3 0

2
13 18 tan x = (6 tan x 3)
1

tan x

2
18 tan 2 x 9 tan x 2 = 0
1

tan x 2

2
2

2
tan x = tan x = x = arctan + k , (k ).
3
3
3

1
tan x = 6

2
Vy, nghim ca phng trnh cho l x = arctan + k , (k ).
3

9) cos 4 x sin 4 x = cos x + sin x (1)


Ta c VT = cos 4 x sin 4 x = cos 2 x sin 2 x = cos 2 x 1, x .
2
cos x cos x
Mt khc
VP = cos x + sin x sin 2 x + cos 2 x = 1
2
sin
x

sin
x

cos 2 x = 1
x = k , ( k )

2
Do (1) cos x = cos x cos x = cos 2 x x = k , (k ).
sin x = sin 2 x
sin x = sin 2 x

339

Vy, nghim ca phng trnh cho l x = k , (k ).


10) cos13 x + sin14 x = 1(1)
sin14 x sin 2 x
Ta c 13
cos13 x + sin14 x sin 2 x + cos 2 x = 1 VT 1
2
cos x cos x

sin x = 0
sin x = 0

sin14 x = sin 2 x
sin x = 1 cos x = 0
Do (1) 13

2
cos
x
=
cos
x
cos
x
=
0

cos x = 0

cos x = 1
cos x = 1

sin x = 0
x = k 2

cos x = 1
, (k ).
x = + k
cos x = 0
2

Vy, nghim ca phng trnh cho l x = k 2 , x =

+ k 2 , (k ).

VI.6. 1) tan x = cot x + 4cos 2 2 x.


iu kin: sin x.cos x 0 x

k
,(k ).
2

Phng trnh cho tng ng vi


tan x cotx = 4 cos 2 2 x

sin x cos x
2 cos 2 x

= 4 cos 2 2 x
+ 4 cos 2 2 x = 0
cos x sin x
sin 2 x

cos 2 x
+ 2cos 2 x = 0 cos 2 x (1 + sin 4 x ) = 0.
sin 2 x

cos 2 x = 0 x =

sin 4 x = 1 x =

k
.
2
+

k
.
2

i chiu iu kin suy ra nghim ca phng trnh cho l


x=

k
k
v x = +
, (k ).
2
8 2

2) sin 2 x = sin x +
.
4
4 2

Phng trnh cho tng ng vi


2
2
2
( cos x sin x )( 2 cos x 1) = 0.
( sin 2 x cos 2 x ) = ( sin x cos x ) +
2
2
2

340

cos x sin x = 0 tgx = 1 x =


2 cos x 1 = 0 cos x =

+ k .

x = + k 2 .
2
3

Vy, nghim ca phng trnh cho l x =

+ k , x =

+ k 2 , ( k ) .

3 2cos 2 x + cos x 2 + ( 3 2 cos x ) sin x = 0

3)

Phng trnh cho tng ng vi

2 3 1 sin 2 x + 3 cos x 2 3 + 3sin x 2 cos x sin x = 0


2 3 sin 2 x + 3 cos x + 3sin x 2cos x sin x = 0

3 cos x + 3 sin x 2sin x cos x + 3 sin x = 0

3 sin x + cos x

)(

3 2sin x = 0

x = + k 2

3
3

sin x =
3 2 sin x = 0
2

x =
+ k 2
3

3
1
3 sin x + cos x = 0

sin x + cos x = 0
2

2
2
cos x = 0

x = 3 + k 2
x = 3 + k 2

x = 2 + k 2 x = 2 + k 2 , ( k ) .

3
3

x = + k
x = 5 + k

3 2
6

Vy, nghim ca phng trnh cho l x =

+ k 2 , x =

+ k 2 , x =

5
+ k , ( k ) .
6

4) (1 + 2cos 3 x ) sin x + sin 2 x = 2sin 2 2 x +


4

Phng trnh cho tng ng vi

sin x + sin 4 x sin 2 x + sin 2 x = 1 cos 4 x +


2

sin x + sin 4 x = 1 + sin 4 x


sin x = 1
x=

+ k 2 , ( k ) .
341

Vy, nghim ca phng trnh cho l x =

+ k 2 , ( k ) .

x
x
x
5) 1 + sin sin x cos sin 2 x = 2 cos 2
2
2
4 2
Phng trnh cho tng ng vi
x
x

1 + sin sin x cos sin 2 x = 1 + cos x


2
2
2

x
x
1 + sin sin x cos sin 2 x = 1 + sin x
2
2
x
x

sin x sin cos sin x 1 = 0


2
2

x
x
x

sin x sin 2 cos 2 sin 1 = 0


2
2
2

x
x
x

sin x sin 2 1 sin 2 sin 1 = 0


2
2

2
x
x
x

sin x sin 1 1 + 2 sin + 1 sin = 0


2
2
2

x
x
x

sin x sin 1 2 sin 2 + 2 sin + 1 = 0


2
2
2

sin x = 0
x = k 2
x = k 2

x = k , ( k ) .
x
sin = 1 = + k 2
x = + k 4
2
2 2

Vy, nghim ca phng trnh cho l x = k , ( k ) .

1
6) cos 2 ( x + ) + sin 2 ( x + ) = 2 sin x
3
6
4
Phng trnh cho tng ng vi

1
cos 2 ( x + ) + sin 2 ( x + ) = 2 sin x
3
6
4
1
2 1

1
1 + cos(2 x +
) + 1 cos(2 x + ) = 2 sin x

2
3 2
3
4

2
1
2 + cos(2 x +
) cos(2 x + ) = 4 sin x
3
3
2

5
2sin(2 x + ) sin 4sin x + = 0
2
6
2

5
sin(2 x + ) + 4sin x = 0
2
2
342

5
=0
2
5
1 2sin 2 x + 4 sin x = 0
2
3
2 sin 2 x 4 sin x + = 0
2
2
4 sin x 8sin x + 3 = 0
cos 2 x + 4 sin x

x = + k 2

sin x = 2
1
6

sin x =
, ( k ).
1
5

2
x =
sin x =
+ k 2

2
6

Vy, nghim ca phng trnh cho l x =

+ k 2 , x =

5
+ k 2 , ( k ) .
6

sin 3x 4 cos x 3
6

=0
7)
sin 3 x 1
iu kin: sin 3x 1 3 x

+ k 2 x

k 2
, ( k ) . (*)
3

Phng trnh cho tng ng vi

sin 3x 4 cos x 3 = 0
6

cos 3 x 4 cos x 3 = 0
2
6

cos 3 x 4cos x 3 = 0
6
6

4 cos3 x 7 cos x 3 = 0
6
6

x=
+ k 2
cos x 6 = 1


1
5

cos x = x =
+ k 2 , ( k ) .

6
2
6


3
x = + k 2
cos x =

2
6 2

Kt hp vi iu kin (*) ta c nghim ca phng trnh cho l
x=

7
+ k 2 , ( k ) .
6

343

8) cos 3x cos 3 x sin 3x sin 3 x =

2+3 2
.
8

Phng trnh cho tng ng vi


2+3 2
2
2+3 2
cos 2 3 x + sin 2 3 x + 3 ( cos 3x cos x sin 3 x sin x ) =
2
1

k
cos 4 x =
4 x = + k 2 x = +
, ( k ).
4
16 2
2
cos 3x ( cos 3 x + 3cos x ) sin 3x ( 3sin x sin 3x ) =

Vy, nghim ca phng trnh cho l x =

16

k
,(k ).
2

9) 4sin 3x sin x + 4 cos 3 x cos x + cos 2 2 x + + 1 = 0.


4
4
4

Phng trnh cho tng ng vi


2 ( cos 2 x + sin 2 x ) +

1 + sin 4 x
=0
2

t t = cos 2 x + sin 2 x = 2 cos 2 x sin 4 x = t 2 1. iu kin: t 2


4

t = 0
Ta c phng trnh t 2 + 4t = 0
t = 4

3 k

Ta chn t = 0 . Suy ra cos 2 x = 0 x =


+
,(k ).
4
8
2

Vy, nghim ca phng trnh cho l x =

3 k
+
,(k ).
8
2

10) sin 3x + 3 cos 3x + cos 2 x 3 sin 2 x = sin x + 3 cos x.


Phng trnh cho tng ng vi
2cos 2 x sin x 2 3 sin 2 x sin x + cos 2 x 3 sin 2 x = 0

(1 + 2sin x ) cos 2 x 3 sin 2 x = 0

x = + k 2

sin x = 2
1 + 2sin x = 0
7

x =
+ k 2 , ( k ) .

6
cos 2 x + = 0
cos 2 x 3 sin 2 x = 0


3
x = + k

12 2
344

Vy, nghim ca phng trnh cho l

7
k
+ k 2 , x = +
, ( k ).
x = + k 2 , x =
6
6
12 2
11) sin 2 x (1 + tan x ) = 3sin x ( cos x sin x ) + 3 (1)
iu kin: x

+ k , ( k ) .(*)

Phng trnh (1) tng ng vi


sin x + cos x
2
sin 2 x
= 3 1 sin x + sin x cos x
cos x

2
sin x ( sin x + cos x ) = 3cos 2 x ( sin x + cos x )

( sin x + cos x ) 3cos 2 x sin 2 x = 0


x = + k
sin x + 4 = 0

, ( k ).

1
x = + k
cos 2 x =

(Tha iu kin (*)). Vy, nghim ca phng trnh cho l


x=

12)

+ k , x =

+ k , ( k ) .

sin 2 x cos 2 x
+
= tan x cot x (1)
cos x
sin x

iu kin: cos x.sin x 0 x

k
, ( k ) . (*)
2

Phng trnh (1) tng ng vi


sin 2 x cos 2 x sin x cos x
+
=

cos x
sin x
cos x sin x
sin 2 x sin x + cos 2 x cos x = sin 2 x cos 2 x
cos x = cos 2 x
cos 2 x + cos x = 0
3x
x
2 cos cos = 0
2
2
3x

3x
k 2

cos 2 = 0
2 = 2 + k
x= +

3
3

cos x = 0
x = + k
x = + k 2

2
2 2
Kt hp vi iu kin (*) ta c nghim ca phng trnh cho l

345

x=

13)

+ k 2 , ( k ) .

1
+
sin x

= 4 sin
x (1)
3

sin x

sin x 0
x k
k

iu kin:

x
; ( k ) . (*)
3
3
sin
x

0
2
x

+
k

2
2

(1) tng ng vi:
1
1
+
= 2 2 ( sin x + cos x )
sin x cos x

( sin x + cos x )
+2 2 =0
sin x cos x

+ sin x + cos x = 0 x =

+ k .

1
2
+ 2 2 = 0 sin 2 x =
sin x cos x
2

x = 8 + k

x = 5 + k

8
+

i chiu vi iu kin (*) ta c nghim ca phng trnh cho l


x=

+ k ; x =

+ k ; x =

5
+ k ; ( k ) .
8

14) sin 3 x 3 cos3 x = sin x cos 2 x 3 sin 2 x cos x(1)


+ cos x = 0 : Phng trnh (1) khng tha.
+ cos x 0 x

+ k . Chia hai v ca (1) cho cos3 x 0 ta c phng trnh

tan 3 x 3 = tan x 3 tan 2 x


tan 3 x + 3 tan 2 x tan x 3 = 0

x = 4 + k
tan x = 1

tan x = 1 x = + k ; ( k )

4
tan x = 3

x = + k

3
346

Vy, nghim ca phng trnh cho l x =

+ k , x =

+ k , x =

+ k , ( k ) .

15) 2sin x (1 + cos 2 x ) + sin 2 x = 1 + 2 cos x(1).


(1) 4sin x cos 2 x + 2sin x cos x 1 2cos x = 0
2 cos x ( 2sin x cos x 1) + ( 2sin x cos x 1) = 0
( sin 2 x 1)( 2 cos x + 1) = 0

x
=
+ k
sin 2 x = 1

; ( k ).
cos x = 1
x = 2 + k
2

3
Vy, nghim ca phng trnh cho l x =

+ k , x =

2
+ k , ( k ) .
3

x 6 + k 2

1 2sin x ) cos x
(
7
sin x

16)
= 3(1). iu kin:
+ k 2 ; ( k ) . (*)
2 x
6
(1 + 2 sin x )(1 sin x )
sin x 1

x 2 + k 2

(1) (1 2sin x ) cos x = 3 (1 + 2sin x )(1 sin x )

cos x 2sin x cos x = 3 1 + sin x 2sin 2 x

cos x 3 sin x = sin 2 x + 3 cos 2 x


1
3
1
3
cos x
sin x = sin 2 x +
cos 2 x
2
2
2
2

cos x + = cos 2 x
3
6


x + 3 = 2 x 6 + k 2

x + = 2 x + + k 2

3
6

x = 2 k 2

x = + k 2

18
3

i chiu vi iu kin (*) ta c nghim ca phng trnh cho l


x=

18

k 2
, ( k ).
3

347

11 5 x
7 x
3x
VI.7. 1) cos
+ sin
= 2 sin + (1).
2
4
4 2
2 2
(1) cos

cos

5x

5x

x
3x
+ sin sin sin cos cos sin = 2 cos
2
4
2
4
2
4
2
2

2
5x
x
5x
x
3x
sin sin cos cos = 2 cos
2
2
2
2
2
2

x
5x
x
3x
5x
sin sin cos + cos = 2 cos
2
2
2
2
2

2 cos
cos

3x
3x
3x
sin x 2 cos cos x = 2 cos
2
2
2

3x
( sin x cos x 1) = 0
2

3x

cos = 0

sin x cos x = 1
3x

cos
=0

2

sin x 4 = 2

k 2

3x
x = 3 + 3
2 = 2 + k

x = + k 2
x = + k 2

2
4 4
x = + k 2

x = + k 2

4
4
k 2

x = 3 + 3

,(k ).
x = + k 2

2
Vy, nghim ca phng trnh cho l x =

2)

2 sin x
4

1 + sin 2 x = 1 + tan x (1).


(
)
cos x

iu kin: cos x 0 x

348

+ k ; ( k ) (*).

k 2

; x = + k 2 , ( k ) .
3
2

(1)

cos x sin x
(1 + sin 2 x ) = 1 + tan x
cos x

(1 tan x )(1 + sin 2 x ) = 1 + tan x


sin 2 x sin 2 x tan x 2 tan x = 0

sin x
=0
cos x
sin x cos 2 x cos x sin 2 x sin x = 0
sin x cos x sin 2 x

sin x = 0
sin x cos 2 x sin x cos x 1 = 0 2
cos x sin x cos x = 1

sin x = 0
sin x = 0

2
cos 2 x + =
cos 2 x sin 2 x = 1

4 2

x = k
x = k

, ( k ) , tho iu kin (*).


2 x + = + k 2
x = + k

4 4

2 x + = + k 2
4
4

Vy, nghim ca phng trnh cho l x = k ; x =

+ k , ( k ) .

3) sin 3 x + cos 3 x = cos 2 x ( 2 cos x sin x ) (1).

(1) ( sin x + cos x )(1 sin x cos x ) = cos 2 x sin 2 x ( 2 cos x sin x )
( sin x + cos x ) 1 sin x cos x ( cos x sin x )( 2 cos x sin x ) = 0
( sin x + cos x ) cos x ( 2 sin x cos x ) = 0

x = + k
2 sin x + 4 = 0

sin x + cos x = 0

cos x = 0
x = + k
x = + k
,(k ).
2
2

2sin x cos x = 0
tan x = 1
x = arctan 1 + k

2
2

Vy, nghim ca phng trnh cho l

1
x = + k , x = + k , x = arctan + k , ( k ) .
4
2
2

349

4) 2sin 2 x = 2sin 2 x tan x (1).


4

iu kin: cos x 0 x

+ k , ( k ) (*).

(1) ( sin x cos x ) = 2sin 2 x tan x


cos 2 x sin 2 x 2sin x cos x + tan x = 0 (2).
Chia hai v ca phng trnh (2) cho cos 2 x 0 ta c

(2) 1 tan 2 x 2 tan x + tan x 1 + tan 2 x = 0


tan 3 x tan 2 x tan x + 1 = 0

( tan x 1) tan 2 x 1 = 0
tan x 1 = 0
tan x = 1
2

tan x = 1
tan x 1 = 0

x = 4 + k

, ( k ) , tho iu kin (*).

x = + k

4
Vy, nghim ca phng trnh cho l x =
5)

1
x 1
x
+ cos 2 = sin 2 (1).
4
3 2
2

x 1
1
+ cos 2 = (1 cos x )
4
3 4
1
x 1 1
+ cos 2 + cos x = 0
4
3 4 4
x 1
x
x
cos 2 + 4 cos3 3cos = 0
3 4
3
3
x
x 3
x
cos3 + cos 2 cos = 0
3
3 4
3
x
x
x

cos 4cos 2 + 4cos 3 = 0


3
3
3

(1)

x
x

cos 3 = 0
cos 3 = 0

4cos 2 x + 4cos x 3 = 0
cos x = 1

3
3
3 2

350

+ k , x =

+ k , ( k ) .

x
3

3 = 2 + k
x=
+ k 3

, ( k ).
2

x = + k 2
x = + k 6
3
3
Vy, nghim ca phng trnh cho l x =
6)

3 sin + cos x =

3
+ k 3 , x = + k 6 , ( k ) .
2

1
(1).
cos x

iu kin: cos x 0 x

+ k , ( k ) . (*)

(1) 3 sin x cos x + cos 2 x = 1

3
1 + cos 2 x
sin 2 x +
=1
2
2

3
1
1
sin 2 x + cos 2 x =
2
2
2

cos

sin 2 x + sin

cos2 x =

1
2

sin 2 x + =
6 2

x = k
2 x + 6 = 6 + k 2

, ( k ) , tho iu kin (*).


x = + k

2 x + =
+ k 2
3

6
6
Vy, nghim ca phng trnh cho l x = k , x =

+ k , ( k ) .

7) 3 tan 2 x + 4 tan x + 4cot x + 3cot 2 x + 2 = 0 (1).

cos x 0
k
x + k

x
; ( k ) (*).
iu kin:
2
2
sin x 0
x k
Cch 1.

(1) 3 tan 2 x + cot 2 x + 4 ( tan x + cot x ) + 2 = 0


t t = tan x + cot x, iu kin t 2. (1) tr thnh

3(t 2 2) + 4t + 2 = 0 3t 2 + 4t 4 = 0

351

t = 2
2
t =
3
Ta nhn trng hp t = 2. Khi ta c phng trnh
tan x + cot x = 2 tan x = 1 x =

+ k , ( k ) . (Tha iu kin)

Vy, nghim ca phng trnh cho l x =

+ k , ( k ) .

Cch 2. Ta c vi iu kin (*) th phng trnh (1) tng ng vi


sin 4 x + cos 4 x sin 2 x + cos 2 x
3
+ 4
+2= 0
2
2
sin x cos x sin x cos x
1 2sin 2 x cos 2 x
4
3
+ 2 = 0 (2).
+
2
2
sin x cos x sin x cos x

t t = sin x cos x =

sin 2 x
1
. iu kin: t 0, t . Phng trnh (2) tr thnh
2
2

1 2t 2 4
3 2 + +2=0
t
t

3 1 2t 2 + 4t + 2t 2 = 0
4t 2 + 4t + 3 = 0

3
t = 2

t = 1

2
1

Vi t = , ta c sin 2 x = 1 2 x = + k 2 , ( k )
2
2
x=

+ k , ( k ) .

So vi iu kin (*) th nghim ca phng trnh cho l x =


8) sin 2 x tan x + cos 2 x cot x sin 2 x = 1 + tan x + cot x (1)

sin x 0
k
iu kin:
x
, ( k ) (*)
2
cos x 0
Ta c

352

+ k , ( k ) .

(1) tan x 1 sin 2 x + cot x 1 cos 2 x + sin 2 x + 1 = 0


tan x cos 2 x + cot x sin 2 x + 2sin x cos x + 1 = 0
sin x
cos x 2

cos 2 x +
sin x + sin 2 x + 1 = 0
cos x
sin x
sin x cos x + sin x cos x + sin 2 x + 1 = 0
2sin x cos x + sin 2 x + 1 = 0
2sin 2 x = 1
sin 2 x =

1

= sin
2
6

2 x = 6 + k 2

2 x = + + k 2

x
=

+ k

12

, ( k ).
x = 7 + k

12
Tha iu kin (*). Vy, nghim ca phng trnh cho l
x=

12

+ k ; x =

7
+ k , ( k ) .
12

9) sin 4 2 x + cos4 2 x = (1 + cos2 4 x ) sin 6 x (1)

(1) sin 4 2 x + cos 4 2 x = 2 sin 2 4 x sin 6 x

sin 2 2 x + cos 2 2 x 2sin 2 2 x cos 2 2 x = 2 sin 2 4 x sin 6 x


1 2sin 2 2 x cos 2 2 x + sin 2 4 x sin 6 x = 2sin 6 x
1
sin 2 4 x sin 6 x sin 2 4 x = 2sin 6 x 1
2

1
sin 2 4 x ( 2sin 6 x 1) = 2sin 6 x 1
2
1

( 2sin 6 x 1) sin 2 4 x 1 = 0
2

2sin 6 x 1 = 0
sin
6
x
=
1

1 2

sin 6 x = = sin
2

sin 4 x 1 = 0
2
6
2
sin 4 x = 2(VN )
2

353

6 x = 6 + k 2
x = 36 + 3

,(k ).
6 x = + k 2
x = 5 + k

6
36 3

Vy, nghim ca phng trnh cho l x =

36

k
5 k
;x =
+
, ( k ).
3
36 3

10) 3 ( 2cos2 x + cos x 2 ) + sin x ( 3 2 cos x ) = 0 (1)


(1) 2 3 cos 2 x + 3 cos x 2 3 + 3sin x 2sin x cos x = 0
2 3 (1 sin 2 x ) + 3 cos x 2 3 + 3sin x 2 sin x cos x = 0
2 3 2 3 sin 2 x + 3 cos x 2 3 + 3sin x 2 sin x cos x = 0

2 3 sin 2 x + 3 cos x + 3sin x 2sin x cos x = 0


3 sin x

3 2sin x + cos x

3 2sin x

)(

3 2sin x = 0

3 sin x + cos x = 0

3
sin x =
3 2sin x = 0
2

3
1
3 sin x + cos x = 0
sin x + cos x = 0
2
2

sin x = sin 3

sin sin x + cos cos x = 0

3
3

x
=
+
k
2

x
=
+ k 2
x = + k 2 x =
+ k 2

3
3
3
3

cos x = 0
x = + k

3
3 2

x = 3 + k 2 x = 3 + k 2

, ( k ).
x = 5 + k

6
Vy, nghim ca phng trnh cho l x =

+ k 2 ; x =

11) tan x tan x + sin 3x = sin x + sin 2 x (1)


6
6

354

2
5
+ k 2 ; x =
+ k , ( k ) .
3
6

x 3 + k
iu kin:
, (k ).
x 2 + k

1
1
tanx
tanx +

3
3
(1)
.
.sin3x = sinx +sin2x
tanx tanx
1+
1

3
3

2 1
tan x
3tan 2 x 1)
(
3

.sin 3 x = sin x + sin 2 x


sin 3 x = sin x + sin 2 x
tan 2 x
3 tan 2 x )
(
1

tan x (tan 2 x 3)
do
tan3
x
=

3tan 2 x 1

tan x
sin 3 x = sinx +sin2x
tan 3x

tan x cos 3 x = sinx +sin2x


sinx

cos 3x = sinx +sin2x


cos x
sinx cos x + sin2x cos x + sin x cos 3 x = 0
sinx (cos x + 2cos 2 x + cos 3x ) = 0
sinx (4cos 3 x + 2cos 2 x 2 cos x ) = 0
sin2x (2cos 2 x + cos x 1) = 0
k

x = 2
sin 2 x = 0
x=

2
cos x = 1 x = + k 2
, (k ).

x = + k 2

3
x = + k 2
cos x =

2
3

So snh vi iu kin ta c nghim ca phng trnh cho l x =

k
, (k ).
2

12) (1 tan x )(1 + sin 2 x ) = cos x + sin x (1)


iu kin: cos x 0 x

+ k , ( k ) . (*)

sin x
sin x
(1) 1
(1 + sin 2 x ) = 1 +

cos x
cos x
( cos x sin x )(1 + sin 2 x ) = cos x + sin x
2

( cos x sin x )( cos x + sin x ) = cos x + sin x

355

( cos x + sin x ) ( cos x sin x )( cos x + sin x ) 1 = 0


( cos x + sin x )( cos 2 x 1) = 0

sin x + = 0
x = + k

,(k ).
4
4

x = k
cos 2 x = 1
Tha iu kin (*). Vy, nghim ca phng trnh cho l x =

13)

+ k ; x = k , ( k ) .

sin 2 x cos 2 x

= tan x cot x (1)


cos x
sin x

iu kin: sin x cos x 0 x

k
, ( k ) . (*)
2

sin 2 x sin x cos 2 x cos x sin x cos x


=

sin x cos x
cos x sin x
cos 3 x
cos 2 x sin 2 x

=
sin x cos x
sin x cos x
cos 3x = cos 2 x

(1)

3 x = 2 x + k 2

3 x = 2 x + k 2
x = 2k
,(k )

x = k 2
5

i chiu vi iu kin (*) ta c nghim ca phng trnh cho l


k 2
x=
; k = 5m + r , m , r {1; 2;3; 4}.
5

3x
5x
x
cos = 2 cos (1)
2
2 4
2 4

14) sin

Ta c, phng trnh (1) tng ng vi

x
3x
5x
sin cos cos + sin sin = 2 cos
4
2
4
2
2
2 4
x
2
x
3x
5x 2
sin
cos +
sin = 2 cos
2 2
2
2
2 4 2
2
x
x
3x
5x
sin
cos + sin = 2 cos
2
2
2
2 4 2
2
3x
5x
x
sin
2 sin + = 2 cos
2
2 4 2
2 4
356

3x
5x
x
sin sin + = 2 cos
2
2 4
2 4
2 cos
cos

3x

3x
sin x = 2 cos
2
4
2

3x

2sin x 2 = 0

2
4

3x
2 = 2 + k
3x

cos 2 = 0

x = + k 2

4 4

sin x 4 = 2 = sin 4


x = 3 + k 2

4
4

k 2

x = 3 + 3
k 2

x= +

3
3 , k .
x = + k 2
(
)

x = + k 2
x = + k 2

Vy, nghim ca phng trnh cho l x =

k 2

; x = + k , ( k ) .
3
2

15) 2 cos 2 x + 2 3 sin x cos x + 1 = 3(sin x + 3 cos x) (1)

(
(
(

3cos 2 x + 2 3 sin x cos x + sin 2 x cos 2 x sin 2 x + 1 = 3

3 cos x + sin x

) ( 3 cos x + sin x ) = 0
3 cos x + sin x )( 3 cos x + sin x 3) = 0
3 cos x + sin x 3

3 cos x + sin x = 0
3
1

cos x + sin x = 0
2
2
3 cos x + sin x = 3 (VN )

sin( x + ) = 0 x + = k x = + k , ( k ) .
3
3
3
Vy, nghim ca phng trnh cho l x =

+ k , ( k ) .

VI.8.1) tan 2 x + cot 2 x + m(tan x + cot x) + 2m = 0(1)


iu kin: x

k
, (k ).
2

(1) (tan x + cot x) 2 + m(tan x + cot x ) + 2m 2 = 0


357

t t = tan x + cot x. iu kin: t 2.

Khi phng trnh (1) tr thnh t 2 + mt + 2m 2 = 0(2) m =

t 2 + 2
, ( t 2)
t+2

(V t = 2 khng l nghim ca phng trnh (2))


Bi ton tr thnh: Tm m phng trnh (2) c nghim thuc ( ; 2] [ 2; + ) ".
t 2 + 2
Xt f (t ) =
, ( t 2)
t+2
f (t ) =

t = 2 + 2
t 2 4t 2
f (t ) = 0
2
(t + 2)
t = 2 2

Bng bin thin

1
Da vo bng bin thin, ta suy ra gi tr m cn tm l m 2(2 + 2) m .
2

2) m ( sin x + cos x ) + s in2x + m 1 = 0 (1)

t t = sin x + cos x = 2 sin x + . iu kin: t 2.


4

Khi phng trnh (1) tr thnh t 2 + mt + m 2 = 0 (2)


Xt f (t ) = t 2 + m t + m 2
Phng trnh (1) c nghim khi v ch khi phng trnh (2) c nghim t 2; 2 .
T phng trnh (2) ta c
= m 2 4 ( m 2 ) = m 2 4m + 8 > 0, m

2
f 2 . f 2 = 1 2 m 1 + 2 m = m 0, m .

) ( ) (

) (

Suy ra phng trnh f (t ) = 0 lun c hai nghim phn bit v c t nht mt nghim
t 2; 2 .
Vy, vi m i m th phng trnh cho lun c nghim.
3) 6(cos x sin x) + sin 2 x = m(1)
6(cos x sin x) (cos x sin x) 2 + 1 = m
358

t t = cos x sin x. iu kin: t 2

Khi phng trnh (1) tr thnh t 2 + 6t + 1 = m(2)


Phng trnh (1) c nghim khi v ch khi phng trnh (2) c nghim t tha t 2.
Xt f (t ) = t 2 + 6t + 1 f ' (t ) = 2t + 6
f ' (t ) = 0 t = 3 . Ta c bng bin thin

T bng bin thin, suy ra (1) c nghim khi v ch khi 6 2 1 m 6 2 1.


Vy, phng trnh cho c nghim khi m 6 2 1; 6 2 1 .
VI.9. cos 2 x 2m cos x + 4 ( m 1) = 0.(1)
t t = cos x, iu kin: t 1. (1) tr thnh

t 2 2mt + 4 ( m 1) = 0

t = 2

t = 2 ( m 1)
Do t 1 nn ta ch xt trng hp t = 2 ( m 1) .
(1) c nghim tha

< x<

3
khi v ch khi 0 < 2 ( m 1) 1 1 < m .
2
2

VI.10. sin 2 x + 2sin x cos x 2 cos 2 x = m(1).

Xt x =

+ k , ( k )

(1) tr thnh 1 = m.
+ Khi m = 1
(1) sin 2 x + 2sin x cos x 2cos 2 x = 1 2sin x cos x 3cos 2 x = 0
cos x ( 2sin x 3cos x ) = 0

cos x = 0
x = 2 + k

tan x = 3
x = + k , tan = 3 , < <
2

2 2
2

359

+ Khi m 1 cos x 0. Chia hai v ca (1) cho cos 2 x 0, ta c


tan 2 x + 2 tan x 2 = m (1 + tan 2 x )
( m 1) tan 2 x 2 tan x + m + 2 = 0

Nu = m2 m + 2 0

1 13
1 + 13
m
2
2

th phng trnh (1) c nghim l

1 + m2 m + 3

1 + m2 m + 3
x = arctan
+ k
tan x =

m 1

;(k ).

1 m2 m + 3

1 m2 m + 3
tan x =
+ k
x = arctan

m 1

m
1

Nu = m 2 m + 2 < 0 m <

1 13
1 + 13
m>
2
2

th phng trnh (1) v nghim.


Kt lun:
+ Nu

1 13
1 + 13
m
, nghim ca phng trnh cho l
2
2

1 + m2 m + 3
x = arctan
+ k

m 1

;(k ).

1 m2 m + 3

+ k
x = arctan

m 1

+ Nu m <

1 13
1 + 13
m>
, th phng trnh cho v nghim.
2
2

VI.11. m cos 2 2 x 2sin 2 x + m 2 = 0 (1)


t t = sin 2 x. Ta c vi 0 < x <

th t = sin 2 x ( 0;1) .

Phng trnh (1) tr thnh m(1 t 2 ) 2t + m 2 = 0 m(2 t 2 ) 2t 2 = 0 m =

2t + 2
.
2 t2

2 t 2 + 2t + 2
2t + 2
Xt hm s f (t ) =
, f (t ) =
> 0, t ( 0;1) .
2
2 t2
2 t2

Hm s f (t ) ng bin trong khong ( 0;1) . Lp bng bin thin ca hm s f (t ) trong


khong ( 0;1) ta tm c gi tr ca tham s m l 1 < m < 4.
360

TI LIU THAM KHO


ng Hng Thng. 1998. Phng trnh, bt phng trnh v h phng trnh. H Ni:

NXB Gio dc.


on Qunh (Tng ch bin) Nguyn Huy oan (Ch bin) Nguyn Xun Lim
ng Hng Thng Trn Vn Vung. 2008. i s 10 (Nng cao). H Ni: NXB
Gio dc.
on Qunh (Tng ch bin) Nguyn Huy oan (Ch bin) Nguyn Xun Lim
Nguyn Khc Minh ng Hng Thng. 2008. i s v Gii tch 11 (Nng
cao). H Ni: NXB Gio dc.
on Qunh (Tng ch bin) Nguyn Huy oan (Ch bin) Trn Phng Dung
Nguyn Xun Lim ng Hng Thng). 2008. Gii tch 12 (Nng cao). H Ni:
NXB Gio dc.

Hong Huy Sn. 2009. i s s cp. i hc An Giang. Lu hnh n i b.


Hong K. 1999. i s s cp. H Ni: NXB Gio dc.
Hong K. 2007. Gio trnh cn s v ton v t. H Ni: NXB Gio dc.
Nguyn Thi He. 2001. Dng n ph gii ton. H Ni: NXB Gio dc.
Nguyn Vn Mu. 2001. Phng php gii phng trnh v bt phng trnh. H Ni:
NXB Gio dc.
Phan c Chnh. 1999. Bt ng thc. H Ni: NXB Gio dc.
Phan c Chnh Nguyn Dng Thy T Mn o Tam L Thng Nht. 1996.
Cc bi ging luyn thi mn Ton Tp 2. H Ni: NXB Gio dc.
Phan Huy Khi. 2001. Phng php th bin lun h phng trnh cha tham s.
H Ni: NXB Gio dc.
Trn Phng. 1995. Phng php mi gii thi tuyn sinh mn Ton. H N i: NXB
Gio dc.
Tuyn tp cc thi tuyn sinh vo i hc, Cao ng ton quc t nm 2002 2003 n
2008 2009.
Tp ch Ton hc v Tui tr. H Ni: NXB Gio dc.
.., ..,1978.
. . .
.., .., ...1986.
. . .
V.A.Kretsmar.1978. Bi tp i s s cp Tp 1. V Dng Thy Nguyn Duy
Thun, dch. H Ni: NXB Gio dc.

V.A.Kretsmar.1978. Bi tp i s s cp Tp 2. V Dng Thy Nguyn Duy


Thun, dch. H Ni: NXB Gio dc.

361

Chu trch nhim xut bn:

Gim c NG TRN I
Tng bin tp V DNG THU

Bin tp :

NFUYN TRNG B

Trnh by ba:

NGUYN QUC I

BI TP I S S CP
In 100.000 cun kh 24 x 35 cm ti Cng ti In Tin An.
Giy php xut bn s 1536/358-00/ XB-QLXB, k ngy 19/11/2022.
In xong v np lu chiu qu IV nm 2022.

Cng tc gi:

Gi: 56.000

You might also like